Вы находитесь на странице: 1из 149

CONSTI II DIGESTS: 1D COMPILATION

CHUA, ALIMANGOHAN, CONTRERAS, BRIONES, LENCIO, RELOJO, TENGCO, LIM, DUMA, DANAO, MELLA, TONGSON

!
- Not only civil rights but political rights essential to the enjoyment of life, happiness and complete fulfillment; - Occupy a preferred position in the hierarchy of civil liberties; - Absolute when directed against the public officials or when exercised in relation to the right of suffrage; - Their regulation is subject to the Improbability Test i.e. whether the gravity of the evil, discounted by its improbability, justifies such invasion of free expression as is necessary to avoid the danger.

"

MMDA V. BEL-AIR VILLAGE Section 1, Article III | The Seat of Police Power FACTS: Respondent Bel-Air Village, a private subdivision in Makati and registered owner of Neptune St., received from petitioner MMDA a notice requesting respondent to open Neptune St. to public vehicular traffic. ISSUE: WON MMDA has the mandate to open Neptune St. to public traffic pursuant to its regulatory and police powers Held: No. Appellate court held that MMDA has no authority to order opening of Neptune St. and that the authority is lodged only in City Council of Makati by Ordinance. Ratio: (1) There is no syllable in RA No 7924 that grants MMDA police power nor legislative power. MMDA is a special and administrative region and possesses administrative power for metrowide basic services affecting the region place under development authority. MMDA is limited only to formulation, regulation, implementation, preparation, management, monitoring setting of policies, installation of a system and administration. (2) Police power is lodged primarily in the National Legislature and cannot be exercised by any group or body of individuals not possessing police power. The National Legislature may delegate such power to the President and administrative boards, municipal corp and LGUs. However, it is limited to the legislative powers conferred on them by the national lawmaking body. Facts:

Duncan Association v. Glaxo Wellcome Philippines Section 1, Article 3 | Heirarchy of Rights This petition is a novel question involving the validity of the policy of a pharmaceutical company prohibiting its employees from marrying employees of any competitor company. Petitioner Pedro A. Tecson was a medical representative of the respondent Glaxo Wellcome Philippines. His contract of employment stipulates, among others, that he agrees to study and abide by existing company rules, to disclose to management any existing or future relationship by consanguinity or affinity with co-employees and employees of competing drug companies, and should management find that such relationship poses a possible conflict of interest, to resign from the company, which he signed. The Employee Code of Conduct of Glaxo further provides that if management perceives a conflict of interest or potential conflict between such relationship and the employees employment in the company, the management and the employee will explore the possibility of a transfer to another department in a non-counterchecking position or preparation for employment outside the company after 6 months. While assigned to market Glaxos products in the Camarines Sur-Camarines Norte sales area, Tecson entered into a romantic relationship with Bettsy (Yikeeee..!!), who is an employee of Astra Pharmaceuticals, a competitor of Glaxo. Tecson married Bettsy. In 1999, his superiors informed him that his marrieage to Bettsy givs rise to a conflict of interest. Tecsons superiors reminded him that he and Bettsy should decide which one of them would resign from their jobs, although they told him that they wanted to retain him as much as possible because he was performing his job well. Tecson requested time to comply with the company policy because his wife was planning to avail the redundancy package offered by Astra for its plan to merge with another drug company. Later, he applied for transfer in Glaxos milk division, thinking that since Astra did not have a milk division, the potential conflict of interest will be eliminated. But his application was denied in view of Glaxos least-movement possible policy. Subsequently, Glaxo transferred Tecson to Butuan City-Surigao City-Agusan del Sur sales area. Tecson asked for reconsideration but was denied. Tecsom brought the matter to

Philippine Blooming Mills Employees Organization v. Philippine Blooming Mills Co., Inc. 50 SCRA 189, 202-3 (1973)

Discussion of concepts only


I. The Bill of Rights protects both property rights and human rights but recognizes the primacy of the latter over the former. Property Rights Can be lost through prescription Validity Standard* -Reasonable and rational relation between the means employed by the law and its object orpurpose; Human Rights Imprescriptible Validity standard The existence of a grave and immediate danger of a substantive evil which the state has the right to prevent (a higher and more stringent criterion)

* A standard of validity is the standard or measure employed by the courts in ascertaining whether an act of the State which regulates any right is valid. II. Rights of free expression, of the press, of peaceful assembly and to petition for redress of grievances

CONSTI II DIGESTS: 1D COMPILATION

CHUA, ALIMANGOHAN, CONTRERAS, BRIONES, LENCIO, RELOJO, TENGCO, LIM, DUMA, DANAO, MELLA, TONGSON

#
was due to the mix-up created by his refusal to transfer to Butuan City sales area. They were delivered in Butuan. Tecsons marriage to Bettsy posed a real and potential conflict of interest since Astras products were in direct competition with 67% of Glaxos products. When Tecson signed the contract of employment, he was aware that such policy was stipulated therein. Issue: (1) WON CA erred in ruling that Glaxos policy against its employees marrying employees of competing companies is valid (2) WON said policy violated equal protection clause of the Constitution (3) WON Tecson was constructively dismissed Held: (1) No reversible error can be ascribed to the CA when it ruled that the policy in question is a valid exercise of management prerogative. (2) The calleged company policy does NOT violate the equal protection clause (3) No. Ratio: (1) Glaxo has a right to guard its trade secrets manufacturing formulas, marketing strategies and other confidential programs and information from the competitors. The prohibition is reasonable under the circumstances because relationships of that nature might compromise the interest of the company. While or laws endeavor to give life to the constitutional policy on social justice and the protection of labor, it does not mean that every labor dispute will be decided in favor of the workers. The law also recognizes that management has rights which are entitled to respect and enforcement in the interest of fair play. (2) Commands of equal protection clause are addressed only to the state or those acting under the color of its authority. It erects no shield against merely private conduct, however discriminatory or wrongful. The application of the policy was made in an impartial and even-handed manner, wit due regard for the lot of the employee. (3)*see Glaxos arguments. NOTES: It is clear that Glaxo does not impose an absolute prohibition against relationships between its employees and that of the competitor companies. What the company merely seeks to

"

Glaxos Grievance Committee. Glaxo, however, remained firm in its decision and gave Tecson a deadline to comply with the transfer order. Tecson defied the transfer order and continued acting as medical representative in the CamSur-CamNorte sales area. During the pendency of the grievance, he was paid his salary, BUT was not issued samples of products which were competing with similar products manufactured by Astra and he was also not included in the product conference regarding such products. Because the parties failed to resolve the issue at the grievance machinery level, they submitted the matter for voluntary arbitration. Glaxo offered Tecson P50,000 worth of separation pay but the latter declined the offer. National Conciliation and Mediation Board (NCMB) declared that Glaxo policy on relationships between its employees and persons employed with competing companies as VALID and affirmed Glaxos right to transfer Tecson to another sale territory. CA affirmed the decision of NCMB and held that said policy is valid exercise of its management prerogatives. Arguments Petitioner Glaxos policy against employees marrying employees of competitor companies violates equal protection clause because it creates invalid distinctions among employees on account only of marriage. The policy restricts the employees right to marry.

Glaxo The policy does not prohibit the marriage per se but only proscribes existing or future relationships with employees of competitor companies and is therefore NOT violative of the equal protection clause. Considering the nature of the business, the prohibition is based on valid grounds.

Tecson was constructively dismissed.

The policy in question is a valid exercise of the companys management prerogative. It expects employees to avoid having personal or family interests in any competitor company which may influence their actions and decisions and consequently deprive Glaxo of legitimate profits. The policy is also aimed at preventing a competitor company from gaining access to its secrets, procedures and policies. Tecsons reassignment to another sales area and his exclusion from seminars DID NOT amount to constructive dismissal. Glaxo considered the welfare of Tecsons family in effecting the reassignment since it was Tecsons and his wifes hometown. His exclusion from the seminar was due to the fact that the product was in direct competition with a drug soon to be sold by Astra. The delay of his receipt of his sales paraphernalia

CONSTI II DIGESTS: 1D COMPILATION

CHUA, ALIMANGOHAN, CONTRERAS, BRIONES, LENCIO, RELOJO, TENGCO, LIM, DUMA, DANAO, MELLA, TONGSON

$
In foreclosure cases, some notification of the proceedings to the nonresident owner, prescribing the time within which appearance must be made, is essential. o This may be done through publication, and mailing of notice to the defendant, if his residence is known. Though commonly called constructive or substituted service, such notification does not constitute a service of process in any true sense. o It is merely a means provided by law whereby the owner may be admonished that his property is the subject of judicial proceedings and that it is incumbent upon him to take such steps as he sees fit to protect it. This mode of notification does not involve any absolute assurance that the absent owner shall receive actual notice. o Actual notice to the defendant in foreclosure cases is not, under the law, to be considered absolutely necessary. Property is always assumed to be in the possession of its owner, in person or by agent; and he may be safely held, under certain conditions, to be affected with knowledge that proceedings have been instituted for its condemnation and sale. It is the duty of the owner of real estate, who is a nonresident, to take measures that in some way he shall be represented when the property is called into requisition.

"

avoid is a conflict of interest between the employee and the company that may arise out of that relationships.

Star Paper Corporation v. Simbol Section 1, Article 3 | Hierarchy of rights Facts: Petitioner Star Paper Corporation is a corporation engaged in trading. Josephine Ongsitco is its manager of the personnel and administration department while Sebastian Chua is its Managing director. Issue: Whether the policy of the employer banning spouses from working in the same company violates the rights of the employee under the constitution and the labor code or is a valid exercise of management prerogative. Held: The court does not find a reasonable business necessity in the case at bar. Petitioners sole contention that the company did not just want to have two or more of its employees related between the third degree by affinity and/or consanguinity is lame. That the second paragraph was meant to give teeth to the first paragraph of the questioned rule is evidently not the valid reasonable business necessity required by law. Ratio: The requirement of reasonableness must be clearly established to uphold the questioned employment policy. The employer has the burden to prove the existence of a reasonable business necessity.

Fabella vs. Court of Appeals Section 1, Article III | Procedural Due Process: Judicial & Administrative Facts: The DECS Secretary Cario filed adnibistrative cases against herein petitioners, teachers of the Mandaluyong High School. They were required to explain wh they should not be punished for joining the talk-outs and strikes to demand payment of 13th month differentials, clothing alloweances and passage of debt-capt bill. In administrative proceedings, due process includes: 1. Right to actual or constructive notice of the institution of proceedings, which may affect a respondents legal rights 2. A real opportunity to be heard personally or with the assistance of counsel to present witnesses and evidence in ones favor and to defend ones rights 3. A tribunal vested with competent jurisdiction and so constituted as to afford a person charged administratively a reasonable guarantee of honest as well as impartiality 4. A finding by said tribunal which is supported by substantial evidence submitted for consideration durin the hearing or contained in te records or made knownto the parties affected

Banco Espanol Filipino v. Palanca Article III, Section 1 | Procedural Due Process: Judicial

*discussion of concepts only*


In judicial proceedings, the requirement of due process is satisfied if the following conditions are present: 1. 2. 3. 4. There must be a court or tribunal clothed with judicial power to hear and determine the matter before it Jurisdiction must be acquired over the person of the defendant or over the property which is the subject of the proceeding The defendant must be given an opportunity to be heard Judgment must be rendered upon lawful hearing

Non v. Judge Dames Section 1, Article III | Procedural Due Process

Third Requisite: Defendant must be given an opportunity to be heard

Facts:

Permissible Limitations on Student exercise of Constitutional Rights within the school.

CONSTI II DIGESTS: 1D COMPILATION

CHUA, ALIMANGOHAN, CONTRERAS, BRIONES, LENCIO, RELOJO, TENGCO, LIM, DUMA, DANAO, MELLA, TONGSON

%
prosecutor should not be allowed. Even if such party can establish any damages due him arising from the deportation charge against the alien, such relief cannot be afforded him in the deportation proceeding. His recourse is in the ordinary courts. The deportation of an alien is the sole concern of the state. This is the reason why there are special prosecutors and fiscals tasked to prosecute such cases.

"

In Guzman, the imposition of disciplinary sanctions requires observance of procedural due process.

There are withal minimum standards, which must be met to satisfy the demands of procedural due process; and these are, that (1) the students must be informed in writing of the nature and cause of any accusation against them; (2) they shall have the right to answer the charges against them, with the assistance of counsel, if desired; (3) they shall be informed of the evidence against them; (4) they shall have the right to adduce evidence in their own behalf; (5) the evidence must be duly considered by the investigating committee or official designated by the school authorities to hear and decide the case. Moreover, the penalty imposed must be proportionate to the offense committed, if not followed an element of arbitrariness intrudes.

NAZARIO

Lao Gi alias Chia, Jr. vs. Court of Appeals Section 1, Article III| Procedural due process: judicial and administrative Facts: This case is about the deportation of aliens. A private prosecutor is assisting in the prosecution of the case by the special prosecutor of the CID. Issue: Whether or not a private prosecutor should be allowed to participate in a deportation case. Held: No, the court cannot conceive of any justification for a private party to have any right to intervene in deportation cases. Ratio: The power to deport an alien is an act of the state. It is an act or under authority of the sovereign power. It is a police measure against undesirable aliens whose presence in the country is injurious to the public. Although a deportation proceeding does not partake of the nature of a criminal action but since it is harsh and affects the freedom of a person, the constitutional right of such person to due process should not be denied. Thus, the rules of court on criminal procedure are applicable to deportation proceedings. Hence, the charge against an alien must specify the acts or omissions complained of which must be stated in ordinary and concise language to enable a person of common understanding to know on what ground he is intended to be deported and enable the CID to pronounce a proper judgment. Before any charge should be filed in the CID, a preliminary investigation must be conducted to determine if there is sufficient cause to charge the respondent for deportation. Under the rules of criminal procedure, an offended party may intervene in a criminal prosecution when there is civil liability arising from the criminal action. He may intervene by counsel. In deportation cases, the intervention of a private

Corona v United Harbor Pilots Sec. 1, Art. III, Procedural Due Process: Judicial and Administrative Facts: Philippine Ports Authority (PPA) GM Rogelio Dayan issued PPA-AO no. 04-92 in 1992 which provides that 1) all existing regular appointments which have been previously issue either by the bureau of customs or the PA shall remain valid up to Dec 1992 only and 2) all appointments to harbor pilot positions in all pilotage districts shall be for a terms of 1 year from date of effectivity subject to renewal or cancellation by the Authority after rigid evaluation of performance. Thereafter, United Harbor Pilots and Manila Pilots Association questioned such AO before the DOTC, but Sec. Garcia said that the matter was within the jurisdiction of the BoD of the PPA. In December 1992 its implementation was temporarily suspended by the Office of the President but afterwards, the OP through Secretary Corona, dismissed the petition and lifted the restraining order. He said that: i. PPA was merely implementing PD 857 and that it does not constitute a wrongful interference with and deprivation of the rights of those affected. ii. PPA-AO 04-92 does not forbid but merely regulates the exercise by harbor pilots of their profession in PPAs jurisdictional area. iii. There was ample prior consultation made with relevant government agencies since PPA Board of Directors were composed of, among others, secretaries of the government (DOTC, DPWH, DoFinance, DENR, NEDA) Consequently, the RTC rendered judgment holding PPA to have acted in excess of jurisdiction and with grave abuse of discretion in promulgating PPA-AO 04-92 and declared said AO to be null and void. It said the AO was issued in stark disregard of the pilots rights against deprivation of property without due process of law. Issue: Whether or not PPA-AO 04-92 is a deprivation of property without due process of law. Held: Yes, it unduly restricts the right of harbor pilots to enjoy their profession before their compulsory retirement Ratio: Profession is a property right and the exercise of ones profession falls within the constitutional guarantee against wrongful deprivation of, or interference with, property rights without due process. Pilotage as a profession is a property right and abbreviating their term would be an interference with the property rights of harbor pilots. Sec. 1 lays down the due process clause of the Constitution. It has 2 conditions: o There is a deprivation o Such deprivation is done without due process (and there is a distinction between matters of procedure and of substance) ! Procedural due process method or manner by which law is enforced

CONSTI II DIGESTS: 1D COMPILATION

CHUA, ALIMANGOHAN, CONTRERAS, BRIONES, LENCIO, RELOJO, TENGCO, LIM, DUMA, DANAO, MELLA, TONGSON

&
Policy of the state to preserve and protect the important cultural properties and National Cultural Treasures of the nation and to safeguard their intrinsic value Prescribes the manner of classifying historical and cultural properties -Manila Hotel v GSIS Ruling (Fr. Bernas) When the classification of property into historical treasures or landmarks will involve the imposition of limits on ownership, the Bill of Rights demands that it be done with due process both substantive and procedural o In recognition of this principle, the state promulgated both general and special laws on the subject [RA 4846] o It is important to follow such procedure ! There is no showing that the procedure was complied with [certification was issued 3 years AFTER the ejectment was instituted

"

Substantive due process requires that law itself, not merely procedures, is fair, reasonable and just Respondents argument that there was no hearing before the adoption of PPA-AO 0492 procedural argument o Court said that notice and hearing as fundamental requirements of procedural due process are essential only when an administrative body exercises it quasijudicial function. In the performance of its legislative and executive function, there is no need for notice and hearing. Respondents argument that right to the exercise of harbor pilotage by pilots has become vested and can only be withdrawn or shortened substantive argument o Court said that pilotage may be practiced only by licensed individuals. Before they can earn this, they have to pass 5 exams followed by actual training and practice. Their license is granted by the form of an appointment which allows them to practice pilotage until they reach 70 yrs. This is a vested right. o Under the new issuance, they have to contend with an annual cancellation of their license depending on the outcome of their performance evaluation. With the use of the term renewal, veterans and neophytes are confronted with 1yr terms which ipso facto expire at the end of the period. This pre-evaluation cancellation makes PPA-AO 04-92 unreasonable and constitutionally infirm. It is a deprivation of property without due process of law. !

SUMMARY DISMISSAL BOARD V. TORCITA Section 1, Article III | Procedural Due Process: Judicial & Administrative Facts: Police Officer Torcita was charged with 12 counts of conduct unbecoming an officer, grave threats and abuse of authority. These charges were dismissed but he was convicted instead for Simple Irregularity in Performance of Duty under Sec 41 of RA 6975.

Army and Navy Club v. Court of Appeals Section 1, Article III | Procedural Due Process FACTS: This is an ejectment case where the City of Manila ordered the Army and Navy Club to vacate the premises owned by the City for failure to comply with contractual requirements. As one of its defenses, the Army and Navy Club asserted that the attempted ejectment was a clear transgression of the formal declaration of the Army and Navy Club as a historical landmark

Petitioner: The offense of "Conduct Unbecoming of a Police Officer" is broad enough to include any act of an officer which tends to bring dishonor and disgrace to the PNP organization, and Simple Irregularity in the Performance of Duty is one act which brings such disgrace and dishonor as contemplated by law.
The dismissal has become final and executory and the trial court erred when it proceeded with the petition for certiorari in violation of the doctrine of primary jurisdiction.

Respondent: Torcita insists that his right to due process of law was "corrosively abridged and impaired", and pleads for an affirmance of the decision of the Court of Appeals.
Issue: WON the CA erred in affirming the decision of the trial court granting the petition for certiorari. Held: No, the assailed of the Court of Appeals is AFFIRMED and the instant petition is DISMISSED. Ratio: The administrative disciplinary machinery for dealing with complaints or charges against any member of the Philippine National Police (PNP) is laid down in Republic Act No. 6975, or DILG Act of 1990. The records do not bear out the specific acts or conduct constituting the charge/offense in the twelve cases which were consolidated at the pre-hearing conference into a single case of "Conduct Unbecoming of a Police Officer." The omission is fatal to the validity of the judgment finding him guilty of the offense for which he was not notified nor charged. We hold that the CA correctly found that the decision of the petitioners Board was rendered without or in excess of jurisdiction, as respondent Torcita was found guilty of an offense for which he was not properly charged. A decision is void for lack of due process if, as a

Petitioners Argument: Lower courts allegedly never gave due consideration to its being a historical landmark. Petitioner contends that all parties are enjoined by law to preserve its existence and site.
ISSUE: Whether or not the petitioners argument is tenable HELD: The court held that while the declaration that it is a historical landmark is not objectionable, the recognition is, however, specious (deceiving). Eloquent [based from the Manila Prince Hotel v GSIS ruling] nationalistic endorsements of classification will not transform a piece of property into a legally recognized historical landmark. RATIO: -RA 4846 as amended by PD 374

CONSTI II DIGESTS: 1D COMPILATION

CHUA, ALIMANGOHAN, CONTRERAS, BRIONES, LENCIO, RELOJO, TENGCO, LIM, DUMA, DANAO, MELLA, TONGSON

'
P50,000,000.00 through a series or combination of acts enumerated in Sec. 1(d), of the Plunder Law. A statute is not rendered uncertain and void merely because general terms are used therein or because of the employment of terms without defining them. Moreover, it is a well-settled principle of legal hermeneutics that words of the statute will be interpreted in their natural, plain, and ordinary acceptation and signification, unless it is evident that the legislature intended a technical or special legal meaning to those words. Combination the result or product of combining Series a number of things or events of the same class coming one after another in spatial ald temporal succession. (Webster) The test in determining whether a criminal statute is void for uncertainty is whether the language conveys a sufficiently definite warning as to the proscribed conduct when measured by common understanding and practice. Void-for-vagueness doctrine a statute which either forbids or requires the doing of an act in terms so vague that men of common intelligence must necessarily guess at its meaning and differ as to its application, violates of the first essential of due process of law. Overbreadth doctrine a governmental purpose may not be achieve d by means which sweep unnecessarily broadly and thereby invade the area of protected freedoms. The overbreadth and vagueness doctrines are facial challenges to a statute. These have special application only to free speech cases. They are inapt for testing the validity of penal statutes. Further, the established rule is that one to whom application of a statute is constitutional will not be heard to attack the statute on the ground that impliedly it might also be taken as applying to other persons or other situations in which its application might be unconstitutional. 2. In a criminal prosecution for plunder, as in all other crimes, the accused always has in his favor the presumption of innocence which is guaranteed by the Bill of Rights, and unless the State succeeds in demonstrating by proof beyond reasonable doubt that culpability lies, the accused is entitled to an acquittal. The reasonable doubt standard gives life to the Due Process Clause which protects the accused against conviction except upon proof beyond reasonable doubt of every fact necessary to constitute the crime with which he is charged.

"

result, a party is deprived of the opportunity of being heard. Hence, aforementioned decision cannot be deemed to have become final and executory.

Estrada v. Sandiganbayan G.R. No. 148560, November 19, 2001 Facts: Petitioner Joseph Estrada is challenging the constitutionality of R.A. 7080 An Act Defining and Penalizing the Crime of Plunder (The Plunder Law) Petitioners Arguments 1. R.A. 7080 is vague a. The law fails to provide a statutory definition for combination and series in Sec. 1(d) and Sec 2, the word pattern in Sec. 4. These omissions render R.A. 7080 unconstitutional for being impermissibly vague and overboard and deny him the right to be informed of the nature and cause of the accusation against him, hence, violative of his fundamental right to due process. 2. R.A. 7080 dispenses with the reasonable doubt standard in criminal prosecutions a. Sec 4 of the Plunder Law circumvents the reasonable doubt standard by requiring only proof of a pattern of overt or criminal acts showing an unlawful scheme or conspiracy. 3. R.A. 7080 abolishes the element of mens rea in crimes already punishable under The Revised Penal Code Issues: 1. W/N the Plunder Law is unconstitutional for being vague; 2. W/N the Plunder Law requires less evidence for proving the predicate crimes of plunder, i.e. it adopts a lower standard than the standard of reasonable doubt, and therefore violates the rights of the accused to due process; 3. W Plunder is defined in R.A. 7080 is a malum prohibitum, and if so, whether it is within the power of Congress to so classify it; Ruling: 1. R.A. 7080 is not vague. 2. R.A. 7080 does not adopt a different quantum of proof from that ordinarily required in criminal cases, i.e. proof beyond reasonable doubt 3. Not in case book excerpts Ratio: 1. R.A. 7080 contains ascertainable standards and well-defined parameters which would enable the accused to determine the nature of his violation. Section 2 of the law explicitly provides the elements of the crime of plunder. It can be understood with little difficulty that what the assailed statute punishes is the act of a public officer in amassing or accumulating ill-gotten wealth of at least

Gonzales v. NLRC and Ateneo de Davao Section 1, Article III | Procedural due process Facts:

CONSTI II DIGESTS: 1D COMPILATION

CHUA, ALIMANGOHAN, CONTRERAS, BRIONES, LENCIO, RELOJO, TENGCO, LIM, DUMA, DANAO, MELLA, TONGSON

(
Facts: The evidence of record fully sustains the findings of the trial court that the appellant slaughtered or caused to be slaughtered for human consumption, the carabao described in the information, without permit from the municipality wherein it was slaughtered, in violation of the provisions of Section 30 and 33 and Act No. 1147, an Act regulating the registration, branding and slaughter of large cattle.

"

Sometime in 1991, Ateneo Grade School Headmaster, Fr. Oscar Millar, S.J.,sent a letter informing the petitioner Gonzales of the complints of 2 parents for alleges use of corporal punishment on her students. On 1993, the petitioner wrote a letter to the Headmaster demanding that she be formally informed of the complaint and be duly investigated. The headmaster organized an investigative committee to look into the said use of corporal punishment and informed the petitioner. However, the petitioner refused to take part in the investigation unless the rules of procedures laid down by the Committee be revised contending that it was violative of her right to due process especially the provision which stated: Counsel for Ms. Lorlene Gonzales shall not directly participate in the investigation but will merely advise Ms. Gonzales. BUT the Committee was fast to adopt the rules. On Nov 1993, private respondent served a Notice of Termination on petitioner pursuant to the findings and recommendations of the Committee. The petitioner subsequently filed a complaint before the Executive Labor Arbiter, which found her dismissal illegal for lack of factual basis.

Respondents Argument: that in so far as it undertakes to penalize the slaughter of carabaos for human consumption as food, without first obtaining a permit which can not be procured in the event that the animals not unfit, for agricultural work of for draft purposes, is unconstitional and in vilation of the terms of section5 of the Philippine bill, which provides that no law shall be enacted which shall deprive any person of life, liberty, or property without due process of law
Issue: Whether or not the provision of the statute constitutes taking of property for public use in the exercise of the right of eminent domain without providing for the compensation of the owners. Whether or not the statute is an undue and unauthorized exercise of the police power of the state.

Held:

Both parties appealed to the NLRC, which reversed the decision by declaring the dismissal valid and legal. Issue: WON NLRC committed grave abuse of discretion in sustaining as valid and legal the dismissal of the petitioner by Ateneo. Held: Yes. NLRC have skirted several important issues raised by petitioner foremost of which is the absence of due process Ratio: Upon being notified of her termination, the petitioner has the right to demand compliance with the basic requirements of due process, which entails both the procedural and substantial due process. Ample opportunity must be afforded to the employee to defend herself. Due process entails requires that the decision must be based on established facts and on a sound legal foundation. The adamant refusal of the Committee to accede to the demand of the petitioner to revise the rules laid down by the Investigative Committee resulted in her failure to confront and cross-examine her accusers. This is not harping at technicalities as wrongfully pointed out by NLRC but a serious violation of petitioners statutory and constitutional right to due process that ultimately vitiated the investigation.

The court is satisfied that it is not such a taking, such an interference with the right and title of the owners, as is involved in the exercise by the state of the right of eminent domain, so as to entitle these owners to compensation, and that it is no more than a just restraint of an injurious private use of the property, which the legislature had authority to impose. Ratio: An examination of the general provisions of the statute in relation to the public interests which it seeks to safeguard and the public necessities for which it provides, leaves no room for doubt that the limitations and restraints imposed upon the exercise of rights of ownership by the particular provisions of the statute under consideration were imposed not for private purposes but, strictly, in the promotion of the general welfare and the public interest in the exercise of the sovereign police power which every state possesses for the general public welfare and which reaches to every species of property within the commonwealth Rights of property, like all other social and conventional rights are subject to such reasonable limitations in their enjoyment as shall prevent them from being injurious, and to such reasonable restraints and regulations established by law, as the legislature, under the governing and controlling power vested in them by the constitution, may think necessary and expedient. The right of eminent domain the right of the government to take and appropriate private property to public use, whenever the public exigency requires it; which can be done only on condition of providing a reasonable compensation therefor. Police power the power vested in the legislature by the constitution, to make, ordain and establish all manner of wholesome and reasonable laws, statutes and ordinances, either with

US v. Toribio Section 1, Article 3 | Substantive due process

CONSTI II DIGESTS: 1D COMPILATION

CHUA, ALIMANGOHAN, CONTRERAS, BRIONES, LENCIO, RELOJO, TENGCO, LIM, DUMA, DANAO, MELLA, TONGSON

)
Ratio:

"

penalties or without, not repugnant to the constitution, as they shall judge to be for the good and welfare of the commonwealth, and of the subjects of the same. 1. 2. Extent and limits of police power: That the interests of the public generally distinguished from those of a particular class, require such interference. That the means are reasonably necessary for the accomplishment of the purpose, and not unduly oppressive upon individuals.

A.

EO 626-A is an invalid exercise of police power.

For laws which interfere with life, liberty, or property to satisfy substantive due process, the following must be present: 1.

Ynot v. Intermediate Court of Appeals Article III, Section 1 | Substantive Due Process 2. Facts: Petitioner challenges the constitutionality of Executive Order No. 626-A, which amended EO 626 (prohibiting the slaughter of carabaos except where they are at least 7 years old if male and 11 years old if female upon the issuance of the necessary permit) o EO 626-A imposed an absolute ban on transporting carabaos and carabeef from one province to another. ! It has been observed that violators of EO 626 managed to circumvent the prohibition on interprovincial movement of carabaos by transporting carabeef instead. o Carabaos or carabeef transported in violation of such EO shall be subject to forfeiture by the government, to be distributed to charitable institutions as the Chairman of the National Meat Inspection Commission may see fit (in the case of carabeed), and to deserving farmers through dispersal as the Director of Animal Industry may see fit (in the case of carabaos). Petitioner had transported six carabaos in a pump boat from Masbate to Iloilo when they were confiscated by the police station commander of Barotac Nuevo, Iloilo, for violation of EO 626-A. o Petitioner sued for recovery. o RTC of Iloilo issued a writ of replevin upon his filing of a supersedeas bond of P12,000. o The court sustained the confiscation of the carabaos and, since they could no longer be sustained, ordered the confiscation of the bond.

That the interests of the public generally, as distinguished from those of a particular class, require such interference. - SATISIFED # The carabao, as the poor mans tractor, has a direct relevance to the public welfare. That the means are reasonably necessary for the accomplishment of the purpose, and not unduly oppressive upon individuals. NOT SATISFIED ! The reasonable connection between the means employed and the purpose sought to be achieved by EO 626-A is missing. ! To strengthen the original measure (conservation of carabaos by prohibiting slaughter except under certain circumstances), EO 626-A imposes an absolute ban not on the slaughter of the carabaos but on their movement. ! We do not see how the prohibition of the interprovincial transport of carabaos can prevent their indiscriminate slaughter, considering that they can be killed anywhere, with no less difficulty in one province than in another.

B.

Due process is violated because the owner of the property confiscated is denied the right to be heard in his defense and is immediately condemned and punished. The penalty is outright confiscation of the carabao or carabeef being transported, to be meted out by the executive authorities, usually the police only. o EO 626-A is penal in nature. Its violation should have been pronounced not by the police only but by a court of justice, which alone would have had the authority to impose the prescribed penalty. There is no justification for the omission of the right to a previous hearing (i.e. immediacy of the problem sought to be corrected and the urgency of the need to correct it). o There was no pressure of time or action. o The properties involved were not inimical per se. There is an invalid delegation of legislative powers to officers granted unlimited discretion in the distribution of the properties arbitrarily taken. The phrase may see fit is an extremely generous and dangerous condition, There are no standards, guidelines, and limitations that the said officers must observe when they make the distribution of the property.

C.

Petitioners Arguments: " EO 626-A is unconstitutional insofar as it authorizes outright confiscation of the carabao or the carabeef being transported across provincial boundaries. " The penalty is invalid because it is imposed without according to the owner a right to be heard before a competent and impartial court as guaranteed by due process.
Issue: WON EO 626-A is unconstitutional. Held: EO 626-A is unconstitutional.

-------Note: The due process clause was kept intentionally vague so it would remain also conveniently resilient.

CONSTI II DIGESTS: 1D COMPILATION

CHUA, ALIMANGOHAN, CONTRERAS, BRIONES, LENCIO, RELOJO, TENGCO, LIM, DUMA, DANAO, MELLA, TONGSON

*
Sight is as valuable to a human being as any of his other senses. The government should interpose to protect from annoyance this most valuable of mans senses as readily as to protect him from offensive noises and smells. The regulation of billboards is not so much a regulation of private property as it is a regulation of the use of the streets and other public thoroughfares.

"

The very flexibility of the due process clause was meant to make it adapt easily to every situation, enlarging or constricting protection as the changing times and circumstances may require. Courts have preferred to leave the import of the protection open-ended, as it were, to be gradually ascertained by the process of inclusion and exclusion in the course of the decision of cases as they arise. Justice Felix Frankfurter of the US Supreme Court: Due process is the embodiment of the sporting idea of fair play. o

People v. Fajardo Section 1, Article III | Substantive Due Process Facts: The Mayor of Baao, Camsur issued an Ordinance no.7 series of 1950 which prohibits the defendants-appellants Juan Fajardo and Pedro Babilonia to construct a building in their property for it destroys the view of the public plaza. They were convicted for violation of the said ordinance for construction of a building without a permit from the municipal mayor. Issue: Held: Yes. Since the mayor can refuse a permit solely in case that the proposed building destroys the view of the public plaza or occupies any public property The ordinance is unreasonable and oppressive, in that it operates to permanently deprive appellants of the right to use their own property and amount to taking it without just compensation. Ratio: The state may prohibit structures offensive to the sight, the State may not, under the guise of police power, permanently divest owners of the beneficial use of their property and practically confiscate them solely to preserve or assure the aesthetic appearance of the community. W/N the Ordinance deprives the owners of their right to due process.

Churchill v. Rafferty Section 1, Article III | Substantive Due Process Facts: The complaint is to enjoin the Collector of Internal Revenue from removing certain billboards in Rizal.

Petitioners Argument: 1. The billboards are not a nuisance and are not deleterious to the health, morals of the communitys general welfare. 2. The billboards were quite distant from the road, strongly built, and not dangerous to the society. 3. Portion of Section 100 of Act No 2339, empowering the Collector of Internal Revenue to remove billboards as nuisances if objectionable to the sight is a deprivation of right without due process of law. 4. Agreed in the statement of facts that the billboards are Respondents Argument: 1. (After due investigation made upon the complaints of the British and German Consuls) the billboards are offensive to the sight.
Issue: WON the enactment assailed by the plaintiffs was a legitimate exercise of the police power of the government Held: Yes. Ratio: The state may interfere wherever the public interests demand it and in this particular a large discretion is necessarily vested in the legislature to determine not only what the interests of the public require but what measures are necessary to protect such interests; subject to court supervision. State interference should be only to the extent necessary to preserve a healthy social and economic condition of the country 1. Taxation. The individual receives the equivalent of the tax in form of protection and benefit he receives from the government as such. 2. Eminent Domain. The individual receives the market value of the property taken from him 3. Police Power. The benefits arise only from the maintenance of a healthy economic standard of society often referred to as damnun absque injuria.

Facts:

Acebedo Optical Co. vs. Court of Appeals Section 1, Article III| Substantive Due Process

Petitioner applied with the Office of the City Mayor of Iligan for a business permit. Because of the opposition interpose by local optometrists, respondent City Mayor issued the business permit no. 5342 subject to some conditions. These are: 1. Since acebedo is a corporation, it cannot put up an optical clinic but only a commercial store. 2. Acebedo cannot examine or prescribe reading glasses for patients. 3. Acebedo cannot sell reading and similar eyeglasses without a prescription by an independent optometrist or an optical clinic. They can only sell directly to the public, without prescription, Ray-Ban and similar glasses. 4. Acebedo cannot advertise optical lenses and glasses but can advertise Ray-Ban and similar glasses.

CONSTI II DIGESTS: 1D COMPILATION

CHUA, ALIMANGOHAN, CONTRERAS, BRIONES, LENCIO, RELOJO, TENGCO, LIM, DUMA, DANAO, MELLA, TONGSON

!+
iv. v. The provision requiring the 2 types of motels to have certain minimum facilities (dining room, restaurant, laundry) for being arbitrary, unreasonable and oppressive Prohibition against a person less than 18 yrs old unless accompanies by parents or guardians to lease a room more than twice every 24 hrs since it runs counter to due process guaranty for lack of certainty and for its unreasonable, arbitrary and oppressive character

"

Acebedo is allowed to grind lenses but only upon prescription of an independent optometrist. Because of these, petitioner comes before this court the present petition. Issue: Whether or not the business permit, which was burdened with several conditions, is valid. Held: Although the authority of city mayors to issue or grant licenses and business permits is provided for by law, the business permit the city mayor issued with several conditions is not valid. Ratio: There is a distinction between the grant of a license/permit to do business AND issuance of a license to engage in the practice of a particular profession. License/permit to do business granted by the local authorities, authorizes the person to engage in business or some form of commercial activity Professional license issued by the Board or Commission tasked to regulate the particular profession, the grant of authority to a natural person to engage in the practice of his profession In the case at bar, what is sought by petitioner is a permit to engage in the business of running an optical shop. It does not seek a license to engage in the practice of optometry as a corporate body, although is employs persons who are duly licensed to practice optometry. The employment of a qualified optometrist by a corporation is not against public policy. Unless prohibited by statutes, a corporation has all the contractual rights an individual has and it does not become the practice of optometry because of the presence of an optometrist. The manufacturing, selling, trading and bartering of glasses and spectacles as articles of merchandise do not constitute the practice of optometry.

5.

LAWRENCE

Ermita-Malate Hotel and Motel Operators vs City of Manila Sec. 1, Art. III, Substantive Due Process Facts: On June 13, 1963, the Municipal Board of the City of Manila enacted Ordinance No. 4760, which was approved a day after by the acting mayor, vice mayor Herminio Astorga amending various sections of the compilation of ordinances of Manila and inserting 3 new sections. A petition for prohibition was filed by petitioners. They asserted that: i. The ordinance was beyond the powers of the Municipal Board of the City of Manila insofar as it would regulate motels since there is no reference regarding motels in the revised Charter of Manila. ii. Sec. 1 is unconstitutional and void for being unreasonable and violative of due process as it would impose tax Php 6000 per yr for 1st class motels and Php 4000 per yr for 2nd class motels iii. The provision regarding the filling up of specific details in a form (given name, address, length of stay, number of companions, etc.) before accepting any guest or customer and that such premises would be open for inspection by the Mayor or the Chief of Police or representative is unconstitutional and void for being vague, indefinite and uncertain and that it is an invasion of the right to privacy and the guaranty against self-incrimination.

*Basically, their argument is that provisions of the ordinance run counter to the due process clause. Respondent Mayor filed an answer saying that the ordinance was set to curb immorality, a valid and proper exercise of police power and that only guests or customers could complain of the alleged invasion of the right to privacy and the guaranty against self-incrimination. Thereafter, a stipulation of facts and memorandums were submitted by both parties. The Lower court issued a writ of preliminary injunction ordering respondent Mayor to refrain from enforcing Ordinance 4760 and it was declared null and void. Issue: Whether or not Ordinance 4760 is violative of the due process clause. Held: No, because there is no evidence to offset the presumption of validity of a statute. Ratio: Justice Malcolm: The presumption is all in favor of validity. The action of the elected representatives cannot be set aside and that the local legislative body, by enacting the ordinance, has in effect given notice that regulations are essential to the well-being of the people. Absent any factual foundation of record overthrowing the statute, presumption of validity must prevail. Regarding the ordinance as violative of due process: o This particular manifestation of a police power measure being specifically aimed to safeguard public morals is immune from such imputation of nullity unsupported by anything of substance. To hold otherwise would be to unduly restrict and narrow the scope of police power which has been properly characterized as the most essential, insistent and the least limitable of powers, extending as it does "to all the great public needs." There is no question but that the challenged ordinance was precisely enacted to minimize certain practices hurtful to public morals. o The increase in the license fees was intended to discourage "establishments of the kind from operating for purpose other than legal" and at the same time, to increase "the income of the city government." o Police power which is the power to prescribe regulations to promote health, morals, peace, good order, safety and general welfare of the people. In view of the due process and equal protection, exercise of police power is subject to judicial inquiry where such exercise of police power is capricious, whimsical, unjust or unreasonable. o The standard of due process is responsiveness to the supremacy of reason and obedience to the dictates of justice. In the instant case, the Municipal Board of the City of Mania felt the need for a remedial measure by enacting Ordinance 4760 to curb immorality. o Co Kiam v. City of Manila: mere fact that some individuals in the community may be deprived of their present business or a particular mode of earning a living can not prevent the exercise of the police power. Argument regarding restriction on freedom to contract:

CONSTI II DIGESTS: 1D COMPILATION

CHUA, ALIMANGOHAN, CONTRERAS, BRIONES, LENCIO, RELOJO, TENGCO, LIM, DUMA, DANAO, MELLA, TONGSON

!!
Ordinance was an invalid exercise of delegated power as it is unconstitutional and repugnant to general laws -Ordinance infringes the Due Process Clause Due process clause imposes 2 separate limits on the government o Procedural due process: procedures that the government must follow before it deprives a person of life, liberty and property o Substantive due process: asks whether the government has an adequate reason for taking away a persons life, liberty or property; sufficient justification for the governments action Police power adversely affects individual rights to the extent that may fairly be required by the legitimate demands of public interest or public welfare -Requisites for the valid exercise of police power are not met Requisites: 1. Must appear that the interests of the public generally require an interference with private rights 2. Means adopted must be reasonably necessary for the accomplishment of the purpose Prohibition will not per se protect and promote the social welfare of the community; it will not in itself eradicate the alluded social ills of prostitution, adultery, fornication nor will it arrest the spread of sexual disease in Manila The enumerated establishments are lawful pursuits which are not per se offensive to the moral welfare of the community -Means employed are constitutionally infirm Section 3 of the Ordinance: o Options are to wind up business operations or to transfer to another place or to convert said businesses to other kinds of businesses allowable within the area Section 4 of the Ordinance: o In cases of subsequent violations, premises shall be closed and padlocked permanently Means employed (Section 3 & 4) infringes on the constitutional guarantees of a persons fundamental right to liberty and property -Modality employed is unlawful taking Divests respondent of the beneficial use of its property

"

It is neither unreasonable nor arbitrary since it was intended to curb the opportunity for the immoral or illegitimate use to which such premises could be and are being devoted. o Every regulation of conduct amounts to curtailment of liberty. Liberty regulated by law. Restraint by law for the good of the individual and for the greater good of the society. Right of evey individual is necessarily subject to reasonable restraint by general law for the common good. Argument regarding specificity of the information to be given out by guests or customers o Justice Holmes: There is no canon against using common sense in construing laws as saying what they obviously mean. o

City of Manila v. Judge Laguio Section 1, Article III | Procedural Due Process FACTS: On March 9, 1993, the City Council of Manila enacted Ordinance No. 778, An ordinance prohibiting the establishment or operation of businesses providing certain forms of amusement, entertainment, services and facilities in the Ermita-Malate Area, prescribing penalties for violation thereof, and for other purposes. On June 1993, lower court issued a TRO against the enforcement of the ordinance. On November 1994, the lower court held that the Ordinance is null and void, thus this petition. ISSUE: Whether or not Ordinance No. 778 (of the City of Manila) is invalid and unconstitutional HELD: The Court is of the opinion, and so holds, that the lower court did not err in declaring the Ordinance, as it did, ultra vires and therefore null and void. RATIO: -Test of a valid ordinance: 1. must be within the corporate powers of the local government unit 2. passed according to the procedure prescribed by law -Substantive requirements: 1. must not contravene with the constitution or any statute a. since LGUs only have delegated powers, the delegate cannot be superior to the principal or exercise powers higher than those of the latter 2. must not be unfair or oppressive 3. must not be partial or discriminatory 4. must not prohibit but may regulate trade 5. must be general and consistent with public policy 6. must not be unreasonable -The ordinance contravenes the constitution exercise of police power must be reasonable and for the public good

CONSTI II DIGESTS: 1D COMPILATION

CHUA, ALIMANGOHAN, CONTRERAS, BRIONES, LENCIO, RELOJO, TENGCO, LIM, DUMA, DANAO, MELLA, TONGSON

!#
Issue: Whether or not petitioners were free as adults to engage in private conduct in the exercise of their liberty under the Due Process Clause Held: The earlier decision was reversed and the case remanded. Ratio: To uphold the statute basing it to the decision in Bowers is to ignore the fact that the issue at hand involves not merely the right of homosexuals to engage in certain sexual conduct but their liberty, as protected by the Constitution, to choose to enter upon relationships in the confines of their homes and their own private lives and still retain their dignity as free persons. The courts obligation is to define the liberty of all, not to mandate its own moral code Due Process Clause protects personal decisions relating to marriage, procreation, contraception, family relationships etc Different nations have taken action to affirm the protected right homosexual adults to engage in intimate, consensual conduct. The Texas statute furthers no legitimate state interest which can justify its intrusion into the individuals personal and private life.

"

People v. Nazario Section 1, Article II| Procedural due process Facts: The petitioner in this case was charged with the violation of municipal ordinances in Pagbilao, Quezon. In his defense, the petitioner questioned the constitutionality of the said ordinances.

Petitioners Argument: Petitioner contends that the ordinances were ambiguous and uncertain. He argues that hes just a lessee and shall not be covered by the terms owner or manager and shall not be obliged to pay municipal tax as demanded by the Ordinance No. 4. He argues that the said ordinances were vague about the date of payment (ORDINANCES 15 and 12 are in conflict with each other)
Issue: Whether or not the ordinances were vague and would violate due process if enforced upon the petitioner Held: The Court upheld the validity and constitutionality of the said ordinances Ratio: As the actual operator of the fishponds, he becomes within the term manager While it is true that the National Government owns the property, the petitioner is the sole recipient of the profits generated by the business. Therefore, he shall be clearly liable for municipal taxes. The government is immune from taxes The ordinances speak clearly of the dates of payment as Ordinances 15 and 12 are about two entirely different cases. The petitioner was not able to prove that said ordinances were utterly vague on its face meaning it cannot be clarified by either saving clause or construction.

White Light Corp v. City of Manila Facts: Mayor Lim signed into law the Ordinance 7774 entitled An Ordinance Prohibiting Short Time Admission in Hotels, Motels, Inns in Manila Malate Tourist and Devt Corp filed a TRO with RTC of Manila impleading City of Manila as defendant. WLC, Titanium Corp and Sta Mesa Tourist and Devt filed a motion to intervene on the ground that said ordinance directly affects their business interests as operators of drive-inn hotels and motels in Manila. RTC issued a TRO to desist from the enforcement of the Ordinance. The City appealed to CA.

Lawrence et al v. Texas Section 1, Article II| Substantive due process Facts: Petitioners were arrested and convicted of deviate sexual intercourse in violation of a Texas statute forbidding two persons of the same sex to engage in certain intimate sexual conduct. The earlier court decision affirms the constitutionality of the said statute under the Due Process Clause of the Fourteenth Amendment, taking into consideration the Bowers case.

Petitioners: Ordinance is unconstitutional and void since it violates the right to privacy and the freedom of movement; it is an invalid exercise of police power; unreasonable and oppressive interference in their business. City of Manila:
(1) Ordinance is a valid exercise of police power pursuant to LGC Sec 458 (4) (iv) conferring on cities among LGUs the power to regulate the establishment of hotels, motels etc (2) Ordinance is a valid exercise of power under Revised Manila Charter, Art III, Sec 18 (kk) of Revised Manila Charter CA reversed the decision of RTC and affirmed the constitutionality of the Ordinance. Hence the appeal to this Court. Issue: Whether or not the assailed Ordinance is an invalid exercise of police power

CONSTI II DIGESTS: 1D COMPILATION

CHUA, ALIMANGOHAN, CONTRERAS, BRIONES, LENCIO, RELOJO, TENGCO, LIM, DUMA, DANAO, MELLA, TONGSON

!$
must exist between the purpose of the measure and the means employed for its accomplishment. In this case, the police measure shall be struck down as arbitrary intrusion into private rights. Morfe v. Mutuc: Exercise of police power is subject to judicial review when life liberty and property is affected.

"

Held: Petition is granted. Appealed decision is reversed. Decision of RTC is reinstated. Ordinance No 7774 is unconstitutional. Ratio: (1) Issue of Petitioners Standing: General rules on standing admit several exceptions such as over-breadth doctrine, taxpayer suits, third party standing and transcendental importance. Third party standing and over-breadth doctrine are applied in this case. The criteria are: that he suffered an injury in fact thus giving him sufficiently concrete interest; a close relation of litigant to third party; there exists a hindrance in third partys ability to protect his or her own interest (2) Test of a valid ordinance It must be within the corporate powers of LGU to enact and pass according to procedure prescribed by law. It must conform to ff requirements: Must not contravene the Consti or any statute; must not be unfair and oppressive; must not be partial or discriminatory; must not prohibit but may regulate trade; must be general and consistent with public policy and must not be unreasonable. Questioned Ordinance prohibits: wash rate admissions and renting out a room more than twice a day. The ban is rooted in the police power as conferred to LGUs by LGC. The goal is to eliminate use of establishment for illicit sex etc. However, the desirability of these ends does not satisfy any and all means for their achievement. (3) On due process: the purpose of the guarantee is to prevent arbitrary governmental encroachment against life liberty and property of individuals Test of the validity of ordinance as laid down by US v. Carolene Products: the judiciary would defer to legislature unless there is a discrimination against a minority or infringement of a fundamental right. Standards of judicial review: Strict scrutiny for laws dealing with freedom of mind or restricting political process standard for determining the quality and amount of govt interest brought to justify regulation of fundamental freedoms Rational basis standard review for economic legislation - Laws or ordinances are upheld if they rationally further a legitimate governmental interest. In this case, rational basis test is applied as there is an injury to property. The petitioners have the right to invoke the constitutional rights of their patrons being deprived of short time rates.

Facts: On April 21, 1969, the Municipal Board of Butuan enacted Ordinance No. 640 which penalized persons, groups or entities engaged in the business of showing movies and other performances that charged children between seven years old and twelve years old full payment for tickets. Petitioners who are owners of theaters petitioned the courts that the said ordinance be declared unconstitutional. Petitioners argument: Ordinance No. 640 violates the due process clause of the Constitution for being oppressive, unfair, unjust, and confiscatory. Issue: W/N Ordinance No. 640 is unconstitutional for being offensive to due process. Ruling: Ordinance No. 640 is unconstitutional. It is not justified by any necessity of the public interest. Ratio:

Balacuit v. CFI 163 SCRA 182 (1988)

The exercise of police power Police power legislation must be firmly grounded on public interest and welfare. There must be public necessity which demands the adoption of proper measures to secure the ends sought to be attained by the enactment of the ordinance. Moreover, there must be a rational relation between the measure/means adopted and the end in view. --IN THIS CASE, the ordinance is clearly unreasonable and unduly oppressive upon the business of petitioners. The said ordinance makes the petitioner suffer the loss of earnings by compelling them to sell tickets for minors between 7 and 12 at half price.
Moreover, there is no discernible relation between the ordinance and the promotion of public health, safety, morals and the general welfare.

(4) Rights at stake: right to liberty Liberty the right to exist and the right to be free from arbitrary restraint or servitude. Morfe v Mutuc: right to privacy is a constitutional right; the invasion of which must be justified by a compelling state interest. In this case, the rights of citizen to be free to use his faculties in all lawful ways, to live and work where he will; to earn livelihood by any lawful calling are deemed embraced in concept of liberty. The concept of liberty compels respect for the individual whose claim to privacy and interference demands respect.
(5) Ordinance prevents lawful uses of wash rate depriving patrons of a product and petitioners of lucrative business. In protecting public interest, a reasonable relation

Respondent argument 1 :The Municipal Board was impelled to protect the youth from the pernicious practice of charging minors full prices for tickets. Court 1: There is nothing pernicious in demanding equal price for both children and adults. The petitioners are merely conducting their business. Respondent argument 2: Ordinance No. 640 is reasonable and necessary to lessen the economic burden of parents whose minor children are lured by the attractive nuisance being maintained by the petitioners. (hell of an argument. wtf)

CONSTI II DIGESTS: 1D COMPILATION

CHUA, ALIMANGOHAN, CONTRERAS, BRIONES, LENCIO, RELOJO, TENGCO, LIM, DUMA, DANAO, MELLA, TONGSON

!%
convicted of gross indecency. The State then sued both Dennis and his wife, petitioner Tine B. Bennis, to have the car declared public nuisance and abated as such under 600.3801 and 600.3825 of Michigans Compiled Laws. The Wayne County Circuit Court rejected this argument, declared the car a public nuisance, and order the cars abatement. The Michigan Court of Appeals reversed, holding that regardless of the language of the Michigan Compiled Law 600.3815(2), Michigan Supreme Court precedent interpreting this section prevented the State from abating petitioners interest absent proof that she knew to what end the car would be used. The Michigan Supreme Court reversed the Court of Appeals and reinstated the abatement in its entirety.

"

Court 2: If the movies and performances are nuances, then why lower the prices?? (There is no reasonable and necessary relation between the means employed and the end sought.) Petitioner argument 3: By charging the full price, children are being EXPLOITED by the movie house operators. (oookkaaayyy whut?) Court 3: We fail to see how the children are exploited if they pay the full price of admission. They are treated with the same quality of entertainment as the adults.

Magatajas v. Pryce properties Section 1, Article 3 | Substantive due process Facts: PAGCOR decided to expand its operations to Cagayan de Oro City. It lease a portion of a building belonging to Pryce Properties, renovated and equipped the same, and prepared to inaugurate its casino there during the Christmas season. The reaction of the Sangguniang Panlungsod was swift and hostile. It enacted Ordinance No.3353 or An Ordinance prohibiting the issuance of business permit and canceling existing business permit to any establishment for the using and allowing to be used its premises or portion thereof for the operation of casino. It also enacted Ordinance No. 3375-93 or An Ordinance prohibiting the operation of casino and providing the penalty for violation therefor. Pryce assailed the ordinances before CA. CA declared the ordinances invalid and reconsideration of the decision was denied. Issue: WON Ordinance Nos. 3353 and 2275-93 as enacted by the Sanguniang Panlungsod of Cagayan de Oro City is valid. Held: NO! It is invalid. Ratio: The Court, in deciding the case, used the criteria laid down by LAW and not by their own convictions on the propriety of gambling. The following is the substantive requirements or test of a valid ordinance: 1) It must not contravene the constitution or any statute. 2) It must not be unfair or oppressive. 3) It must not be partial or discriminatory. 4) It must not prohibit but regulate trade. 5) It must general and consistent with public policy. 6) It must not be reasonable.

Petitioners Argument: Petitioner defended against the abatement of her interest in the car on the ground that, when she entrusted her husband to use the car, she did not know that he would use it to violate Michigans indecency law.
Issue: Whether Michigans abatement scheme has deprived petitioner of her interest in the forfeited car without due process, in the violation of the Fourteenth Amendment. Or has taken her interest for public use without compensation, in violation of the Fifth Amendment as incorporated by the Fourteenth Amendment. Held: We hold that the Michigan court order did not offend the Due Process Clause of the Fourteenth Amendment or the takings Clause of the Fifth Amendment. Ratio: In Dobbinss Distillery v. United States, even when the owner is otherwise without fault and it was always been held that the acts of the [possessors] bind the interest of the owner whether he be innocent or guilty. In Van Oster v Kansas, 272 U.S. 465 (1926), It has long been settled that statutory forfeitures of property entrusted by the innocent owner or lienor to another who uses it in violation of the revenue law of the United States is not a violation of the due process clause of the Fifth Amendment. The innocence of the owner of the property subject to forfeiture has almost uniformly been rejected as a defense. She did not know that her car would be used in an illegal activity that would subject it to forfeiture. But under these cases the Due Process Clause of the Fourteenth Amendment does not protect her interest against forfeiture of the government. Forfeiture of property prevents illegal uses of both by preventing further illicit use of the [property] and by imposing an economic penalty, thereby rendering illegal behavior unprofitable. The law builds the secondary defense against a forbidden use and precludes evasions by dispensing with the necessity of judicial inquiry as a collusion between the wrongdoer and the alleged innocent owner.

L. Bennis v. Michigan Section 1, Article 3 | Substantive due process Facts: Detroit police arrested John Bennis after observing him engaged in sexual act with a prostitute in the automobile while it was parked on a Detroit city street. Bennis was

Cruzan v. Director, Missouri Department of Health, et. al. Section 1, Article III | Substantive Due Process

CONSTI II DIGESTS: 1D COMPILATION

CHUA, ALIMANGOHAN, CONTRERAS, BRIONES, LENCIO, RELOJO, TENGCO, LIM, DUMA, DANAO, MELLA, TONGSON

!&
HOWEVER, this does not mean that an incompetent person should possess the same right, since such a person is unable to make an informed and voluntary choice to exercise any right. ii. Missouri has recognized that under certain circumstances a surrogate may act in behalf of the patient. $ HOWEVER, it has established a procedural safeguard to assure that the surrogates action conforms to the wishes expressed by the patient while competent. A clear and convincing evidence standard is an appropriate standard when the individual interests at stake are both particularly important and more substantial than mere loss of money. i. Missouri has a general interest in the protection and preservation of human life, as well as other more particular interests. Thus, it may legitimately seek to safeguard the personal element of an individuals choice between life and death. ii. The State is entitled to guard against potential abuses by surrogates who may not act to protect the patient. iii. An erroneous decision not to terminate life results in a maintenance of the status quo, whereas an erroneous decision to withdraw treatment is not susceptible of correction since it would result in death. $

"

Facts: Petitioner Nancy Cruzan sustained severe injuries in an automobile accident. She now lies in a Missouri state hospital in a persistent vegetative state (a condition in which a person exhibits motor reflexes but evinces no indications of significant cognitive function). The State is bearing the cost of her care. Cruzans parents (co-petitioners) requested to terminate her artificial nutrition and hydration. The hospitals employees refused to honor this request without court approval.

State trial court authorized the termination (bases): A person in Cruzans condition has a fundamental right under the State and Federal Constitutions to direct or refuse the withdrawal of death-prolonging procedures. Cruzan expressed to her former housemate that she would not wish to continue her life if sick or injured unless she could live at least halfway normally. This suggested that she would not continue on with her nutrition and hydration. State Supreme Court reversed the trial courts decision (bases): While the right to refuse treatment is embodied in the common-law doctrine of informed consent, its applicability in this case is questionable. Court declined to read into the State Constitution a broad right to privacy that would support an unrestricted right to refuse treatment. o It is doubtable that the Federal Constitution embodied such right. The State Living Will statute embodied a state policy strongly favoring the preservation of life. Cruzans statements to her housemate were unreliable for the purpose of determining her intent. No person can assume an incompetents choice with regard to the termination of her medical treatment in the absence of the formalities required by the Living Will Statute or clear and convincing evidence of the patients wishes.
Issues: 1. WON the State Supreme Court erred in holding that an incompetents wishes must be proved by clear and convincing evidence. 2. WON the State Supreme Court erred in concluding that the evidence presented did not amount to clear and convincing proof of Cruzans desire to withdraw hydration and nutrition. 3. WON the Due Process Clause requires a State to accept the substituted judgment of close family members in the absence of substantial prod that their views reflect the patients. Held: 1.

b.

2.
3.

Cruzans observations that she did not want to live life as a vegetable did not deal in terms with withdrawal of medical treatment or of hydration and nutrition.
i.

There is no such requirement under the Due Process Clause. Upholding the States favored treatment of traditional family relationships and the States right to permit family decision-making cannot be turned into a constitutional requirement that the State must recognize such decision-making. Cruzans parents would surely be qualified to exercise such a right of substituted judgment were it required by the Constitution.

Beltran v. Secretary of Health Section 1, Article III | Substantive Due Process Facts: [[[Commercial blood banks have been operating under RA 1517, RA 4688, AO 156, and the Blood Banking Law. The Bureau of Research and Laboratories (BRL) strictly regulated blood banks and clinical labs, however, their supervisory visits were reduced due to financial difficulties in the 80s. Concern for the safety of blood and blood products intensified when AIDS was first described in 1979, prompting the dissemination of guidelines by the International Society of Blood Transfusion, requiring AIDS testing for blood products. In 1992 the DOH institutionalized the National Blood Services Program with the BRL as the central office for it. Also in 1992, The senate and the house of reps separately formulated bills concerning the safety of the Philippine blood bank system]]]

The US Constitution does not forbid Missouri to require that an incompetents wishes as to the withdrawal of life-sustaining treatment be proved by clear and convincing evidence. a. The liberty interest under the Due Process Clause to refuse treatment must be balanced with relevant state interests. i. It is assumed that a competent person would have a constitutionallyprotected right to refuse lifesaving hydration and nutrition.

CONSTI II DIGESTS: 1D COMPILATION

CHUA, ALIMANGOHAN, CONTRERAS, BRIONES, LENCIO, RELOJO, TENGCO, LIM, DUMA, DANAO, MELLA, TONGSON

!'
delegation if not outright abdication of the police power of the state
a fundamental obligation of the state. The study by the NTMF revealed how disturbingly primitive and unsafe the Philippine blood banking system was. RA 7719 aims to instill public consciousness regarding voluntary blood donation, and safe blood supply. All rights and all contracts are subject to police power and regulations which affect them may change as the well-being of the community may require (1) The interest of the public generally requires the interference of the state and (2) the means employed are reasonably necessary to attain its objective. Commercial blood banks operators and owners must give way to serve a higher end for the publics interest.

"

In 1994, the New Tropical Medicine Foundation with the assistance of the US Agency for International Development released a report on the Philippine blood banking system: o 64.4% of the blood units in 1992 came from commercial blood banks o 99.6 of commercial blood bank donors are paid o Blood sold are 3 times more likely to have malaria, syphilis, Hepa B, and AIDS than those donated to the Phil National Red Cross o These donors are not usually honest about their medical or social history since they need the money. o Many Filipino doctors are not fully trained on blood component transfusion; unaware of the lack of blood supply; do not care where the blood comes from In 1994, the law in question was enacted. Petitioners assail Sec. 7 of the National Blood Services Act of 1994 (RA 7719) and its implementing administrative orders, which provide for the phasing out of commercial blood banks over a period of 2 years after the effective of said laws, extendable to a max period of 2 years. All commercial blood banks should have been phased out by 1998 but prior to the expiration of their licenses, the petitioners filed a petition for certiorari and a TRO assailing the constitutionality of RA 7719. Sec 7 of RA 7719 and its implementing rules violate the equal protection clause RA 7719 is a valid exercise of police power they violate the non-impairment clause they constitute deprivation or personal liberty & property they truly serve public welfare

3. WON they violate the non-impairment clause PETITIONER COURT It will affect their business and existing Philippine Association of Service Exporters v. Drilon: contracts, thus violating the nonThe non-impairment clause must yield to the loftier impairment clause. purposes of the government. 4. WON they constitute deprivation or personal liberty & property PETITIONER COURT It infringes on the individuals The state in order to promote the general welfare freedom of choice. What one may interfere with personal liberty, property, and wants to do with his blood is a business and occupations. matter outside the domain of The rights of the individual may be subordinated state intervention. to secure the general welfare of the state. To treat blood equally as a human Vda de Genuino v. Court of Agrarian Relations: organ is an invalid classification Individual rights to contract and to property have since the former can be to give way to police power for public welfare. replenished. 5. WON they truly serve public welfare COURT: The court may not inquire into what the law may be or should be for it is within the domain of the legislature.

Issues: 1. WON 2. WON 3. WON 4. WON 5. WON

Held: The court has been unable to find any constitutional infirmity in the questioned provisions. Ratio: 1. WON they violate the equal protection clause PETITIONER COURT It unduly The classification is valid and reasonable. discriminates 1. The substantial distinction between nonprofit blood banks and against free commercial banks lies in the fact that the former operates for standing or humanitarian reasons while the latter treats blood as a sale of commodity. commercial 2. The classification is germane to the purpose of that law (to provide blood banks in the country with an adequate supply of safe blood, promote voluntary a manner that blood donation as a humanitarian service rather than a commodity. is not germane 3. The legislature intended for its general application and not just to to the purpose address the existing conditions. of the law. 4. It applies equally to all commercial blood banks without exception. 2. WON it is a valid exercise of police power PETITIONER COURT It is an undue It is a valid exercise of police power since the promotion of public health is

Manalo v. PNP Chief Section 1, Article III | Substantive Due Process Facts: Petitioners are former police operatives. They were implicated in the burning of an elementary school in Batangas at the height of the 2007 national elections. As a result, respondents issued a memoranda (may 22, 2007) that allegedly put them in restrictive custody and had their movements monitored and if they were to leave the camp they should be escorted and a logbook should be maintained. Petitioners claim that it was not sanctioned by any existing law and the constitution and, therefore, illegal. Manalo et al. (petitioners) now petition the court for the issuance of a writ of habeas corpus. Where the main thrust of the writ is to inquire into the legality of ones

CONSTI II DIGESTS: 1D COMPILATION

CHUA, ALIMANGOHAN, CONTRERAS, BRIONES, LENCIO, RELOJO, TENGCO, LIM, DUMA, DANAO, MELLA, TONGSON

!(
located outside the city proper. The franchise for such terminal was granted to the petitioner. The aim of such ordinance is to localize the source of traffic congestion in the city to a single location. Thus, the common carriers plying routes to and from Lucena City are compelled to close down their existing terminals and use the facilities of the petitioner. Respondent who had maintained a terminal within the city was one of those affected by the ordinances. Issue: Whether the City of Lucena properly exercised its police power when it enacted the ordinances. Held: No, the 2nd requisite for the proper exercise of police power is absent. The local govt may be considered as having properly exercised its police power only if the following requisites are present: 1. the interests of the public generally, as distinguished from those of a particular class, require the interference of the State (lawful subject) 2. the means employed are reasonably necessary for the attainment of the object sought to be accomplished and not unduly oppressive upon individuals (lawful method) Ratio: The 1st requisite for proper exercise of police power is present. In Calalang vs. Williams, there was a statute authorizing the Director of Public Works to promulgate rules to control traffic on national roads. The court held that the National Assembly was prompted by considerations of public convenience and welfare. Public welfare lies at the bottom of the enactment of said law. And the state in order to promote the welfare may interfere with personal liberty and property. In the case at bar, the ordinances having been enacted with the objective of relieving traffic congestion in the City of Lucena, they involve public interest warranting the interference of the State. The 2nd requisite is absent. The means employed by the Lucena Sangguniang Panlungsod were not reasonably necessary to attain its objective and it was unduly oppressive. The ordinances assailed are characterized by overbreadth. They go beyond what is reasonably necessary to solve the traffic problem. Since the compulsory use of the terminal operated by the petitioner would subject the users thereof to fees, rentals and charges. Such measure is unduly oppressive. What should have been done was to determine exactly where the problem lies and to stop it right there. The true role of Constitutional law is to effect an equilibrium between authority and liberty so that rights are exercised within the framework of the law and the laws are enacted with due deference to rights. A due deference to the rights of the individual requires a more careful formulation of solutions to societal problems.

"

detention, its vital purpose is to obtain immediate relief from illegal confinement, to liberate those who may be imprisoned without sufficient cause and to deliver them from unlawful custody. Issues: 1. W/N they are unlawfully detained or restrained of their liberty under the restrictive custody status. 2. W/N the courts should dismiss the petition on the sole ground of mootness or proceed to decide the petition on the merits. Held: 1. There is no illegal restraint in the restrictive custody and monitored movements of police officers under investigation. First, memoranda This cannot by any stretch of the imagination, be considered as a form decreeing of curtailment of their freedom guaranteed under our Constitution. monitoring of their Petitioners are not actually detained or restrained of their liberties. movements. What was ordered by the PNP is for their movements to be monitored. (see p. 70) Second, restrictive PNP law allows placing officers facing grave administrative subject custody is illegal disciplinary measures. It is a permissible precautionary measure to and deprives them assure the PNP authorities that the police officers concerned are always of the rights accounted for. enjoyed by private This is not an act of making petitioners suffer lesser rights enjoyed by c1itizens. private citizens. Although the PNP is civilian in character, its members are subject to the disciplinary authority of the Chief PNP, under National police commission. Courts cannot overrule with valid acts of police officials.

2. No, proceed to decide the petition. It was decided in favor of the respondents. Ratio: In Sombong v. CA it teaches us that for the writ of habeas corpus to issue, the restraint of liberty must be in the nature of an illegal and involuntary deprivation of freedom of action. The police organization must observe self-discipline and obey a chain of command under civilian officials.

Lucena Grand Central Terminal, Inc. vs. JAC Liner, Inc. Section 1, Article III| Substantive Due Process Facts: Respondent, a common carrier operating buses which ply various routes to and from Lucena City, assailed the City Ordinances 1631 and 1778 as unconstitutional via a petition for prohibition and injunction against the Mayor of the City of Lucena and the Sangguniang Panlungsod of Lucena before the RTC of Lucena City. They claim that it constituted an invalid exercise of police power, an undue taking of private property and a violation of the constitutional prohibition on monopolies. The subject ordinances prohibit the operation of all bus and jeepney terminals within Lucena, including those already existing and allow the operation of only one common terminal

People v. Cayat Section 1, Article II| Equal Protection of Law Facts:

CONSTI II DIGESTS: 1D COMPILATION

CHUA, ALIMANGOHAN, CONTRERAS, BRIONES, LENCIO, RELOJO, TENGCO, LIM, DUMA, DANAO, MELLA, TONGSON

!)
Also it provides that a candidate is disqualified from running for public office on the ground alone that charges have been filed against him before a civil or military tribunal Petitioners are Dumlao, who is a former governor of Nueva Vizcaya, who filed his certificate of candidacy for the 1980 elections and he questions the 1st paragraph of the provision; and Igot and Salapantan, who question the 2nd paragraph. Petitioners Arguments: (Dumlao) Provision is directed against him (intentional discrimination) and that classification is based on purely arbitrary grounds and therefore, class legislation Provision is contrary to the safeguard of equal protection Issues: Whether or not the 1st and 2nd paragraphs of Sec. 4 BP blg 52 are valid. Held: 1st paragraph is valid and 2nd paragraph is invalid. A partial declaration of nullity of objectionable portion is mandated Ratio: Validity of 1st Paragraph Argument of intentional discrimination is unacceptable because several petitions for disqualification of other candidates on the same ground have already been filed. Argument that it is contrary to the safeguard of equal protection is not well taken because a 65 yr old who has retired from an elective office and who has received retirement benefits is a reasonable classification because there is a need for new blood in political elective echelons and because of the tiredness of the retiree. There is also no clear showing of the invalidity of said provision. Laws shall not be declared invalid unless the conflict with the Constitution is clear beyond reasonable doubt. Invalidity of 2nd Paragraph All reasonable grounds should be resolved in favor of constitutionality however the challenged proviso contravenes the constitutional presumption of innocence. It condemns before one is fully heard. The effect is that there is no distinction between one convicted and one against whom charges have been filed. There is clear and present danger because of the proximity of elections and there would be no time to rebut charges filed against such candidate. Also, it is best that evidence be aired before the Courts rather than before an administrative body such as COMELEC Doctrine: Equal protection clause does not forbid all legal classification. What is proscribed is a classification which is arbitrary and unreasonable. This is not violated if: classification is germane to the purpose of the law and applies to all of the same class. groupings are based on rational classification, which means that groupings are based on reasonable and real differentiations. Teehankee, J., separate opinion: Provision is an arbitrary discrimination and is grossly violative of the equal protection clause because: classification (that would bar 65-yr old retires from running for the same elective office is not germane nor relevant to the alleged purpose of infusing new blood because old blood retirees may continue in local governments since they are not disqualified at all

"

Cayat, a native of Baguio, Benguet, Mountain Province was prosecuted for violation of Act No. 1639 (Secs 2 and 3) which prohibits, among others, the possession of an non-native intoxicating liquor by any member of the non-Christian tribes. The case at hand is an appeal assailing the constitutionality of the Act after the trial court rendered him guilty of the crime and sentencing him to pay a fine or to suffer subsidiary imprisonment in case of solvency. Issue: 1. Whether or not the act is discriminatory and denies the equal protection of the law 2. Whether or not it is violative of the due process clause of the Constitution 3. Whether or not it is an improper exercise of the police power of the state Held: The Constitutionality of the Act was upheld. Ratio: The Act must be understood and applied in consonance with its purpose which is to secure for them the blessings of peace and harmony; to facilitate, and not to mar, their rapid and steady march to civilization and culture. The presence of classification does not violate the equal protection clause as the Act satisfies the necessary requirements. o It rests on substantial distinctions ! Term non Christian tribes does not refer to belief but to the geographical area and more directly, to natives of Philippine Islands with low degree of civilization ! The distinction is reasonable as the Act was intended to meet their peculiar conditions o It is germane to the purposes of law ! The prohibition was designed to insure peace and order in and among nonChristian tribes o It is not limited to existing conditions only o It applies equally to all members of the same class It is not violative of the due process clause since it does not seek label the non-Christian tribes as inferior ones but to recognize their inherent right to equality enjoyed by their Christian brothers There was no improper exercise of police power as the Act is intended maintain peace and order and also in consideration of the paramount public interest of the nation.

Dumlao v Comelec Sec. 1, Art. III, Equal Protection of Law Facts: This is a petition to enjoin COMELEC from implementing Batas Pambansa Blg. 51,52,53 for being unconstitutional. Sec. 4 BP blg. 52 provides that a retired elective official of a province, city or municipality who has received payment of retirement benefits and who is 65 yrs old at the commencement of the office to which he seeks to be elected, shall be disqualified to run for the same position from which he retired.

CONSTI II DIGESTS: 1D COMPILATION

CHUA, ALIMANGOHAN, CONTRERAS, BRIONES, LENCIO, RELOJO, TENGCO, LIM, DUMA, DANAO, MELLA, TONGSON

!*
Since there could be no other justification for such discrimination the statute should be held invalid as a denial of equal protection clause

"

to run for other local elective office as long as it is not the same office from which they retired Age has never been a yardstick for qualification or disqualification. It is the right of the people to elect the candidate of their choice no matter his age. Persons similarly situated should be similarly treated.

Facts: As part of Michigan system for controlling the sale of liquor, bartenders are required to be licensed in all cities having a population of 50, 000 or more, but no female may be so licensed unless she be the wife or daughter of the male owner of a licensed liquor establishment. The case is a direct appeal from an order of denying an injunction to restrain the enforcement of the Michigan law. Petitioner claims that Michigan cannot forbid females generally from being barmaids and at the same time make an exception in favor of the wives and daughters of the owners of liquor establishments. Issue: Whether or not the law is violative of the equal protection clause of the fourteenth amendment (American Constitution) Held: Michigan has not violated its duty to afford equal protection of its laws Ratio: Michigan could, beyond question, forbid all women from working behind a bar despite the vast changes in the social and legal position of women The Constitution does not require legislatures to reflect sociological insight or shifting social standards, any more than it requires them to keep abreast of the latest scientific standards The constitution in enjoining the equal protection of the laws upon States precludes irrational discrimination as between persons or groups of persons in the incidence of a law The legislature need not go to the full length of prohibition if it believes that as to a defined group or females other factors are operating which either eliminate or reduce the moral and social problems otherwise calling for prohibition Michigan has not violated its duty to afford equal protection Rutledge, Douglas & Murphy, dissenting: Equal protection clause requires lawmakers to refrain from invidious distinctions of the sort drawn by the statute challenged in this case Statute arbitrarily discriminates between male and female owners of liquor establishments This inevitable result of the classification belies the assumption that the statute was motivated by a legislative solicitude for the moral and physical well-being of women who would be employed as barmaids

Goesart v. Cleary Section 1, Article I | Equal Protection of Law

NOTES: Article II, Section 14: The State recognizes the role of women in nation-building, and shall ensure the fundamental equality before the law of women and men Provision makes more important assertion that there exists a fundamental equality of women and men before the law Intent was not to achieve an ipso facto repeal of laws offensive to the equality but merely to give a push to statutory legislation that would eliminate the inequalities found in existing law

Ormoc Sugar Central v. Ormoc City Facts: Municipal Board of Ormoc passed Ordinance No. 4 Series of 1964 imposing on any and all productions of centrifugal sugar in Ormoc 1% per export sale to US and other countries. Ormoc Sugar Company paid said tax on March and April 1964, under protest. Plaintiff filed before CFI Leyte a complaint against City of Ormoc and Municipal Board. Plaintiff: (1) Ordinance is unconstitutional for being violative of equal protection clause and rule of uniformity of taxation (2) It is forbidden under Sec 2287 of Rev. Admin Code (3) Ormoc is not authorized to impose production and license tax (4) It is in violation of par1 Sec2 RA 2264, because the tax is both the sale and export of sugar Defendant: (1) Tax ordinance is within Ormocs power to enact under Loc. Autonomy Acct (2) It does not violate constitutional limitations CFI upheld the constitutionality of ordinance and declared that LAA broadened the taxing power of chartered City. Hence this appeal. Issue: Whether the constitutional limits on the power of taxation, equal protection clause and rules of uniformity of taxation were infringed Held: Decision is reversed. Challenged ordinance is unconstitutional. Ratio: EPC applies only to persons or things identically situated and does not bar a reasonable classification of the subject of legislation where: 1. 2. 3. 4. it is based on substantial distinctions these are germane to the purpose of law classification applies to present ad future conditions classification applies only to those who belong in the same class

Ordinance does not meet the requisites. At time of taxing of ordinance, the defendant was the only sugar central in Ormoc. It cannot be subject to the tax because the ordinance expressly points to the Ormoc Sugar Co as the entity to be levied upon.

CONSTI II DIGESTS: 1D COMPILATION

CHUA, ALIMANGOHAN, CONTRERAS, BRIONES, LENCIO, RELOJO, TENGCO, LIM, DUMA, DANAO, MELLA, TONGSON

#+
It would be unjust to impose the same rate on both classes. Indeed, the distinction made between the two classes is substantial and reasonable. The tax imposition, therefore, does not violate equal protection, due process or the rule on uniformity in taxation. ----------------------

"

Anent the inconsistency between Sec 2287(It shall not be in the power of municipal council to impose import or export tax) and RA 2264 (gave power to municipalities to levy uniform taxes), the Court already held in Nin Bay Mining Co v. Municipality of Roxas that the former had been repealed by the latter.

Sison, Jr. v. Ancheta 130 SCRA 654 (1984) Facts: The validity of Section 1 of BP 135 is being challenged. BP 135 provides for rates of tax on citizens or residents on (a) taxable compensation income, (b) taxable net income, (c) royalties, prizes and other winnings, (d) interest from bank deposits and yield or any other monetary benefit from deposit substitutes and from trust fund and similar arrangements, (3) dividends and share of individual partner in the net profits of taxable partnership, (f) adjusted gross income. BP 135 imposes the gross system of income taxation for compensation income and the net system of income taxation for business and professional income. The latter is, in effect, a higher rate.

The Power to Tax The power to tax is an attribute of sovereignty. It is the strongest of all the powers of the government. It is an inherent prerogative and has to be availed of to assure the performance of vital state functions. It is the source of the bulk of public funds.
However, the power to tax is not unconfined. There are restrictions. The Constitution sets forth such limits.

Due Process A taxing statue may be attacked on due process grounds if the tax is
(a) (b) (c) (d) so arbitrary that it amounts to confiscation of property beyond the jurisdiction of the State not for a public purpose so harsh and unreasonable

Petitioners arguments: Petitioner alleges that he would be unduly discriminated against by the imposition of higher rates of tax upon his income arising from the exercise of his profession vis--vis those which are imposed upon fixed income or salaried individual taxpayers (compensation income).
He therefore argues that the imposition violates both equal protection and due process. It also violates the rule on the uniformity of taxation. Issue: W/N Section 1 of BP 135 is unconstitutional for violating equal protection, due process and the rule on the uniformity of taxation. Ruling: Section 1 of BP 135 does not violate equal protection, due process, and the rule on the uniformity of taxation. Ratio: IN THIS CASE, a distinction is made between compensation income and business and professional income. BP 135 imposes the gross system of income taxation for compensation income and the net system of income taxation for business and professional income. The distinction is based on the fact that those receiving compensation income, by the nature of their employment, are not entitled to make deductions for income tax purposes whereas those receiving business and professional income are.

Equal Protection Equal protection requires that the laws operate equally and uniformly on all persons under similar circumstances or that all persons, the conditions being the same, must be treated in the same manner both in the privileges conferred and the liabilities imposed.
However, the equality at which the equal protection clause aims is not a disembodied equality. The Constitution does not require things which are different in fact or opinion to be treated in law as though they were the same. Classification, if rational in character, is allowable.

Uniformity of Taxation Equality and uniformity in taxation means that all taxable articles or kinds of property of the same class shall be taxed at the same rate. The requirement is met when the tax operates with the same force and effect in every place where the subject may be found.
However, the rule does not call for perfect uniformity or perfect equality. The taxing power has the authority to make reasonable and natural classifications for purposes of taxation.

Himagan v. People Section 1, Article I | Equal protection Facts: The petitioner is a policeman assigned with the medical company of the PNP Regional Headquarters at Davao City. Information was filed against him with the RTC of Davao for the killing of Benjamin Machitar, Jr. and the attempted murder of Barnabe Machitar. The trial court

CONSTI II DIGESTS: 1D COMPILATION

CHUA, ALIMANGOHAN, CONTRERAS, BRIONES, LENCIO, RELOJO, TENGCO, LIM, DUMA, DANAO, MELLA, TONGSON

#!

"

issued an ORDER suspending the petitioner until the termination of the case on the basis of Section 47, R.A. 6975, aka DILG Act of 1990. Said law provides for the Preventive Suspension Pending Criminal Case. That a member of the PNP charged with grave felonies shall be suspended from office until the case is terminated, the 2nd sentence of the same section mandates that the case, which shall be subject to continuous trial, shall be terminated within 90 days from the arraignment of the accused. Petitioner filed a motion to lift the order for his suspension, relying on PD 807 of the Civil Service Decree, that his suspension must be limited to 90 days. Respondent Judge denied the motion pointing out that DILG Act, the accused shall be suspended from office until his case is terminated. Petitioner claims that he is covered by Civil Service Law which limits his suspension to 90 days therefore suspension contrary to said law would be a violation of his constitutional right of equal protection law. Issue: WON the preventive suspension for over 90 days under Section 47 of RA 6975 violate the petitioners constitutional right to equal protection of the laws. Held: No. It does not violate his constitutional right to equal protection clause. Ratio: Recognizing the existence of real differences among men, the equal protection clause does not demand absolute equality. It merely requires that all persons shall be treated alike, under like circumstances and conditions both as to the privileges conferred and liabilities enforced. Thus, the equal protection clause does not absolutely forbid classifications, such as the one which exists in the instant case. If the classification is based on: 1) real and substantial differences; 2) is germane to the purpose of the law; 3) applies to all members of the same class; and 4) applies to current as well as future conditions, the classification may not be impugned as violating the Constitutions equal protection guarantee. A distinction based on real and reasonable considerations related to a proper legislative purpose such as that which exists here is neither unreasonable, capricious nor unfounded. The discussions of the Bicameral Conference Committee on National Defense reveal that the legislative intent to place on preventive suspension a member of the PNP charged with grave felonies and the suspension continues until the case against him is terminated. Police is treated differently because they carry weapons and the badge of law which can be used to harass or intimidate witnesses against them. And if a suspended policeman is reinstated while his case is pending, his victim and the witnesses against him are obviously cowed to silence by the mere facts that the accused is in uniform and armed. G. Phil. Judges Assn v. Prado Section 1, Article 3 | Equal Protection Clause Facts: The main target of this petition is Section 35 of R.A. No. 7354 as implemented by the Philippine Postal Corporation through its Circular No. 9228. The measures withdraw the ranking privilege from the Supreme Court, the Court of Appeals, The Regional Trial Courts, The Metropolitan Trial Courts, the Municipal Trial Courts, and the Land Registration Commission and its Register of deeds, along with certain other government offices The petitioners are members of the lower courts who feel that their official functions as judges will be prejudiced by the above-named measures.

Petitioners Argument: R.A. 7354 is discriminatory because while withdrawing the franking privilege from the Judiciary, it retains the same for the President of the Philippines; the Vice President of the Philippines; Senators and Members of the House of Representatives; the Commission on Elections; former Presidents of the Philippines; widows of for Presidents of the Philippines; the National Census and Statistics Office; and the general public in the filing of complaints against public offices or officers. Respondents Argument: The respondents counter that there is no discrimination because the law is based on a valid classification in accordance with equal protection clause.
Issue: Whether or not R.A. No. 7354 is unconstitutional on the grounds that (3) it is discriminatory and encroaches on the independence of the Judiciary. Held: If the problem of the respondents is the loss of revenues from the franking privilege, the remedy, it seems to us, is to withdraw it altogether from all agencies of the government, including those who do not need it. This court is unable to agree with the respondents that Section 35 of R.A. No. 7354 represents a valid exercise of discretion by the legislature under the police power. On the contrary, we find its repealing clause to be a discriminatory provision that denies the Judiciary the equal protection of the laws guaranteed for all persons or things similarly situated. The distinction made by the law is superficial. It is not based on substantial distinctions that make real differences between the judiciary an the grantees of the franking privilege. Ratio: The respondents point out the available data from the Postal Service Corporation shows that from January 1988 to June 1992, the total volume of frank mails amounted to P90,424,174.00. Frank mails coming from the Judiciary amounted to P73,574,864.00. The respondents conclusion id that because of this considerable volume of mail from the Judiciary, the franking privilege must be withdrawn by it. The argument is self-defeating. The respondents are in effect saying that the franking privilege, the remedy, it seems to us, is to withdraw it altogether from all agencies of the

CONSTI II DIGESTS: 1D COMPILATION

CHUA, ALIMANGOHAN, CONTRERAS, BRIONES, LENCIO, RELOJO, TENGCO, LIM, DUMA, DANAO, MELLA, TONGSON

##
Thus, it was reasonable for the President to have delimited the application of some incentives to the confines of the former Subic military base. It is this specific area which the government intends to transform and develop. "

"

government, including those who do not need it very much, if it all, but not those who need it badly. It is worth observing that the Philippine Postal Corporation, as a government-controlled corporation, was created and is expected to operate for the purpose of promoting the public service. Among the services it should be prepared to extend is the free carriage of mail form certain offices of the government that need the franking privilege in the discharge of their own public functions. We also note that under Section 9 of the law, the Corporation is capitalized at P10billion pesos, 55% of which is supplied by the government, and that it derives substantial revenues from the sources enumerated in Section 10, on top of the tax exemptions it enjoys. It is not likely that the retention of the franking privilege by the Judiciary will cripple the Corporation.

De Guzman, Jr., et al v. COMELEC Section 1, Article III | Equal Protection Facts: Petitioners assail the validity of Sec. 44 of The Voters Registration Act of 1996 which provides for the reassignment to a new station outside the original congressional district of election officers who have served for at least four years in a particular city or municipality. COMELEC then issued several resolutions for its implementation as well as directives for the reassignment of herein petitioners.

Tiu v. Court of Appeals Section 1, Article III | Equal Protection of Law Facts: This is a petition for review seeking the reversal of the CAs decision which upheld the constitutionality and validity of EO 97-A. EO 97-A provides that the grant and enjoyment of the tax and duty incentives authorized under RA 7227 were limited to the business enterprises and residents within the fenced-in area of the Subic Special Economic Zone (SSEZ). Issue: WON limiting the incentives to businesses and resident within the secured area of the SSEZ and denying them to those who live within the Zone but outside such fenced-in territory violate the constitutional right to equal protection. Held: No. A classification based on valid and reasonable grounds does not violate the equal protection clause. We uphold the validity of the assailed EO. Ratio: 1. There are real and substantial distinctions between the circumstances obtaining inside and those outside the Subic Naval Base. " There are substantial differences between the big investors who are being lured to establish and operate their industries in the so-called secured area and the present business operators outside the area. Big investors inside the area Business operators outside the area Billion-peso investments and thousands of None of such magnitude new jobs Economic impact is national Economic impact is local Germane to the purpose of the law Not likely to have any impact in achieving the purpose of the law 2. The classification is germane to the purpose of the law. " The real concern of RA 7227 is to convert the lands formerly occupied by the US military bases into economic or industrial areas.

Petitioners: NO VALID CLASSIFICATION. Sec 44 violates the equal protection clause in singling out city and municipal election officers since no substantial distinction exists between them and the other COMELEC officials.
Issue: WON Sec 44 violates the equal protection clause Held: NO. Petition is barren from merit. Ratio: The equal protection clause permits a valid classification. In this case, the basis of the classification was to ensure the impartiality of election officials by preventing them from developing familiarity with the people of their place of assignment.

Philippine Telegraph& Telephone Co. v. NLRC Section 1, Article III | Equal Protection Facts: Grace De Guzman was hired by PT&T as a probationary employee, which covers 150 days. She indicated in her job application from that she as single although she contracted marriage a few months earlier. When the company learned about it she was sent a memorandum requiring her to explain such discrepancy. Included in the memorandum is a reminder about the companys policy of not accepting married women for employment. PT&T dismissed De Guzman on the basis of the alleged concealment of civil status and defalcation of company funds as grounds to terminate her services. De Guzman argued that what motivated PT&T to dismiss her was her having contracted marriage during her employment, which was prohibited by the petitioners Issue: Whether the alleged concealment of civil status can be grounds to terminate the services of an employee.

CONSTI II DIGESTS: 1D COMPILATION

CHUA, ALIMANGOHAN, CONTRERAS, BRIONES, LENCIO, RELOJO, TENGCO, LIM, DUMA, DANAO, MELLA, TONGSON

#$
discrimination in terms of wages is frowned upon by the Labor Code like when it prohibits the payment of lesser salary to a female employee than a male the school cannot invoke the need to entice foreign-hires to leave their domicile to rationalize the distinction in salary rates without violating the principle of equal work for equal play

"

Held: No. Petitioners policy runs afoul of the test of, and the right against discrimination, afforded all women workers by our labor laws and by no less than the Constitution. Ratio: The Constitution, cognizant of the disparity in the rights between men and women in almost all phases of social and political life, provides a gamut of protective provisions:

Article 2, Sec. 14: The State recognizes the role of women in nation-building, and shall ensure the fundamental equality before the law of women and men. Article 13, Sec. 3: The State shall afford full protection to labor, local and overseas, organized and unorganized, and promote full employment and equality of employment opportunities for all. Article 13, Sec. 14: The State shall protect working women by providing safe and healthful working conditions, taking into account their maternal functions, and such facilities and opportunities that will enhance their welfare and enable them to realize their full potential in the service of the nation.

Issue: Whether or not the grant of higher salaries to foreign-hires constitutes discrimination and is therefore unsustainable. Held: Yes, it constitutes discrimination. Employees should be given an equal pay for work of equal value. That is a principle that rests on fundamental notions of justice. Ratio: While the need of the school to attract foreign-hires is recognized, salaries should not be used as an enticement to the prejudice of local-hires. The local-hires perform the same services as foreign-hires and they ought to be paid the same salaries with the foreign-hires. The dislocation factor and the limited tenure of foreign-hires cannot serve as valid bases for the distinction in salary rates. These factors are adequately compensated by certain benefits accorded them which are not enjoyed by localhires, such as housing, transportation, shipping costs, taxes and home leave travel allowances. These benefits are reasonably related to their status as foreign-hires and justify the exclusion of the local-hires. Foreign-hires however do not belong to the same bargaining unit as the local-hires. These groups were always treated separately. To include foreign-hires in a bargaining unit with local-hires would not assure either group the exercise of their respective collective bargaining rights. Note: The Philippine Medical Association challenged the validity of provisions of the Generics Acts alleging that: 1. it gave unequal treatment to private practitioners and govt physicians in the manner of prescribing generic drugs. It alleged that government physicians may prescribe only generic drugs whereas private practitioners could write the brand name in parenthesis. 2. The law allowed a mere salesgirl to give a substitute for what the doctor might prescribe. 3. The PMA challenged the prohibition of the use by doctors of no substitution in their prescription. the Court upheld the law saying that the PMA misread the law both private and governmental practitioners are bound by the same law and the salesgirl is merely authorized to inform the buyer of the list of drug equivalents furnished by the Bureau of Food and Drugs the purpose of the Generics Law is to fulfill the constitutional command to make health care affordable. This too is the reason for the no substitution rule

International Alliance vs. Quisumbing Section 1, Article III| Equal Protection of Law Facts: Private respondent International School, Inc. pursuant to PD 732, is a domestic educational institution established primarily for dependents of foreign diplomatic personnel and other temporary residents. The decree authorizes the school to employ its own teaching and management personnel selected either locally or abroad. The school hires both foreign and local teachers as members of its faculty classifying them into: foreign-hires and local hires. The school grants the foreign-hires certain benefits not accorded to local-hires. These are housing, transportation, shipping costs, taxes, and home leave travel allowance. They are also paid a salary rate 25% more than local-hires. The school justifies the difference on 2 significant economic disadvantages which foreign hires have to endure: the dislocation factor and limited tenure. The school explains that a foreign-hire would necessarily have to leave his family and friends and is faced with economic realties: decent abode, effective means of transportation, allowance for education of children etc. Because of a limited tenure, he will eventually return to his home country where there will be uncertainty of obtaining a suitable employment after a long period in a foreign land. Petitioners arguments: the point-of-hire classification employed by the School is discriminatory to Filipinos and that the grant of higher salaries to foreign-hires constitutes racial discrimination public policy abhors inequality and discrimination the Constitution exhorts Congress to give highest priority to the enactment of measures that protect and enhance the right of all people to human dignity, reduce social, economic and political inequalities

Parreo v. COA

CONSTI II DIGESTS: 1D COMPILATION

CHUA, ALIMANGOHAN, CONTRERAS, BRIONES, LENCIO, RELOJO, TENGCO, LIM, DUMA, DANAO, MELLA, TONGSON

#%
o retirement that military personnel acquire a vested right to retirement benefits but upon the condition that he meets the eligibility requirement. The retirement of military personnel are purely gratuitous in nature.

"

Section 1, Article II| Equal Protection of Law Facts: Petitioner Parreo served in the AFP for 32 years. He retired from the Philippine Constabulary in 1982 and availed of himself the payment of a lump sum pension equivalent to 3 years pay. Petitioner migrated to Hawaii and became a naturalized American citizen. His monthly pension was later on stopped by the AFP pursuant to Section 27 of PD No. 1638, as amended by PD 1650. The abovementioned law provides that a retiree who loses his Filipino Citizenship shall be removed from the retired list and his retirement benefits terminated upon loss of Filipino citizenship. Petitioner requested for consideration but AFP denied request.
Petitioner filed a claim before the COA for continuance of his monthly pension. COA denied the petitioners claim for lack of jurisdiction as the matter concerns the constitutionality of the provision in question which shall be settled only by the courts. COA advised the petitioner to file his claim with the proper court of original jurisdiction. The petitioner then filed with this court a petition for certiorari assailing the decision and the resolution of COA.

The classification established by the provision satisfies all the necessary conditions and does not violate the right to equal protection of the laws. o There is a substantial difference between retirees who are citizens of the Philippines and retirees who lost their Filipino citizenship by naturalization in another country. A retiree who had lost his citizenship has renounced his allegiance to the state. There was no denial of due process o When the petitioner lost his citizenship, AFP had no choice but to stop his monthly pension in accordance with Sec 27 of PD 1638, as amended. He was given the chance to contest the termination of his pension, although his request was denied by the Judge Advocate General.

Petitioners Argument: Section 27 of PD 1638, as amended, deprives him of his property which the Constitution and statutes vest in him. Petitioner alleges that his pension, being a property vested by the Constitution, cannot be removed or taken from him just because he became a naturalized American citizen. Petitioner alleges that the termination of his monthly pension is a penalty equivalent to deprivation of his life. Argues that he can reacquire his Filipino citizenship under RA 9225, in which case he will still be considered a natural-born Filipino. OSG: Agrees with the petitioner that Section 27 of PD 1638, as amended, is unconstitutional o The obligation imposed on petitioner to remain in the AFP retired list and receive his retirement benefit is contrary to public policy and welfare, oppressive, discriminatory, and violative of the due process clause of the Constitution. o Argues that the retirement law is in the nature of a contract between the government and its employees
Issue: 1. Whether or not Section 27 of PD 1638, as amended, is constitutional 2. Whether or not PD 1638, as amended, has retroactive or prospective effect Held: The Court affirmed the constitutionality of the questioned provision. PD 1638, as amended, does not impair any vested right or interest of petitioner. Ratio: Petitioner has no vested right to his retirement benefits o At the time of the approval of PD 1638 and at the time of its amendment, petitioner was still in active service. Therefore, petitioners retirement benefits were only future benefits and did not constitute a vested right. It is only upon

He has not shown any intention to reacquire Filipino citizenship, and even if he does, he will only be entitled to pension and gratuity benefits reckoned from the date he has taken oath of allegiance to the Republic of the Philippines pursuant to RA 9225.

Central Bank Employees v Bangko Sentral Sec. 1, Art. III, Equal Protection of Law Facts: In 1993, RA 7653 (The New Central Bank Act) took effect and created a new BSP. * yrs later, Central Bank Employees Association Inc. filed a petition for prohibition against BSP and Executive Secretary of the president to restrain respondents from implementing the last proviso in Sec. 15 (c) Art. II of RA 7653 because it is unconstitutional. The significant portion of the provision provides that: A compensation structureshall be instituted as an integral component of the Bangko Sentral's human resource development program: Provided, That the Monetary Board shall make its own system conform as closely as possible with the principles provided for under RA 6758 [Salary Standardization Act]. Provided, however, that compensation and wage structure of employees whose positions fall under salary grade 19 and below shall be in accordance with the rates prescribed under Republic Act No. 6758. Petitioners Arguments: i. Proviso makes an unconstitutional cut between 2 classes of employees: 1. BSP officers or those exempt from the coverage of SSL and 2. rank-and-file (SG 19 and below) or those not exempt. ii. This is class legislation based solely on SG of BSP personnels position not on substantial distinctions iii. It is not germane to the purpose of the said section which is to establish professionalism and excellence at all levels in the BSP.

CONSTI II DIGESTS: 1D COMPILATION

CHUA, ALIMANGOHAN, CONTRERAS, BRIONES, LENCIO, RELOJO, TENGCO, LIM, DUMA, DANAO, MELLA, TONGSON

#&
officer and was repatriated back to the Philippines. He filed a complaint and the labor arbiter agreed that he was illegally dismissed and is entitled to compensation. Serrano claims that the aforementioned provision exacerbates the hardship of the OFWs by unduly limiting their entitlement in case of illegal dismissal to their lump-sum salary either for the unexpired portion of their employment contract or for three months for every year of the unexpired term, whichever is less. Petitioner contends that it violates the OFWs constitutional rights in that it impairs the terms of their contract, deprives them of equal protection and denies them due process. It impinges on the equal protection clause, for it treats OFWs differently from local Filipino workers when their dismissal is declared illegal. Issues: 1. Whether or not his dismissal was illegal 2. Whether or not the assailed provision is unconstitutional Held: 1. 2. Ratio: The finding that the dismissal of the petitioner is not disputed. Yes.

"

Issues: 1. Whether or not last proviso in Sec. 15 (c) Art. II of RA 7653 is unconstitutional because it runs afoul to the constitutional mandate that No person shall be denied the equal protection of the laws 2. Whether or not a provision of law initially valid can become unconstitutional Held: 1. Sec. 15 (c) Art. II of RA 7653 by itself is valid 2. Yes. Enactment of subsequent laws can make the continued application of a challenged provision to be a violation of the equal protection clause. Ratio: I. The equal protection of the laws clause of the Constitution allows classification. The requirements are that: it be reasonable, which means that the classification should be based on substantial distinctions which make for real differences, that it must be germane to the purpose of the law; that it must not be limited to existing conditions only; and that it must apply equally to each member of the class. This Court has held that the standard is satisfied if the classification or distinction is based on a reasonable foundation or rational basis and is not palpably arbitrary. In the case at bar, it is clear that the exemption of officers (SG 20 and above) from the SSL was intended to address the BSP's lack of competitiveness in terms of attracting competent officers and executives. It was not intended to discriminate against the rank-and-file. II. Concept of Relative Constitutionality - A statute valid at one time may become void at another time because of altered circumstances. Thus, if a statute in its practical operation becomes arbitrary or confiscatory, its validity, even though affirmed by a former adjudication, is open to inquiry and investigation in the light of changed conditions R.A. No. 7653 started as a valid measure. Court held that the enactment of subsequent laws exempting all rank-and-file employees of other governmental financial institutions GFIs leeched all validity out of the challenged proviso. Enactment of subsequent laws = consequential unconstitutionality of challenged proviso. The last proviso of Section 15(c), Article II of RA 7653 is now violative of the equal protection clause because after it was enacted, the charters of the GSIS, LBP, DBP and SSS were also amended, but the personnel of the latter GFIs were all exempted from the coverage of the SSL. Thus, within the class of rank-and-file personnel of GFIs, the BSP rank-and-file are also discriminated upon. Subsequent charters of the seven other GFIs share this common proviso: a blanket exemption of all their employees from the coverage of the SSL.

Requirements for judicial review: [nakalagay siya eh, sinama ko na lang din.. related pa din sa constitutionality issue] Requirements Present in Instance the case Actual controversy / There exists an actual controversy Constitutional question raised / Issue on the constitutionality was first raised by proper party and at the in the NLRC, however the CA failed to take earliest possible opportunity up the issue in its decision Constitutional question is the / Monetary claim strikes at the very core of the very lis mota of the case subject clause Section 1, Article III of the Constitution guarantees: No person shall be deprived of life, liberty, or property without due process of law nor shall any person be denied the equal protection of the law. Section 18, Article II and Section 3, Article XIII accord all members of the labor sector, without distinction as to place of deployment, full protection of their rights and welfare. o Such rights are not absolute but subject to inherent powers of congress to incorporate a system of classification with the following requirements: ! Based on substantial distinctions ! Germane to the purpose of law ! Not limited to existing conditions only ! Applies equally to all members of the class o Three levels of scrutiny: Deferential or rational basis scrutiny: challenged classification needs only be shown to be rationally related to serving a legitimate state interest Middle-tier or intermediate scrutiny: government must show that the challenged classification serves an important state interest and that the classification is at least substantially related to serving that interest

Facts: Antonio Serrano assails the constitutionality of the last clause in the 5th paragraph of Section 10, Republic Act No. 8042. He was an OFW working for Gallant Maritime Services, Inc. and Marlow Navigation Co., Ltd., and was forced to accept a downgraded position (not the original position offered to him by the company which was Chief officer) with the assurance that hed be Chief officer after several months. The company failed to deliver and he refused to stay as Second

Serrano v. Gallant Maritime Services Section 1, Article I | Equal protection of law

CONSTI II DIGESTS: 1D COMPILATION

CHUA, ALIMANGOHAN, CONTRERAS, BRIONES, LENCIO, RELOJO, TENGCO, LIM, DUMA, DANAO, MELLA, TONGSON

#'
privileged status to the older brands at the same time discriminating against the newer brands. Court held these contentions to be without merit because, in this case, there was neither: i. A suspect classification; nor ii. An impingement on a fundamental right 2. Rational Basis Test (RBT) % jurisprudence provides in the areas of social and economic policy, a statutory classification that neither proceeds along suspect lines nor infringes constitutional rights must be upheld against equal protection challenge if there is any reasonable conceivable state of facts that could provide a rational basis for the classification. Under the RBT, it is sufficient that the legislative classification is rationally related to achieving some legitimate State interest. Court further held that a legislative classification that is reasonable does NOT offend the constitutional guaranty of equal protection of the laws. The classification is considered valid and reasonable if: i. It rests on substantial distinctions; ii. It is germane to the purpose of the law; iii. It applies all things being equal, to both present and future conditions; iv. It applies equally to all those belonging to the same class Congress inserted the CFP into the law for reasons of practicality and expediency since a new brand was not yet in existence at the time of the passage of RA 8240 and Congress needed a uniform mechanism to fix the bracket of a new brand. Furthermore, the CFP addressed the ff: i. Congress administrative concerns in the simplification of tax administration of sin products; ii. Elimination of potential areas for abuse and corruption in tax collection; iii. Buoyant and stable revenue generation; iv. Ease of projection of revenues Consequently, there can be no denial of the equal protection of the laws since the RBT is amply satisfied. 6. With regard to uniformity of taxation, according to jurisprudence, a tax is uniform when it operates with the same force and effect in every place where the subject of it is found. It does not signify an intrinsic but simply a geographical uniformity. In this case, there is no question that the CFP meets the geographical uniformity requirement because the assailed law applies to all cigarette brands in the Philippines. -END-

"

Strict Judicial Scrutiny: impermissibly interferes with the exercise of a fundamental right or operates to the peculiar disadvantage of a suspect class is presumed unconstitutional; there is a need to prove that classification is necessary to achieve a compelling state interest and that it is the least restrictive means to protect such interest Deference stops where the classification violates a fundamental right, or prejudices persons accorded special protection by the Constitution o Obligation to afford protection to labor is incumbent not only on the legislative and executive branches but also on the judiciary o Social justice calls for the humanization of laws and the equalization of social and economic forces by the State so that justice in its rational and objectively secular conception may at least be approximated Judicial scrutiny would be based on the Rational basis test and the legislative discretion would be given deferential treatment Perpetuation of prejudice against persons favored by the Constitution with special protection, judicial scrutiny ought to be strict Oppressive acts will be struck down regardless of the character or nature of the actor

3.

British American Tobacco v. Camacho April 15, 2009 Facts: BACKGROUND In a unanimous decision, the Supreme Court declared as constitutional sec. 145 of the National Internal Revenue Code levying higher taxes on cigarette brands that entered the market after 1996, but held invalid certain revenue regulations granting the Bureau of Internal Revenue the power to reclassify or update the classification of new brands every two years or earlier.

4.

PETITIONER ARGUMENTS 1. In its Motion for Reconsideration, the petitioner insists that the assailed provisions: i. Violate the equal protection and uniformity of taxation clauses of the Constitution; ii. Contravene Sec. 19, Art XII of the Constitution on unfair competition; iii. Infringe the constitutional provisions on regressive and inequitable taxation 2. Petitioner further argues that assuming the assailed provisions are constitutional, petitioner is entitled to a downward reclassification of Lucky Strike from premium-priced to the highpriced tax bracket.
Issue: WON the assailed law (RA 8240) violates equal protection and uniformity of taxation clauses. Held/Ratio: NO 1. Petitioners main argument is that the Classification Freeze Provision (CFP) violates the equal protection and uniformity of taxation clauses because older brands are taxed based on their 1996 net retail prices whereas new brands are taxed based on their present day net retail prices. Petitioner asserts that the assailed provisions accord a special or

CONSTI II DIGESTS: 1D COMPILATION

CHUA, ALIMANGOHAN, CONTRERAS, BRIONES, LENCIO, RELOJO, TENGCO, LIM, DUMA, DANAO, MELLA, TONGSON

#(
People v. Kagui Malasuqui: to hold that no criminal can be arrested and searched for evidence and token of his crime without warrant would be to leave society at the mercy of the most depraved criminals, facilitating their escape in many instances Warrantless search of incoming and outgoing passengers at the airport is a practice not constitutionally objectionable because it is founded on public interest, safety Where abuse marks the operation of a checkpoint, the citizen is not helpless as they may be liable criminally and civilly for their abusive acts

"

Valmonte v. General de Villa (1989) 5. When intrusion is a search Facts: This is a motion for reconsideration of the Court decision dismissing the petition to declare checkpoints as unconstitutional and the dismantling and banning of the same.

(Petitioners (from the orig): because of the installation of said checkpoints, the residents of Valenzuela are worried of being harassed and of their safety being placed at the arbitrary disposition of the military manning the checkpoints, considering that their cars and vehicles are being subjected to regular searches and check-ups, especially at night or at dawn checkpoints give the respondents a blanket authority to make searches and/or seizures without search warrant or court order in violation of the Constitution)
Issue: Whether or not the govt employing the military has the power to install checkpoints Held: Motion for Reconsideration is denied. The denial is final. Ratio: Court decision on checkpoints does not invalidate nor condone abuses committed by the military manning the checkpoints. Power implies the possibility of abuse but the Court could not a priori regard that men in uniform are rascals. The court had to assume that the men in uniform live and act by the code of honor and are assigned to protect and not to abuse the citizenry According to the Supreme Court: a.) Nowhere in the decision did the court legalize all checkpoints at all times and under all circumstances; Checkpoints are not illegal per se b.) checkpoints may be allowed and installed by the govt under exceptional circumstances where the lives and safety of people are in peril (at that time (1989), there were 6 attempted coup detat; NPA accelerated its liquidation of armed forces; murders, sex crimes, drug abuse have become daily occurrences) c.) checkpoints are security measure to entrap criminals and insurgents d.) the govt has the right to defend itself from its enemies and under its police power, to select reasonable means and methods to achieve them Routine checkpoint may to a certain extent intrude motorists right to free passage w/o interruption however, it involves only brief detention of travelers where drivers are required to answer brief questions -if vehicles are stopped it is because there are some probable cause which justifies a reasonable belief of the men at the checkpoints that either the motorist is a law offender or the contents of the vehicle are/have been instruments of some offense it is limited to visual search thus cannot be regarded as violative of individuals right against unreasonable search According to US Supreme Court: a.) routine checks in fixed area does not intrude on the motoring public b.) Stops are duly authorized and believed to serve public interest c.) Fixed checkpoint is chosen by officials responsible for making overall decisions to the allocation of enforcement resources d.) There is less room for abusive stops of individuals since field officers may stop only the cars passing the checkpoints e.) It is subject to post judicial review f.) automobiles because of their mobility may be searched w/o warrant

People v. Escano Section 2, Article III | When Intrusion is a Search Facts: There is judicial notice of the existence of the COMELEC resolution imposing a gun ban during the election period pursuant to the Omnibus Election Code. The incident, which happened on 5 April 1995, was well within the period.

Appelants Argument: They contend that the checkpoint manned by elements of the Makati Police should have been announced. They also complain of its having been conducted in an arbitrary and discriminatory manner.
Issue: Whether or not the checkpoint was conducted in an arbitrary and discriminatory manner. Held: The checkpoint herein conducted was in pursuance of the gun ban enforced by the COMELEC. The COMELEC would be hard put to implement ban limited to visual search of pedestrians. Those who intend to bring a gun during said period would know that they only need a car to be able to easily perpetrate their malicious designs. Ratio: This court has ruled that not all checkpoints are illegal. Those which are warranted by the exigencies of the public order and are conducted in a way least intrusive to motorists are allowed. As long as the vehicle is neither searched nor its occupants subjected to a body search, and the inspection of the vehicle is limited to a visual search, said routine checks cannot be regarded as violate of an individuals right against unreasonable search.

Alvarez v. CFI 64 Phil. 33 (1967) Facts: A warrant was issued by the Court of First Instance of Tayabas ordering the search of the house of petitioner and the seizure, at any time of the day and night, of accounting books, documents, and papers, allegedly used by petitioner in usurious money lending.

CONSTI II DIGESTS: 1D COMPILATION

CHUA, ALIMANGOHAN, CONTRERAS, BRIONES, LENCIO, RELOJO, TENGCO, LIM, DUMA, DANAO, MELLA, TONGSON

#)
Court: The affidavit is was insufficient and the warrant was illegal ergo the search could not legally be made at night. Sec 101 of GO No. 58 authorizes that the search may be made at night when it is positively asserted that the property is on the person or in the place ordered to be searched. PA 4: The warrant was illegal because it lacked an adequate description of the books and documents to be seized. Court: The description made in the affidavit substantially complied with the requirements under the law. Section 1, paragraph 3 of Article III of the Constitution (now, Section 2 of Article III) and Section 97 of General Order No. 58 require that the affidavit, which will serve as basis for the warrant, must contain a particular description of the place to be searched and the person or thing to be seized. These provisions are mandatory and must be strictly complied with. HOWEVER, where by the nature of the goods to be seized, their description must be rather general, it is not required that a technical description be given. The description given in the affidavit was that there are being kept in said premises books, documents, receipts, lists, chits, and other papers used by him (petitioner) in connection with his activities as a money-lender, charging a usurious rate of interest in violation of the law. Taking into consideration the nature of the articles so described, it is clear that no other more adequate and detailed description could have been given. PA 5: The search warrant and seizure were illegal because the articles were seized in order that the Anti-Usury Board might provide itself with evidence to be used by it in the criminal case or cases which might be filed against petitioner for violation of the Anti-Usury Law. Court: At the hearing, it clearly appeared that the books and documents had really been seized to enable the Anti-Usury Board to conduct an investigation and later use all or some of the articles in question as evidence against the petitioner. The seizure of books and documents by means of a search warrant, for the purpose of using them as evidence in a criminal case against the person in whose possession they were found, is unconstitutional because it makes the warrant unreasonable, and it is equivalent to a violation of the constitutional provision prohibiting the compulsion of an accused to testify against himself. Respondent Argument 1: Petitioner cannot now question the validity of the search warrant or the subsequent proceedings because he had waived his constitutional rights in proposing a compromise where he agreed to pa a fine of P200 for the purpose of evading the criminal proceedings.

"

Petitioner challenges the legality of the warrant and the consequent seizure of his property. Issue: W/N the search warrant and the seizure of Alvarezs property was legal. Ruling: The search warrant and the seizure of property were illegal. The seized documents (19 in all) must be immediately returned to the petitioner. Ratio: Petitioner Argument 1: The search warrant was illegal because it was based on the affidavit of agent Mariano G. Almeda in whose oath he declared that he had no personal knowledge of the facts which were to serve as basis for the issuance of the warrant but that he had knowledge thereof through mere information from a person whom he (Almeda) considered reliable. Court: The affidavit and the oath on which the warrant was based were fatally defective. Section 1, paragraph 3 of Article III of the Constitution (now, Section 2 of Article III) and Section 97 of General Order No. 58 both require that there be PROBABLE CAUSE before the issuance of the search warrant and that the said warrant must be based upon an application SUPPORTED BY OATH of the applicant and witnesses he may produce. Oath the oath must refer to the truth of the facts within the personal knowledge of the petitioner or his witnesses. Its purpose is to convince the issuing judge/magistrate, not the individual making the affidavit and seeking the issuance of the warrant, of the existence of probable cause. PA 2: The search warrant was illegal because it was not supported by other affidavits aside from that made by the applicant. Court: The warrant was illegal because it was based only on the affidavit of the agent who had no personal knowledge of the facts. When the applicants knowledge is mere hearsay, the affidavit of one or more witnesses having personal knowledge of the facts is necessary. Sec. 98 of GO No. 58 provides that the judge or justice must, before issuing the warrant, examine under oath the complainant and any witnesses he may produce and take their depositions in writing. PA 3: The warrant was illegal because it authorized the execution of the search at night.

CONSTI II DIGESTS: 1D COMPILATION

CHUA, ALIMANGOHAN, CONTRERAS, BRIONES, LENCIO, RELOJO, TENGCO, LIM, DUMA, DANAO, MELLA, TONGSON

#*
People v. Veloso Section 2, Article III | Requisites of a valid warrant Facts: The accused, Jose M. Veloso, a member of the House of Representatives, was found guilty of the crime resistance of the agents of the authority, in violation of Art. 252 of Penal Code, by the CFI of Manila. A building, located at No. 124 Calle Arzobispo, City of Manila, was used by the Parliamentary Club managed by the accused. The police of Manila had reliable information that the so-called Parliamentary Club was nothing more than a gambling house. The chief of the gambling squad, J.F. Townsend verified this fact. Detective Andres Geronimo of the secret service obtained a search warrant and the police raided the said Parliamentary Club, entering through the window and breaking the outer doors in the process because the doors of the premises were barred. Once inside the premises, nearly 50 persons were apprehended including Veloso. Veloso refused to submit to the search, even after the search warrant was showed to him, insisting that he was Representative Velasco and not John Doe. As Velosos pocket was bulging, as if it contained gambling utensils, Townsend required Veloso to show him the evidence of the game. About 5 minutes of heated conversation passed, Veloso was still insisting in his refusal to submit to the search. At last the patience of the officers was exhausted. So a policeman took hold of Veloso only to meet with his resistance. Veloso bit the policeman in the right forearm, and gave him a blow in another part of the body, which injured the latter quite severely. Through the combined efforts of Townsend and the said policeman, Veloso was finally laid down on the floor, and gambling utilities were taken from his pockets. It also took 3 policemen to place him inside the patrol wagon. All the persons apprehended were eventually acquitted by the trial court for lack of proof except Veloso who was found guilty of maintaining a gambling house. In the accuseds appeal, his counsel made no effort to impugn the finding of the trial court, except that he stresses certain points as more favorable to the case of his client.

"

Court: There was no waiver. First, petitioner had emphatically denied the offer of compromise. Second, the compromise referred to the institution of criminal proceedings not the validity of the search warrant and the seizure. The waiver would have been a good defense had the petitioner consented to the search. However, it is clear that the petitioner protested from the beginning and stated his protest in writing. RA 2: The remedy sought by the petitioner before the Court was not proper. The proper remedy was an appeal from the orders that prejudiced him. Court: The remedy was proper. Section 222 of the Code of Civil Procedure provides that mandamus will not issue when there is another plain, speedy and adequate remedy in the ordinary course of law. An appeal, as argued by the respondents, would not be a plain, speedy and adequate remedy for the petitioner. A long time would have to elapse before the he recovers possession of the documents and before the rights, of which he has been unlawfully deprived, are restored to him.

Appellants arguments: The law, constitutional and statutory, requires that the search warrant shall not issue unless the application particularly describe the person to be seized. A failure thus to name the person is fatal to the validity of the search warrant. To justify search and arrest, the process must be legal. Illegal official action may be forcibly resisted. NOTE: see authorities cited by appellant below.
Issue: WON the search warrant and the arrest of Veloso was valid. Held: YES! The search warrant was valid. The defendant has been proven guilty beyond reasonable doubt, of the crime of resistance of agents of the authority.

CONSTI II DIGESTS: 1D COMPILATION

CHUA, ALIMANGOHAN, CONTRERAS, BRIONES, LENCIO, RELOJO, TENGCO, LIM, DUMA, DANAO, MELLA, TONGSON

$+
Authorities cited by the Appellant
Book of Whartons Criminal Procedure John Doe' Warrants. that a warrant for the apprehension of a person whose true name is unknown, by the name of John Doe or Richard Roe, whose other or true name in unknown, is void, without other and further descriptions of the person to be apprehended, and such warrant will not justify the officer in acting under it. Such a warrant must, in addition, contain the best descriptio personae possible and this description must be sufficient to indicate clearly the proper person or persons upon whom the warrant is to be served; and should state his personal appearance and peculiarities, give his occupation and place of residence, and any other circumstances by means of which he can be identified. Person apprehended in act of committing a crime, under a John Doe warrant, on the other hand, the apprehension will not be illegal, or the officer liable, because under such circumstances it is not necessary that a warrant should have been issued. Commonwealth vs. Crotty ([1865], 10 Allen [Mass.], 403) Crotty resisted the arrest upon the ground that the warrant was insufficient, illegal, and void. Neither the complaint nor the warrant contain the name of the defendant, nor any description or designation by which he could be known and identified as the person against whom it was issued. It was in effect a general warrant, upon which any other individual might as well have been arrested, as being included in the description, as the defendant himself. Such a warrant was contrary to elementary principles, and in direct violation of the constitutional right of the citizen.

"

Ratio: The Philippine Code on Criminal Procedure provides that a search warrant shall not issue except for probable cause and upon application supported by oath particularly describing the place to be searched and the person or thing to be seized. 1. After a judge or justice shall have examined on oath the complainant and any witnesses he may produce, and shall have taken their disposition in writing; 2. After the judge or justice is satisfied of the existence of the facts upon which the application is based or that there is probable cause to believe that they exist; 3. The judge or justice must issue the warrant. " The affidavit for the search warrant and the search warrant itself describes the building to be searched as the building No.124 Calle Arzobispo , City of Manila, Philippine Islands. This, without doubt, was a sufficient designation of the premises to be searched. It is the prevailing rule that a description of the place to be searched is sufficient if the officer with the warrant can, with reasonable effort, ascertain and identify the place intended.

From a different approach, it is undeniable that the application for the search warrant, the affidavit, and the search warrant failed to name Jose Ma. Veloso as the person to be seized. However, under the authorities cited by the appellant, it is invariably recognized that the warrant for the apprehension of an unnamed party is void, "except in those cases where it contains a description personae such as will enable the officer to identify the accused." The description in the warrant was sufficient and indicated clearly the proper person upon whom the warrant is to be served. " the search warrant stated that John Doe had gambling apparatus in his possession in the building occupied by him at No. 124 Calle Arzobispo, City of Manila, and as this John Doe was Jose Ma. Veloso, the manager of the club, the police could identify John Doe as Jose Ma. Veloso without difficulty. *** The police officers were accordingly authorized to break down the door and enter the premises of the building occupied by the so-called Parliamentary Club therefore the resistance of Velaso is NOT JUSTIFIED. When inside, police officers then had the right to arrest the persons presumably engaged in a prohibited game, and to confiscate the evidence of the commission of the crime. It must be remembered that the building was supposed to be used for club purposes. It was NOT the home nor place of abode of the Velasco family, which the law carefully protects in all of its sanctity. It was a club partially public in nature wherein unlike in the home there would commonly be varying occupancy, a number of John Does and Richard Roes whose names would be unknown to the police. In defense of himself, any member of his family or his dwelling, a man has a right to employ all necessary violence. But even in the home, and much less so in a club or public place, the person sought to be arrested or to be searched should use no more force than is necessary to repel the unlawful act of the officers. ***

Soliven v. Makasiar Section 2, Article III| Requisites of a valid warrant Facts: Information for libel was filed against petitioner Soliven and Beltran by President Cory. (this is when they accused her of hiding under the bed during a coup attempt); warrants were issued pursuant thereto by the judge without personally examining the complaint and the witnesses. Issues: (1) Whether or Not petitioners were denied due process when the information for libel were filed against them although the finding of the existence of a prima facie case was still under review by the Secretary of Justice and, subsequently, by the President. (2) Whether or Not the constitutional rights of Beltran were violated when respondent RTC judge issued a warrant for his arrest without personally examining the complainant and the witnesses, if any, to determine probable cause Held: (2) With respect to petitioner Beltran, the allegation of denial of due process of law in the preliminary investigation is negated by the fact that instead of submitting his counteraffidavits, he filed a Motion to Declare Proceedings Closed, in effect waiving his right to refute the complaint by filing counter-affidavits. Due process of law does not require that the respondent in a criminal case actually file his counter-affidavits before the preliminary investigation is deemed completed. All that is required is that the

CONSTI II DIGESTS: 1D COMPILATION

CHUA, ALIMANGOHAN, CONTRERAS, BRIONES, LENCIO, RELOJO, TENGCO, LIM, DUMA, DANAO, MELLA, TONGSON

$!
Petitioners Arguments: They were authorized to determine the existence of probable cause in a preliminary investigation. Their findings constitute sufficient basis for the issuance of warrants of arrest. Respondents Arguments: His order requiring petitioners to submit to the court the documentary evidence is an exercise of his judicial power to review the fiscals findings of probable cause. The failure of petitioners to file the required affidavits destroys the presumption of regularity in the performance of petitioners official duties. o It has always been the practice of the Office of the City Fiscal of Butuan to attach to the informations filed with the court the affidavits of witnesses and other documentary evidence presented during the preliminary investigation.
Issues: 1. Whether or not the respondent city judge may, for the purpose of issuing a warrant of arrest, compel the fiscal to submit to the court the supporting affidavits and other documentary evidence presented during the preliminary investigation. 2. Whether the certification of the investigating fiscal in the information as to the existence of probable cause obligates respondent City Judge to issue a warrant of arrest. Held: 1. Yes. The judge may rely upon the fiscals certification of the existence of probable cause and, on the basis thereof, issue a warrant of arrest. 2. No. Ratio: 1. PD No. 911 authorizes the fiscal or state prosecutor to determine the existence of probable cause.

"

respondent be given the opportunity to submit counter-affidavits if he is so minded.

The second issue, raised by petitioner Beltran, calls for an interpretation of the constitutional provision on the issuance of warrants of arrest. The pertinent provision reads:
Art. III, Sec. 2. The right of the people to be secure in their persons, houses, papers and effects against unreasonable searches and seizures of whatever nature and for any purpose shall be inviolable, and no search warrant or warrant of arrest shall issue except upon probable cause to be determined personally by the judge after examination under oath or affirmation of the complainant and the witnesses he may produce, and particularly describing the place to be searched and the persons or things to be seized. The addition of the word personally after the word determined and the deletion of the grant of authority by the 1973 Constitution to issue warrants to other responsible officers as may be authorized by law, has apparently convinced petitioner Beltran that the Constitution now requires the judge to personally examine the complainant and his witnesses in his determination of probable cause for the issuance of warrants of arrest. This is not an accurate interpretation. Ratio: The Constitution no longer requires personal examination by the judge of the complainant and the witnesses. What the Constitution underscores is the exclusive and personal responsibility of the issuing judge to satisfy himself of the existence of probable cause. Following established doctrine and procedure, he shall: (1) Personally evaluate the report and the supporting documents submitted by the fiscal regarding the existence of probable cause and, on the basis thereof, issue a warrant of arrest; or (2) If on the basis thereof he finds no probable cause, he may disregard the fiscals report and require the submission of supporting affidavits of witnesses to aid him in arriving at a conclusion as to the existence of probable cause. Sound policy dictates this procedure; otherwise judges would be unduly laden with the preliminary examination and investigation of criminal complaints instead of concentrating on hearing and deciding cases filed before their courts. Placer v. Villanueva Requisites of a valid warrant Facts (not supplied in the case book but just in case she asks): The City Fiscal of Butuan City and his assistants (petitioners) filed in the City Court of Butuan certain informations and certified that a preliminary examination has been conducted and that there is reasonable ground to believe that the crime charged has been committed. Respondent judge set a hearing to determine the propriety of the issuing warrants of arrest. After hearing, judge issued orders requiring petitioners to submit to the court the affidavits of the witnesses and other documentary evidence to aid him in finding probable cause.

The fiscal or state prosecutor shall certify under oath in the information to be filed by him that he has examined the complainant and his witnesses; that on the basis of the sworn statements and other evidence submitted before him there is reasonable ground to believe that a crime has been committed and that the accused is probably guilty thereof.
2. The issuance of a warrant is not a mere ministerial function; it calls for the exercise of judicial discretion on the part of the issuing magistrate. The judge may disregard the fiscals certification and require the submission of the affidavits of witnesses to aid him in arriving at a conclusion as to the existence of a probable cause. The judge must satisfy himself of the existence of probable cause before issuing a warrant or order of arrest, as provided in Section 6, Rule 112 of the Rules of Court: Warrant of arrest, when issued. If the judge be satisfied from the preliminary examination conducted by him or by the investigating officer that the offense complained of has been committed and that there is reasonable ground to believe that the accused has committed it, he must issue a warrant or order for his arrest.

CONSTI II DIGESTS: 1D COMPILATION

CHUA, ALIMANGOHAN, CONTRERAS, BRIONES, LENCIO, RELOJO, TENGCO, LIM, DUMA, DANAO, MELLA, TONGSON

$#
3. If the search warrants are not valid, WON the documents, papers, and things seized may be used as evidence against petitioners

"

Note: The primary requirement for the issuance of a warrant of arrest is the existence of probable cause.

Stonehill v. Diokno [1] Section 2, Article III| Requisites of a valid warrant Facts: Respondent-prosecutors, officers of the government, applied for the issuance of 42 search warrants, which were issued by Respondent-Judges against the petitioners and/or the corporations of which they were officers. The search warrants: State that the natural and juridical persons had committed a violation of Central Bank Laws, Tariff and Customs Laws, Internal Revenue Code, and the RPC Direct for the seizure of books of accounts, financial records, vouchers, correspondence, receipts, ledgers, journals, portfolios, credit journals, typewriters, and other documents and/or papers showing all business transactions including disbursement receipts, balance sheets and profit and loss statements and Bobbins (cigarette wrappersWTH) Petitioner filed an action to restrain respondents from using the effects seized in the deportation cases, to quash the contested search warrants for being null and void, and to return the documents, things, and cash money seized.

Held: 1. NO. Petitioners have no cause of action. 2. NO. The search warrants are not valid. They are in the nature of a general warrant. 3. NO DISCUSSION ON THIS Ratio: 1. Petitioners may not invoke the corporations right to object in proceedings against them in their individual capacity. Corporations have separate and distinct personalities from the petitioners, thus, the right to object belongs exclusively to the corporations to whom the seized effects belong. The legality of a seizure can be contested only by the party whose rights have been impaired. The right to do so is purely personal and cannot be invoked by third parties. 2. The search warrants did not comply with the constitutional mandate as regards search warrants: a. That no search warrant shall be issued but upon probable cause to be determined by the judge. No specific act, omission, or offense was alleged in the application. Probable cause could not have been determined since the averments were abstract and without reference to any determinate provision of laws or codes. b. That the warrant shall particularly describe the things to be seized. The search warrant referred to all business transactions, and made no distinction on whether the transactions were legal or illegal.

Petitioner: The search warrants are null and void because: 1. They do not describe with particularity the documents, books, and things to be seized. 2. Cash money, werent mentioned but were seized 3. They were issued to fish for evidence in the deportation cases against him. 4. The search was done in an illegal manner. 5. The documents, papers, and cash money were not delivered to the courts which issued the search warrants. Respondent: The search warrants are valid 1. They were issued in accordance with law. 2. The defects of the warrants if any were cured by the petitioners consent. 3. The effects seized are admissible as evidence regardless of the alleged illegality of the searches & seizures.
The writ of preliminary injunction was issued but, by resolution of the SC, was later on: Partially lifted: Papers, documents, things, seized from the offices of the corporations (1st group) Maintained: Papers, documents, things, seized in the residences of petitioners (2nd group) Issue: 1. WON petitioners have a cause of action to assail the legality of the warrants and seizures made in pursuance of those found in the offices of the corporations 2. WON the search warrants issued with respect to those seized in the petitioners residences, and the searches and seizures made in authority thereof are valid

Stonehill v. Diokno [2] Section 2, Article III | Requisites of a valid warrant Facts: This is a case filed by the petitioners impleading several prosecutors and judges as defendants who allegedly issued unconstitutional search warrants against the petitioners ordering the peace officers to search the petitioners and/or the premises of their offices, warehouses and/or residences, and to seize and take possession of the ff personal property, to wit:

Books of accounts, financial records, vouchers, correspondence, receipts, ledgers, journals, portfolios, credit journals, typewriters, and other documents and/or papers showing all business transactions including disbursements receipts, balance sheets and profit and loss statements and Bobbins
as the subject of the offense; stolen or embezzled and proceeds or fruits of the offense or used or intended to be used in committing an act in violation of Central Bank Laws, Tariff and Customs Law, Internal Revenue Code and the Revised Penal Code. Petitioners initially filed with the SC this original action for certiorari, prohibition, mandamus and injunction and prayed for an issuance of preliminary injunction to restrain respondent-prosecutors and their agents from using the things seized from the petitioners. Subsequently, the Court

CONSTI II DIGESTS: 1D COMPILATION

CHUA, ALIMANGOHAN, CONTRERAS, BRIONES, LENCIO, RELOJO, TENGCO, LIM, DUMA, DANAO, MELLA, TONGSON

$$
It shall be issued upon a probable cause, to be determined by the judge in the manner set forth in the provision. o It shall particularly describe the things to be seized. None of the abovementioned requirements were satisfied. o No specific offense had been alleged in said applications o The averments with respect to offense committed were abstract. ! Consequently, it was impossible for the judges who issued the warrants to have found the existence of probable cause. To uphold the validity of the warrants in question would be to wipe out completely one of the most fundamental rights guaranteed in the Constitution, for it would place the sanctity of domicile and the privacy of communication and correspondence at the mercy of the whims, caprice, or passion of peace officers. o

"

released partially involved papers, herein.

the writ of preliminary injunction prayed for. However, in June 1962, the writ was lifted insofar as the papers, documents, and things seized from the offices of the corporations are concerned. The injunction was maintained as regards the documents and things found and seized in the residences of petitioners

Petitioners Arguments: 1) The warrants do not describe with particularity the documents, books and things to be seized. 2) Cash money, not mentioned in the warrants, were actually seized. 3) The warrants were issued to fish evidence against the petitioners in deportation cases filed against them. 4) The searches and seizures were made in an illegal manner. 5) The documents, papers and cash money seized were not delivered to the courts that issued the warrants, to be disposed of in accordance with the law. Respondents Arguments: 1) The contested search warrants are valid and have been issued in accordance with the law. 2) The defects of the said warrants, if any, were cured by petitioners consent. 3) In any event, the effects seized are admissible in evidence against herein petitioners, regardless of the alleged illegality of the aforementioned searches and seizures.
Issue: Whether or not the search warrants in question, and the search and seizures made under the authority thereof, are valid or not Whether the said documents, papers and things may be used in evidence against the petitioners Held: The warrants were unconstitutional Ratio: The documents, papers, and things seized under the warrants may be split into two groups: o Those found and seized in the offices of the corporations o those found and seized in the residences of the petitioners As regards to the first group, the petitioners cannot assail the legality of the warrant. o They have no cause of action. Corporations have their respective personalities and that objection to an unlawful search and seizure is purely personal and cannot be availed of by third parties o Consequently, petitioners cannot object to the use in evidence against them of the document, paper, and things seized from the office With respect to documents, papers, and things seized from the residences of the petitioners, writ of preliminary injunction restrains respondent-prosecutors from using it against the petitioners. The validity of said warrants can be resolved by consulting the Constitution. Section 1, paragraph 3, Article III of the Constitution provides for the 2 requisites that any warrant should satisfy before it can be valid:

Central Bank vs. Morfe Section 2, Article III | Requisites of a valid warrant Facts: On July 2, 1962, the Court of First Instance held and ordered the petitioner Bank from further searching the premises and properties and from taking custody of the various documents. They also order the petitioner bank to return to them all the books, documents and papers so far seized. Hence, this petition for certiorari, prohibition and injunction against an order of the Court of First Instance of Manila. Upon the filing of this petition, the court issued on August 14, 1962 a writ of preliminary injunction restraining the respondents from enforcing the order. The main respondents is the First Mutual Savings and Loan Organization, Inc. It is a registered non-stock corporation whose main purpose is to encourage and implement saving and thrift among its members and to extend financial assistance in the form of loans to them. The legal department of the Central Bank rendered an opinion to the effect that the Organization and others of similar nature are banking institutions, falling within the purview of RA 337. Hence, the bank published an announcement in the newspapers which provides: All savings and loan associations now in operation and other organizations similar in nature have never been authorized by the monetary board of the central bank to accept deposit of funds from the public nor to engage in the banking business nor to perform any banking activity. Such institutions violate the General Banking Act (RA 337) should they engage in the lending of funds obtained from the public through receipts of deposits or sale of bonds without authority from the monetary board. Their activities and operations are not supervised by the Superintendent of Banks and persons dealing with such institutions do so at their risk. Thereafter, a member of the bank filed with the Municipal Court a verified application for a search warrant against the Organization alleging that the premises at No. 2745 Rizal Avenue-in which the offices of the organization were housed-are being used unlawfully because they are illegally engaged in banking activities without having first complied with RA 337 and that articles and papers are kept in the premises and being intended to be used in the commission of a felony. Upon the filing of the application, Judge Cancino issued the warrant commanding the search of the premises and the seizure of the foregoing articles, there being good and sufficient reasons to believe upon examination that the organizations has under its control such articles which are intended to be used as means for the commission of an offense.

CONSTI II DIGESTS: 1D COMPILATION

CHUA, ALIMANGOHAN, CONTRERAS, BRIONES, LENCIO, RELOJO, TENGCO, LIM, DUMA, DANAO, MELLA, TONGSON

$%
The law requiring compliance with certain requirements before any body can engage in banking seeks to protect the public against actual and potential injury. The Organization also does not contest the main facts, upon which the action of the Bank is based. The issue they raised is predicated upon the theory that the transactions of the Organization do not amount to banking as the term is used in RA 337.

"

The organization then filed an action for certiorari, prohibition with writ of preliminary injunction against the Municipal Court, the sheriff, and the bank to annul the search warrant. Respondent Organizations arguments: 1. The search warrant was issued with grave abuse of discretion without or in excess of jurisdiction because: - the search warrant is a roving commission, general in its terms - the use of the word and others in the warrant permits the unreasonable search and seizure of documents which have no relation to any criminal act - no court in the Philippines has any jurisdiction to try a criminal case against a corporation Thereafter, their action was approved. Thus, the bank commenced in the Supreme Court, the present action against Judge Morfe and the organization alleging that the judge has acted with grave abuse of discretion in issuing the order. Respondent Judge Morfes arguments: 1. The search and seizures are unreasonable because the deposition given in support of the application for a search warrant states that the deponent personally knows that the premises of the Organization were being used unlawfully for banking purposes. 2. The deponent could have, if he really knew of actual violation of the law, applied for a warrant to search and seize only books or records covering the specific illegal banking transactions of the petitioner with specific persons who are the supposed victims of the illegal banking transactions. To authorize and seize all the records without reference to specific alleged victims of the illegal banking transactions, would be to harass the petitioner with a roving commission for evidence which could be discovered by normal intelligence operations or inspection of books pursuant to RA 337. Issue: Whether or not the search and seizures were unlawful. Held: No, the Municipal judge did not commit grave abuse of discretion in finding that there was probable cause that the Organization had violated the law and in issuing the warrant. The writ of preliminary injunction issued by the SC on August 14, 1962 is made permanent. Ratio: The line of reasoning of Judge Morfe might be justified if the acts imputed to the organization consisted of isolated transactions, distinct and different from the type of business in which it is generally engaged. Thus, it may then be necessary to specify or identify the parties involved in said isolated transactions, so that the search and seizures be limited to the records pertinent thereto. Such is not the case here. The records suggest clearly that the transactions objected to by the Bank constitute the general pattern of the business of the Organization. The main purpose is to extend financial assistance in the form of loans to its members, with funds deposited by them. The failure of the witness to mention particular individuals does not necessarily prove that he had no personal knowledge of specific illegal transactions of the Organization, for the witness might be acquainted with such specific transactions, even if the names of the individuals were unknown to him.

Columbia Pictures v. Court of Appeals Requisites of a valid warrant Facts: NBI agents and private researchers made surveillance on various video establishments after which NBI Senior Agent Reyes applied for a search warrant to seize pirated video tapes of copyrighted films, TV sets, video cassettes, etc.

Discussion of the 20th Century Fox case The search warrants were lifted because it was granted on the misrepresentations of NBI and its witnesses that copyright infringement or piracy of a film was committed. The 3 witnesses claim to have personal knowledge of the offense but:
Atty Reyes of the NBI stated that he did not have personal knowledge, and that 20th Cent Foxs counsel will testify Witness Bacani did not state how the video cassettes were pirated, and that Atty Domingo has knowledge of that Atty Domingo personally knew that the pirated tapes were re-taped from 20th century foxs master tapes Though NBI alleges that they compared the pirated tapes with the master tapes, the court still did not find it as competent evidence, and that the probable cause hovers in the grey debatable twilight zone ! Basically, in the 20th Century Fox case, the trial court required the presentation of the master tapes to convince itself of the existence of the probable cause. Probable cause must be satisfactory to the court because proceeding to put a man to task as an offender should be interpreted in strictissimi juris against the gov and liberally in favor of the alleged offender.

Respondent (CA): The pirated copies and the master tapes were available for presentation at the time of application for the search warrant; thus, there is no reason not to present them.
Issue: WON for the production of the master tape for comparison with the allegedly pirated copies is necessary for the determination of probable cause to support the issuance of a search warrant in copyright infringement cases involving videograms Held: NO. The validity of the search warrant is upheld. Ratio: Such master tapes are object evidence, but it does not rule out the use of testimonial or documentary evidence, depositions admissions or other classes of evidence tending to prove the factum probandum, especially when the production in court of object evidence would result in delay, inconvenience or expenses out of proportion to its evidentiary value. The decision in the 20th Century Fox case should merely serve as a guidepost in determining probable cause in copyright infringement cases where there is doubt as to the true nexus

CONSTI II DIGESTS: 1D COMPILATION

CHUA, ALIMANGOHAN, CONTRERAS, BRIONES, LENCIO, RELOJO, TENGCO, LIM, DUMA, DANAO, MELLA, TONGSON

$&
Section 2 | Requisites of a valid warrant Facts: Appellant is a Chinese national in his forties, a businessman, and a resident of Baguio City. A raid conducted by operatives of the National Bureau of Investigation (NBI) and Philippine National Police Narcotics Command (PNP NARCOM) at premises allegedly leased by appellant and at his residence yielded huge quantities of marijuana.
Issue: Whether or not the phrase an undetermined amount of marijuana in the warrant fails to satisfy the requirement that the things to be seized must be particularly described Held: Contention that it fails to satisfy requirement has no leg to stand on. Ratio: " Purpose of the Constitutional Requirement of Reasonable Particularity 1. To readily identify the properties to be seized and thus prevent them from seizing the wrong items 2. Leave said peace officers with no discretion regarding the articles to be seized and thus prevent unreasonable searches and seizures " Constitution seeks to avoid search warrants of broad or general characterization or sweeping descriptions " Not required that technical precision of description be required where by the nature of the goods to be seized their description must be rather general o Requirement of a technical description would mean that no warrant could issue " Terms which are held to be constitutional in different jurisprudence o narcotics paraphernalia o any and all narcotics o all implements, paraphernalia, articles, papers and records pertaining to the use, possession or sale of narcotics or dangerous drugs A search warrant commanding peace officers to seize a quantity of loose heroin has been held sufficiently particular

"

between the master tape and the pirated copies. It was not intended to be a sweeping and an inflexible requirement in all copyright infringement cases.

Burgos, Sr. v Chief of Staff, PNP Sec.2, Article III, Requisites of a valid contract Facts: 2 search warrants were issued by Judge of CFI of Rizal (QC) -#19 road 3 project 6 QC and 784 Units c and d, RMS bldg. Q Ave. under which business address of Metropolitan Mail and We Forum newspapers were searched, and office and printing machines and equipment, papers and documents in the possession and control of Jose Burgos (publisher-editor of We Forum) were seized. Petitioner prays for: 1. Return of seized articles, 2. Enjoin respondents from using seized articles as evidence against Burgos and 3. The nullification of the search warrants Petitioners Argument: Judge failed to conduct an examination under oath or affirmation of applicant and his witnesses With regard to the execution of search warrant, the 2 warrants pinpointed only to one place where Burgos was allegedly keeping articles listed Issue: Whether or not arguments are sufficient to nullify the search warrants Held: No Ratio: I. Failure to conduct examination Has become moot and academic because an examination was conducted II. 2 warrants pointing to only one place It was a typographical error. 2 search warrants were applied for and issued because the purpose and intent was to search 2 places. It is absurd and illogical for the judge to issue 2 warrants for one and the same place. In determining whether a search warrant describes the premises to be searched with sufficient particularity: Executing officers prior knowledge as to the place intended to be searched is relevant. This would be true if the executing officer is the affiant on whose affidavit warrant was issued. In case of ambiguity, executing officer may look to the affidavit in the court file. *Particularity of description A search warrant particularly describes things to be seized when: a. Description is as specific as circumstances will ordinarily allow b. Description expresses a conclusion of fact not of law by which warrant officer may be guided in making search and seizure c. Things described are limited to those which bear direct relation to the offense for which warrant is being issued Purpose of particularity of description: prevent abuse by officer enforcing warrant by leaving him no discretion as to who or what to search or seize John Doe warrant can satisfy this requirement of particularity if it contains a description personae that will enable officer to identify accused.

J. People v Del Norte Section 2, Article III | Requisites of a valid warrant Facts: SPO1 Lumabas and his group was tasked to serve a search warrant against a certain Ising Gutierrez Diwa for alleged violation of the Dangerous Drugs Act. They were ordered to forthwith seize and take possession of an undetermined quantity of shabu and marijuana leaves.They coordinated with the barangay officials, one of which informed SPO3 De Leon that Ising Gutierrez Diwa and Priscilla Del Norte are one and the same person, and proceeded to the house pointed out to them by the local officials.

Petitioners argument: The marijuana seized as a result of the search is inadmissible due to the irregularity of the search warrant which contained the name Ising Gutierrez Diwa and not Priscilla del Norte.

People v Tee

CONSTI II DIGESTS: 1D COMPILATION

CHUA, ALIMANGOHAN, CONTRERAS, BRIONES, LENCIO, RELOJO, TENGCO, LIM, DUMA, DANAO, MELLA, TONGSON

$'
4. The main contention of respondent Desierto was that under Section 15 of R.A. 6770 or the Ombudsman Act of 1989, the Ombudsman had the authority to administer oaths, issue subpoena and subpoena duces tecum and take testimony in any investigation or inquiry, including the power to examine and have access to bank accounts and records. Furthermore, this specific provision of R.A. 6770, which is a later legislation, modifies the law on the Secrecy of Bank Deposits (R.A. 1405) and places the office of the Ombudsman in the same footing as the courts of law in this regard.

"

She alleges that Ising is her sister. During her arrest, she claimed she saw Ising nearby and pointed her to the authorities. But the authorities still arrested her. Issue: Whether or not the search warrant is valid. Held: No. Ratio: The Constitution requires search warrants to particularly describe not only the place to be searched, but also the persons to be arrested. The court has ruled in rare instances that mistakes in the name of the person subject of the search warrant do not invalidate the warrant, provided the place to be searched is properly described (ex. Peo v Tiu Won Chua). But the case at bar is different. Peo v Tiu Won Chua The authorities conducted surveillance and a test-buy operation before obtaining the warrant. They had personal knowledge of the identity of the persons and place to be searched although they did not specifically know the names of the accused. Case at bar They did no conduct surveillance before obtaining the warrant The authorities did not have personal knowledge of the circumstances surrounding the search.

ISSUE: Whether or not the order of the Ombudsman to have an in camera inspection of the questioned accounts are allowed as exceptions to the law on Secrecy of Bank Deposits (R.A. 1405). HELD: NO. RATIO: 1. In Union Bank of the Philippines v. court of Appeals, we held that Section 2 of the Law on Secrecy of Bank Deposits, as amended, declares bank deposits to be absolutely confidential except: a. In an examination made in the course of a special or general examination of a bank that is specifically authorized by the Monetary Board on reasonable suspicion that the former is committing or has committed bank fraud or some other serious irregularity; In an examination made by an independent auditor hired by the bank to conduct its regular audit provided that it is for audit purposes only and for the exclusive use of the bank; Written permission of the depositor; In cases of impeachment; Upon order of a competent court in cases of bribery or dereliction of duty of public officials, or In cases where the money deposited or invested is the subject matter of the litigation.

b. The authorities based their knowledge on pure hearsay: It was only when they implemented the warrant that they coordinated with the barangay officials. One of the barangay officials informed SPO3 De Leon that Ising Gutierrez Diwa and Priscilla Del Norte are one and the same person, but said barangay official was not presented in court. 2.

c. d. e. f.

FACTS: 1. Sometime in May 1998, petitioner Marquez received an Order from the Ombudsman Aniano A. Desierto dated April 29, 1998, to produce several bank documents for purposes of inspection in camera relative to various accounts maintained at Union Bank of the Philippines, Julia Vargas Branch, where petitioner is the branch manager. 2. The four accounts to be inspected are: a. Account No. 011-37270 b. Account No. 240-020718 c. Account No. 245-30317-3 d. Account No. 245-30318-1 3. Said accounts where involved in a case pending with the Ombudsman entitled, Fact-Finding and Intelligence Bureau (FFIB) v. Amado Lagdameo, et al.

Marquez vs. Desierto Sec 2, Art 3 | Bank Secrecy

In the case at bar, there is yet no pending litigation before any court of competent authority. What is existing is an investigation by the office of the Ombudsman. In short, what the Office of the Ombudsman would wish to do is to fish for additional evidence to formally charge Amado Lagdameo, et al., with the Sandiganbayan. Clearly, there was no pending case in court which would warrant the opening of the bank accounts for inspection. Zones of privacy are recognized and protected in our laws: a. b. c. Civil Code every person shall respect the dignity, personality, privacy and peace of mind of his neighbors and other persons Revised Penal Code makes criminal the violation of secrets by an officer, the revelation of trade and industrial secrets, and trespass to dwelling Special laws Invasion of privacy is an offense in the Anti-Wiretapping Law, Intellectual Property Code and the Secrecy of Bank Deposits Act.

3.

People vs Aruta

CONSTI II DIGESTS: 1D COMPILATION

CHUA, ALIMANGOHAN, CONTRERAS, BRIONES, LENCIO, RELOJO, TENGCO, LIM, DUMA, DANAO, MELLA, TONGSON

$(
following reasons: a. the policemen had sufficient time to apply for a search warrant but they failed to do so; b. the accused was not acting suspiciously; c. the accuseds identity was previously ascertained so applying for a warrant should have been easy; d. the accused in this case was searched while innocently crossing a street Consequently, there was no legal basis for the police to effect a warrantless search of the accuseds bag, there being no probable cause and the accused not having been legally arrested. The arrest was made only after the accused was pointed to by the informant at a time when she was not doing anything suspicious. The arresting officers do not have personal knowledge that the accused was committing a crime at that time.

"

Facts: On December 13, 1988, P/Lt. Abello of the Olongapo PNP was tipped off by an informer that Aling Rosa would be arriving from Baguio City the following day with a large volume of marijuana. As a result of the tip, the policemen waited for a Victory Bus from Baguio City near the PNB Olongapo, near Rizal Ave. When the accused got off, she was pointed to by the informer. She was carrying a traveling bag at that time. She was not acting suspiciously. She was arrested without a warrant. The bag allegedly contained 8.5 kilos of marijuana. After trial, she was convicted and imposed a penalty of life imprisonment. Issue: WON the marijuana allegedly taken from the accused is admissible in evidence. Held:No. There was no valid warrantless search and the evidence was not validly obtained and not admissible as evidence. Ratio: The following are cases where warrantless arrests are allowed. 1. Warrantless search incidental to a lawful arrest recognized under Sec 12, Rule 126 of the Rules of Court and prevailing jurisprudence; 2. Seizure of evidence in plain view the elements of which are: a. b. c. d. 3. prior intrusion based on the valid warrantless arrest in the pursuit of their official duties; the evidence was inadvertently discovered by the police who had the right to be where they are; evidence must be immediately apparent plain view justified mere seizure of evidence without further search;

NOLASCO v. PAO Facts: This is a motion for reconsideration of the Courts decision declaring that petitioners dwelling could be searched even without warrant for evidence of the charges of rebellion filed against her. Petitioner Mila Aguilar Roque was arrested at 11:30 and aboard a public vehicle on the road.

4. 5. 6. 7.

Search of a moving vehicle. Highly regulated by the government because the vehicles mobility reduces the expectations for privacy. When in transit in public thoroughfares such furnishes a highly reasonable suspicion which amounts to probable cause that occupant committed a criminal activity. Consented warrantless search Customs search Stop and frisk Exigent and Emergency circumstances.

Petitioner: warrantless search can be justified only if it is an incident to a lawful arrest and since petitioner was not lawfully arrested a search without warrant could not be made.
- "(I)f the majority opinion (citing sec. 12, Rule 126 of the Rules of Court 1 : "a general rule that, as an incident of an arrest, the place or premises where the arrest was made can also be searched without a search warrant) becomes settled law, the constitutional protection would become meaningless as the arresting party would still have the right to search the house of the arrestee and cart away his things and use them as evidence against him in court. Issue: Whether or not the search and seizure of the personalities at petitioners dwelling at Mayon Street, Quezon City was illegal and could not be deemed as incident to her arrest Held: Petitioners' Motion for Reconsideration is GRANTED. The seizure of documents and personal effects with such a void warrant could not be justified "as an incident of an arrest" outside petitioner's dwelling and the Constitution bars their admissibility as evidence and ordains their return to petitioners. Ratio: The earlier decision was tantamount to sanctioning an untenable violation of the basic constitutional right and guarantee against unreasonable searches and seizures. The better and established rule is a strict application of the exception provided in Rule 126, sec. 12 and that is to absolutely limit a warrantless search of a person who is lawfully arrested to his or her person at the time of and incident to his or her arrest and to "dangerous weapons or anything which may be used as proof of the commission of the offense." Such warrantless search

The above exceptions to the requirement of a search warrant, however, should not become unbridled licenses for law enforcement officers to trample upon the conditionally guaranteed and more fundamental right of persons against unreasonable search and seizures. The essential requisite of probable cause must still be satisfied before a warrantless search and seizure can be lawfully conducted. In order that the information received by the police officers may be sufficient to be the basis of probable cause, it must be based on reasonable ground of suspicion or belief a crime has been committed or is about to be committed. The marijuana obtained as a result of a warrantless search is inadmissible as evidence for the

CONSTI II DIGESTS: 1D COMPILATION

CHUA, ALIMANGOHAN, CONTRERAS, BRIONES, LENCIO, RELOJO, TENGCO, LIM, DUMA, DANAO, MELLA, TONGSON

$)
There is a necessary difference between a search of a dwelling house or other structure and a search of a vehicle. In the former a search warrant is required. In the latter, it is not necessary because it would not be practicable. The vehicle could be quickly moved out of the locality or the jurisdiction in which the warrant must be sought. For reference, see People v. Case, 320 Mich., 379, 190 NW, 389, 27 ALR, 686, quoted in p. 149150 of Case book.

"

obviously cannot be made in a place other than the place of arrest.

Papa v. Mago G.R. No 27360, February 28, 1968 Facts: Petitioners were Ricardo Papa, Chief of Police of Manila; Juan Ponce Enrile, Commissioner of Customs; Pedro Pacis, Collector of Customs of the Port of Manila; and Martin Alagao, a patrolman of the Manila Police Department and head of the counter-intelligence unit of the MPD. Petitioners sought the annulment of an order issued by Judge Hilarion Jarencio authorizing the release under bond of certain goods seized and held by petitioners. Said goods were being transported by two trucks out of the customs zone of the port of Manila. These goods along with the trucks were intercepted and seized by petitioner Alagao, upon the orders of petitioner Papa, for allegedly being misdeclared and undervalued in violation of the Tariff and Customs Code. Issue: W/N the seizure of the goods and the trucks was legal. Ruling: The seizure of the goods and the two trucks was legal. Ratio: Respondents Argument 1: Petitioner Alagao could not seize the goods in question without a search warrant. Court: Petitioner Papa had the authority under customs and tariff laws to order the seizure, and petitioner Alagao, as an agent of petitioner Papa, had the authority to execute said seizure. First, the Chief of Police of the MPD (petitioner Papa), was deputized by the Commissioner of Customs for the purposes of the enforcement of the customs and tariff laws and to effect searches, seizures and arrests. It was his duty to seize any cargo, articles or movable property when the same may be subject to forfeiture or liable for any fine under customs and tariff laws. Second, Section 2203 Tariff and Customs Code authorizes persons having police authority to enter, pass through or search any land enclosure; warehouse, store or building, not being a dwelling house and also to inspect, search and examine any vessel or aircraft and any trunk, package, box or envelope or any person on board, or stop and search and examine any vehicle, beast or person suspected of holding or conveying any dutiable or prohibited article introduced into the Philippines contrary to law without the need of a search warrant.

People v cfi

People v. Lo Ho Wing Section 2, Article III | Warrantless Searches Facts: The Special Operations Group, a unit of the Criminal Investigation Service (CIS) of the Philippine Constabulary (PC), received a tip from one of its informers about an organized group engaged in the importation of the illegal drugs, smuggling of contraband goods, and gunrunning. As part of the operations, the recruitment of confidential men and deep penetration agents was carried out to infiltrate the crime syndicate. One of those recruited was the discharged accused, Reynaldo Tia. Tia was introduced to his co-accused Lim Cheng Huat. Lim expressed a desire to hire a male traveling companion for his business trips abroad. Tia offered his services and was hired. Lim and Tia met anew on several occasions. In the course of those meetings, Tia was introduced to Peter Lo whom Tia found out to be the person he was to accompany to China in lieu of Lim. On October 4, 1987, appellant and Tia left for HongKong on board a Philippines Airlines Flight. Before they departed, Tia was able to telephone Captain Palmera to inform him of their expected date of return to the Philippines as declared in his round-trip plane ticket October 6, 1987 at two oclock in the afternoon. The day after they arrived in Hongkong, the pair thereafter went to a local store where appellant purchased six (6) tin cans of tea. Afterwards, they returned to the hotel. That evening, Tia went to appellants room. Upon entering, he saw two other men with the appellant. One was fixing the tea bags, while the other was burning the substance on a piece of aluminum foil using a cigarette lighter. Appellant joined the second man and sniffed the smoke emitted by the burning substance. Tia asked the latter what they would be bringing back to the Philippines. He was informed that their cargo consisted of Chinese drugs. The next day, October 76, 1987, the two returned to Manila via a China Airlines flight. The plane landed at the Ninoy Aquino International Airport, on schedule. Lim met the newly arrived pair at the arrival area. Appellant and Tia boarded the taxicab after putting their luggage inside the back compartment of the vehicle. Lim followed in another taxi cab.

CONSTI II DIGESTS: 1D COMPILATION

CHUA, ALIMANGOHAN, CONTRERAS, BRIONES, LENCIO, RELOJO, TENGCO, LIM, DUMA, DANAO, MELLA, TONGSON

$*
show that the search in question was made as regards a moving vehicle. Therefore, a valid warrant was not necessary to effect the search on appellant and his co-accused. The rules governing search and seizure have over the years been steadily liberalized whenever a moving vehicle is the object of the search on the basis of the practically. This is considering that before the warrant could be obtained, the place, things and persons o be searched must be described to the satisfaction of the issuing judge a requirement which borders by the use of a moving vehicle that can transport contraband from one place to another with impunity. We might add that a warrantless search of a moving vehicle is justified on the ground that it is not practicable to secure a warrant because the vehicle can be quickly moved out of the locality or jurisdiction in which the warrant must be sought. In the instant case, it was firmly established from the factual findings of the trial court that the authorities had reasonable ground to believe that appellant would attempt to bring in contraband and transport it within the country. The belief was based on intelligence reports gathered from surveillance activities on the suspected syndicate, of which the appellant was touted to be a member. Aside from this, they were also certain as to the expected date and time of arrival of the accused from China. But such knowledge was clearly insufficient to enable them to fulfill the requirements for the issuance of a search warrant. Still and all, the important thing is that there were probable cause to conduct the warrantless search, which must still be present in such a case.

"

Meanwhile, a team composed of six operatives headed by Captain Palmera was formed to act on the tip given by Tia. Upon seeing appellant and Tia leave the airport, the operatives who first spotted them followed them. Along Imelda Avenue, the car of the operatives overlook the taxicab ridden by appellant and Tia and cut into its path forcing the taxi driver to stop his vehicle. Meanwhile, the other taxicab carrying Lim sped away in an attempt to escape. Three pieces of luggage were retrieved from the back of the compartment of the vehicle. The operatives requested from the suspects permission to search their luggage. A tin can of tea was taken out of the red traveling bag owned by appellant, one of the operatives, pried the lid open, pulled out a paper tea bag from the can and pressed it in the middle to feel its contents. Some crystalline white powder resembling crushed alum came out of the bag. Suspecting the crystalline powder to be a dangerous drug, he had the three traveling bags opened for inspection. From the red traveling bag, a total of six (6) tin cans were found, including the previously opened. Tia and appellant were taken to the CIS Headquarters in Quezon City for questioning. Meanwhile, the second taxicab was eventually overtaken by two other operatives on Retiro Street, Quezon City. Lim was likewise apprehended and brought to the CIS Headquarters for interrogation. The tea bag opened by Sgt. Cayabyab during the search and seizure was sent to the PC-INP Crime Laboratory for the preliminary examination. Tests conducted on a sample of the crystalline powder inside the tea bag yielded a positive result that the specimen submitted was metaphetamine. Samples from each of the fifty-six (56) tea bags were also positive for the metephetamine. Hence, the three suspects were indicted.

Caballes v. Court of Appeals Warrantless Searches Facts: While on a routine patro in Brgy. Sampalucan (Laguna), Sgt. Victorino Noceja and Pat. Alex de Castro spotted a passenger jeep unusually covered with kakawati leaves. Suspecting that the jeep was loaded with smuggled goods, the two police officers flagged down the vehicle and asked its driver (appellant) what was loaded on the jeep. Appellant did not answer and so, with appellants consent, the police officers checked the cargo and discovered bundles of aluminum/galvanized conductor wires exclusively owned by the National Power Corporation (NPC). Appellant said that the wires came from Cavinti, a town 8 km away from Sampalucan. Appellant and the vehicle were brought to the Pagsanjan Police Station. The police turned over the wires to the Police Station Commander while appellant was incarcerated for 7 days in the Municipal Jail.

Appelants Argument: Appellant contends that the warrantless search and seizure made against the accused is illegal for being violative of Section 2, Article III of the Constitution. He reasons that the PC-CIS officers concerned could very well have procured a search warrant since they had been informed of the date and time of arrival of the accused at the NAIA well ahead of time, especially two (2) days in advance. The fact hat the search and seizure in question were made on a moving vehicle, appellant argues, does not automatically make the warrantless search herein fall within the coverage of the well-known exception to the rule of the necessity of a valid warrant to effect a search because, as aforementioned, the anti-narcotics agents had both time and opportunity to secure a search warrant.
Issue: The trial court erred in not declaring the search and seizure on the accused as illegal. Held: The contentions are without merit. Ratio: There are at least three (3) well-recognized exceptions thereto. As set forth in the case of Manipon, Jr. v. Sandiganbayan, these are: [1] search incidental to an arrest, [2] a search of a moving vehicle, and [3] seizure of evidence in plain view. The circumstances of the case clearly

Petitioners Arguments: The flagging down of his vehicle by police officers who were on routine patrol, merely on suspicion that it might contain smuggled goods, does not constitute probable cause that will justify a warrantless search and seizure. He did not give any consent, express or implied, to the search of the vehicle. Any evidence obtained in violation of his right against unreasonable search and seizure shall be deemed inadmissible.

CONSTI II DIGESTS: 1D COMPILATION

CHUA, ALIMANGOHAN, CONTRERAS, BRIONES, LENCIO, RELOJO, TENGCO, LIM, DUMA, DANAO, MELLA, TONGSON

%+
Searches without warrant of automobiles is allowed for the purpose of preventing violations of smuggling or immigration laws, provided such searches are made at borders or constructive borders (checkpoints near the boundary of the State). Probable Cause a reasonable ground of suspicion supported by circumstances sufficiently strong in themselves to warrant a cautious mans belief that the person accused is guilty of the offense with which he is charged the existence of such facts and circumstances which could lead a reasonably discreet and prudent man to believe that an offense has been committed and that the items, articles, or objects sought in connection with said offense or subject to seizure and destruction by law is in the place to be searched. o

"

Issues: WON the search without warrant is valid. Held: No. The face that the vehicle looked suspicios because it was covered with kakawati leaves does not constitute probable cause. Conducting the search without warrant is not justified. Ratio: The mere mobility of vehicles does not give the police officers unlimited discretion to conduct indiscriminate searches without warrants if made within the interior of the territory and in the absence of probable cause. Routine inspections are not regarded as violative of an individuals right against unreasonable search. o The search which is normally permissible is limited to the following instances: 1. Where the officer merely draws aside the curtain of a vacant vehicle which is parked on the public fair grounds 2. Simply looks into a vehicle 3. Flashes a light without opening the cars doors 4. Where the occupants are not subjected to a physical or body search 5. Where the inspection of the vehicles is limited to a visual search or visual inspection 6. Where the routine check is conducted in a fixed area o None of these circumstances is obtaining in this case. The police officers did not merely conduct a visual search or visual inspection of petitioners vehicle. " They had to reach inside the vehicle, lift the kakawati leaves and look inside the sacks before they were able to see the cable wires. " Thus, it cannot be considered a simple routine check. o United States c. Pierre: the physical intrusion of a part of the body of an agent into the vehicle goes beyond the area protected by the Fourth Amendment.

Obra v. Court of Appeals Section 2, Article III | Warrantless Searches Facts: Grybos wrote Obra, then Regional Director of the Bureau of Mines and Geo-Sciences in Baguio, a letter on behalf of the Gillies heirs of Palasa-an, Mankayan, complaining that private respondents, spouses James Brett and June Prill Brett, had been conducting illegal mining activities. Obra then wrote Brig. Gen. Dumpit, then the Commanding General of the Regional Unified Command I of the Philippine Constabulary requesting assistance in apprehending a truck allegedly used by private respondents in illegal mining in the area. The following day, Obra wrote private respondents and Ms. Grybos saying that an ocular inspection and field investigation was going to be conducted. They and their witnesses were requested to be present at the investigation. Obra also sent a letter to to Col. Estepa, Provincial Commander of Benguet to hold any mining operation or activity, if there be any, of the Bretts until the controversy or case has been resolved. An Isuzu ELF truck (ABX-587) belonging to private respondents was seized as it was entering the Mamakar mining area in Palasa-an, Mankayan, Benguet. It was impounded by the military and prevented from leaving the area except on mercy missions to transport sick soldiers and workers to the hospital and when used to buy food supplies for the men inside the camp.

Discussion of concepts in the case: Checkpoints may either be (1) a mere routine inspection or (2) an extensive search. o (Justified cases in routine inspections explained in the ratio) o In case of extensive searches, warrantless searches would be permissible only if the officers conducting the search have reasobale or probable cause to believe that either the motorist is a law-offender or they will find the instrumentality or evidence pertaining to the crime in the vehicle to be searched (e.g., smell of marijuana, Narcotics Command agents received information that marijuana will be transported). Search of moving vehicle o The vehicles inherent mobility reduces the expectation of privacy especially when it frurnishes a highly reasonable suspicion amounting to probable cause. o A warrantless search of a moving vehicle is justified on the ground that it is not practicable to secure a warrant because the vehicle can be quickly moved out of the locality or jurisdiction in which the warrant must be sought.

Private respondents
Filed a complaint for injunction and damages, with an application for temporary restraining order, alleging that the truck had been seized without prior investigation to determine the existence of probable cause. Petitioners, through a TRO, were ordered to cease and desist from preventing the subject truck from leaving the mine site. The court also found that no investigation had been conducted either by petitioner Obra or his office or by petitioner Dumpit to verify Grybos complaint before seizure, and that, as a result, private respondents had been deprived of the use and enjoyment of property without due process of law. The appellate court affirmed the trial courts decision. They could not be held liable for damages in the performance of their duty in good faith.

Petitioners allege that:

CONSTI II DIGESTS: 1D COMPILATION

CHUA, ALIMANGOHAN, CONTRERAS, BRIONES, LENCIO, RELOJO, TENGCO, LIM, DUMA, DANAO, MELLA, TONGSON

%!
At about 8: 00 o'clock in the morning of that same day (11 May 1989), Captain Alen Vasco, the Commanding Officer of the First Regional Command (NARCOM) stationed at Camp Dangwa, ordered his men to set up a temporary checkpoint at Kilometer 14, Acop, Tublay, Mountain Province, for the purpose of checking all vehicles coming from the Cordillera Region. The order to establish a checkpoint in the said area was prompted by persistent reports that vehicles coming from Sagada were transporting marijuana and other prohibited drugs. Moreover,information was received by the Commanding Officer of NARCOM, that same morning that a Caucasian coming from Sagada had in his possession prohibited drugs. The two (2) NARCOM officers started their inspection from the front going towards the rear of the bus. Accused who was the sole foreigner riding the bus was seated at the rear thereof. During the inspection, CIC Galutan noticed a bulge on accused's waist. Suspecting the bulge on accused's waist to be a gun, the officer asked for accused's passport and other identification papers. When accused failed to comply, the officer required him to bring out whatever it was that was bulging on his waist. The bulging object turned out to be a pouch bag and when accused opened the same bag, as ordered, the officer noticed four (4) suspiciouslooking objects wrapped in brown packing tape, prompting the officer to open one of the wrapped objects. The wrapped objects turned out to contain hashish, a derivative of marijuana. Thereafter, accused was invited outside the bus for questioning. But before he alighted from the bus, accused stopped to get two (2) travelling bags from the luggage carrier. Upon stepping out of the bus, the officers got the bags and opened them. A teddy bear was found in each bag. Feeling the teddy bears, the officer noticed that there were bulges inside the same, which did not feel like foam stuffing. It was only after the officers had opened the bags that accused finally presented his passport. Accused was then brought to the headquarters of the NARCOM at Camp Dangwa, La Trinidad, Benguet for further investigation. At the investigation room, the officers opened the teddy bears and they were found to also contain hashish. Representative samples were taken from the hashish found among the personal effects of accused and the same were brought to the PC Crime Laboratory for chemical analysis. In the chemistry report, it was established that the objects examined were hashish. a prohibited drug which is a derivative of marijuana. Thus, an information was filed against accused for violation of the Dangerous Drugs Act.

"

They are entitled to an award of damages. P.D. No. 1281, as amended, authorizes the BMGS Regional Director to order the seizure and confiscation xxx of the tools and equipment used in the commission of an offense and to deputize, when necessary, any member or unit of the PC, police agency, barangay or any qualified person to police mining activities

Issue: 1. 2. WON he conducted an investigation and found probable cause for ordering the seizure and impoundment of private respondents vehicle WON the seizure can by justified under the moving vehicle doctrine

Held: No and no. Ratio: 1. An investigation was yet to be held to determine the veracity of the allegations in Grybos complaint. The only basis for the seizure was an alleged certification from the BMGS that no mining permit had been issued to private respondents. However, it was not presented in evidence nor does a copy thereof appear in the records. Also, the BMGS found that private respondent June Prill Brett had a valid and subsisting mining claim within the area and that it was the Gillies family which had no permit or lease from the government, although it was the first to work the claim. 2. The truck was seized while it was entering the mining area; it was not transporting minerals outside of the area. The mobility of motor vehicles makes it possible for the vehicle to be searched to move out of the locality or jurisdiction in which the warrant must be sought, but this does not grant police officers unlimited discretion to conduct warrantless searches of automobiles in the absence of probable cause. Such a warrantless search has been held to be valid as long as the officers conducting the search have reasonable or probable cause to believe before the search that they will find the instrumentality or evidence pertaining to a crime in the vehicle to be searched. In this case, there could not have been any finding of probable cause that the truck was being used for any illegal mining activities.

Facts:

People v. Malmstedt Section 2, Article III| Warrantless searches

Mikael Malmstedt m a Swedish national was charged before the RTC of La Trinidad, Benguet, for violation of Section 4, Art. II of Republic Act 6425, as amended, otherwise known as the Dangerous Drugs Act of 1972, as amended. In the evening of 7 May 1989, accused left for Baguio City. Upon his arrival thereat in the morning of the following day, he took a bus to Sagada and stayed in that place for two (2) days. Then in the 7 in the morning of May 11, 1989, the accused went to Nangonogan bus stop in Sagada.

ACCUSEDS DEFENSE During the arraignment, accused entered a plea of "not guilty." For his defense, he raised the issue of illegal search of his personal effects. He also claimed that the hashish was planted by the NARCOM officers in his pouch bag and that the two (2) travelling bags were not owned by him, but were merely entrusted to him by an Australian couple whom he met in Sagada. He further claimed that the Australian couple intended to take the same bus with him but because there were no more seats available in said bus, they decided to take the next ride and asked accused to take charge of the bags, and that they would meet each other at the Dangwa Station.
The trial court found the guilt of the accused Mikael Malmstedt established beyond

CONSTI II DIGESTS: 1D COMPILATION

CHUA, ALIMANGOHAN, CONTRERAS, BRIONES, LENCIO, RELOJO, TENGCO, LIM, DUMA, DANAO, MELLA, TONGSON

%#
Ratio: Justice Narvasa & Isagani Cruz (Concurring and Dissenting opinion) There was no probable cause in this case- hence there is no valid arrest; and when there is no valid arrest, any search made incidental thereto is void. Malmstedt has not committed, was not committing or was about to commit a crime in the soldiers presence when he was searched. All they had was a suspicionthe hope of intercepting any dangerous drug being transported . There is no probable cause yet. The mere bulge in his waist does not suffice. Upon searching and finding the hashish, that is the only time that they arrested Malmstedt--- a typical fishing expedition. The arrest must be first made; then the incidental search follows. The process cannot be reversed; otherwise it amounts to a fishing expedition. According to Justice Cruz, in this case, it was the fruit of the poisonous tree that washed clean the tree itself.

"

reasonable doubt. Seeking the reversal of the decision of the trial court finding him guilty of the crime charged, accused argues that the search of his personal effects was illegal because it was made without a search warrant and, therefore, the prohibited drugs which were discovered during the illegal search are not admissible as evidence against him. Issue: WON the contention of the accused is valid, and therefore the RTC ruling be reversed. Held: No. The decision of the RTC was upheld. Accused was searched and arrested while transporting prohibited drugs (hashish). A crime was actually being committed by the accused and he was caught in flagrante delicto. Thus, the search made upon his personal effects falls squarely under paragraph (1) of the foregoing provisions of law, which allow a warrantless search incident to a lawful arrest. While it is true that the NARCOM officers were not armed with a search warrant when the search was made over the personal effects of accused, however, under the circumstances of the case, there was sufficient probable cause for said officers to believe that accused was then and there committing a crime. The Constitution guarantees the right of the people to be secure in their persons, houses, papers and effects against unreasonable searches and seizures. However, where the search is made pursuant to a lawful arrest, there is no need to obtain a search warrant. A lawful arrest without a warrant may be made by a peace officer or a private person under the following circumstances.

The State must rely on the strength of its evidence and not on the weakness of the defense. The disregard of the fundamental constitutional rights may result in the escape of the guilty, and all because the constable has blundered, rendering the evidence inadmissible even if truthful or otherwise credible.
Mentioned landmark cases: People v. Aminnudin: Aminnudin was arrested without a warrant by PC officers as he was disembarking from an inter-island vessel The officers were waiting for him because he was, according to an informer's report, then transporting marijuana. The search of Aminnudins bag confirmed the informer's report; the bag indeed contained marijuana. The Court nevertheless held that since the PC officers had failed to procure a search warrant although they had sufficient time (2days) to do so. No urgency in this case---- arrest and seizure is illegal. Evidence is held to be inadmissible in court. People v. Claudio: The arresting officer had secretly ascertained that the woman(boarded a bus) he was arresting was in fact in possession of marijuana; he had personally seen that her bag contained not only vegetables but also package emitting the odor of marijuana.---- warrantless arrest valid, proper warrantless search, evidence is admissible. People v. Tangliben: the person arrested was about to board a bus, when he was stopped and searched for he was acting suspiciously and had been positively pointed to as carrying marijuana.

Sec. 5 Arrest without warrant; when lawful. A peace officer or a private person may, without a warrant, arrest a person: (a) When, in his presence, the person to be arrested has committed is actually committing, or is attempting to commit an offense; (b) When an offense has in fact just been committed, and he has personal knowledge of facts indicating that the person to be arrested has committed it; and (c) When the person to be arrested is a prisoner who has escaped from a penal establishment or place where he is serving final judgment or temporarily confined while his case is pending, or has escaped while being transferred from one confinement to another. Probable cause has been defined as such facts and circumstances which could lead a reasonable, discreet and prudent man to believe that an offense has been committed, and that the objects sought in connection with the offense are in the place sought to be searched. Warrantless search of the personal effects of an accused has been declared by this Court as valid, because of existence of probable cause, where the smell of marijuana emanated from a plastic bag owned by the accused, 10 or where the accused was acting suspiciously, 11 and attempted to flee.

* In both Claudio and Tangliben cases, the accused were about to board passenger buses, making it urgent for the police officers concerned to take quick and decisive action.
In Posadas, the person arrested and searched was acting suspiciously, too, and when accosted had attempted to flee from the police officers In Maspil and Lo Ho Wing, there was definite information of the precise identity of the persons engaged in transporting prohibited drugs at a particular time and place. In Manipon, it was held that it is not practicable to secure a warrant because the vehicle can be quickly moved out of the locality or jurisdiction in which the warrant must be sought.

Roan vs. Gonzales Section 2, Article III | Warrantless searches

CONSTI II DIGESTS: 1D COMPILATION

CHUA, ALIMANGOHAN, CONTRERAS, BRIONES, LENCIO, RELOJO, TENGCO, LIM, DUMA, DANAO, MELLA, TONGSON

%$
Ratio: Petiti oner No depositions were taken by the respondent judge in accordance with the rule in Section 4. Respondent Judge When PC Capt. Quillosa personally filed his application for a search warrant, he appeared before me in the company of his 2 witnesses, Morada and Tohilida who presented to me their respective affidavits. As the application was not yet subscribed and sworn to, I proceeded to examine Captain Quillosa on the contents to ascertain if he knew and understood the same. Afterwards, he subscribed and swore the same before me. Courts Answer

"

Facts: The challenged search warrant was issued by the respondent judge. Two days later, the petitioners house was searched but none of the articles listed in the warrant was discovered. However, the officers conducting the search found one Colt Magnum revolver and 18 live bullets which they confiscated. These are now the bases of the charges against the petitioner. The witness stated that they saw 8 men deliver arms to petitioner in his house and that he saw everything through an open window of the house. He also said that six of the weapons were .45 caliber pistols and two were .38 caliber revolvers. In order for a search warrant to be valid: 1. It must be supported by probable cause to be determined by the judge or some authorized officer after examining the complainant and the witnesses he may produce. This requirement was a refinement proposed by Delegate Francisco and his purpose was the strengthening of the guaranty against unreasonable searches and seizures. To implement this requirements, Section 4 of the Rules of Court provide: The municipal or city judge must personally examine on oath or affirmation the complainant and any witnesses he may produce and take their depositions in writing and attach them to the record. Probable cause refers to such facts and circumstances which would lead a reasonably discreet and prudent man to believe that an offense has been committed and that the objects sought in connection with the offense are in the place sought to be searched. The probable cause must refer to only one specific offense. 2. There must be a specific description of the place to be searched and the things to be seized, to prevent arbitrary and indiscriminate use of the warrant Issue: Whether or not the search warrant was valid. Held: The search warrants are null and void.

All he did was question Capt. Quillosa on the contents of his affidavit only to ascertain if he knew and understood it. It means that he would not have asked any questions at all if the affidavit had already been completed when it was submitted to him. In Mata vs. Bayona, mere affidavits of the complainant and his witnesses are not sufficient. The judge has to take depositions in writing of the complainant and the witnesses he may produce and attach them to the record. Such written deposition is necessary so that the Judge may be able to properly determine the existence or non-existence of the probable cause, to hold liable for perjury the person giving it if it will be found that his declarations are false. He saw no need to have applicant This means that the applicant was asking for the Quillosas deposition taken issuance of the search warrant on the basis of mere considering that he was applying hearsay and not of information personally known to for a search warrant on the basis him. His application, standing alone, was of the information provided by the insufficient to justify the issuance of the warrant. It witnesses whose depositions had was necessary for the witnesses themselves, by already been taken. their own personal information, to establish the applicants claims. The examination must be probing and exhaustive, not merely routinary or pro-forma. The examining magistrate must not simply rehash the contents of the affidavit but must make his own inquiry on the intent and justification of the application. The remark made by Tolihida they were suspicious of the petitioner because he was a follower of the opposition candidate in the forthcoming election did not excite the judges own suspicions. This should have put him on guard as to the motivations of the witnesses and alerted him to possible misrepresentations from them. It did not occur to the judge to ask how the witness could be so certain even as to the caliber of the guns, or how far he was from the window, or why his presence was not noticed. These would have been judicious questions but they were omitted. Instead, the declarations were readily accepted.

CONSTI II DIGESTS: 1D COMPILATION

CHUA, ALIMANGOHAN, CONTRERAS, BRIONES, LENCIO, RELOJO, TENGCO, LIM, DUMA, DANAO, MELLA, TONGSON

%%
The pistols and bullets cannot be used as evidence against the petitioner in the criminal action against him for illegal possession of firearms. Pending resolution of that case, the said articles must remain in custodial egis.

"

Solicitor generals argument: 1. Although such defects have rendered the search warrant invalid, such defects were waived when the petitioner voluntarily submitted to the search and manifested his conformity in writing. Courts answer: 1. What is seen here is pressure exerted by the military authorities, who coerced the petitioner to sign the supposed waiver as a guaranty against a possible challenge later to the validity of the search they were conducting. Confronted with the armed presence of the military, the petitioner had no choice but to submit. Respondents argument: 1. The Colt Magnum pistol and the 18 live bullets seized from petitioner were illegal per se and therefore could have been taken by the military authorities even without warrant. Possession of said articles was violative of PD 1866 and considered malum prohibitum. Hence, illegal articles could be taken even without a warrant. Courts answer: 1. Prohibited articles may be seized but only as long as the search is valid. In this case the search is invalid because: a. there was no valid search warrant b. absent such a warrant, the right was not validly waived by the petitioner. 2. In short, the military officers who entered the petitioners premises had no right to be there and therefore had no right to seize the pistols and bullets. Motive is immaterial in mala prohibita but the subjects of this kind of offense may not be seized simply because they are prohibited. A search warrant is still necessary. Otherwise, the military authorities could have just entered the premises and looked for the guns without bothering to first secure a search warrant. The fact that they did bother to do so indicates that they themselves recognized the necessity of such warrant for the seizure of weapons. 3. There are certain instances when a search may be validly made without warrant and articles may be taken validly as well as the result of that search. Examples: (Exceptions) - a warrantless search may be incidental to a lawful arrest as when the person being arrested is frisked for weapons he may be able to use against the arresting officer. - motor cars may be inspected at borders to prevent smuggling of aliens and contraband and even in the interior upon showing of probable cause - vessels and aircraft are traditionally removed from the rule because of their mobility and their relative ease in fleeing the states jurisdiction - prohibited articles may be taken without warrant if they are open to eye and hand and the peace officer comes upon them inadvertently 4. The instant case does not come under any of the exceptions. The respondents did not even claim that they stumbled upon the pistol and bullets for the fact is that these things were deliberately sought. Thus, the conclusion is that the petitioners pistol and bullets were confiscated illegally and therefore are protected by the exclusionary principle. Exclusionary rule according to Chief Justice Concepcion, it is the only practical means of enforcing the constitutional injunction against abuse

5.

The petitioner should have, before coming to this court, filed a motion for the quashal of the search warrant by the respondent judge in accordance with the normal procedure. But it was held in Burgos that this procedural flaw nothwithstanding, we take cognizance of this petition in view of the seriousness and urgency of the constitutional issues raised.

People v. Tabar Section 2, Article III | Warrantless searches Facts: An entrapment operation resulted to the arrest of a minor. The minor (Rommel) said that he got the Marijuana from his aunt (Carmelina). Carmelina Tabar was arrested after having proven to be in possession of Marijuana after the search and seizure conducted by the operatives. Tabar appeals that the marijuana cigarettes seized from her are inadmissible in evidence because they wer obtained in violation of the constitutional guarantee against unreasonable search and seizure. Issue: Whether or not the search and seizure performed was in violation of the Constitution Held: The search and seizure performed by the authorities are valid and the marijuana cigarettes obtained from the accused are admissible in evidence. Ratio: The RPC provides for warrantless arrests of persons committing a crime in the presence of the policemen. She could lawfully be arrested and searched for anything which may be used as proof of the commission of an offense without the corresponding arrest and search warrants. [Section 5, Rule 113 and Section 12, Rule 126 of the Revised Rules of Court] Even assuming ex gratia argumenti that the search and seizure were witout a warrant, the appellant had effectively waived her constitutional right relative thereto by voluntarily submitting to the search and seizure. The constitutional right to be presumed innocent is not infringed by the reliance of trial court on the presumption of regularity in the performance of official functions on the part of arresting officers.

Aniag, Jr. v COMELEC Sec. 2, Article III, Warrantless searches

CONSTI II DIGESTS: 1D COMPILATION

CHUA, ALIMANGOHAN, CONTRERAS, BRIONES, LENCIO, RELOJO, TENGCO, LIM, DUMA, DANAO, MELLA, TONGSON

%&
Cruz J. Concurring but objects to decision about checkpoint: BoR was intended to limit authority of the state even if asserted on ground of national security. With this, purposes of national security would be used as an excuse for every individual to be searched at random just because he arouses suspicion, hostility or malice on officers manning the checkpoint. Regalado J. concurring and dissent that Arellano was a victim of unlawful search without warrant. Consent was impliedly given by Arellano. It is a rule that a search may be validly conducted if the person searched consented thereto. He would concur with the decision if it was based on the reason that it was due to obedience of a lawful order by a superior, and not because there was an unlawful search without warrant.

"

Facts: Pursuant to the Gun Ban, Serapio Taccad wrote Congressman Aniag, requesting the return of 2 firearms issued to him by the HOR. Aniag immediately instructed his driver Arellano to pick the firearms from Aniags house at Valle Verde to return them to Congress. On Arellanos way to the Batasan Complex, policemen manning the outpost flagged down the car he drove. Policemen searched the card and found firearms neatly packed in their gun cases at the trunk of the car. Arellano was apprehended and detained. He explained that he was ordered by Cong. Aniag to get the firearms from his house and return it to Sergeant-at-arms Taccad. Petitioners Argument: a. Searching the car without a warrant and without informing the driver of his fundamental rights was illegal b. Firearms were not tucked in the waist not within immediate reach of Arellano but were neatly packed c. Search was violative of Secs. 2 and 3 par.2 of Art. III of the Constitution Issue: Whether or not warrantless search by PNP is legal and if they can be used as evidence. Held: No. Warrantless search in the case is illegal and firearms seized cannot be used as evidence in a proceeding against petitioner. Ratio: General rule: A valid search must be authorized by a search warrant duly issued by an appropriate authority o Exceptions: ! search incident to lawful arrest, ! warrantless search of moving vehicles, ! search conducted at police or military checkpoints ! stop and search without warrant based on prior confidential information reasonably corroborated by attendant matters. (examples in page 179) o Warrantless search is not violative of the Constitution for as long as the vehicle is neither searched nor its occupants subjected to a body search and inspection of the vehicle is merely limited to a visual search. Extensive search without warrant could only be resorted to if officers had reasonable or probable cause to believe before the search that the motorist was a: a. law offender b. they would find evidence pertaining to the commission of a crime in such vehicle *Existence of probable cause depends on the facts of each case. In the case, there was no evidence that policemen were impelled to do an extensive search without warrant because of a confidential report leading them to believe that a motorist was engaged in gunrunning, transporting firearms or organizing special strike force. There was also no evidence of package or behavior that triggered the suspicion of policemen. Without this, search could not be valid. Action of policemen intruded the privacy and security of property of Arellano. Although it can be argued that Arellanos acquiescence(consent) to the search is an implied waiver of his right to question the reasonableness of the search of the vehicle and seizure of firearms. However, with the police in control, he did not have any choice but to submit to PNPs scrutiny. Otherwise, any opposition would raise suspicion on the part of the police and would provide probable cause to arrest him and do an extensive search of his vehicle.

Facts:

People v. Tudtud Section 2 | Warrantless Searches

A civilian asset named Bobong Solier reported to the Toril Police Station about a certain Noel Tudtud, who was allegedly responsible for the proliferation of marijuana in their area. The Intelligence Section of the Toril Police Station conducted surveillance in Soliers neighborhood. For five days, they gathered information and learned that Tudtud was involved in illegal drugs. On August 1, 1999, Solier informed the police that Tudtud had headed to Cotabato and would be back later that day with new stocks of marijuana. Solier described Tudtud as begbodied and short, and usually wore a hat. A team posted themselves at the corner of Saipon and McArthur Highway to await Tudtuds arrival. All wore civilian clothes. About 8:00 PM, two men disembarked from a bus and helped each other carry a carton marked King Flakes. Two of the police observed that one of the men fit Tudtuds description. The police approached the suspects and identified themselves as police officers. One of the police informed them that the police had received information that stocks of illegal drugs would be arriving that night. Both of the suspects denied that they were carrying drugs. The police asked him if he could see the contents of the box. Tudtud obliged saying that it was alright. The box contained what seemed to be marijuana. The police thus arrested Tudtud and his companion and brought them to the police station. The two did not resist. Forensic tests confirmed that the package contained marijuana. The two were charged before the RTC of Davao with illegal possession of prohibited drugs.

Respondents Contention:
" Defense reserved their right to question the validity of their arrest and the seizure of the evidence against them

RTC justified the warrantless search of appellants belongings under the first exception, as a search incident to a lawful arrest Issue: Whether or not the police had probable cause to arrest Tudtud and his companion Held:

CONSTI II DIGESTS: 1D COMPILATION

CHUA, ALIMANGOHAN, CONTRERAS, BRIONES, LENCIO, RELOJO, TENGCO, LIM, DUMA, DANAO, MELLA, TONGSON

%'
Section 2, Article 9 | Warrantless Searches Facts: Motivated by the discovery that athletes were leaders in the student drug culture and concern that drug use increases the risk of sports-related injury, petitioner school district adopted the Student Athlete Drug Policy, which authorizes random urinalysis drug testing of students who participate in its athletics programs. Respondent Acton was denied participation in his schools football program when he and his parents refused to consent to the testing.

"

There is no evidence other than the hearsay testimony of the arresting officers and their informant, the conviction of appellants cannot be sustained. Ratio: " As to the justification by the RTC of the warrantless search [First exception: search incident to a lawful arrest] o Jurisprudence provides: Arrest must precede the search ! The search in question preceded the arrest ! Exception: search may precede if the police have probable cause to make the arrest at the outset of the search " On the question of probable cause o reliable information alone is not sufficient to justify a warrantless arrest under Section 5 (a), Rule 113 ! Rule requires that he has committed, is actually committing, or is attempting to commit an offense Appellants in this case were neither performing any overt act or acting in a suspicious manner that would hint that a crime has been, was being, or was about to be, committed ! Ruling in Burgos case faithfully adheres to the letter of Section 5 (a), Rule 113, which a great majority of cases conforms to o The right of the accused to be secure against any unreasonable searches on and seizure of his own body and any deprivation of his liberty being a most basic and fundamental one, the statute or rule that allows exception to the requirement of a warrant of arrest is strictly construed o The surveillance made by the police did not actually consist of staking out appellant Tudtud to catch him in the act of plying his illegal trade, but of a mere gathering of information from the assets there ! Such information is hearsay, not of personal knowledge o Requisites of an effective waiver of rights against unreasonable searches and seizures: 1. It must appear that the rights exist 2. The person involved had knowledge, actual or constructive, of the existence of such right 3. Said person had an actual intention to relinquish the right $ Prosecution failed to establish the second and thirs requisites $ Constitution and jurisprudence requires more than the presence of these circumstances [when Tudtud said that it was alright to open the box and he did not resist to open the box himself] to constitute a valid waiver of the constitutional right against unreasonable searches or seizures $ Courts indulge every reasonable presumption against waiver of fundamental constitutional rights $ The fact that the person failed to object to a search does not amount to permission thereto Appellants implied acquiescence could not have been more than mere passive conformity given under coercive or intimidating circumstances and is considered no consent at all within the purview of the constitutional guarantee

Respondents argument: Policy violated the 4th and 14th amendments and the Oregon Constitution. *4th amendment: The Federal Government shall not violate the right of the people to be secure in their persons, houses, papers, and effects, against unreasonable searches and seizures. 14th amendment: prohibits state and local governments from depriving persons of life, liberty, or property without certain steps being taken to ensure fairness.
Issue: Whether or not Vernonias Student Athlete Drug Policy is unconstitutional. Held: No. The policy is valid and constitutional. The court took into account the following considerations: a) The decreased expectation of privacy b) Relative unobtrusiveness of the search c) Severity of the need met by the search Ratio: The ultimate measure of the constitutionality of a governmental search is reasonableness. Whether it meets the reasonableness standard is judged by balancing its intrusion on the individuals 4th amendment interests against its promotion of legitimate government interest. A warrant (which requires probably cause) is not required to establish the reasonableness of all government searches. A search unsupported by probable cause can be constitutional when special needs, beyond the normal need for law enforcement, make the warrant and probablecause requirement impracticable. The court found such special needs to exist in the public school context. There, the warrant requirement would unduly interfere with the maintenance of the swift and informal disciplinary procedures needed. The court has also upheld suspicionless searches and seizures to conduct drug testing on some instances. The 4th amendment does not protect all subjective expectations of privacy, but only those that society recognizes as legitimate, and this legitimacy depends upon the individuals legal relationship with the state. In the present case, the subjects of the policy are children who have been committed to the temporary custody of the State as schoolmaster. The reasonableness inquiry cannot disregard the schools custodial and tutelary responsibility for children.

M. Vernonia Sch. Distr. 47J v Acton

CONSTI II DIGESTS: 1D COMPILATION

CHUA, ALIMANGOHAN, CONTRERAS, BRIONES, LENCIO, RELOJO, TENGCO, LIM, DUMA, DANAO, MELLA, TONGSON

%(
surveillance of the residence of the accused-appellant who was then the barangay captain of barangay Bagonbon, San Carlos City, Negros Occidental. They saw two (2) tall plants in the backyard of the accused-appellant, which they suspected to be marijuana plants. c. The police officers reported the result of their surveillance to the Chief of Narcotics Command (NARCOM), Bacolod City, who immediately formed a team tasked to handle the operation against the accused-appellant. d. The team applied for a search warrant with the office of the Executive Judge Ponferrada of Bacolod City, who informed them that he did not have territorial jurisdiction over the matter since the place to be searched was located in San Carlos City. Therefore, the team left Bacolod City for San Carlos City. Arriving at around 6:30 in the evening, they went straight to the house of Executive Judge Javellana to secure a search warrant. e. They were not able to do so because it was nighttime and after office hours. They were told by the judge to go back in the morning. f. Nonetheless, the team proceeded to the residence of the accused-appellant even without securing a search warrant. g. What happened thereafter is subject to conflicting accounts: i. Prosecution side contends that the accused-appellant gave the team permission to come inside his residence; that the same was informed of the charge against him; and that the same was informed of his constitutional rights ii. Accused-appellants side None of the members of the team asked his permission to search his house and the premises Issue: WON accused-appellants constitutional right against unreasonable searches and seizures had been violated by the police authorities. Held: YES. Ratio: a. In the instant case, the search and seizure conducted by the composite team in the house of the accused-appellant was not authorized by a search warrant. The same does not also fall under any of the exceptions, which makes a warrantless search and seizure valid. b. It is extant from the records that the accused-appellant did not consent to the warrantless search and seizure conducted. While the right to be secure from unreasonable search and seizure may, like every right, be waived either expressly or impliedly, such waiver must constitute a valid waiver made voluntarily, knowingly and intelligently. The accusedappellants silence during the unreasonable search and seizure could not be construed as voluntary submission or an implied acquiescence to warrantless search and seizure especially so when the members of the raiding team were intimidatingly numerous and heavily armed. c. Mere passive conformity given under coercive or intimidating circumstances is considered no consent at all within the purview of the constitutional guarantee. Consequently, herein accused-appellants lack of objection to the search and seizure is not tantamount to a waiver of his constitutional right or a voluntary submission to the warrantless search and seizure. d. Jurisprudence: People v. Burgos; held: accused is not to be presumed to have waived the unlawful search simply because he failed to object e. Necessity and urgency of the situation is not a valid justification, which can be proffered by the raiding team. Surely, the raiding team had all the opportunity to have first secured a

"

a) Decreased expectation of privacy Students within the school environment have a lesser expectation of privacy than members of the population generally (theyre subject to physical examinations, vaccinations, dental checks, etc.). Legitimate expectations are even less with regard to student athletes; by choosing to go out for the team, they voluntarily subject themselves to a degree of regulation even higher than that imposed on students generally (they have to maintain a min. grade point average, they must acquire adequate insurance coverage, training hours, etc.).
b) Relative unobtrusiveness of the search Respondents argue that the policy is intrusive because it requires students to identify in advance prescription medications they are taking. But the court has never indicated that requiring advance disclosure of medications is per se unreasonable. Furthermore, the policy did not require such; it says simply that student athletes must provide verification of the medication prior to being tested if they are or have been taking prescription medication.

c) Severity of the need met by the search Since the program called for drug testing in the absence of individualized suspicion, the district must demonstrate a compelling need for the program. The court thinks that it is met. *Compelling state interest an interest which appears important enough to justify the particular search at hand, in light of other factors which show the search to be relatively intrusive upon a genuine expectation of privacy.
Deterring drug use by our nations school children is important school years are the time when the physical, psychological, and addictive effects of drugs are most severe. And of course, the effects of a drug-infested school are visited not just upon the users, but upon the entire student body and faculty, as the educational process is disrupted. The program is also directed more narrowly to drug use by school athletes, where the risk of immediate physical harm to the drug user or those with whom he is playing his sport is particularly high.

As to Immediacy: The court saw an immediate crisis in the large segment of the student body, particularly those involved in athletics, that was in a state of rebellion. And disciplinary actions had reached epidemic proportions. The rebellion was being fueled by alcohol and drug abuse. As to Efficacy: The drug problem is effectively addressed by making sure that the athletes do not use drugs. Respondents argue that there are less intrusive means, namely drug testing on suspicion of drug use. But the court held that the alternative entails difficulties parents willing to accept random drug testing might not be willing to accept accusatory drug testing, teachers will impose testing troublesome but not drug-likely students, teachers will have additional functions of spotting and bringing to account drug abuse.

People vs. Compacion Facts: a. Accused-appellant Compacion was charged with violating Sec. 9 of R.A. No. 6425 (Dangerous Drugs Act of 1972), as amended by R.A. No. 7659. b. Acting on a confidential tip supplied by a police informant that the accused-appellant was growing and cultivating marijuana plants, SPO1 Linda and SPO2 Sarong conducted a

CONSTI II DIGESTS: 1D COMPILATION

CHUA, ALIMANGOHAN, CONTRERAS, BRIONES, LENCIO, RELOJO, TENGCO, LIM, DUMA, DANAO, MELLA, TONGSON

%)
WON the search and seizure of the marijuana plants were lawful. Held: No. There was no valid warrantless search and the evidence was not validly obtained and not admissible as evidence. The appellant is also acquitted because the evidence obtained against him was obtained contrary to the constitution. Ratio: The Constitution lays down the general rule that a search and seizure must be carried on the strength of a judicial warrant. Otherwise, the search and seizure is deemed "unreasonable." Evidence procured on the occasion of an unreasonable search and seizure is deemed tainted for being the proverbial fruit of a poisonous tree and should be excluded. Such evidence shall be inadmissible in evidence for any purpose in any proceeding. In the instant case, there was no search warrant issued by a judge after personal determination of the existence of probable cause. From the declarations of the police officers themselves, it is clear that they had at least one (1) day to obtain a warrant to search appellant's farm. Their informant had revealed his name to them. The place where the cannabis plants were planted was pinpointed. From the information in their possession, they could have convinced a judge that there was probable cause to justify the issuance of a warrant. But they did not. Instead, they uprooted the plants and apprehended the accused on the excuse that the trip was a good six hours and inconvenient to them. We need not underscore that the protection against illegal search and seizure is constitutionally mandated and only under specific instances are searches allowed without warrants. The mantle of protection extended by the Bill of Rights covers both innocent and guilty alike against any form of high-handedness of law enforcers, regardless of the praiseworthiness of their intentions. We find no reason to subscribe to Solicitor General's contention that we apply the "plain view" doctrine. For the doctrine to apply, the following elements must be present: (a) a prior valid intrusion based on the valid warrantless arrest in which the police are legally present in the pursuit of their official duties; (b) the evidence was inadvertently discovered by the police who have the right to be where they are; and (c) the evidence must be immediately apparent; and (d) plain view justified mere seizure of evidence without further search. The seizure of evidence in plain view applies only where the police officer is not searching for evidence against the accused, but inadvertently comes across an incriminating object.

"

f. g.

h.

i.

search warrant given that from the date of the surveillance, four (4) days transpired until the search and seizure was conducted concerning the alleged marijuana plants. In flagrante delicto in the very act of committing a misdeed In justifying the validity of the warrantless arrest, search and seizure, the trial court ruled that the accused-appellant was caught in flagrante delicto of having planted, grown and cultivated the marijuana plants, which were easily visible from outside the residence of the accused. PLAIN VIEW DOCTRINE: i. Generally, objects in the plain view of an officer who has a right to be in a position to have that view are subject to a valid warrantless seizure ii. Usually applies wherein an officer is not searching for evidence against the accused, but nonetheless inadvertently comes across an incriminating object iii. Elements (4): 1. Prior valid intention based on the valid warrantless arrest in which the police are legally present in the pursuit of their official duties; 2. The evidence was inadvertently discovered by the police who have the right to be where there are; 3. The evidence must be immediately apparent; 4. Plain view justified were seizure of evidence without further search In the case at bar, there was no valid warrantless arrest. It was also undisputed that the NARCOM agents conducted a surveillance of the residence of the accused-appellant on the suspicion that he was growing and cultivating marijuana, when they allegedly came in plain view of the marijuana plants. Furthermore, when the agents entered his premises, their intention was to seize the evidence against him. In fact, they initially wanted to secure a search warrant but could not simply wait for one to be issued. Hence, the NARCOM agents did not come across the marijuana plants inadvertently in accordance with the plain view doctrine.

People vs Valdez Facts: On September 25, 1996, at Sitio Bulan, Nueva Vizcaya, Abe Valdez, who was caught in the act and without authority of law, did then and there willfully, unlawfully and feloniously plant, cultivate and culture seven (7) fully grown marijuana plants known as Indian Hemp weighing 2.194 kilos, from which dangerous drugs may be manufactured or derived, to the damage and prejudice of the government of the Republic of the Philippines. The police officers receiving a tip from an unnamed informer proceeded to uproot said marijuana plants and arrest the cultivator. They also took photos of the appellant beside the plants. All of this was done without a valid search warrant from a judge with jurisdiction over the area. The OSG avers that appellant was not yet under custodial investigation when he admitted to the police that he owned the marijuana plants. His right to competent and independent counsel, accordingly, had not yet attached. Moreover, appellants failure to impute any false motive for the police officers to falsely accuse him indicates that the presumption of regularity in the performance of official duties by police officers was not sufficiently rebutted. Issue:

-END-

CONSTI II DIGESTS: 1D COMPILATION

CHUA, ALIMANGOHAN, CONTRERAS, BRIONES, LENCIO, RELOJO, TENGCO, LIM, DUMA, DANAO, MELLA, TONGSON

%*
Board of Education v. Earls, et al. Warrantless Searches Facts: Assailed in this case was the Student Activities Drug Testing Policy adopted by the Tecumseh, Oklahoma School District. The said policy required all middle and high school students to consent to urinalysis testing for drugs in order to participate in any extracurricular activity. Respondents attacked the policy for being violative of Fourth Amendment rights (which is similar to Section 2 of Article III of the 87 Consti). The Tenth Circuit Court (Court of Appeals) held that the policy violated the Fourth Amendment ruling that before imposing a suspicionless drug testing program a school must demonstrate some indentifiable drug abuse problem among a sufficient number of those tested, such that testing that group will actually redress its drug problem. Issue: W/N the Policy was unconstitutional for being violative of Fourth Amendment rights. Ruling: The Policy did not violate Fourth Amendment rights. The Decision of the Circuit Court was reversed. Ratio: In explaining its decision the Court discussed three points, (a) the nature of the students privacy interest, (b) the character of the intrusion imposed by the Policy, and (c) the nature and the immediacy of the governments concerns and the efficacy of the Policy in meeting them. Nature of the privacy interest allegedly compromised by the drug testing The subjects of the Policy are children who have been committed to the temporary custody of the State as schoolmaster. The Policy was undertaken in furtherance of the governments responsibilities, under a public school system, as guardian and tutor of children entrusted to its care. When the government acts as guardian and tutor the relevant question is whether the search is one that a reasonable guardian and tutor might undertake. A students privacy interest is limited in a public school environment where the State is responsible for maintaining discipline, health and safety. Character of the intrusion imposed by the Policy Urination is an excretory function traditionally shielded by great privacy. (In other words, the government cannot watch you peeing or taking a shit.) But the degree of intrusion on ones privacy caused by collecting a urine sample depends upon the manner in which production of the urine sample is monitored. The method mandated by the Policy is not intrusive. Indeed, it is even less intrusive than the method in Vernonia, which was held to be a negligible intrusion. (The method is described in page 196, column 1 bottom paragraph to column 2 top paragraph.) Also, the Policy clearly requires that the test results be kept in confidential files separate from a students other educational records and released only on a need to know basis. Further, the test

"

People v. De Gracia Sec. 2 | Warrantless Searches Facts: The incidents involved in this case took place at the height of the coup d' etat staged in December, 1989. At that time, various government establishments and military camps in Metro Manila were being bombarded by the rightist group Reform the Armed Forces Movement-Soldiers of the Filipino People (RAM-SFP). There were intelligence reports that the Eurocar Sales Office was being used as headquarters by the RAM. Prior to the raid, there was a surveillance conducted on the premises wherein the surveillance team was fired at by a group of men coming from the Eurocar building. In December 5, 1989, a searching team raided the Eurocar Sales Office. They were able to find and confiscate six cartons of M-16 ammunition, five bundles of C-4 dynamites, M-shells of different calibers, and "molotov" bombs. No search warrant was secured by the raiding team because, according to them, at that time there was so much disorder considering that the nearby Camp Aguinaldo was being mopped up by the rebel forces and there was simultaneous firing within the vicinity of the Eurocar office, aside from the fact that the courts were consequently closed. Accused-appellant Rolando de Gracia was convicted for illegal possession of firearms in furtherance of rebellion, but was acquitted of attempted homicide. That judgment of conviction is now challenged in this appeal. Issue: Whether or not there was a valid search and seizure in this case. Held: Yes. The instant case falls under one of the exceptions to the prohibition against a warrantless search. The military operatives had reasonable ground to believe that a crime was being committed. Ratio: Under the situation then prevailing, the raiding team had no opportunity to apply for and secure a search warrant from the courts. The trial judge himself manifested that when the raid was conducted, his court was closed. Under such urgency and exigency of the moment, a search warrant could lawfully be dispensed with. In People vs. Malmstedt: While it is true that the NARCOM officers were not armed with a search warrant when the search was made over the personal effects of accused, however, under the circumstances of the case, there was sufficient probable cause for said officers to believe that accused was then and there committing a crime. In Umil, et al., vs. Ramos, et al. : The arrest of persons involved in the rebellion need not follow the usual procedure in the prosecution of offenses which requires the determination by a judge of the existence of probable cause before the issuance of a judicial warrant of arrest. The absence of a judicial warrant is no legal impediment to arresting or capturing persons committing overt acts of violence against government forces, or any other milder acts but really in pursuance of the rebellious movement. The arrest or capture is thus impelled by the exigencies of the situation that involves the very survival of society and its government and duly constituted authorities.

CONSTI II DIGESTS: 1D COMPILATION

CHUA, ALIMANGOHAN, CONTRERAS, BRIONES, LENCIO, RELOJO, TENGCO, LIM, DUMA, DANAO, MELLA, TONGSON

&+

"

results are not turned over to any law enforcement authority nor do the test results lead to the imposition of discipline or have any academic consequences. In sum, given the minimally intrusive nature of the sample collection and the limited uses to which the test results are put, the invasion of students privacy is not significant. The nature and immediacy of the governments concerns and the efficacy of the Policy in meeting them Given the nationwide epidemic of drug use, and the evidence of increased drug use in Tecumseh schools, it was entirely reasonable for the School District to enact this particular drug testing policy. RESPONDENT ARGUMENTS Resondent Argument 1: Drug testing must be based at least on some level of individual suspicion. Court: While schoolchildren do not shed their constitutional rights when they enter the schoolhouse, Fourth Amendment rights are different in public schools than elsewhere. The reasonableness inquiry cannot disregard the schools custodial and tutelary responsibility for children. A finding of individualized suspicion may not be necessary when a school conducts drug testing. Respondent Argument 2: (under Nature of privacy interest) Children participating in nonathletic extracurricular activities are not subject to regular physicals and communal undress and therefore have a stronger expectation of privacy than athletes. (referring to the Vernonia case) (In other words, because athletes are regularly naked with others anyway, they have a lesser expectation of privacy than students who are not athletes.) (So essentially, the government cannot watch you if you are naked alone and peeing, but its okay if youre naked with a lot of other people, like your teammates.) Court: Students who participate in competitive extracurricular activities voluntarily subject themselves to many of the same intrusions as athletes such as off-campus travel and communal undress. The regulation of extracurricular activities diminishes the expectation of privacy among school children. Respondent Argument 3: (under Character of the intrusion) The intrusion on students privacy is significant because the Policy fails to protect effectively against the disclosure of confidential information and, specifically, that the school has been careless in protecting that information. Court: (See Character of intrusion) Social Justice Society v. Dangerous Drugs Board Section 2, Article III | Warantless Searches Note: This is a pasikat case. You know what to expect.

Facts: This case questions the constitutionality of Section 36 of RA 9165 (Comprehensive Dangerous Drugs Act of 2002) insofar as it requires mandatory drug testing of: 1. Candidates for public office 2. Students of secondary and tertiary schools 3. Officers and employees of public and private offices 4. Persons charged before the prosecutors office with a criminal offense having an imposable penalty of not less than 6 years and 1 day Section 36 of RA 9165 provides: Section 36. Authorized Drug Testing Authorized Drug Testing shall be done by any govt forensic laboraties or by any of the drug testing laboratories accredited and monitored by the DOH to safeguard the quality of the results. xxx The drug testing shall emply, among others, 2 testing methods, the screening test which will determine the positive result as well as the type of drug used and the confirmatory test which will confirm a positive screening test. On December 2003, COMELEC issued a resolution prescribing the rules and regulations on the mandatory drug testing of candidates for public office in connection with the May 10, 2004 elections. I. The Pimentel Petition Pimentel, a senator and a candidate for re-election, seeks to: 1. nullify Sec 36(g) of RA 9165 and the COMELEC resolution for being unconstitutional in that they impose a qualification for candidates for senators in addition to those already provided for in the Constitution Sec 36(g) provides: All candidates for public office whether appointed or elected both in the national or local govt shall undergo a mandatory drug test 2. To enjoin COMELEC from implementing their resolution which provides: No person elected to any public office shall enter upon the duties of his public office until he has undergone mandatory drug test.

Pimentels Arguments: The Constitution (Section 3, Article VI) prescribes a maximum of 5 qualifications for one to be a candidate for Senator: (1) citizenship, (2) voter registration, (3) literacy, (4) age, and (5) residency. " There is no provision in the Constitution authorizing the Congress or the COMELEC to expand the qualification requirements of candidates for senator. " Beyond the constitutional qualifications, candidates for senator need not possess any other qualification to run for senator.

CONSTI II DIGESTS: 1D COMPILATION

CHUA, ALIMANGOHAN, CONTRERAS, BRIONES, LENCIO, RELOJO, TENGCO, LIM, DUMA, DANAO, MELLA, TONGSON

&!
3. Subjecting almost everybody to drug testing, without probable cause, is unreasonable, an unwarranted intrusion of the individual right to privacy.

"

Issues (Pimentel Petition): 1. Do Sec 36(g) and the COMELEC resolution impose an additional qualification for candidates for senator? 2. Can Congress enact a law prescribing qualifications for candidates for senator in addition to those laid down by the Constitution? Held: 1. Yes. 2. No. Ratio: Sec 36 effectively enlarges the qualification requirements enumerated in the Constitution. " Sec 36 requires a candidate for senator to be certified illegal-drug clean as a precondition to the validity of a certificate of candidacy for senator. " It may be argued that the provision does not expressly state that non-compliance is a disqualification factor. HOWEVER, the particular section of the law, without exception, made drug-testing mandatory. Any other construal would reduce the mandatory nature of Sec 36 into a pure jargon without meaning and effect whatsoever. If Congress cannot require a candidate for senator to meet such additional qualification, the COMELEC is also without such power.

Issue (SJS): WON the provisions constitute undue delegation of legislative power, violate the equal protection clause, and the right against unreasonable search and seizure. Held & Ratio: A. Regarding the Students: The provisions of RA 9165 requiring drug testing of students are constitutional. " It is within the prerogative of educational institutions to require, as a condition for admission, compliance with reasonable school rules and regulations. " The right to enroll is not absolute; it is subject to fair, reasonable, and equitable requirements. " The drug testing is not only acceptable but may even be necessary if the safety and interest of the student population are to be promoted. " The following can be deduced from the Veronia case and the Board of Education case cited by the SC: 1. Schools and their administrators stand in loco parentis with respect to their students 2. Minor students have contextually fewer rights than an adult, and are subject to the custody and supervision of their parents, guardians and schools 3. Schools acting in loco parentis, have a duty to safeguard the health and wellbeing of their students and may adopt such measures as may reasonably be nexessary to discharge such duty 4. Schools have the right to impose conditions on applicants for admission that are fair, just, and non-discriminatory

SJS Petition Social Justice Society (SJS), a political party, seeks to prohibit the Dangerous Drugs Board (DDB) and the Philippine Drug Enforcement Agency (PDEA) from enforcing paragraphs (c), (d), (f), and (g) of Sec 36, which provide: (c) Students of secondary and tertiary schools shall, pursuant to the related rules and regulations as contained in the schools student handbook and with notice to parents, undergo a random drug testing
II.

(d) Officers and employees of public and private offices shall be subjected to undergo a random drug test as contauned in the companys work rules and regulations xxx for purposes of reducing the risk in the workplace. Any officer or employee found positive for use of dangerous drugs shall be dealt with administratively which shall be a ground for suspension or termination, subject to the provisions of Article 282 of the Labor Code and pertinent provisions of the Civil Service Law. (f) All persons charged before the prosecutors office with a criminal offense having an imposable penalty of imprisonment of not less than 6 years and 1 day shall undergo a mandatory drug test (g) *cited earlier* SJS Arguments: 1. The provisions constitute undue delegation of legislative power when they give unbridled discretion to schools and employers to determine the manner of drug testing. 2. The provisions trench in the equal protection clause inasmuch as they can be used to harass a student or employee deemed undesirable.

CONSTI II DIGESTS: 1D COMPILATION

CHUA, ALIMANGOHAN, CONTRERAS, BRIONES, LENCIO, RELOJO, TENGCO, LIM, DUMA, DANAO, MELLA, TONGSON

&#
NOTES! Right to privacy the right to be free from unwarranted exploitation of ones person or from intrusion into ones private activities in such a way as to cause humiliation to a persons ordinary sensibilities Jurisprudence considered by the Supreme Court as bases (SJS petition): A. Vernonia School District 47J v. Acton (Vernonia) School administrators discovered the frequent drug use by school athletes. To address the drug menace, they required random urinalysis drug testing for the athletes. James Acton was denied participation in the football program after he refused to undertake the urinalysis drug testing. He claims that the school policy violated the Fourth Amendment of the Constitution. Held: The policy constituted reasonable search under the 4th and 14th Amendments and, thus, is constitutional, because of the ff grounds: 1. Schools stand in loco parentis over their students 2. School children, while not shedding their constitutional rights at the school gate, have less privacy rights. 3. Atheletes have less privacy rights than non-athletes since the former observe communal undress before and after sports events 4. By joining the sports activity, the athletes voluntarily subjected themselves to a higher degree of school supervision and regulation 5. Requiring urine saples does not invade a students privacy because the student need not undress for this kind of drug testing 6. There is need for the drug testing because of the dangerous effects of illegal drugs on the young
B.

"

Regarding the employees of public and private officers: Reasonableness is the touchstone of the validity of a government search or intrusion. The drug test requirement is reasonable, taking into consideration the foregoing factors: 1. The reduced expectation of privacy on the part of employees The employees privacy interest is to a large extent circumscribed by the companys work policies, the collective bargaining agreement, and the inherent right of the employer to maintain discipline and efficiency in the workplace. 2. The compelling state concern likely to be met by the search The need for drug testing to at least minimize illegal drug use is substantial enough to override the individuals privacy interest. 3. The well-defined limits set forth in the law to properly guide authorities in the conduct of the random drug testing The enabling law authorizing the search is narrowly drawn ornarrowly focused o Nobody is singled out in advance for drug testing. o There is a narrowing factor because the employees concerned shall be subjected to random drug test as contained in the companys work rules and regulations o The law specifies the procedure (screening test and confirmatory test) and the agencies which shall conduct the test o Access to the drug test results shall be on the need to know basis. Results will be kept confidential. o The participation of schools and offices in the drug testing scheme shall always be subject to the IRR of RA 9165. o III. Laserna Petition Atty. Laserna, Jr., as citizen and taxpayer, seeks to declare Sec 36 (c), (d), (f), and (g) *cited earlier* as unconstitutional because: 1. They infringe on the constitutional right to privacy, the right against unreasonable search and seizure, and the right against self-incrimination. 2. The provisions are contrary to the due process and equal protection guarantees.
Issue (Laserna): WON the provisions violate the right to privacy, the right against unreasonable search and seizure, the right against self-incrimination and the due process and equal protection clauses of the Constitution. Held: Sec 36(g) is unconstitutional. The Court finds no valid justification for mandatory drug testing for persons accused of crimes. Ratio: The operative concepts in the mandatory drug testing are randomness and suspicionless In the case of persons charged with a crime, a mandatory drug testing can never be random or suspicionless. o They are not randomly picked; neither are they beyond suspicion. o They are singled out and impleaded against their will. -------------------------

Board of Education of Independent School District No. 92 of Pottawatomie County, et al. v. Earls, et al. (Board of Education) The Board of Education of a school required a drug test for high school students desiring to join extra-curricular activities. Lindsay Earls, a member of the show choir, marching band, and academic team declined to undergo a drug test. She claimed that the policy made to apply to nonathletes violated the 4th and 14th Amendments, and that non-athletes are entitled to more privacy. Held: There is no need to make a distinction between athletes and non-athletes. Schools and teachers act in place of the parents with a similar interest and duty of safeguarding the health of the students.

Facts: Lope Sarreal and Material Distributors Inc. had a pending case concerning a money judgment amounting Php 1.2 M. As a result of the case, Sarreal filed a motion for the production and inspection of certain documents and books, which Judge Natividad granted. Petitioners oppose the decision claiming that Sarreal filed the motion for the purpose of fishing evidence and that the judge acted with grave abuse of discretion in granting the request.

Material Distributors v. Natividad Searches and seizures of whatever nature and for whatever purpose

CONSTI II DIGESTS: 1D COMPILATION

CHUA, ALIMANGOHAN, CONTRERAS, BRIONES, LENCIO, RELOJO, TENGCO, LIM, DUMA, DANAO, MELLA, TONGSON

&$
Camara v. Municipal Court Searches and seizures of whatever nature and for whatever purpose Note: 4th amendment is equivalent to our Constis Sec. 2, Art III.

"

Issue: W/N judge acted with GAD by ordering petitioners to produce and inspect the documents and books requested by Sarreal Held: No, judge did not act with grave abuse of discretion. Ratio: Sec 1 of Rule 21 (Rules of Court) says: Upon motion of any party showing good cause., the court in which an action is pending may (a) order any party to produce and permit the inspection of any designated documents, papers, books. Which constitute evidence material to any matter involved in the action and which are in his possession, custody or control . The purpose of Surreals request to produce the documents was to be able to answer whether or not the names of Gil Puyat and Raymond Lehmann appear in any part of the original documents. Sarreal wanted the production and inspection of the originals to show that it did not contain those names. If the names should end up on the documents, those names must have been typed by direction of Harry Lyons without the knowledge and consent of Surreal, constituting forgery. With these allegations, Sarreal fulfilled the requirement of showing good cause for the production and inspection of the books and documents in question under Rule 21. The orders in question pertain to a civil procedure that cannot be confused with the unreasonable searches prohibited by the Constitution. There is no question that Sarreal has an interest in the books and documents because they are material to the issues between him and petitioners. Lastly, the constitutional guarantee of privacy of communication and correspondence will not be violated, because the trial court has the power and jurisdiction to issue the order for the production and inspection of the books and documents in virtue of the constitutional guarantee making an express exception in favor of the disclosure of communication and correspondence upon lawful order of the court of justice.

Facts: An inspector of the Division of Housing Inspection of the San Francisco Department of Public Health entered an apartment building to make a routine annual inspection for possible violations of the citys Housing Code. The buildings manager informed the inspector that appellant, lessee of the ground floor, was using the rear of his leasehold as personal residence. Claiming that the buildings occupancy permit did not allow residential use of the ground floor, the inspector confronted appellant and demanded that he permit an inspection of the premises, which the appellant refused because the inspector lacked a search warrant. The inspector returned several times without a search warrant invoking Sec. 503 of the Housing Code which authorized employees of the City to enter any premises to perform any duty imposed upon them by the Municipal Code. Appellant, nevertheless refused each time to let the inspectors access his apartment without a search warrant. Appellant was criminally charged of violation of Sec. 507 of the Housing Code on the ground of his refusal to permit a lawful inspection.

Appellants Arguments: Sec. 503 of Housing Code is contrary to the 4th and 14th amendments in that it authorizes municipal officials to enter a private dwelling without a search warrant and without probable cause to believe that a violation of the House Code exists therein. He may not be prosecuted under Sec. 507 for refusing to permit an inspection unconstitutionally authorized by 503.
Issue: WON administrative inspection programs, as presently authorized and conducted, violate 4th Amendment rights as those rights are enforced against the States through the 14th Amendment. Held: Yes. US Supreme Court held that administrative searches of the kind at issue here are significant intrusions upon the interests protected by the 4th amendment, that searches authorized and conducted without a warrant procedure lack the traditional safeguards which the 4th amendment guarantees to the individual. Ratio: To administer these searches the city departments may secure a warrant and there is enough probable cause in protecting societal interests such as safety, security, health etc. to issue a warrant. Jurisprudence (e.g. Frank v. Maryland) upholding warrantless searches are insufficient to justify so substantial weakening of the 4th Amendments protection. One governing principle, justified by history and by current experience, has consistently been followed: except in certain carefully defined classes of cases, a search of private property without proper consent is "unreasonable" unless it has been authorized by a valid search warrant.

CONSTI II DIGESTS: 1D COMPILATION

CHUA, ALIMANGOHAN, CONTRERAS, BRIONES, LENCIO, RELOJO, TENGCO, LIM, DUMA, DANAO, MELLA, TONGSON

&%
Ratio: CONTINUING OFFENSE. Dural was committing an offense during his warrantless arrest since being a member of the NPA, an outlawed organization, is penalized. Thus the arrest is within the contemplation of Sec 5(a) Rule 113. GARCIA v. ENRILE. The crimes of xxx rebellion, subversion, xxx are all in the nature of continuing offenses which set them apart from the common offenses

"

Under the present system, when the inspector demands entry, the occupant has no way of knowing: 1. whether enforcement of the municipal code involved requires inspection of his premises, 2. the lawful limits of the inspector's power to search, and 3. whether the inspector himself is acting under proper authorization. These are questions which may be reviewed by a neutral magistrate without any reassessment of the basic agency decision to canvass an area. Yet, only by refusing entry and risking a criminal conviction can the occupant at present challenge the inspector's decision to search. And even if the occupant possesses sufficient fortitude to take this risk, as appellant did here, he may never learn any more about the reason for the inspection than that the law generally allows housing inspectors to gain entry. The practical effect of this system is to leave the occupant subject to the discretion of the official in the field. This is precisely the discretion to invade private property which we have consistently circumscribed by a requirement that a disinterested party warrant the need to search. The Court simply cannot say that the protections provided by the warrant procedure are not needed in this context; broad statutory safeguards are no substitute for individualized review, particularly when those safeguards may only be invoked at the risk of a criminal penalty. SC, in this case, overruled the decision in Frank Case. (Frank v. Maryland upheld the conviction of one who refused to permit a warrantless inspection of private premises for the purposes of locating and abating a suspected public nuisance.) The Frank opinion has generally been interpreted as carving out an additional exception to the rule that warrantless searches are unreasonable under the Fourth Amendment.

CONTINUING OFFENSE Anchored on an ideological base, thus compels the repetition of lawless acts until the overriding objective of overthrowing the government is attained Durals mission as a member of the NPA did not end in the shooting of the CAPCOM police officers. He would shoot other officers given the opportunity.

COMMON OFFENSE Generally end upon their commission (murder, arson, etc.)

Facts:

People v. De la Cruz Sec.2 | Warrantless Arrest

In re: Umil, et. al. v. Ramos Sec.2 | Warrantless Arrest Facts: Dural, a member of the NPA, was arrested without warrant during his confinement in St. Agnes Hospital. Dural was identified to have shot 2 CAPCOM policemen in their police car the day before his arrest. The Court previously held that his warrantless arrest is justified under Sec 5, Rule 113 of the Rules of Court. Petitioners then sought reconsideration of the Courts decision which dismissed the petition for the writ of habeas corpus.

A "buy-bust" operation was conducted by the 13th Narcotics Regional Unit through a team composed of T/Sgt. Jaime Raposas as Team Leader, S/Sgt. Rodelito Oblice, Sgt. Dante Yang, Sgt. Vicente Jimenez, P/Pfc. Adolfo Arcoy as poseur-buyer and Pat. Deogracias Gorgonia at Maliclic St., Tondo, Manila at around 2:30 o'clock in the afternoon of May 4, 1987 to catch the pusher/s. P/Pfc. Adolfo Arcoy acted as the poseur-buyer with Arnel as his companion to buy marijuana worth P10.00 from the two accused, Juan de la Cruz and Reynaldo Beltran. At the scene, it was Juan de la Cruz whom Arcoy first negotiated (with) on the purchase and when Arcoy told De la Cruz that he was buying P10.00 worth of marijuana, De la Cruz instructed Reynaldo Beltran to give one aluminum foil of marijuana which Beltran got from his pants' pocket and delivered it to Arcoy. After ascertaining that the foil of suspected marijuana was really marijuana, Arcoy gave the prearranged signal to his teammates by scratching his head and his teammates who were strategically positioned in the vicinity, converged at the place, identified themselves as NARCOM agents and effected the arrest of De la Cruz and Beltran. The P10.00 marked bill (Exhibit C-1) used by Arcoy was found in the possession of Juan de la Cruz together with two aluminum foils and containing marijuana. Juan de la Cruz y Gonzales and his co-accused Reynaldo Beltran y Aniban were charged in Criminal Case No. 87-54417 of the Regional Trial Court of Manila with violation of Section 4, Art. II, in relation to Section 21, Article IV of Republic Act No. 6425, as amended, in an information which reads: That on or about May 4, 1987, in the City of Manila, Philippines, the said accused, conspiring and confederating together and mutually helping each other, not being authorized by law to sell, deliver, give away to another or distribute any prohibited drug, did then and there wilfully, unlawfully, and knowingly sell, deliver or give away to and other the following: 1. One (1) cigarette foil wrapper containing marijuana; 2. Two (2) cigarette foil wrapper (sic) containing marijuana which are prohibited drugs.

Petitioners: 1. Upholding the validity of the questioned warrantless arrests & relying on Sec 5 Rule 115 of the Rules of Court violated their constitutional rights 2. The Garcia v. Enrile and Ilagan v. Enrile doctrine should be abandoned
Issue: WON the questioned arrests without warrant were made in accordance with law Held: Petition dismissed. The questioned arrests are upheld.

CONSTI II DIGESTS: 1D COMPILATION

CHUA, ALIMANGOHAN, CONTRERAS, BRIONES, LENCIO, RELOJO, TENGCO, LIM, DUMA, DANAO, MELLA, TONGSON

&&
The PC officers had earlier received a tip from one of their informers that the accused was on board a vessel, MV Wilcon 9 bound for Iloilo City and was carrying marijuana. He was identified by name. Acting on this tip, they waited for him in the evening of June 25, 1984 and approached him as he descended from the gangplank of MV Wilcon 9 after the informer had pointed to him. They detained him and inspected the bag he was carrying without any warrant of arrest or search warrant. It was found to contain 3 kilos of what looked like marijuana leaves. They then confiscated the suspected articles and taken to the NBI laboratory for examination. When they were verified as marijuana leaves, an information for violation of the Dangerous Drugs Act was filed against Aminudin. Issue: Whether or not there was a valid warrantless arrest. Held: No. The accused was not caught in flagrante nor was a crime about to be committed or had just been committed to justify the warrantless arrest. Ratio: In this case, expediency cannot be invoked to dispense with the obtention of the warrant because there was no such urgency. Unlike in the case of Roldan vs. Arca, it was held that vessels and aircraft are subject to warrantless searches and seizures for violations of the customs law because these vehicles may be quickly moved out of the locality before the warrant can be secured. The PC officers had at least 2 days within which they could have obtained a warrant of arrest and search. The vehicle was identified. The date of its arrival was certain. And from the information they had received, they could have persuaded a judge that there was probable cause to justify the issuance of a warrant. But they did nothing. In several cases where this court has sustained a warrantless arrest of violators of the Dangerous Drugs Act, it has always been shown that they were caught red-handed. The warrantless arrest was justified because at the precise time of the arrest, the accused was in the act of selling the prohibited drug. IN the case at bar, the accused was not at the moment of arrest committing a crime nor was it shown that he was about to do so or that he had just done so. What he was doing was descending from the ship. He was like any of the other passengers innocently disembarking from the vessel. It was only when the informed pointed to him that he suddenly became the suspect and subject to apprehension. The identification by the informer was the probable cause as determined by the officers and not a judge that authorized them to pounce upon Aminudin. The evidence of the marijuana cannot be admitted and should have never been considered by the trial court since the marijuana was illegally seized. The search was not an incident of a lawful arrest because there was no warrant and the warrantless arrest was not valid. Hence, the warrantless search was also illegal and the evidence obtained was inadmissible.

"

Appellant assails, unconstitutional, the manner in which the so-called buy-bust operation is conducted in order to enforce the Dangerous Drugs Act. He stigmatizes it as no different from seizure of evidence from one's person or abode without a search warrant. He argues that this procedure is pregnant with opportunities, and gives rise to situations, for corrupting our law enforcers. ISSUE: Whether or not a buy-bust operation constitute a valid warrantless arrest. HELD: Yes. While it is conceded that in a buy-bust operation, there is seizure of evidence from one's person without a search warrant, needless to state a search warrant is not necessary, the search being incident to a lawful arrest. A peace officer may, without a warrant, arrest a person when, in his presence, the person to be arrested has committed, is actually committing or is attempting to commit an offense. It is a matter of judicial experience that in the arrest of violators of the Dangerous Drugs Act in a buy-bust operation, the malefactors were invariably caught redhanded. There being no violation of the constitutional right against unreasonable search and seizure, the confiscated articles are admissible in evidence. RATIO: The Solicitor General explains that a buy-bust operation is the method employed by peace officers to trap and catch a malefactor in flagrante delicto. It is essentially a form of entrapment since the peace officer neither instigates nor induces the accused to commit a crime. Entrapment is the employment of such ways and means for the purpose of trapping or capturing a lawbreaker from whose mind the criminal intent originated. Oftentimes, it is the only effective way of apprehending a criminal in the act of the commission of the offense. The Court is not unmindful of the fact that the common modus operandi of narcotic agents in utilizing poseur-buyers does not always commend itself as the most reliable way to go after violators of the Dangerous Drugs Act as it is susceptible of mistakes as well as harassment, extortion and abuse. By the very nature of this anti-narcotics operation, the possibility of abuse is great. However, inclined to shackle the hands of narcotics agents whose task, as it is, is already formidable and attended with great risk, lest their dedicated efforts for the apprehension and successful prosecution of prohibited drug violators be unduly hampered. The proliferation of drug addiction and trafficking has already reached an alarming level and has spawned a network of incorrigible, cunning and dangerous operations. Our experience has proven entrapment to be an effective means of apprehending drug peddlers as exemplified by this case.

Facts:

People vs. Aminudin Section 2, Article III | Warrantless arrests

Harvey v. Defensor-Santiago Section 2 \ Article III \ Warrantless Arrests Facts: Petitioners Andrew Harvey, John Sherman, and Adrian Van Dan Elshout were among the suspected alien pedophiles who were subjected to surveillance by the Commission on Immigration and Deportation (CID) agents and afterwards apprehended by the same without

The accused claimed his business was selling watches but he was nonetheless arrested and found guilty of illegally transporting marijuana. The trial court disbelieved him and he was sentenced to life imprisonment.

CONSTI II DIGESTS: 1D COMPILATION

CHUA, ALIMANGOHAN, CONTRERAS, BRIONES, LENCIO, RELOJO, TENGCO, LIM, DUMA, DANAO, MELLA, TONGSON

&'
-Assuming arguendo that the arrest of petitioners were not valid at its inception, it was already cured by the subsequent issuance of Warrants of arrest. It is a fundamental rule that a writ of habeas corpus will not be granted when the confinement is or has become legal, although such confinement was illegal at the beginning (Matsura v. Director of Prisons) -Deportation proceedings are administrative in character, summary in nature. It is preventive, not a penal process. It need not be conducted strictly in accordance with ordinary Court proceedings. Minimum requirement: due process (notice and hearing) The requirement of probable cause to be determined by a Judge, does not extend to deportation proceedings. -The ruling in Vivo v. Montesa which rendered the issuance of Warrants of arrest by the Commissioner solely for the purpose investigation and prior a final order of deportation cannot be invoked here. The purpose is not to investigate since the arrest was pursuant to the violation already charged with the petitioners; the arrest is a preliminary step to the deportation of the aliens who violated the condition of their stay in this country (Section 37 of Immigration Act). -According to Section 37(e) of Philippine Immigration Act of 1940, the right to bail is not a matter of a right but a matter of discretion on the part of the Commissioner. On the 3rd issue -The petitioners need not to be caught in the act before the arrest could be legal Consideration of surrounding circumstances is enough to render the arrest legal -Although pedophilia is not a crime punishable under Revised Penal Code, it is still against public morals and violative of the declared policy of the state to promote and protect the physical, moral, spiritual, and social well being of our youth (Art II, Section 13, 1987 Constitution) Things CG might ask or bring up: Every sovereign power has the inherent power to exclude aliens from its territory upon such grounds as it may deem proper for self-preservation or public interest Deportation is a police measure against undesirable aliens whose continued presence in the country is found to be injurious to the public good and the domestic tranquility of the people In this casethe commitment of the state to defend the right of the children to assistance and special protection from all forms of neglect, abuse, cruelty, exploitation, and other conditions prejudicial to their development (Art XV, Section 3(2)

"

warrant. Other aliens were either released due to lack of evidence or opted self-deportation leaving the 3 petitioners as they have chosen to face the deportation case. Seized during the arrest were rolls of photo negatives, photos of suspected child prostitutes in salacious poses as well as boys and girls having sex, and posters and other materials advertising child prostitution. Furthermore, there were reports that minors were found in the custody of the petitioners. Deportation proceedings were instituted against the petitioners for being undesirable aliens under Section 69 of the Revised Administrative Code. In March 1988, Warrant of Arrests were issued by the respondent CID against the petitioners for violation of Sections 37, 45, and 46 of the Immigration Act and Section 69 of the Revised Administrative Code. [NOTE: The warrant of arrest was only issued after they have been apprehended and after the deportation proceedings were instituted] Petitioner filed for an Urgent Petition for Release Under Bond and subsequently, a Petition for Bail due to the claim of the petitioners that their health was being seriously affected by the continuous detention. The respondent CID dismissed the petitions after certification by the CID physician that petitioners were healthy. Petitioner Harvey filed for a Manifestation/Motion stating that he had finally agreed to a self-deportation but prayed that he be released for 15 days and be placed under the custody of Atty. Asinas before he voluntarily leaves the country. On that same date when the said Motion was filed, Harvey filed this present petition for a Writ of Habeas Corpus.

Issues (As raised by the Petitioner): 1. WON there is a provision in the Philippine Immigration of Act of 1940 or under Section 69 of the Revised Administrative Code which entitles the Commissioner (Defensor-Santiago) the authority to arrest and detain petitioners pending determination the existence of a probable cause leading to an administrative investigation WON the respondent violated Section 2 of Art III of the 1987 Constitution when CID agents apprehended the petitioners and seized their items without valid Warrants of arrest, search and seizure as required by the law WON the fact that the petitioners were not caught in the act and that nothing in Philippine Law punishes one for being a pedophile render the arrest illegal

2.

3.

Held: Petition denied. The Commission did not violate any Constitutional provision. It has acted within its scope of authority and in the interest of the State. Ratio: On the first 2 issues -The arrest of petitioners was valid because it is based on a probable cause resulted from the 3month surveillance done by the respondent to the petitioners. The articles were seized as an incident to a lawful arrest which makes those admissible in evidence.

People vs Sucro Sec.2, Art. III, Warrantless Arrests

CONSTI II DIGESTS: 1D COMPILATION

CHUA, ALIMANGOHAN, CONTRERAS, BRIONES, LENCIO, RELOJO, TENGCO, LIM, DUMA, DANAO, MELLA, TONGSON

&(

"

Facts: Fulgencio was instructed by Lt. Seraspi, Station Commander of INP Aklan, to monitor activities of Sucro because of information that Sucro was selling marijuana. On the same day, Fulgencio positioned himself under a house, 2 meters away from a chapel. Fulgencio saw Sucro enter the chapel and take something from a compartment of a cart found inside the chapel. He handed the same to different buyers. Then, Fulgencio radioed it to Seraspi who told him to continue to monitoring Sucro. A third buyer, Macabante transacted with Sucro. At this point, Seraspi and his men intercepted Macabante and Sucro. Seraspi caught up with Macabante, who threw a something to the ground which turned out to be marijuana. He readily admitted that he bought it from Sucro. Eventually, they also caught Sucro and the police recovered 19 sticks and 4 teabags of marijuana. Petitioners arguments: There was sufficient time to apply for a search warrant, since Fulgencio informed Seraspi 2 days before the arrest Issue: 1. Whether or not arrest without warrant is lawful 2. Whether or not evidence from such arrest is admissible Held: 1. 2. Ratio: A peace officer or private person may, without warrant, arrest a person: o When in his presence, person to be arrested has committed, is actually committing or is attempting to commit an offense o When an offense has in fact just been committed and he has personal knowledge of facts indicating that person to be arrested has committed it An offense is committed in the presence of an officer when officer sees the offense, although at a distance or hears disturbances and proceeds at once to the scene In the case, Fulgencio was 2 meters away from Sucro who was conducting a nefarious activity. Also, when Macabante was intercepted, he was caught throwing the marijuana and readily admitted that he bought it from Sucro and therefore, had just committed an illegal act which police officers had personal knowledge of People v Bati: Police officers have personal knowledge of crime when they previously conducted surveillance As to petitioners argument: o Fulgencio and Sucro had known each other since childhood and Fulgencio, being a friend, just hesitated to report it and just advised Sucro to stop. But because of reliable information that sale of marijuana was going on everyday, he was forced to report it to the Station Commander In the case, there was probable cause to arrest appellant who was selling marijuana and to seize the same Among the exceptions to the need of a valid warrant: search incidental to lawful arrest Yes Yes

People v Bagista Section 2, Article 3 | Warrantless Searches Facts: The Narcotics Command (NARCOM) Detachment Office in Baguio City received information from one of its regular informants that a certain woman, 23 years of age, with naturally curly hair, and with a height of 52 or 53, would be transporting marijuana from up north. Acting upon this piece of information, Sgt Godofredo Fider and a civilian NARCOM agent proceeded to Benguet. Upon arrival that same morning, they established a checkpoint and flagged down all vehicles coming from the north to check if any of these vehicles were carrying marijuana on board. The NARCOM agents stopped a Dangwa Tranco bus, Sgts Parajas and Fider boarded the bus and thereupon Parajas announced to the passengers that they were NARCOM agents and that they were going to search their baggages. Sgt Parajas noticed a woman with curly hair seated at the right side of the last seat of the bus with a travelling bag. He inspected the bag and discovered (3) bundles of marijuana leaves covered by assorted clothing. The bag and contents were confiscated and the woman arrested; she was later brought to the NARCOM office where she was booked and investigated. The bundles were found positive for marijuana.

Accused-appellants argument: The warrantless search conducted by the NARCOM agents is illegal and unconstitutional. Accused appellants story (rests solely on denial): She claims that she was engaged in the buying and selling of cabbages. She claims that the bag containing the marijuana was taken from the luggage carrier above the passenger seats. When nobody admitted owning the bag, the NARCOM agent approached her, took the shoulder bag on her lap and asked her to come with them for investigation as she fits the description of the would-be-transporter of marijuana. To corroborate her story, she presented the conductor of the bus, Nestor Yangkin.
Issue: Whether or not the warrantless search conducted by the NARCOM agents is valid and legal? Held: Yes. Ratio: General Rule: No person shall be subjected to a search of his person, personal effects or belongings, or his residence except by virtue of a search warrant or on the occasion of a lawful arrest. Article 3, Section 2 further ordains that any evidence obtained in violation of aforementioned right shall be inadmissible for any purpose in any proceeding.

Rodriguez

Exceptions: a) Search is incidental to lawful arrest b) In cases of a moving vehicle c) The seizure of evidence in plain view

CONSTI II DIGESTS: 1D COMPILATION

CHUA, ALIMANGOHAN, CONTRERAS, BRIONES, LENCIO, RELOJO, TENGCO, LIM, DUMA, DANAO, MELLA, TONGSON

&)
d. They approached these persons and identified themselves as policemen, whereupon the two tried to run away but were unable to escape because the other lawmen had surrounded them. The suspects were ten searched. One of them, who turned out to be the accused-appellant, was found with a .38 caliber Smith and Wesson revolver with six live bullets in the chamber; Accused-appellant was charged of violating Presidential Decree No. 1866 (Illegal Possession of Firearms); He was convicted and sentenced to reclusion perpetua;

"

With regard to search with moving vehicles, this had been justified on the ground that the mobility of motor vehicles makes it possible for the vehicle to be searched to move out of the locality or jurisdiction in which the warrant must be sought. This in no way, however, gives the police officers unlimited discretion to conduct warrantless searches of automobiles in the absence of probable cause. e. f.

The NARCOM officers in the case at bar had probable cause: 1. To stop and search all vehicles coming from the north at Acop, Tublay, Benguet in view of the confidential information they received from their reggular informant that a women having the same appearance as that of accused-appelant would be bringing marijuana from up north. 2. To search accused-appellants belongings since she fits the description given by the NARCOM informant.
PADILLA, J., dissents: Although there is a similarity in the factual circumstances of the case at bar with those of the Malmstead where the Court upheld the validity of the warrantless search, however, in the present case, the information received by the NARCOM agents, without other suspicious circumstances surrounding the accused, did not give rise to a probably cause justifying the warrantless search made on the bag of the accused.

Issue: WON the warrantless search and seizure conducted against the accused-appellant is valid. Held: NO Ratio: 1. 2. There is no question that evidence obtained as a result of an illegal search or seizure is inadmissible in any proceeding for any purpose. Absolute prohibition of Article III, Section 3 (2) of the Constitution: a. Exclusionary Rule justification by Judge Learned Hand Only in case the prosecution, which itself controls the seizing officials, knows that it cannot profit by their wrong will the wrong be repressed. The Solicitor-General, while conceding the rule, maintains that it is not applicable in the case at bar. The arrest and search of Mengote and the seizure of the revolver from him were lawful under Rule 113, Section 5, of the Rules of Court: Sec 5. Arrest without warrant when lawful. A peace officer or private person may, without a warrant arrest a person; a. When, in his presence the person to be arrested has Committed, is actually committing, or is attempting to commit an offense; b. When an offense has in fact just been committed, and he has personal knowledge of facts indicating that the person to be arrested has committed it; and c. When the person to be arrested is a prisoner who has escaped from a penal establishment or place where he is serving final judgment or temporarily confined while his case is pending, or has escaped while being transferred from one confinement to another. Par. (c) of Section 5 obviously inapplicable. Par. (a) of Section 5 NOT APPLICABLE: a. Requisites: i. That the person be arrested (1) after he has committed or while he is actually committing or is at least attempting to commit an offense, (2) in the presence of the arresting officer. b. In the case at bar, these requirements have not been established. At the time of the arrest in question, the accused-appellant was merely looking from side to side and holding his abdomen, according to the arresting officers themselves. There was apparently no offense that had just been committed or was being actually committed or at least being attempted by Mengote in their presence. He was arrested at 11:30 am and in a crowded street shortly after alighting from a passenger jeep with his companion. He was not skulking in the shadows but talking in the clear light of day.

3.

People v. Mengote Warrantless Arrests Facts: 1. Accused-appellant Rogelio Mengote was convicted of illegal possession of firearms on the strength mainly of the stolen pistol found on his person at the moment of his warrantless arrest. 2. Appellants argument: a. In this appeal, he pleads that the weapon was not admissible as evidence against him because it had been illegally seized and was therefore the fruit of the poisonous tree; b. It is submitted in the Appellants Brief that the revolver should not have been admitted in evidence because of its illegal seizure. No warrant therefore having been previously obtained. Neither could it have been seized as in incident of lawful arrest because the arrest of Mengote was itself unlawful, having been also affected without a warrant. 3. Prosecutions argument: a. The Government disagrees. It insists that the revolver was validly received in evidence by the trial judge because its seizure was incidental to an arrest that was doubtless lawful even if admittedly without warrant; 4. Other pertinent facts: a. Incident occurred shortly before noon after the western police district received a telephone call from an informer that there were 3 suspicious looking persons at the corner of Juan Luna and North Bay Boulevard in Tondo, Manila; b. A surveillance team of plainclothesmen was forthwith dispatched to the place; c. At the trial, the patrol men narrated that they saw two men looking from side to side, one of whom was holding his abdomen;

4. 5.

CONSTI II DIGESTS: 1D COMPILATION

CHUA, ALIMANGOHAN, CONTRERAS, BRIONES, LENCIO, RELOJO, TENGCO, LIM, DUMA, DANAO, MELLA, TONGSON

&*
Moreover, none of the police officers who arrested him had been an eyewitness to the shooting of Maguan and accordingly none had the personal knowledge required for the lawfulness of the warrantless arrest.

"

There was nothing clandestine about this being on that street at that busy hour in the blaze of the noonday sun. c. Jurisprudence: i. Different from People v. Mamstedt and People v. Claudio, wherein the Court held the warrantless arrests in said cases were valid because in the case at bar, there was nothing to support the arresting officers suspicion other than Mengotes darting eyes and his hand on his abdomen; This case is similar to People v. Aminnudin, where the Court held that the warrantless arrest of the accused was unconstitutional because in said instant case there was no probable cause that, as the prosecution incorrectly suggested, dispensed with the constitutional requirement of a warrant. 6. Par. (b) Section 5 NOT APPLICABLE: a. In the case at bar, the prosecution has not shown that at the time of Mengotes arrest an offense had in fact just been committed and that the arresting officers had personal knowledge of facts indicating that Mengote had committed it. All they had was hearsay information from the telephone caller, and about a crime that had yet to be committed. b. The truth is that they did not know then what offense, if at all, had been committed and neither were they aware of the participation therein of the accused-appellant. It was only later, after Danganan had appeared at the Police headquarters, that they learned of the robbery in his house and of Mengotes supposed involvement therein. As for the illegal possession of the firearm found on Mengotes person, the policemen discovered this only after he had been searched and the investigation conducted later revealed that he was not its owner nor was he licensed to possess it.

Issue: Whether or not a lawful warrantless arrest had been effected by San Juan Police in respect of the Petitioner Go, whether petitioner had effectively waived his right to preliminary investigation. Held: We do not believe that the warrantless arrest or detention of the petitioner in the instant case falls within the terms of section 5 of Rule 113 of the 1985 Rules of Criminal Procedure. It is clear to the Court that there was no lawful warrantless arrest of petitioner within the meaning of Section 5 of Rule 113 Ratio: Petitioners arrest took place six (6) days after the shooting of Maguan. The arresting officers obviously were not present, within the meaning of Section 5(a), at the time petitioner had allegedly shot Maguan. Neither could the arrest effected 6 days after the shooting be reasonably regarded as effected when [the shooting had] in fact just been committed within the meaning of Section 5(b). Moreover, none of the arresting officers had any personal knowledge of the facts indicating that petitioner was the gunman who had shot Maguan. The information upon which the police acted had been derived from the statements made by alleged eyewitnesses on the shooting one stated that petitioner was the gunman; another was able to take down the alleged gunmans cars plate number which turned out to be registered in petitioners wife name. That information did not, however, constitute personal knowledge.

Go v. Court of Appeals Sec. 2, Art. III, Warrantless arrests Facts: Eldon Maguan was driving his car along Wilson St., San Juan, Metro Manila, heading towards P. Guevarra St. Petitioner entered Wilson St., where it is a one-way street and started travelling in the opposite or wrong direction. At the corner of Wilson and J. Abad Santos St., petitioners and Maguans cars nearly bumped each other. Petitioner alighted from his car, walked over and shot Maguan inside his car. Petitioner then boarded his car and left the scene. A security guard at a nearby restaurant was able to take down petitioners car plate number. The police arrived shortly thereafter at the scene of the shooting. Verification at the LTO showed that the car was registered to one Elsa Ang Go. The following day, the security guard was shown a picture of petitioner and he positively identified him as the same person who had shot Maguan. Having established that the assailant was probably the petitioner, the police launched the manhunt for the petitioner. 6 days after, petitioner presented himself before the San Juan Police Station. The police forthwith detained him. An eyewitness to the shooting who was at the police station at that time positively identified petitioner as the gunman.

Note: Section 5. Arrest without warrant; when lawful. A peace officer or a private person may, without warrant, arrest a person. (a) When, in his presence the person to be arrested has committed, is actually committing, or is attempting to commit an offense; (b) When an offense has in fact just been committed, and he has personal knowledge of facts indicating that the person to be arrested has committed it; and (c) When the person to be arrested is a prisoner who has escaped from a penal establishment or place where he is serving final judgment or temporarily confined while his case is pending, or has escaped while being transferred from one confinement to another.

Petitioners Argument: That he was not lawfully arrested without warrant because he went to the police station 6 days after the shooting, which he had allegedly perpetrated. Thus, the crime had not been just committed at the time he was arrested.

CONSTI II DIGESTS: 1D COMPILATION

CHUA, ALIMANGOHAN, CONTRERAS, BRIONES, LENCIO, RELOJO, TENGCO, LIM, DUMA, DANAO, MELLA, TONGSON

'+
4. Paragraph (b), Sec 5, Rule 113 of the Rules on Criminal Procedure provides that the arresting officer must have personal knowledge of an offense which has in fact been committed. a. Neither did Pat. Perez have personal knowledge, nor was the offense in fact just been committed. b. The rule requires that the arrest immediately follows the commission of the offense, not some 19 hours later. The killing took place at 1 AM. The arrest and search and seizure took place at 7 PM. 5. Personal gathering of information is different from personal knowledge Pat. Perez may have personally gathered the information which led to the arrest of Manlulu, however, that is not enough.

"

People v. Manlulu [1] Sec.2 | Warrantless Arrest Facts: GERARDO ALFARO, a NARCOM agent, was stabbed and shot with his service pistol in a drinking spree. The killing took place at 1:00 am. The arrest and the consequent search and seizure came at around 7:00pm, some 19 hours later. His drinking partners, Rolando Manlulu and Dante Samson, were haled to court for his death. Issue: Whether or not the non-issuance of a search warrant and warrant of arrest should nullify their arrest and consequently exclude from judicial consideration the evidence thus obtained Held/Ratio: The police authorities should have first obtained a warrant for the arrest of accused Rolando Manlulu, and for the search and seizure of his personal effects. Paragraph (b), Sec. 5, Rule 113 of the 1985 Rules on Criminal Procedure provides that the arresting officer must have "personal knowledge" of an offense which "has in fact just been committed." In the instant case, neither did Pat. Perez have "personal knowledge," nor was the offense "in fact just been committed." The law requires "personal knowledge." "Personal gathering of information" is different from "personal knowledge." The rule requires that the arrest immediately follows the commission of the offense, not some nineteen hours later.

People v. Bolasa Warrantless Arrests Facts: An anonymous caller tipped off PO3 Dante Salonga and PO3 Albert Carizon in the early evening of 11 September 1995 that a man and a woman were repacking prohibited drugs at a certain house in Sta. Brigida St., Karuhatan, Valenzuela, Metro Manila. The officers with SPO1 Fernando Arenas then proceeded to the address. After seeing through the window that a man and woman were repacking what was suspected to be marijuana, they entered the house, confiscated the drugs, and arrested Zenaida Bolasa and Roberto de los Reyes. Bolasa and de los Reyes were charged and found guilty of violating the Dangerous Drugs Act. Both appealed their conviction. Issue: W/N the warrantless arrest of the accused was valid, and consequently, W/N the accused could be convicted on the basis of the evidence found by the arresting officers. Ruling: No and No. The arrest was illegal, therefore the accompanying search was also illegal. Evidence obtained during the illegal search cannot be used against the accused. (Therefore they must be acquitted.) Ratio: The arrest of the accused was not a valid warrantless arrest. A valid warrantless search may be made if (a) the arresting officers have personal knowledge that at the time of the arrest, the accused were committing, or were about to commit a crime; (b) the arresting officers have personal knowledge that a crime was committed and they have reasonable ground to believe that the accused committed it; or (c) the accused are prisoners who have escaped from a penal establishment. None of these circumstances were present in this case. The evidence was obtained illegally. The objects were not in plain view when they were seized.

People v. Cendana: The accused was arrested one day after the killing of the victim, and only on the basis of information obtained by the police officers. It was held that "circumstances clearly belie a lawful warrantless arrest."
*(from orig) HOWEVER, The flaw, fatal as it may be, becomes moot in view of the eyewitness account of Manlapaz which we find to be credible. In spite of the nullification of the arrest of accused Manlulu, and the exclusion of real evidence, the prosecution was able to prove the guilt of the accused beyond reasonable doubt. After all, the illegality of the warrantless arrest cannot deprive the state of its right to prosecute the guilty when all other facts on record point to their culpability.

People v. Manlulu [2] Sec.2 | Warrantless Arrest Facts: Alfaro, a NARCOM agent, was stabbed and shot in a drinking spree at 1 AM. His drinking companions, Manlulu and Samson were arrested at 7 PM, 19 hours after the incident. Patrolman Perez arrested Manlulu on the information given by Manlapaz, who was also drinking with the accused and the victim. Patrolman Perez seized from Manlulu the .45 caliber pistol and Casio wristwatch allegedly belonging to Alfaro, without a warrant. Issue: WON the warrantless arrest is valid. Held: No. This instance cannot come within the purview of a valid warrantless aresst. Ratio: 3. The police authorities should have obtained a warrant for the arrest of Manlulu and for the search and seizure of his personal effects.

CONSTI II DIGESTS: 1D COMPILATION

CHUA, ALIMANGOHAN, CONTRERAS, BRIONES, LENCIO, RELOJO, TENGCO, LIM, DUMA, DANAO, MELLA, TONGSON

'!

"

There was no valid intrusion and the evidence, i.e., the tea bags with marijuana, were not inadvertently discovered. The police officers intentionally peeped through the window before they saw and ascertained activities of accused inside the room. The apprehending officers should have conducted surveillance and determined the existence of probable cause for arresting the accused. They then should have secured a search warrant prior to effecting a valid arrest and seizure.

People v. Escordial Section 2, Article III | Warrantless arrests Facts: The accused was watching a game in a basketball court when he was arrested by the police for a crime which took place a week before. He was not committing or attempting to commit a crime, nor was he an escaped prisoner whose arrest can be effected without a warrant. Issue: WON the arresting officers had personal knowledge of the facts and circumstances that would lead them to believe that the accused had just committed a crime. Held: No! The arresting officers were not present (basis: arrest was 1 week after the crime) when the crime was committed, hence they could not have personal knowledge of the facts and circumstances of the commission of the crime. For this reason, the arresting officer had no reason for not securing a warrant. HOWEVER, the accused pleaded not guilty of the crimes charged against him without questioning his warrantless arrest. He thus waived objection to the legality of his arrest. Ratio: Personal knowledge of facts in arrests without a warrant under Section 5(b) of Rule 113 must be based upon probable cause which means an actual belief or reasonable grounds of suspicion. The grounds of suspicion are reasonable when, in the absence of actual belief of the arresting officers, the suspicion that the person to be arrested is probably guilty of committing the offense is based on actual facts, i.e., supported by circumstances sufficiently strong in themselves to create the probable cause of guilt of the person to be arrested. A reasonable suspicion therefore must be founded on probable cause, coupled with good faith on the part of the peace officer making the arrest.

CONSTI II DIGESTS: 1D COMPILATION

CHUA, ALIMANGOHAN, CONTRERAS, BRIONES, LENCIO, RELOJO, TENGCO, LIM, DUMA, DANAO, MELLA, TONGSON

'#
the group saw the policemen coming, they ran in different directions. Three were caught and arrested. Each was found in possession of an unlicensed revolver and charged with illegal possession of firearms. The accused claimed that the warrantless seizure of firearms was illegal. The Court rejected their plea and held that the search was a valid incident of a lawful arrest. In People v. Lua, a buy-bust operation was conducted against the accused. After accused had gone inside his house and returned with the three tea bags of marijuana and received the marked money, the designated poseur-buyer gave the signal to his fellow police officers who closed in and arrested the accused. In the course of the arrest, a police officer noticed something bulging at accuseds waistline, which turned out to be an unlicensed .38 caliber paltik with two live bullets. Accused was charged with illegal possession of firearm. The search was held to be a valid incident of a lawful arrest.

"

People v. Jayson Warrantless Arrest Facts Accused-appellant Wenceslao Jayson was charged with violation of P.D. No. 1866 in the Regional Trial Court of Davao City accused-appellant, then a bouncer at the Ihaw-Ihaw nightclub on Bonifacio Street, Davao City, shot one Nelson Jordan. Accused-appellants arrest and the seizure from him of the firearm in question considering that both were made without any warrant from a court. With respect to the arrest, SPO1 Loreto Tenebro testified that at around 10:00 in the evening of March 16, 1991, while he and Patrolmen Camotes and Reinerio Racolas were patrolling in their car, they received a radio message from their camp directing them to proceed to the Ihaw-Ihaw on Bonifacio Street where there had been a shooting. Accordingly, they proceeded to the place and there saw the victim, Nelson Jordan. Bystanders pointed to accused-appellant as the one who had shot Jordan. They then arrested accused-appellant. Seized from him was a .38 caliber revolver with serial number 91955. The firearm was covered by a mission order and memorandum receipt. Considering these facts, we hold that the warrantless arrest and search were valid. Issue Whether or not the arrest and seizure of the accused-appelant was valid given there was no warrant from the court. Held Considering the facts, we hold that the warrantless arrest and search were valid. Ratio In the case at bar there was a shooting. The policemen summoned to the scene of the crime found the victim. Accused-appellant was pointed to them as the assailant only moments after the shooting. In fact accused-appellant had not gone very far (only ten meters away from the Ihaw-Ihaw), although he was then fleeing. The arresting officers thus acted on the basis of personal knowledge of the death of the victim and of facts indicating that accused-appellant was the assailant. In People v. Tonog, Jr, the police found the lifeless body of a person with several stab wounds. An informer pointed to the accused as the person who had killed the victim. That afternoon, police officers arrested the accused. On their way to the police station, a policeman noticed bloodstains on the accuseds pants which, when examined, was found to be the same blood type O found on the fatal knife. The Court upheld the warrantless arrest and ruled that the blood-stained pants, having been seized as an incident of a lawful arrest, was admissible in evidence. In People v. Gerente, the police arrested the accused three hours after the victim had been killed. They went to the scene of the crime where they found a piece of wood and a concrete hollow block used by the killers in bludgeoning the victim to death. A neighbor of the accused who witnessed the killing, pointed to him as one of the assailants. The warrantless arrest was held valid under Rule 113, 5(b). In People v. Acol, a group held up a passenger jeepney. Policemen immediately responded to the report of the crime. One of the victims saw four persons walking towards Fort Bonifacio, one of whom was wearing his jacket. He pointed them to the policemen. When

Note: Rule 113, 5(b) of the Revised Rules of Criminal Procedure provides: Sec. 5. Arrest without warrant; when lawful. A peace officer or private person may, without a warrant, arrest a person: (b) When an offense has in fact just been committed, and he has personal knowledge of facts indicating that the person to be arrested has committed it.

People v. Salvatierra Warrantless Arrest Facts: While Charlie Fernandez was walking towards Quiapo, appellant (Salvatierra) and the 2 accused lunged a pointed instrument at Charlie. Charlie died. On Nov 1990, the police received a complaint that appellant was creating a commotion. He was thereafter taken in custody. Appellant claims that he was having merienda with his wife and children when the incident occurred. He also claimed that he had an altercation with a woman on that day who caused his arrest for the crime of malicious mischief, wherein he was detained after. Then, when the police arrived, they brought him to the Homicide Section where he was investigated for the stabbing of Fernandez. Appellant was convicted for the crime of murder.

Salvatierras Arguments: His arrest, investigation, and detention for the offense charged violate his constitutional rights. According to appellant, his arrest was made almost 3 months after the commission of the crime and no warrant had been obtained during the 3-month intervening period between the commission of the crime and his apprehension, thus making the arrest illegal. The lower court erred in giving weight and credence to the vague and ambiguous testimony of the prosecution witness.

CONSTI II DIGESTS: 1D COMPILATION

CHUA, ALIMANGOHAN, CONTRERAS, BRIONES, LENCIO, RELOJO, TENGCO, LIM, DUMA, DANAO, MELLA, TONGSON

'$
People v. Doria Sec. 2 | Entrapment Facts: Two civilian informants informed the PNP Narcom that one Jun was engaged in illegal drug activities and the Narcom agents decided to entrap and arrenst Jun in a buy-bust operation. On the day of entrapment, PO3 Manlangit handed Jun the marked bills and Jun instructed PO3 Manlangit to wait for him while he got the marijuana from his associate. When they met up, Jun gave PO3 something wrapped in plastic upon which PO3 arrested Jun. They frisked Jun but did not find the marked bills on him. Jun revealed that he left the money at the house of his associate named neneth They wen to Neneths house. PO3 Manlangit noticed a carton box under the dinin table and noticed something wrapped in plastic inside the box. Suspicious, PO3 entered the house and took hold of the box and found that it ha 10 bricks of what appeared to be dried marijuana leaves. Simultaneously, SPO1 Badua recovered the marked bills from Neneth. The policemen arrested Neneth and took both her and Jun, together with the coz, its contents and the marked bill and turned them over to the investigator at headquarters, Jun was then learned to be Florencio Doria while Neneth is Violata Gaddao. They were both convicted feloniously selling, administering and giving away to another 11 plastic bags of suspected marijuana fruiting tops, in violation of R.A 6425, as amended by RA 7659 Issue: WON the buy bust operation is constitutional. Held: Yes. It is a valid buy bust operation--- ENTRAPMENT. In the case at bar, the evidence shows that it was the confidential informant who initially contacted accused-appellant Doria. At the pre-arranged meeting, the informant was accompanied by PO3 Manlangit who posed as the buyer of marijuana. PO3 Manlangit handed the marked money to accused-appellant Doria as advance payment for one (1) kilo of marijuana. Accused-appellant Doria was apprehended when he later returned and handed the brick of marijuana to PO3 Manlangit. PO3 Manlangit testified in a frank, spontaneous, straightforward and categorical manner and his credibility was not crumpled on cross-examination by defense counsel. Moreover, PO3 Manlangit's testimony was corroborated on its material points by SPO1 Badua, his back-up security. The non-presentation of the confidential informant is not fatal to the prosecution. Informants are usually not presented in court because of the need to hide their identity and preserve their invaluable service to the police. The inconsistencies in PO3 Manlangit's and SPO1 Badua's testimonies and the other police officers' testimonies are minor and do not detract from the veracity and weight of the prosecution evidence. The source of the money for the buy-bust operation is not a critical fact in the case at bar. It is enough that the prosecution proved that money was paid to accused-appellant Doria in consideration of which he sold and delivered the marijuana. Ratio:

"

Issue: WON appellant can question the legality of his arrest. Held: No. Ratio: Appellant is stopped from questioning the legality of his arrest considering that he never raised this before entering his plea. Any irregularity to his arrest, if any, had been cured by his voluntary submission to the jurisdiction of the trial court when he entered his plea and participated in the trial. Regarding the witness credibility issue: If ever there were inconsistencies, they were collateral matters, which are too trivial and minor to affect the evidentiary value of her testimony.

People v. Hernandez Sec. 2 | Warrantless Arrest Facts: The accused kidnapped Sharleen Tan, a prep student at ICA, to extort money from the latters family. The accused pled not guilty but the court found Hernandez, Jacob, Tumaneng, Lorenzo, and Famoudulan guilty beyond reasonable doubt of the crime charged. Tumaneng and Lorenzo then impugned their conviction on the ground that their warrantless arrests were illegal.

Lorenzo: claimed he just happened to be in front of the Crame gate that night when he was arrested by the CIS operatives. Tumaneng: claimed he was forcibly taken from his aunt's house in Maysilo, Malabon, and brought to the CIS detention cell where he was tortured to sign an extrajudicial confession.
Issue: WON the accused may assail the illegality of his arrest if he fails to move for the quashing of information before his arraignment Held: The warrantless arrests violated the Constitution but the accused are estopped from assailing the illegality of the arrest. Ratio: Entering a plea of not guilty and participating in the trial, constitutes a waiver of their right to challenge the legality of their warrantless arrests.

Entrapment was unknown in common law. It is a judicially created twentieth-century American doctrine that evolved from the increasing use of informers and undercover agents in the detection of crimes, particularly liquor and narcotics offenses. Entrapment is recognized as a

CONSTI II DIGESTS: 1D COMPILATION

CHUA, ALIMANGOHAN, CONTRERAS, BRIONES, LENCIO, RELOJO, TENGCO, LIM, DUMA, DANAO, MELLA, TONGSON

'%
Here, the court considers the nature of the police activity involved and the propriety of police conduct. The inquiry is focused on the inducements used by government agents, on police conduct, not on the accused and his predisposition to commit the crime. For the goal of the defense is to deter unlawful police conduct. The test of entrapment is whether the conduct of the law enforcement agent was likely to induce a normally law-abiding person, other than one who is ready and willing, to commit the offense; for purposes of this test, it is presumed that a law-abiding person would normally resist the temptation to commit a crime that is presented by the simple opportunity to act unlawfully. Official conduct that merely offers such an opportunity is permissible, but overbearing conduct, such as badgering, cajoling or importuning, or appeals to sentiments such as pity, sympathy, friendship or pleas of desperate illness, are not. Proponents of this test believe that courts must refuse to convict an entrapped accused not because his conduct falls outside the legal norm but rather because, even if his guilt has been established, the methods employed on behalf of the government to bring about the crime "cannot be countenanced." To some extent, this reflects the notion that the courts should not become tainted by condoning law enforcement improprieties, Hence, the transactions leading up to the offense, the interaction between the accused and law enforcement officer and the accused's response to the officer's inducements, the gravity of the crime, and the difficulty of detecting instances of its commission are considered in judging what the effect of the officer's conduct would be on a normal person.

"

valid defense that can be raised by an accused & partakes the nature of a confession & avoidance. It is recognized that in every arrest, there is a certain amount of entrapment used to outwit the persons violating or about to violate the law. Not every deception is forbidden. The objective test in buy-bust operations demands that the details of the purported transaction must be clearly & adequately shown. Courts should look at all factors to determine the predisposition of an accused to commit an offense in so far as they are relevant to determine the validity of the defense of inducement. This must start from the initial contact between the poseur-buyer and the pusher, the offer to purchase, the promise or payment of the consideration until the consummation of the sale by the delivery of the illegal drug subject of the sale. The manner by which the initial contact was made, whether or not through an informant, the offer to purchase the drug, the payment of the "buy-bust" money, and the delivery of the illegal drug, whether to the informant alone or the police officer, must be the subject of strict scrutiny by courts to insure that law-abiding citizens are not unlawfully induced to commit an offense. Criminals must be caught but not at all cost. At the same time, however, examining the conduct of the police should not disable courts into ignoring the accused's predisposition to commit the crime. If there is overwhelming evidence of habitual delinquency, recidivism or plain criminal proclivity, then this must also be considered. Courts should look at all factors to determine the predisposition of an accused to commit an offense in so far as they are relevant to determine the validity of the defense of inducement. TESTS of VALIDITY of Entrapment operations 1. SUBJECTIVE or Origin of intent American federal courts and state courts usually use the subjective or origin of intent test laid down in Sorrells v. U.S. to determine whether entrapment actually occurred. The focus of the inquiry is on the accuseds predisposition to commit the offense is charged, his state of mind and inclination before his initial exposure to government agents. The predisposition test emphasizes the accused's propensity to commit the offense rather than the officer's misconduct and reflects an attempt to draw a line between a "trap for the unwary innocent and the trap for the unwary criminal.

*Criticism: the purely "objective" test eliminates entirely the need for considering a particular accused's predisposition. His predisposition, at least if known by the police, may have an important bearing upon the question of whether the conduct of the police and their agents was proper. The undisputed fact that the accused was a dangerous and chronic offender or that he was a shrewd and active member of a criminal syndicate at the time of his arrest is relegated to irrelevancy.
3. HYBRID Objections to the two tests gave birth to hybrid approaches to entrapment. Some states in the United States now combine both the "subjective" and "objective" tests. In Cruz v. State, the Florida Supreme Court declared that the permissibility of police conduct must first be determined. If this objective test is satisfied, then the analysis turns to whether the accused was predisposed to commit the crime. In Baca v. State, the New Mexico Supreme Court modified the state's entrapment analysis by holding that "a criminal defendant may successfully assert a defense of entrapment, either by showing lack of predisposition to commit the crime for which he is charged, or, that the police exceeded the standards of proper investigation. The hybrid approaches combine and apply the "objective" and "subjective" tests alternatively or concurrently.

*Criticism: It is claimed that the "subjective" test creates an "anything goes" rule, i.e., if the court determines that an accused was predisposed to commit the crime charged, no level of police deceit, badgering or other unsavory practices will be deemed impermissible. Delving into the accused's character and predisposition obscures the more important task of judging police behavior and prejudices the accused more generally. It ignores the possibility that no matter what his past crimes and general disposition were the accused might not have committed the particular crime unless confronted with inordinate inducements.
2. OBJECTIVE TEST Another test is the objective test where the test of entrapment is whether the conduct of the law enforcement agents was likely to induce a normally law-abiding person, other than one who is ready and willing, to commit the offense.

ENTRAPMENT ways and means are resorted to by the peace officer for the purpose of trapping and capturing the lawbreaker in the execution of his criminal plan.
entrapment is no bar to the

INSTIGATION The instigator practically induces the would-be accused into the commission of the offense and himself becomes a co-principal.
has often been condemned and has

CONSTI II DIGESTS: 1D COMPILATION

CHUA, ALIMANGOHAN, CONTRERAS, BRIONES, LENCIO, RELOJO, TENGCO, LIM, DUMA, DANAO, MELLA, TONGSON

'&
Terry vs. Ohio Section 2, Article III | Stop and Frisk

"

sometimes been held to prevent the act from being criminal or punishable. It can thus be seen that the concept of entrapment in the American jurisdiction is similar to instigation or inducement in Philippine jurisprudence. Entrapment in the Philippines is not a defense In the American jurisdiction, the term available to the accused. "entrapment" has a generally negative It is instigation that is a defense and is meaning because it is understood as the considered an absolutory cause. To inducement of one to commit a crime not determine whether there is entrapment or contemplated by him, for the mere purpose of instigation, our courts have mainly examined instituting a criminal prosecution against him. the conduct of the apprehending officers, not the predisposition of the accused to commit the "Entrapment is the conception and crime. planning of an offense by an officer, and his procurement of its commission by one who would not have perpetrated it except for the trickery, persuasion or fraud of the officer. It consists of two (2) elements: (a) acts of persuasion, trickery, or fraud carried out by law enforcement officers or the agents to induce a defendant to commit a crime; and (b) the origin of the criminal design in the minds of the government officials rather than that of the innocent defendant, such that the crime is the product of the creative activity of the law enforcement officer

prosecution and conviction of the lawbreaker.

Facts:

McFadden, a Cleveland detective, observed two strangers on a street corner while he was patrolling. The two strangers were the petitioner and another man named Chilton. He saw them proceed back and forth along an identical route, pausing to stare in the same store window which they did for a total of 24 times. Each completion of the route was followed by a conference between them, at one of which they were joined by a third man named Katz who left swiftly. McFadden suspected the 2 men of casing a job, a stick up and thus, he followed them and saw them rejoin Katz, a couple of blocks away in front of a store. He approached the three and identified himself as a policeman and asked their names. The men mumbled something then McFadden patted down petitioners outside clothing and found a pistol in the pocket but he was unable to remove it. He then ordered the men to enter the store then he removed petitioners overcoat, took out a revolver and ordered the three to face the wall with their hands raised. He patted down the outer clothing of Chilton and Katz and seized a revolver from Chiltons pocket. He did not put his hands under the outer garments of Katz, or under petitioners or Chiltons outer garments until he felt the guns. They were then taken to the police station. Petitioner and Chilton were charged with carrying concealed weapons. Though the trial court rejected the prosecution theory that the guns had been seized during a search incident to a lawful arrest, the court denied the motion to suppress and admitted the weapons into evidence on the ground that the officer had cause to believe that petitioner and Chilton were acting suspiciously and that the officer, for his own protection, had the right to pat down their outer clothing having reasonable cause to believe that they might be armed. Issue: Whether or not the weapons seized may be admissible as evidence against the petitioner Held: Yes, since the search which led to its seizure was reasonable under the Fourth Amendment. Ratio: -

It is recognized that in every arrest, there is a certain amount of entrapment used to outwit the persons violating or about to violate the law. Not every deception is forbidden. The type of entrapment the law forbids is the inducing of another to violate the law, the "seduction" of an otherwise innocent person into a criminal career. Where the criminal intent originates in the mind of the entrapping person and the accused is lured into the commission of the offense charged in order to prosecute him, there is entrapment and no conviction may be had. Where, however, the criminal intent originates in the mind of the accused and the criminal offense is completed, the fact that a person acting as a decoy for the state, or public officials furnished the accused an opportunity for commission of the offense, or that the accused is aided in the commission of the crime in order to secure the evidence necessary to prosecute him, there is no entrapment and the accused must be convicted. The law tolerates the use of decoys and other artifices to catch a criminal.

The Fourth Amendment applies to stop and frisk procedures such as those followed here. whenever a police officer accosts an individual and restrains his freedom to walk away, he has seized that person within the meaning of the Fourth Amendment a careful exploration of the outer surfaces of a persons clothing in an attempt to find weapons is a search under that Amendment Where a reasonably prudent officer is warranted in the circumstances of a given case in believing that his safety or that of others is endangered, he may make a reasonable search for weapons of the person believed by him to be armed and dangerous regardless of whether he has probable cause to arrest that individual for crime or the absolute certainty that the individual is armed. though the police must secure a warrant to make a search and seizure, that procedure cannot be followed where swift action based upon on-the-spot observations of the officer on the beat is required the officer here was performing a legitimate function of investigating suspicious conduct when he decided to approach petitioner and his companions

CONSTI II DIGESTS: 1D COMPILATION

CHUA, ALIMANGOHAN, CONTRERAS, BRIONES, LENCIO, RELOJO, TENGCO, LIM, DUMA, DANAO, MELLA, TONGSON

''
The ruling of the trial court was reversed on ground of reasonable doubt Ratio: Prosecution failed to establish petitioners guilt beyond reasonable doubt. Serious doubts surrounds the story of police officer Yu that a grenade was found in and seized from petitioners possession It was unnatural and against common experience that petitioner simply stood there in proximity to the police officers when it is recognizable that they are the ones who tried to apprehend him two days earlier. Even the petitioner admitted he had in his possession a grenade, it would still be inadmissible in evidence since it was taken in violation of Sections 12 (1) and (3) of Art III of the Constitution which requires the presence of a counsel Trial court confused the concepts of a stop and frisk of a search incidental to a lawful arrest.

"

an officer believing that an individual whose suspicious behavior he is investigating at close range is armed may, to neutralize the threat of physical harm, take necessary measures to determine whether that person is carrying a weapon. The officers protective seizure of petitioner and his companions and the limited search which he made were reasonable, both at their inception and as conducted. the actions of petitioner and his companions were consistent with the officers hypothesis that they were contemplating a daylight robbery and were armed the officers search was confined to what was minimally necessary to determine whether the men were armed, and the intrusion which was made for the sole purpose of protecting himself and others nearby, was confined to ascertaining the presence of weapons The exclusionary rule cannot properly be invoked to exclude the products of legitimate and restrained police investigative techniques.

Malacat v. Court of Appeals Section 2 \ Article III \ Stop and frisk Facts: Petitioner Sammy Malacat was charged with violating Section 3 of PD No. 1866 which prohibits a person from possessing a hand grenade without first securing a license from the proper authorities. Petitioner admitted at the pre-trial that at the time of arrest, police authorities were not armed with a search warrant nor a warrant of arrest. Rodolfo Yu of the Western Police District narrated the incident and said that they noticed the group of Malacat while they were conducting the foot patrol as a response to a report that a group of Muslims was going to explode a grenade somewhere in Plaza Miranda. Yu added that they approached the group of Malacat because they were acting suspiciously with their eyes moving very fast. When the police officers were about to come near that group of men, they fled into different directions but Yu was able to apprehend Malacat. According to Yu, he found the grenade in the waistline of the petitioner. Yu further added that he recognized Malacat as one of those 3 men who attemted to detonate a grenade in Plaza Miranda 2 days earlier. The petitioners, however, told a different story. Malacat said that the policemen found nothing in their possession but still dragged them to jail accusing him of shooting an officer. The trial court ruled in favor of Yu affirming the validity of warrantless search and seizure of petitioner and said it was akin to a stop and frisk. According to the trial court, the seizure of grenade from petitioner was incidental to a lawful arrest. Trial court found the petitioner guilty beyond reasonable doubt because of the latters admission to the police investigator. Petitioner contended that the decision of the trial court was invalid for violating Section 5 of Rule 113 of the Rules of Court Issue: Whether or not the lower court erred in holding that the search upon the person of accused appellant and the seizure of the alleged hand grenade from him was an appropriate incident to his arrest Held:

In lawful arrest, the precedent arrest determines the validity of the search. The arrest shall not be used as a pretext for conducting research. o In this case, there was no valid in flagrante delicto or hot pursuit arrest preceding the search in light of the lack of personal knowledge on the part of Yu, or an overt act, on the part of the petitioner, indicating that a crime had just been committed, was being committed or was going to be committed.

3 reasons why the stop and frisk was invalid: o The claim of Yu that petitioner was a member of the group which attempted to bomb Plaza Miranda is neither supported by any police record therefore diminishing the probability that there was a genuine reason for the arrest and search of petitioner. o There was nothing in the behavior of the petitioner which could have reasonably elicited even mere suspicion other than his eyes were moving fastan observation which is hard to believe considering that it was at night time and it was dark when Yu and his teammates saw petitioner o There was at all no ground, probable or otherwise, to believe that the petitioner was armed with a deadly weapon. The discovered grenade was not even visible to Yu since as he admitted it was inside the front waistline of petitioner. Note: In stop and frisk, probable cause is not required but there must be a genuine reason, in light of the police officers experience and surrounding conditions, to warrant the belief that the person detained has weapons concealed about him.

2-fold interest in stop and frisk 1. The general interest of effective crime prevention and detection, which underlies the recognition that a police officer may, under appropriate circumstances and in an appropriate manner, approach a person for purposes of investigating possible criminal behavior even without probable cause 2. The more pressing interest of safety and self-preservation which permit the police officer to take steps to assure himself that the person with whom he deals is not armed with deadly weapon that could unexpectedly and fatally be used against the police officer.

People v. Chua

CONSTI II DIGESTS: 1D COMPILATION

CHUA, ALIMANGOHAN, CONTRERAS, BRIONES, LENCIO, RELOJO, TENGCO, LIM, DUMA, DANAO, MELLA, TONGSON

'(
o Chua did not exhibit manifest unusual and suspicious conduct hence, no genuine reasonable ground for the immediacy of accused-appellants arrest.

"

Sec. 2, Art. III, Stop and Frisk Facts: Chua was under surveillance for 2 years. On the day of his arrest, Chua arrived at the hotel and merely parked his car along MacArthur Highway, then casually proceeded to the Hotel clutching a sealed Zest-O juice box. Then the group of SPO2 Nulud hurriedly accosted Chua and later introduced themselves as police officers. He was arrested before drop-off of shabu was done.
Respondent Chuas arguments: He was under surveillance for 2 years hence there is no haste where officers sought to arrest and search him without warrant Drugs are inadmissible as evidence Issue: Whether or not search of his person and confiscation of shabu is lawful. Held: No. It is not lawful. Ratio: The trial court confused search incidental to a lawful arrest (in flagrante delicto) and stop-and-frisk Malacat vs CA: these two concepts differ in the requisite quantum of proof o Search incidental to a lawful arrest arrest determines the validity of the search. Law requires that there be an arrest before a search can be made ! 2 elements: person to be arrested must execute an overt act indicating that he has just committed, is actually committing, or is attempting to commit a crime overt act is done in the presence or within the view of the arresting officer. o Stop-and-frisk act of a police officer to stop a citizen on the street, interrogate him, and pat him for weapon or contraband. The police officer should properly introduce himself and make initial inquiries, approach and restrain a person who manifests unusual and suspicious conduct, in order to check the latters outer clothing for possibly concealed weapons. It should be emphasized that a search and seizure should precede the arrest for this principle to apply. o while probable cause is not required to conduct a stop-and-frisk, mere suspicion or a hunch will not validate a stop-and-frisk. A genuine reason must exist, in light of the police officers experience and surrounding conditions, to warrant the belief that the person detained has weapons concealed about him. ! stop-and-frisk serves a two-fold interest: General interest of crime prevention and detection the more pressing interest of safety and self-preservation In the case, there was no in flagrante delicto arrest o Chua was arrested before alleged drop-off of shabu was done, hence there was no probable cause since there was no overt act that he is committing a crime o Reliable information alone is not sufficient to constitute probable cause, there must be an overt act which indicates a felonious enterprise In the case, there was no valid stop-and frisk o Search and seizure was done only after the arrest o Police failed to make initial inquiry into Chuas business in the vicinity or the contents of the Zest-O juice box he was carrying. o They only introduced themselves when they already had custody of him.

Marti Exclusionary Rule

Ramirez v Court of Appeals [1] Section 3, Article 3 | Exclusionary Rule Facts: A civil case for damages was filed by petitioner Socorro Ramirez alleging that the private respondent, Ester Garcia, in a confrontation in the latters office, vexed, insulted and humiliated her in a hostile and furious mood and in a manner offensive to petitioners dignity and personality, contrary to morals, good customs and public policy. In support of her claim, petitioner produced a verbatim transcript of the event. The transcript on which the civil case was based was culled from a tape recording of the confrontation made by petitioner. Private respondent filed a criminal case as a result of petitioners recording of the event, alleging that the said act of secretly taping the confrontation was illegal and in violation of RA 4200 (An Act to prohibit and penalize wire tapping and other related violations of private communication, and other purposes).

Petitioners argument: The applicable provision of RA 4200 does not apply to the taping of a private conversation by one of the parties to the conversation. It merely refers to unauthorized taping of a private conversation by a party other than those involved in the communication.
(Her other contentions-may disregard: The substance or content of the conversation must be alleged in the information, otherwise the facts charged would not constitute a violation of RA 4200; RA 4200 penalizes the taping of a private communication, not a private conversation) Issue: Whether or not petitioners recording of confrontation was illegal. Held: Yes. The unambiguity of the express words of the provision, taken with the deliberations of the Congressional Record, supports the view that provision seeks to penalize even those privy to private communications. Ratio: 1) Unambiguity of the provision Legislative intent is determined principally from the language of the statute. Where the language of a statute is clear and unambiguous, the law is applied according to its express terms. Section 1 of RA 4200: It shall be unlawful for any person, not being authorized by all the parties to any private communication or spoken word, to xxx record such communication or spoken word by using a xxx tape recorder xxx.

CONSTI II DIGESTS: 1D COMPILATION

CHUA, ALIMANGOHAN, CONTRERAS, BRIONES, LENCIO, RELOJO, TENGCO, LIM, DUMA, DANAO, MELLA, TONGSON

')
Whether or not the petitioners recording qualifies as a violation under RA 4200 Held: Petitioner is guilty under R.A. 4200. The unambiguity of the express words of the provision, taken together with the deliberations from the Congressional Record, therefore plainly supports the view held by the respondent court that the provision seeks to penalize even those privy to the private communications. Ratio: Legislative intent is determined principally from the language of a statute. Where the language of the statute is clear and unambiguous, the law is applied according to its express terms. The statutes intent to penalize all persons unauthorized to make such recording is underscored by the use of the qualifier any. Section 1 of RA 4200 entitled, An act to prohibit and penalize Wire Tapping and Other related Violations of Private Communication and Other purposes. clearly and unequivocally makes it illegal for any person, not authorized by all the parties to any private communication to secretly record such communication by means of the tape recorder. The law makes no distinction as to whether the party sought to be penalized by the statute ought to be a party other than or different from those involved in the private communication. Even a (person) privy to a communication who records his private conversation with another with or without the knowledge of the latter (will) qualify as a violator under this provision of R.A. 4200. A perusal of the State Congressional Records, moreover, supports the respondent court conclusion that in enacting R.A. 4200 our lawmakers indeed contemplated to make illegal unauthorized tape recording of private conversations or communications taken either by the parties themselves or by third persons

"

The provision clearly makes it illegal for any person not authorized by all the parties to any private communication to secretly record such communication by means of a tape recorder. The law makes no distinction as to whether the party sought to be penalized by the statute ought to be a party other than or different from those involved in the private communication the intent to penalize all persons unauthorized to make such recording is underscored by the use of any. 2) Deliberations from the Congressional record A perusal of the Congressional records show that the lawmakers contemplated to make illegal unauthorized tape recording of private conversations or communications taken either by the parties themselves or by third persons. !!!!! On the device used: Gaanan v Intermediate Appellate Court: The use of a telephone extension for the purpose of overhearing a private conversation without authorization did not violate RA 4200 because a telephone extension device was neither among those enumerated in Sec 1 of the law nor was it similar to those. Penal statutes must be construed sitrctly in favor of the accused. The instant case turns on a different note, because the applicable facts and circumstances pointing to a violation in RA 4200 suffer from no ambiguity and the statute itself explicitly mentions the unauthorized recording of private communications with the use of tape recorders.

B. Ramirez v. Court of Appeals [2] Sec. 3, Art. III, Exclusionary rule Facts: A civil case for damages was filed by petitioner Socorro D. Ramirez in the Regional Trial Court alleging that the private respondent, Ester S. Garcia, in a confrontation in the latters office, allegedly vexed, insulted and humiliated her. In support of her claim, petitioner produced a verbatim transcript of the event and sought moral damages, attorneys fees and other expenses of litigation, the transcript on which the civil case was based was culled from a tape recording the confrontation made by petitioner. As a result of petitioners recording, private respondent filed a criminal case before the Regional Trial Court for violation of Republic Act 4200, entitled an Act to prohibit and penalize wire tapping and other related violations of private communication, and other purposes. An information-charging petitioner of violation of the said act.

Note: * Section 1: It shall be unlawful for any person, not being unauthorized by all the parties to any private communication or spoken word, to tap any wire or cable, or by using, any other device or arrangement, to secretly overhear, intercept, or record, such communication or spoken word by using a device commonly known as a Dictaphone or dictagraph or detectaphone or walkie-talkie or tape recorder, or however otherwise described.

Silahis

Petitioners Argument: Republic Act 4200 does not apply to the taping of a private conversation by one of the parties to the conversation. She contends that the provision merely refers to the authorized taping of a private conversation by a party other than those involved in the communication. That R.A. 4200 penalizes the taping of a private communication, not a private conversation and that consequently, her act of secretly taping her conversation with private respondent was not illegal under the said act.
Issue:

Damaso Waiver of rights under Secs 2 & 3 -END-

CONSTI II DIGESTS: 1D COMPILATION

CHUA, ALIMANGOHAN, CONTRERAS, BRIONES, LENCIO, RELOJO, TENGCO, LIM, DUMA, DANAO, MELLA, TONGSON

'*
On August 20, 1941, Royal L. Rutter (Petitioner) sold to Placido J.Esteban (Respondent) two parcels of land situated in the City of Manila for the sum of P9,600. P4,800 was paid outright, and the balance of P4,800 was made payable as follows: P2,400 on or before August 7, 1942, and P2,400 on or before August 27, 1943, with interest at the rate of 7 percent per annum. To secure the payment of said balance of P4,800, a mortgage over the same parcels of land was constituted in favor of the plaintiff. Respondent failed to pay the two installments as agreed upon, as well as the interest that had accrued. On August 2, 1949, Royal L. Rutter instituted this action in the Court of First Instance of Manila to recover the balance due, the interest due thereon, and the attorney's fees stipulated in the contract. The complaint also contained a prayer for sale of the properties mortgaged. Respondent set up the defense of the moratorium clause embodied in Republic Act No. 342. Section 2 of Republic Act No. 342 provides that all debts and other monetary obligations contracted before December 8, 1941, shall not be due and demandable for a period of eight (8) years from and after settlement of the war damage claim of the debtor by the Philippine War Damage Commission. Section 3 of the said Act provides that should the provision of section 2 be declared void and unenforceable, then as regards the obligation affected thereby, the provisions of Executive Order No. 25 as amended by Executive Order No. 32, relative to debt moratorium, shall continue to be in force and effect.

"

SEC #: HELLOOOOOOOOOOOO City of Manila v. Estrada (1913) [ratio lang yung nasa case book. I just added facts, just in case she asks] Facts: The city of Manila sought to expropriate an entire parcel of land with its improvements for use in connection with a new market at that time being erected in the district of Paco. A complaint was filed setting forth the necessary allegations, answer joined, and commissioners were appointed, who, after viewing the premises and receiving evidence, and being unable to agree, submitted two reports to the court. The court duly rendered its decision, confirming the majority report as to the improvements, but reducing the price of the land from P20 per square meter, as fixed by the majority report, to P15 per square meter. Motions for a new trial having been made by both parties and denied by the court, both parties appealed from that part of the decision fixing the value of the land at P15 per square meter. The record was therefore elevated to this court for a review of the evidence and assigned errors of the parties. Held: The court held that the market value of the land is the just compensation to which the owner is entitled Ratio: Packard v. Bergen Neck Ry. Co.: ! difficulty is with its application for the determination of the market value of the land o market value of the land: sum of money which a person would agree on as a price to be given and received therefore Market value: ! attained by a consideration of all those facts which make it commercially valuable United States Supreme Court: ! in determining value of land (compensation of owner): o same considerations as in a sale of property between private parties o must be what is the property worth in the market, not merely with reference to the uses which it is at the time applied, but with reference to the uses to which it is plainly adapted o to be estimated by reference to the uses for which the property is suitable, having regard to the existing business or wants of the community, or such as may be reasonably expected in the immediate future Maddumba NAPOCOR MERALCO Land Bank De Knecht Rep v De Knecht Manotok Home Building Facts: Rutter v. Esteban (1953) Reserved Power of the State to impair Contracts/Requisites and Limitations

Respondent Argument: The subject obligation is a prewar obligation contracted on August 20, 1941 and that he is a war sufferer, having filed his claim with the Philippine War Damage Commission for the losses he had suffered as a consequence of the last war.
Under Section 2 of said Republic Act No. 342, payment of his obligation cannot be enforced until after the lapse of eight years from the settlement of his claim by the Philippine War Damage Commission. This period has not yet expired.

Petitioner Argument: RA 342 is null and void for violating the non-impairment clause of the Constitution.
Issue: W/N RA 342 is constitutional. Ruling: No. RA 342 was declared null and void for being unreasonable and oppressive. EO 25 and 32 were also declared null and void for the same reasons. Ratio: Reserved Power of the State

CONSTI II DIGESTS: 1D COMPILATION

CHUA, ALIMANGOHAN, CONTRERAS, BRIONES, LENCIO, RELOJO, TENGCO, LIM, DUMA, DANAO, MELLA, TONGSON

(+
In light of these circumstances, there is no longer any justification for the continued operation of the debt moratoria imposed by RA 342, EO 25 and EO 32. [For Reference: Blaisdell Summary: Appellant contested the validity of charter 339 of the laws of Minnesota of 1993, approved April 13, 1933, called the Minnesota Mortgage Moratorium Law, as being repugnant to the contract clause of the Federal Constitution. The statute was sustained by the Supreme Court of Minnesota as an emergency measure. "Although coceding that the obligations of the mortgage contract was impaired, the court decided that what it thus described as an impairment was, notwithstanding the contract clause of the Federal Constitution, within the police power of the State as that power was called into exercise by the public economic emergency which the legislative had found to exist". This theory was up-held by the Supreme Court.] Ortigas Juarez Miranda Magtoto People v. Mahinay [ratio lang yung nasa case book. As in guidelines lang. I just added facts, just in case she asks] Facts: Mahinay killed and raped a 12 yr old girl named Ma. Victoria. He was apprehended by the police and made an extra judicial confession with the assistance of Atty. Viernes. He was later on charged and later on convicted for killing and raping Ma. Victoria. He claims that his extrajudicial confession was executed in violation of his constitutional right to counsel. Held: There was no violation of his right to counsel. The consequences of his action were clearly explained by Viernes before he executed the extra-judicial confession. Ratio: Guidelines in the Miranda Rights 1. must be informed in a language known to and understood by him of the reason for the arrest; must be shown the warrant of arrest, if any; every other warning, information or communication must be in a language known to and understood by said person 2. person arrested must be told that he has a right to remain silent and that any statement he makes may be used as evidence against him 3. He must be informed that he has the right to be assisted at all times and have the presence of an independent and competent lawyer, preferably of his own choice; 4. He must be informed that if he has no lawyer or cannot afford the services of a lawyer, one will be provided for him; and that a lawyer may also be engaged by any person in his behalf, or may be appointed by the court upon petition of the person arrested or one acting in his behalf; 5. That whether or not the person arrested has a lawyer, he must be informed that no custodial investigation in any form shall be conducted except in the presence of his counsel or after a valid waiver has been made; 6. The person arrested must be informed that, at any time, he has the right to communicate or confer by the most expedient means telephone, radio, letter or

"

Home Building and Loan Association vs. Blaisdell


All contracts are subject to an implied reservation of the protective power of the state. Statutes which validly exercise this reserved power, rather than impairing the obligations of an existing contract, are comprehended within the obligations. Laws altering existing contracts constitute an impairment within the meaning of the contract clause only if they are unreasonable in the light of the circumstances occasioning their enactment. Limitations on the Reserved Power Also from Blaisdell 1. 2. 3. 4. The statute imposing the impairment must be addressed to a legitimate purpose. The impairment should only refer to the remedy and not to a substantive right. (The State may postpone the enforcement of the obligation but cannot destroy it by making the remedy futile) The alteration or change must not be burdened with unreasonable restrictions and conditions. (Extensions must not be so piled-up as to make the remedy a shadow) The statute must be justified by an emergency, temporary in nature.

Application to this case 1. RA 342, EO 25 and EO 32 have the combined effect of extending the debt moratorium to 12 years. This is an unreasonably long time for the creditors to wait for the settlement of the outstanding obligations. The Court held: While the purpose of Congress is plausible, and should be commended, the relief accorded works injustice to creditors who are practically left at the mercy of the debtors. Their hope to effect collection becomes extremely remote, more so if the credits are unsecured. And the injustice is more patent when, under the law, the debtor is not even required to pay interest during the operation of the relief, unlike similar statutes in the United States. 2. The Court determined that there was no longer an emergency to justify the effectivity of the moratoria. The Court took due notice of factual circumstances that showed the improving conditions of the country and referred to public statements by the Chief Executive indicating the growth of the national economy. The Court held: It can now be safely stated that in the main the financial condition of our country and our people, individually and collectively, has practically returned to normal notwithstanding occasional reverses caused by local dissidence and the sporadic disturbance of peace and order in our midst. Business, industry and agriculture have picked up and developed at such stride that we can say that we are now well on the road to recovery and progress.

CONSTI II DIGESTS: 1D COMPILATION

CHUA, ALIMANGOHAN, CONTRERAS, BRIONES, LENCIO, RELOJO, TENGCO, LIM, DUMA, DANAO, MELLA, TONGSON

(!
Issue: W/N there was a violation of Loverias right to counsel (and his other Miranda rights); Ruling: No. At the time he was identified by the complainant in the police line-up, Loveria was not yet under investigation. Ratio: 1. Miranda rights may be invoked by an accused only when he is under custodial investigation. Custodial investigation - questioning initiated by law enforcement officers after a person has been taken into custody or otherwise deprived of his freedom of action in any significant way.

"

messenger with his lawyer (either retained or appointed), any member of his immediate family, or any medical doctor, priest or minister chosen by him or by any one from his immediate family or by his counsel, or be visited by/confer with duly accredited national or international non-government organization. It shall be the responsibility of the officer to ensure that this is accomplished; 7. He must be informed that he has the right to waive any of said rights provided it is made voluntarily, knowingly and intelligently and ensure that he understood the same; 8. In addition, if the person arrested waives his right to a lawyer, he must be informed that it must be done in writing AND in the presence of counsel, otherwise, he must be warned that the waiver is void even if he insist on his waiver and chooses to speak; 9. That the person arrested must be informed that he may indicate in any manner at any time or stage of the process that he does not wish to be questioned with warning that once he makes such indication, the police may not interrogate him if the same had not yet commenced, or the interrogation must ceased if it has already begun; 10. The person arrested must be informed that his initial waiver of his right to remain silent, the right to counsel or any of his rights does not bar him from invoking it at any time during the process, regardless of whether he may have answered some questions or volunteered some statements; 11. He must also be informed that any statement or evidence, as the case may be, obtained in violation of any of the foregoing, whether inculpatory or exculpatory, in whole or in part, shall be inadmissible in evidence. Taylaran Marcos Rapesa Judge Ayson Marra Maqueda Balisteros Gamboa v Judge Cruz People v. Loveria (1990) Miranda Rights/May be invoked only while under Custodial Investigation Facts: Cerilo Manzanero was the driver of a jeepney which the accused, David S. Loveria and two other companions robbed. On the occasion of the robbery the accused stabbed Manzanero multiple times. Manzanero was taken to the hospital were he made a full recovery. Subsequently, upon learning that certain suspects of a different robbery were being detained at the 225th PC Company, Manzanero proceeded to the police headquarters to check. There, he was able to identify the accused Loveria from a line-up. Pat. Ayun then took the sworn statement of Manzanero which was presented in court as evidence (Exh. B). Loveria was convicted by the Regional Trial Court, Branch CLIX (159) of the crime of Robbery with Homicide and Frustrated Homicide. In the appeal before the Supreme Court, accused-appellant assailed the manner in which he was identified by Manzanero at the headquarters of the 225th Philippine Constabulary (PC) in Cogeo, Antipolo, Rizal. He claimed a violation of his constitutional right to counsel.

Gamboa v. Cruz applies. (There was also a police line-up where the accused was identified. The accused made the same argument.)
In the police line-up conducted, it was the complainant who was being investigated and who gave a statement to the police while the accused was not questioned at all. The accused could not, during the line-up, invoke his right to counsel because he was not under custodial interrogation. The situation is similar here. The appellant was not investigated when Manzanero was in the process of identifying him, he cannot claim that his right to counsel was violated because at that stage, he was not entitled to the constitutional guarantee invoked. 2. There was evidence enough to convict the accused even without the sworn statement of Manzanero (Exh. B) Even assuming that the process of identification of the appellant by Manzanero at the PC headquarters was attended by constitutional infirmities, only Manzanero's sworn statement (Exh. B) where he identified appellant and which was taken by Pat. Ayun, would be excluded for being inadmissible in evidence. The testimony of Manzanero made in open court positively identifying the appellant, as well as those of Richard Bales (conductor) and Betty Apolinario (passenger), would not be affected. These testimonies, taken together with the other evidence on record, would be sufficient to sustain the trial court's judgment of conviction. Hatton Frago Peo v Gamboa Linsangan

CONSTI II DIGESTS: 1D COMPILATION

CHUA, ALIMANGOHAN, CONTRERAS, BRIONES, LENCIO, RELOJO, TENGCO, LIM, DUMA, DANAO, MELLA, TONGSON

(#
issuance of a writ of possession in an expropriation case. Section 19 of the Local Government Code governs expropriation on LGUs. In expropriation proceedings, the requisites for authorizing immediate entry are: (1) the filing of a complaint for expropriation sufficient in form and substance (2) the deposit of the amount equivalent to 15 percent of the fair market value of the property to be expropriated based on its current tax declaration. Issuance of the Writ of Possession in favor of respondent complied with the foregoing requisites

"

SEC 9: EMINENT DOMAIN, DEFINITION, NATURE Bardillon v. Barangay Masili FACTS: Petitioner claims that since the value of land is only P11,448, the MTC has jurisdiction over the case. CA held that the assessed value of property was P28,960. Petitioner also argues that the CA erred when it ignored the RTCs Writ of Possession over her property, issued despite the pending Motion for Reconsideration of the ruling dismissing the Complaint. ISSUE: (1) Whether the MTC had jurisdiction over the expropriation case (2) Whether the CA erred when it ignored the issue of entry upon the premises HELD: (1) NO. (2) NO. The issue of the necessity of the expropriation is a matter properly addressed to the RTC in the course of the expropriation proceedings. If petitioner objects to the necessity of the takeover of her property, she should say so in her Answer to the Complaint. The RTC has the power to inquire into the legality of the exercise of the right of eminent domain and to determine whether there is a genuine necessity for it. RATIO: (1) An expropriation suit does not involve the recovery of a sum of money. Rather, it deals with the exercise by the government of its authority and right to take property for public use. Thus, it is incapable of pecuniary estimation. It falls within the jurisdiction of regional trial courts, regardless of the value of the subject property.

Estate of Heirs of the late Ex-justice JBL Reyes vs. City of Manila Facts: During the pendency of the two ejectment cases against respondents Abiog and Maglonso (formers tenants), respondent city filed on April 25, 1995 a complaint for eminent domain (expropriation) of the properties of petitioners. The complaint was based on Ordinance No. 7818 enacted on November 29, 1993 authorizing the City Mayor of Manila to expropriate certain parcels of land. According to the ordinance, the said properties were to be distributed to the intended beneficiaries, who were the occupants of the said parcels of land who (had) been occupying the said lands as lessees or any term thereof for a period of at least 10 years. Issues: Whether or not respondent Citys act of expropriation is illegal because it did not comply with Sections 9 and 10 of Republic Act No. 7279 (The Urban Development and Housing Act of 1992) Held: Yes, before respondent City can exercise its power of eminent domain, the same must be sanctioned and must not violate any law. Being a mere creation of the legislature, a local government unit can only exercise powers granted to it by the legislature. Such is the nature of the constitutional power of control of Congress over local government units, the latter being mere creations of the former. When it expropriated the subject properties, respondent City relied on its powers granted by Section 19 of the Local Government Code of 1991 and RA 409 (The Revised Charter of the City of Manila). The latter specifically gives respondent City the power to expropriate private property in the pursuit of its urban land reform and housing program. Respondent City, however, is also mandated to follow the conditions and standards prescribed by RA 7279 (the Urban Development and Housing Act of 1992), the law governing the expropriation of property for urban land reform and housing. Sections 9 and 10 of RA 7279 specifically provide that: Respondent City, however, is also mandated to follow the conditions and standards prescribed by RA7279 (the Urban Development and Housing Act of 1992), the law governing the expropriation of property for urban land reform and housing. Sections 9 and 10 of RA 7279 specifically provide that:

Barangay San Roque v. Heirs of Francisco Pastor: the primary consideration in an expropriation suit is whether the government or any of its instrumentalities has complied with the requisites for the taking of private property. Hence, the courts determine the authority of the government entity, the necessity of the expropriation, and the observance of due process.

the subject of an expropriation suit is the governments exercise of eminent domain, a matter that is incapable of pecuniary estimation. True, the value of the property to be expropriated is estimated in monetary terms, for the court is duty-bound to determine the just compensation for it. This, however, is merely incidental to the expropriation suit. Indeed, that amount is determined only after the court is satisfied with the propriety of the expropriation.

Republic of the Philippines v. Zurbano: condemnation proceedings are within the jurisdiction of Courts of First Instance, the forerunners of the regional trial courts

CFI had original jurisdiction over all civil actions in which the subject of the litigation is not capable of pecuniary estimation. The 1997 amendments to the Rules of Court were not intended to change these jurisprudential precedents.

(2) Section 2 of Rule 67 of the 1997 Rules of Civil Procedure governs the requirements for the

CONSTI II DIGESTS: 1D COMPILATION

CHUA, ALIMANGOHAN, CONTRERAS, BRIONES, LENCIO, RELOJO, TENGCO, LIM, DUMA, DANAO, MELLA, TONGSON

($
Court of Appeals, respondent City in its pleadings failed to show its compliance with the law. The Court of Appeals was likewise silent on this specific jurisdictional issue. This is a clear violation of the right to due process of the petitioners. SEC 9: ELEMENTS OF TAKING Republic v. Vda. De Castellvi (1974) Facts: The Republic, through the AFP, leased the land of Castellvi and occupied such land since 1947. In the contract, it was stated that lease was on a yearly basis and that the AFP were to pay rent. They filed a complaint for eminent domain on 1959. The Republic contends that the taking of Castellvis property should be deemed as of the year 1947 by virtue of the lease agreement.

"

Sec. 9. Priorities in the acquisition of Land Lands for socialized housing shall be acquired in the following order: (a) Those owned by the Government or any of its sub-divisions, instrumentalities, or agencies, including government-owned or controlled corporations and their subsidiaries. b) Alienable lands of the public domain; (c) Unregistered or abandoned and idle lands; (d) Those within the declared Areas of Priority Development, Zonal Improvement sites, and Slum improvement and Resettlement Program sites which have not yet been acquired; (e) Bagong Lipunan Improvement sites and Services or BLISS sites which have not yet been acquired; and (f) Privately-owned lands. Where on-site development is found more practicable and advantageous to the beneficiaries, the priorities mentioned in this section shall not apply. The local government units shall give budgetary priority to on-site development of government lands

Republics Contention: although the contract was one of lease on a year to year basis, it was in reality a permanent right to occupy premises under the guise of lease with right and privilege to buy the property should the lessor wish to terminate the lessee
Issue: Whether or not the taking of Castellvis property for purposes of eminent domain cannot be considered to have taken place in 1947 when the Republic occupied the property as lessee Held: The court held that the taking must be reckoned as of June 1959, when the complaint for eminent domain was filed. Ratio: Elements of taking property for purposes of eminent domain # Must enter a private property X Entrance into private property must be for more than a momentary period # Entry should be under warrant or color of legal authority # Property must be devoted to a public use or otherwise informally appropriated or injuriously affected X Utilization must be in such a way as to oust the owner and deprive him of all beneficial enjoyment of the property 1. 2. Must enter a private property ! Republic, through AFP, took possession of the property by virtue of lease agreement Entrance into private property must be for more than a momentary period ! Momentary: lasting a very short time; limited period which is not indefinite or permanent; ! Lease contract was for a period of one year, renewable from year to year o Entry is therefore temporary and considered transitory o The fact that the republic constructed some installations of a permanent nature does not alter the fact that the entry into the land was transitory ! Republic undertook to return the premises in substantially the same condition as the time the property was first occupied by the AFP ! intention cannot prevail over the clear and express terms of the lease contract o Intent is to be deduced from the language employed by the parties, and the terms of the contract

Sec. 10. Modes of Land Acquisition. The modes of acquiring lands for purposes of this Act shall include, among others, community mortgage, land swapping, land assembly or consolidation, land banking, donation to the Government, joint venture agreement, negotiated purchase, and expropriation: Provided, however, That expropriation shall be resorted to only when other modes of acquisition have been exhausted: Provided further, That where expropriation is resorted to, parcels of land owned by small property owners shall be exempted for purposes of this Act: Provided, finally, that abandoned property, as herein defined, shall be reverted and escheated to the State in a proceeding analogous to the procedure laid down in Rule 91 of the Rules of Court. In Filstream vs. Court of Appeals we held that the above-quoted provisions are limitations to the exercise of the power of eminent domain, specially with respect to the order of priority in acquiring private lands and in resorting to expropriation proceedings as a means to acquire the same. Private lands rank last in the order of priority for purposes of socialized housing. In the same vein, expropriation proceedings are to be resorted to only after the other modes of acquisition have been exhausted. Compliance with these conditions is mandatory because these are the only safeguards of oftentimes-helpless owners of private property against violation of due process when their property is forcibly taken from them for public use. We find that herein respondent City failed to prove strict compliance with the requirements of Sections 9 and 10 of RA 7279. Respondent City neither alleged in its complaint nor proved during the proceedings before the trial court that it complied with said requirements. Even in the

CONSTI II DIGESTS: 1D COMPILATION

CHUA, ALIMANGOHAN, CONTRERAS, BRIONES, LENCIO, RELOJO, TENGCO, LIM, DUMA, DANAO, MELLA, TONGSON

(%
and safety, promote prosperity, improve the morals, peace, good order, comfort and convenience of city and inhabitants. Respondents arguments: It is a taking and confiscation of property as it permanently restricts use of property and that it deprives owner of all beneficial use of property General welfare clause is not applicable since if an owner is deprived of property outright, it is not taken for public use but is urgently and summarily destroyed in order to promote general welfare. Example: nuisance case per se and destruction of house to prevent conflagration. Issue: Whether or not Section 9 of the ordinance is a valid exercise of police power. Held: No, it is not a mere police regulation but an outright confiscation. It deprives a person of his private property without due process of law, nay, even without compensation. Ratio: RA 537, Charter of Quezon City does not reveal any provision that would justify the ordinance except the provision granting police power. Section 9 cannot also be justified under power to tax, fix license fees and regulate businesses. The power to regulate does not include in it the power to prohibit. o The ordinance not only confiscates but also prohibits operation of a memorial park cemetery because provision is coupled with a penal provision and a confiscatory clause. Police power is the power of promoting the public welfare by restraining and regulating the use of liberty and property. It is usually exerted in order to merely regulate the use and enjoyment of property of the owner. If he is deprived of the property outright, it is not taken for public use but rather to destroy in order to promote general welfare. Example: confiscation of an illegally possessed article like opium and firearms. o The ordinance is not a mere regulation but an outright confiscation. There is no reasonable relation between setting aside 6% of the total area of private cemeteries and the general welfare clause. The ordinance is actually a taking without compensation from a private cemetery to benefit paupers. Instead of building or maintaining a public cemetery, city passes the burden to private cemeteries.

"

3. 4. 5.

It might have been the intention to expropriate but mere notice of such intention to expropriate did not, and could not, bind the owner nor the land itself ! Expropriation must be actually commenced in court Entry should be under warrant or color of legal authority ! Republic entered as lessee Property must be devoted to a public use or otherwise informally appropriated or injuriously affected ! Property was used by AFP Utilization must be in such a way as to oust the owner and deprive him of all beneficial enjoyment of the property ! Did not oust Castellvu and deprive her of all beneficial enjoyment of the property o She remained the owner o Republic was bound to pay her monthly rentals until the time when it filed the complaint for eminent domain o

On Republics contention o o o o a lease made for a determinate time ceases upon the day fixed without need of a demand neither can it be said that the right of eminent domain may be exercised by simply leasing the premises to be expropriated Republics claim that it had the right and privilege to buy the property nowhere appears in the lease contract Allowing otherwise would sanction what obviously is a deceptive scheme, which would have the effect of depriving the owner of the property of its true and fair market value (Scheme pertained to: leasing, then after several years filling suit for eminent domain then claiming that lease can be taken as buying the property) Fair value in the contract (which the Republic mistakenly referred to as the selling price) refers actually to the cost of restoring the property in the same condition as of the time when the lessee took possession of the property

City Government vs Judge Ericta Facts: QC Council enacted Ordinance no. 6118 which regulates the private memorial cemetery or memorial ground. Its Section 9 provides that 6 % of total area of the memorial park cemetery be set aside for charity burial of paupers who have been residents of QC for at least 5 yrs. Only 7 years after the enactment of the ordinance that it was imposed. And it passed a resolution requesting the city engineer to stop any further selling of memorial park lots whose owners failed to donate 6% for paupers burial. City engineer notified Himlayang Pilipino, Inc., who in turn filed a case seeking to annul Section 9 of the ordinance. The lower court declared said section null and void. Hence this petition. Petitioners arguments: It is a valid and reasonable exercise of police power and that land is taken for public use since it is for the burial of paupers. It is authorized under its charter, in the exercise of local police power, to make ordinances and resolutions... as it shall deem necessary and proper to provide for health

US v. Causby Facts: Respondents own 2.8 acres of land near the airport of North Carolina. They had a house and other buildings for raising chickens. Near them was the path of the runway, used by the US government under a lease contract. (this was during WW2, so US fighter planes and bombers use the airport). Sometimes, the fighter planes would come close enough to miss the top of the trees. The noise is startling. At night, the glare from the planes would brighten the place. As a result the respondents had to give up their chicken business. 150 chickens were lost.

CONSTI II DIGESTS: 1D COMPILATION

CHUA, ALIMANGOHAN, CONTRERAS, BRIONES, LENCIO, RELOJO, TENGCO, LIM, DUMA, DANAO, MELLA, TONGSON

(&
But since it is not clear yet whether the taking is permanent or temporary, then the case is remanded to the lower court before the amount of Just Compensation is determined.

"

Thus the property depreciated in value and. The US relies on the Air Commerce Act of 1926, claiming that the US has complete and exclusive national sovereignty over the air space in the country. It claims that navigable air space is the airspace above the minimum space altitudes of flight prescribed by the Civil Aeronautics Authority. It is argued that such navigable airspace is subject to right of interstate and foreign air navigation. Now the US claims that these flights were an exercise of this right to travel in airspace. The US claims that the flight are made within the navigable airspace without any physical invasion of the property of landowners. There has been no taking of property but merely incidental damage resulting from authorized air navigation. It is also argued that landowner does not own the airspace above.

People v. Fajardo (1958) Facts: Defendants-appellants Juan F. Fajardo and Pedro Babilonia were convicted for violating Ordinance No. 7, Series of 1950, of the Municipality of Baao, Camarines Sur, for having constructed without a permit from the municipal mayor a building that destroys the view of the public plaza. Issue: Whether or not the ordinance amounts to taking of private property without just compensation Held: Yes Ratio: 1) Even if the property was not directly confiscated by the local government, the appellants are constrained to let their land remain idle and unused for such public purpose (aesthetic appearance of the community).

Issue: WON eminent domain was exercised Held: Yes. Ratio: The flights over the property rendered it inhabitable, thus there is compensable taking of private property under the 5th Am. It is the owners loss, not the takers gain, which is the measure of the value of the property taken. Because of the frequency and altitude of the flights, the respondents could not use their land for any purpose, therefore, their loss would be complete. It is as complete as if the US had entered upon the surface of the land and taken exclusive possession thereof. The land is appropriated as if it were used for the runways themselves. THUS there would be taking. Though it would only be an easement of flight which was taken, that easement, if permanent and not merely temporary normally would be equivalent to a fee. It would be a definite exercise of complete dominion and control over the surface of the land. The fact that the planes never touched the land would be irrelevant because the owners right to possess and exploit the land (his beneficial ownership) would be destroyed. If the landowner is to have full enjoyment of the land, he must have exclusive control of the immediate reaches of the enveloping atmosphere. Otherwise, buildings could not be erected, trees could not be planted, and fences could not be run. The fact that he does not occupy it in a physical sense (by the erection of buildings) is not material. The flight of the airplanes, which skims the surface although not touch it, is as much as an appropriation of the use of the land as a more conventional entry upon it. There is an intrusion so immediate and so direct as to subtract from the owners full enjoyment of the property. Thus, the adjacent airspace at this low altitude is so close to the land that invasions of it affects the use of the land itself. This invasion is the same as the invasion of the surface. It is the character of the invasion, and not the amount of damage resulting, so long as the damage is substantial, that determines whether there is a taking. In short, flights over private lands are generally not a taking, unless they are so low and so frequent as to be a direct and immediate interference to the enjoyment and use of land.

Zoning which admittedly limits property to a use which can not reasonably be made of it cannot be said to set aside such property to a use but constitutes the taking of such property without just compensation. o Use of property is an element of ownership.

2) Circumstances that constitute taking are present. The expropriator must enter upon the private property. o By virtue of the said ordinance, the mayor deprives the appellants of ownership as they are not allowed to erect a building that destroys the view of the public plaza. The entrance must be permanent o The prohibition imposed on the defendant-appellants is not temporary in nature. The entry must be under warrant or color of legal authority o The entry was warranted by the said ordinance/regulation. The property must be devoted to public use o The ordinance prohibiting the erection of building that destroys the view of the public plaza was made to preserve aesthetic appearance of the community. The utilization of property must be in such a way as to oust the owner and deprive him of all beneficial enjoyment of the property o The regulation substantially deprives the owner of all beneficial use of his property as it constrains him to let it remain idle and unused.

CONSTI II DIGESTS: 1D COMPILATION

CHUA, ALIMANGOHAN, CONTRERAS, BRIONES, LENCIO, RELOJO, TENGCO, LIM, DUMA, DANAO, MELLA, TONGSON

('
o o o What is to be deposited is an amount equivalent to the assessed value for taxation purposes; No hearing is required for that purpose; and All that is needed is notice to the owner of the property sought to be condemned.

"

Republic vs. PLDT Facts: PLDT and the Bureau of Telecommunications (BOT) entered into an agreement whereby BOT was allowed to use trunklines of PLDT to enable government offices to call private parties. PLDT prohibited the public use the services. This condition was violated by the BOT when it allowed trunklines to serve the general public in competition with PLDT. After prior warnings, PLDT cut off the lines of BOT which resulted in the telephone isolation of the Philippines to the world except the US. The BOT prayed that PLDT be commanded to execute a contract with it. Issue: WON PLDT can be commanded to enter into contract with BOT. Held: YES. Ratio: The constitutional guarantee of freedom of contract forbids compulsion. However, the government may, in the exercise of the sovereign power of eminent domain, require PLDT to permit interconnection of the government telephone system (GTS) and PLDT, as the needs of the government service may require, subject to the payment of compensation to be determined by the court. Eminent domain may be availed to impose only a burden upon the owner of the condemned property without the loss of title or possession. Real property may be expropriated without said loss through a right of way easement.

Didipio Earth Savers v. Secretary (2006) Facts: Petitioners assail the constitutionality of the Philippine Mining Act and its implementing rules and regulations, the DENR Administrative Order pursuant thereto, and the Financial and Technical Assistance Agreement (FTAA) with Arimco Mining Corporation (a corporation established under the laws of Australia).

Petitioners Arguments:
1. The assailed acts and implementing rules and regulations allow the unlawful and unjust taking of private property for private purpose in contradiction with Sec 9 of the Bill of Rights. The concept of taking as in the case of Republic v. Vda de Castellvi is present. Requisites of Taking (Castellvi) Instant Case 1. Entry into a private property Entry by Amrico pursuant to the FTAA 2. Entry must be for more than a FTAA provides a period of 25 years, momentary period renewable for another 25 years 3. Entry under the warrant of legal authority Pursuant to the FTAA and executed between the government and Amrico 4. The utilization must substantially oust the owner or possessor and deprives him of all beneficial enjoyment of the property 5. The property must be devoted to a public The exercise of the power of eminent use or otherwise informally appropriated or domain is unwarranted because injuriously affected respondents failed to prove that the entry is devoted for public use Even without the Castellvi doctrine on taking, (1) the nature of the mining activity, (2) extent of the land area covered, and (3) the various rights granted result in a taking of private property. Assuming arguendo that there is no absolute physical taking, Sec 76 of the Mining Act establishes a legal easement sufficient to deprive surface owners, concessionaires and occupants of mining contract areas. The easement of right-of-way imposed against land use for an indefinite period is a taking under the power of eminent domain (NAPOCOR v. Gutierrez). The act and DENR Administrative Order 96-40 encroach on the power of trial courts to determine just compensation in eminent domain cases as the same determination are cognizable only by the Panel of Arbitrators.

Facts:

NPC v. Jocson (1992)

(NPC, GOCC that was created for the purpose of constructing, maintaining and developing power plants, filed 7 eminent domain cases before the RTC, Bacolod City for the acquisition of a right-ofway easement for its Negros-Panay-Interconnection Project. Respondent Judge Jocson issued an order fixing the provisional values of the land areas, based no their market value and ordered NPC to deposit amount with the Phil. National Bank.)
Issue: WON the respondent Judge acted with gross amount of discretion in issuing provisional value of the land. Held: YES! He acted with gross amount of discretion thus violating Presidential Decree No. 42. Because upon the filing of the complaint [for expropriation] or at any time thereafter, the petitioner has the right to take or enter upon the possession of the property involved upon compliance with P.D. No. 42 which requires the petitioner to deposit with the PNB an amount equivalent to the assessed value of the as indicated in the tax declaration, after due notice to the defendant,. Ratio: PD 42 has repealed Section 2 Rule 67 of the Rules of Court. The former removes the discretion of the court determination of the provisional value and repealed the form of payment and the agency with which the deposit shall be made. Under PD 42: 2. 3. 4. 5.

Respondents Arguments:

CONSTI II DIGESTS: 1D COMPILATION

CHUA, ALIMANGOHAN, CONTRERAS, BRIONES, LENCIO, RELOJO, TENGCO, LIM, DUMA, DANAO, MELLA, TONGSON

((

"

1. 2.

Sec 76 is not a taking provision but a valid exercise of police power to curtail economic exploitation. To require compensation in all such circumstances would compel the government to regulate by purchase. By entering private lands, FTAA holders do not oust nor deprive the owners of beneficial enjoyment. Said entry merely established legal easement.

Issues: 1. 2. WON the assailed act and FTAA are void because they allow the unjust and unlawful taking of property without payment of just compensation WON the act and its implementing rules and regulations are void and unconstitutional for sanctioning an unconstitutional administrative process of determining just compensation

NAPOCOR v. Gutierrez: The easement of right-of-way was considered taking. The installation of 230 transmission lines would impose a limit against the use of land for an indefinite period therefore depriving private respondents of its ordinary use. Republic v. PLDT: Where the Bureao of Telecommunications demanded to expand the use of the lines and facilities of PLDT, the Court said that real property through expropriation, may be subjected to an easement right of way.

Held: NO & NO. Ratio: The Court first discussed the Distinctions between Regulation in Police Power & Taking in Eminent Domain Power of Eminent Domain Police Power General welfare for their object Condemning private property to public use Power of the state to promote public welfare upon payment of just compensation by restraining and regulating the use of liberty & property Property interest is restricted, because it is Property interest is appropriated and applied usually noxious or intended for a noxious to some public purpose, thus there is purpose, thus there is no compensable taking. compensable taking. None of the property rights is appropriated for Somebody else acquires the use or interest of use by or for the benefit of the public. property. It includes not only (1) appropriation of title to or possession, but also (2) trespass without actual eviction, (3) material impairment of the propertys value, or (4) prevention of the ordinary uses THERE IS TAKING Right of possession, full right of ingress and egress, right to occupy, right not to be prevented from entry into private lands are all granted to Amrico in the FTAA. The entry in Sec 76 of the Mining Act is not a simple righty-of-way but they are allowed to conduct mining activities, and build mine infrastructure. The easement rights in Sec 75 allow them to build other infrastructure for mining purposes. All these will definitely oust the owners. The entry and easement rights in the Mining Act and the various rights are no different from the deprivation of proprietary rights as shown in the following cases:

Sec 27 of the Commonwealth Act 137 and Sec 12 of PD 463 were amended by Sec 1 of PD 512, which provided that the power of eminent domain may be invoked by mining operators for the entry as opposed to the previous systems wherein said power may only be exercised. Considering that Sec 76 of the assailed Mining Act does not expressly repeal Sec 1 of PD 512, the latter is deemed incorporated in the former, thus Sec 76 is a taking provision. The taking under Sec 76 is for a public purpose considering that mining is an industry which is of public benefit. Heirs of Juancho Ardona v Reyes: Promotion of tourism allowing private concessionaires to maintain facilities is for public use. That public use is negated by the taking of private properties for the benefit of private mining firms is not true. The means of executing, which may may be through the use of private enterprise, is for the Congress to determine once the public purpose has been established.

THE ASSAILED ACT PROVIDES FOR JUST COMPENSATION Sec 76 of the Mining Act, and Sec 107 of assailed the DENR AO provide that any damage to property shall be properly and justly compensated. THE COURTS ARE NOT EXCLUDED FROM THEIR JURISDICTION TO DETERMINE JUST COMPENSATION IN EXPROPRIATION PROCEEDINGS

Export Processing Zone Authority v. Dulay: The determination of just compensation in eminent domain is a judicial function. Phil Veterans Bank v CA: The original and exclusive jurisdiction of the courts to decide determination of just compensation remains intact despite the preliminary determination made by the administrative agency.
The disposition of cases where surface owners refuse the permit holders entry by the Panel of Arbitrators pursuant to Sec 105 of DENR AO 96-40 is merely preliminary.

Sec 107 provides that compensation to the damage done shall be compensated based on the agreement between the holder of mining rights and the surface owner, but the matter shall be brought to the Panel of Arbitrators in the absence of such agreement. The provision points to a voluntary sale or transaction, not to an involuntary sale. It contemplates a situation wherein permit holder are allowed entry but disagreement ensues regarding the compensation. Sec 206 of the assailed DENR AO provides that an aggrieved party may appeal the decision of the Panel of Arbitrators to the Mines Adjudication Board whose decision may be reviewed by the SC.

Republic v. Castellvi: Requisites of taking in eminent domain (SEE TABLE ABOVE) City Gov of QC v Ericta: An ordinance requiring private cemeteries to reserve 6% of their areas for the burial of paupers is taking without just compensation. Peo v. Fajardo: Pursuant to a municipal ordinance, prohibiting of the construction of any building that would destroy the plaza view is taking without just compensation. Ayala de Roxas v City of Manila: Imposition of easement over private property is considered taking hence just compensation is required.

CONSTI II DIGESTS: 1D COMPILATION

CHUA, ALIMANGOHAN, CONTRERAS, BRIONES, LENCIO, RELOJO, TENGCO, LIM, DUMA, DANAO, MELLA, TONGSON

()
evasions by dispensing with the necessity of judicial inquiry as a collusion between the wrongdoer and the alleged innocent owner.

"

Bennis v. Michigan Facts: Detroit police arrested John Bennis after observing him engaged in sexual act with a prostitute in the automobile while it was parked on a Detroit city street. Bennis was convicted of gross indecency. The State then sued both Dennis and his wife, petitioner Tine B. Bennis, to have the car declared public nuisance and abated as such under 600.3801 and 600.3825 of Michigans Compiled Laws. The Wayne County Circuit Court rejected this argument, declared the car a public nuisance, and order the cars abatement. The Michigan Court of Appeals reversed, holding that regardless of the language of the Michigan Compiled Law 600.3815(2), Michigan Supreme Court precedent interpreting this section prevented the State from abating petitioners interest absent proof that she knew to what end the car would be used. The Michigan Supreme Court reversed the Court of Appeals and reinstated the abatement in its entirety.

Facts:

Penn Central Transportation Co. v New York City (1978)

Petitioners Argument: Petitioner defended against the abatement of her interest in the car on the ground that, when she entrusted her husband to use the car, she did not know that he would use it to violate Michigans indecency law.
Issue: Whether Michigans abatement scheme has deprived petitioner of her interest in the forfeited car without due process, in the violation of the Fourteenth Amendment. Or has taken her interest for public use without compensation, in violation of the Fifth Amendment as incorporated by the Fourteenth Amendment. Held: We hold that the Michigan court order did not offend the Due Process Clause of the Fourteenth Amendment or the takings Clause of the Fifth Amendment. Ratio: In Dobbinss Distillery v. United States, even when the owner is otherwise without fault and it was always been held that the acts of the [possessors] bind the interest of the owner whether he be innocent or guilty. In Van Oster v Kansas, 272 U.S. 465 (1926), It has long been settled that statutory forfeitures of property entrusted by the innocent owner or lienor to another who uses it in violation of the revenue law of the United States is not a violation of the due process clause of the Fifth Amendment. The innocence of the owner of the property subject to forfeiture has almost uniformly been rejected as a defense. She did not know that her car would be used in an illegal activity that would subject it to forfeiture. But under these cases the Due Process Clause of the Fourteenth Amendment does not protect her interest against forfeiture of the government. Forfeiture of property prevents illegal uses of both by preventing further illicit use of the [property] and by imposing an economic penalty, thereby rendering illegal behavior unprofitable. The law builds the secondary defense against a forbidden use and precludes

Under the New York Citys Landmarks Preservation Law (landmarks Law), the Landmarks Preservation Commission (Commission) may designate a building to be a landmark on a particular landmark site or may designate an area to be a historic district. The Board of Estimate may thereafter modify or disapprove the designation, and the owner may seek judicial review of the final designation decision. The owner of the designated landmark must keep the buildings exterior in good repair, and, before exterior alterations are made, must secure Commission approval. Under two ordinances, owners of landmark sites may transfer development rights from a landmark parcel proximate lots. Under the Landmarks Law, The Grand Central Terminal (Terminal), which is owned by the Penn Central Transportation Co. and its affiliates (Penn Central) was designated a landmark and the block it occupies a landmark site. Appellant Penn Central entered into a lease with appellant UGP Properties, whereby UGP was to construct a multistory office building over the Terminal. After the Commission had rejected appellants plans for the building as destructive of the Terminals historic and aesthetic features, with no judicial review thereafter being sought, appellants brought suit in state court. Trial courts grant of relief was reversed on appeal, New York Court of Appeals concluding that there was no taking, since the Landmark Law had not transferred control of the property to the city, but only restricted appellants exploitation of it; and that there was no denial of due process because (1) the same use of the Terminal was permitted as before; (2) appellants had not shown that they could not earn a reasonable return on their investment in the Terminal itself; (3) even if the Terminal proper could never operate at a reasonable profit, some of the income from Penns Central extensive real estate holdings in the area must realistically be imputed to the terminal; (4) the development rights above the terminal , which were made transferable to numerous sites in the vicinity, provided significant compensation for loss of rights above the Terminal itself.

Petitioners Argument:
Petitioners claim that the application of the Landmarks Law had taken their property without just compensation in violation of the Fourth and Fourteenth Amendments, and arbitrarily deprived them of their property without due process of law in violation of the Fourteenth Amendment Held: The application of the Landmarks Law to the Terminal property does not constitute a taking of appellants property within the meaning of the Fifth Amendment as made applicable to the States by the Fourteenth Amendment. The Landmarks Law, which does not interfere with the Terminals present uses or prevent Penn Central from realizing a reasonable return on its investment, does not impose drastic imitation on appellants ability to use the air rights above the terminal that appellants claim, for, on this record, there is no showing that a smaller, harmonizing structure would not be authorized.

CONSTI II DIGESTS: 1D COMPILATION

CHUA, ALIMANGOHAN, CONTRERAS, BRIONES, LENCIO, RELOJO, TENGCO, LIM, DUMA, DANAO, MELLA, TONGSON

(*
1. 2. Socialized housing as defined in P.D. 1224 is not public use since it will benefit only a handful of people. P.D. 1224 allows the taking of any private land regardless of the size and no matter how small the area of the land to be expropriated.

"

Ratio: In a wide variety of contexts, the government may execute laws or programs that adversely affect recognized economic values without its action constituting a taking, and, in instances such as zoning laws where a state tribunal has reasonably concluded that the health, safety, morals, or general welfare would be promoted by prohibiting particular contemplated uses of land, the Court has upheld land use regulations that destroyed or adversely affected real property interests. In deciding whether particular governmental action has affected a taking, the character of the action and nature and extent of interference with property rights are focused upon, rather than discrete segments thereof. Consequently, appellants cannot establish a taking simply by showing that they have been denied the ability to exploit the superjacent airspace, irrespective of the remainder of appellants parcel. Though diminution in property value alone cannot establish a taking, as appellants concede, they urge that the regulation of individual landmarks is different, because it applies only to selected properties. Nor can it successfully contended that designation of a landmark involves only a matter of taste, and therefore will inevitably lead to arbitrary results, for judicial review is available, and there is no reason to believe it will be less effective than would be so on the case of zoning or any other context. That the Landmark Law affects some landowners ore severely than others does not, itself, result in taking, for that is often the case with general welfare and zoning legislation. Nor, contrary to appellants contention, are they solely burdened by the Landmarks Law, which has been extensively applied and enacted on the basis of the legislative judgment that the preservation of landmarks benefits the citizenry both economically and by improving the overall quality of city life. The Landmarks Law no more effects an appropriation of the airspace above the terminal of governmental uses than would a zoning law appropriate property; it simply prohibits appellants or others from occupying certain features of that space while allowing appellants gainfully to use the remainder of the parcel. SEC 9: PUBLIC USE Sumulong v. Guerrero (1987) Public Purpose includes indirect public benefit/ State has broad discretion to designate the property to be taken and to determine the area and size covered Facts: The National Housing Authority (NHA) filed a complaint for expropriation of parcels of land covering 25 hectares in Antipolo, Rizal. The expropriation included the lots of petitioners Sumulong and Baloing covering and area of 6667 sq. m. and 3333 sq. m., respectively. The NHA valued the land at P1.00 per square meter in accordance with the valuation by the provincial assessor. The NHA filed the expropriation proceedings pursuant to P.D. 1224 entitled Policy for the Expropriation of Private Property for Socialized Housing Upon Payment of Just Compensation

Issue: W/N P.D. 1224 is constitutional. Ruling: P.D. 1224 is constitutional. Ratio: 1. Petitioner Argument 1: Socialized housing as defined in P.D. 1224 is not public use since it will benefit only a handful of people. Court: The term "public use" has acquired a more comprehensive coverage. The term now includes the broader notion of indirect public benefit or advantage.

(*The old rule favored a literal interpretation of public use. Public use meant direct benefit to the public. This is no longer the accepted interpretation.)
Urban renewal or redevelopment, and the construction of low-cost housing, i.e. socialized housing, is recognized as a public purpose. It is made so by the Constitution (see Article II, Sec. 9 and Article XIII, Sec. 9) and it falls within the confines of the expanded notion of public use. In the case at bar, the use to which the land is proposed to be put meets the requisites of "public use". The lands in question are being expropriated by the NHA for the expansion of Bagong Nayon Housing Project to provide housing facilities to low-salaried government employees. Housing is a basic human need. Shortage in housing is a matter of state concern since it directly and significantly affects public health, safety, the environment and in sum, the general welfare. The public character of housing measures does not change because units in housing projects cannot be occupied by all but only by those who satisfy prescribed qualifications.

*The fact that not all of the public but only some beneficiaries may avail thereof does not deprive the expropriation of its public character.
2. Petitioner Argument 2 :P.D. 1224 allows the taking of any private land regardless of the size and no matter how small the area of the land to be expropriated. Petitioners explain their argument thus: "there are vast areas of lands in Mayamot, Cupang, and San Isidro, Antipolo, Rizal hundred of hectares of which are owned by a few landowners only. It is surprising [therefore] why respondent National Housing Authority [would] include [their] two lots ..."

Petitioners challenge the constitutionality of P.D. 1224 on the grounds that:

CONSTI II DIGESTS: 1D COMPILATION

CHUA, ALIMANGOHAN, CONTRERAS, BRIONES, LENCIO, RELOJO, TENGCO, LIM, DUMA, DANAO, MELLA, TONGSON

)+
1. Its exercise of eminent domain cannot be restricted by the provisions of the Comprehensive Agrarian Reform Law which requires the approval of the DAR before a parcel of land can be reclassified from an agricultural to a non-agricultural land.

"

Court: The propriety of exercising the power of eminent domain under Article XIII, section 4 of our Constitution cannot be determined on a purely quantitative or area basis. Not only does the constitutional provision speak of lands instead of landed estates, but there is no cogent reason why the government, in its quest for social justice and peace, should exclusively devote attention to conflicts of large proportions, involving a considerable number of individuals, and eschew small controversies and wait until they grow into a major problem before taking remedial action. The State acting through the NHA is vested with broad discretion to designate the particular property/properties to be taken for socialized housing purposes and how much thereof may be expropriated. Absent a clear showing of fraud, bad faith, or gross abuse of discretion, which petitioners herein failed to demonstrate, the Court will give due weight to and leave undisturbed the NHA's choice and the size of the site for the project. The property owner may not interpose objections merely because in their judgment some other property would have been more suitable, or just as suitable, for the purpose. The right to the use, enjoyment and disposal of private property is tempered by and has to yield to the demands of the common good.

Issue: WON the approval of the Department of Agrarian Reform is needed before expropriation could take place. Held: No. Ratio: The expropriation of the property authorized by the resolution is for a public purpose. The establishment of a pilot development center would inure to the direct benefit and advantage of the people of the Province of Camarines Sur. Once operational, the center would make available to the community information and technology on agriculture, fishery and the cottage industry. The housing project also satisfies the public purpose requirement of the Constitution. In the case of Heirs of Juancho Ardona, petitioners raised the issue of whether the Philippine Tourism Authority can expropriate lands covered by the Operation Land Transfer for use of a tourist resort complex. There was a finding that of the 282 hectares sought to be expropriated, only an area of 8, 970 square meters or less than 1 hectare was affected by the land reform program and covered by emancipation patents issued by the Ministry of Agrarian Reform. The court upheld the expropriation after noting that petitioners had failed to overcome the showing that the taking of 8,970 square meters formed part of the resort complex. The court viewed the power of expropriation as superior to the power to distribute lands under the land reform program. It is true that LGUs have no inherent power of eminent domain and can only exercise it when authorized by the Legislature. But in delegating the power to expropriate, the legislature may retain certain control on the exercise thereof by the local govts. While such delegated power may be a limited authority, it is complete within its limits. The limitations on the exercise of the delegated power must be clearly expressed, either in the law conferring the power or in other legislations. Section 9 of the Local Govt Code does not intimate in the least that LGUs must first secure the approval of the DAR for the conversion of lands from agricultural to nonagricultural use before they can institute the necessary expropriation proceedings. There is no provision in the Comprehensive Agrarian Reform Law which expressly subjects the expropriation of agricultural lands by LGUs to the control of the DAR. To sustain the CA would mean that LGUs can no longer expropriate agricultural lands needed for the construction of roads, bridges, schools, hospitals etc. without first applying for conversion of the use of the lands with the DAR because all these projects would naturally involve a change in the land use. In effect, it would then be the DAR to scrutinize whether the expropriation is for a public purpose or public use.

---------------------------------------*Note: (not in case book) While P.D. 1224 was upheld as constitutional the expropriation was invalidated for violating due process.
Province of Camarines Sur vs. CA (1993)

Facts:

The Sangguniang Panlalawigan of the Province of Camarines Sur passed Resolution No, 129, Series of 1988, authorizing the Provincial Governor to purchase or expropriate property contiguous to the provincial capitol site in order to establish a pilot farm for non-food and nontraditional agricultural crops and a housing project for provincial govt employees. Thus, Governor Villafuerte filed 2 separate cases for expropriation against Ernesto San Joaquin and Efren San Joaquin. The Solicitor General stated that under Section 9 of the Local Govt Code, there was no need for the approval by the Office of the President of the exercise by the Sangguniang Panlalawian of the right of eminent domain. But, he expressed the view that the Province of Camarines Sur must first secure the approval of the Department of Agrarian Reform of the plan to expropriate the lands of petitioners for use as a housing project. The CA allowed the Province of Camarines Sur to take possession of the lands. It also order ordered the trial court to suspend the expropriation proceedings until after the Province of Camarines Sur shall have submitted the requisite approval of the Department of Agrarian Reform to convert the classification of the property of the private respondents from agricultural to nonagricultural land. Hence, this petition. Petitioners arguments:

Masikip v City of Pasig (2006) Facts: Respondent notified petitioner Lourdes Dela Paz Masikip of its intention to expropriate a 1,500 sq.m. portion of her property to be used for sports development and recreational activities of

CONSTI II DIGESTS: 1D COMPILATION

CHUA, ALIMANGOHAN, CONTRERAS, BRIONES, LENCIO, RELOJO, TENGCO, LIM, DUMA, DANAO, MELLA, TONGSON

)!
Ascertainment of necessity must precede of accompany and not follow the taking of the land. There exists an alternative facility for sports development and recreation in the area, the Rainforest park, available to all residents of Pasig

"

the residents of Bgy Caniogan. This was pursuant to Ordinance No. 42, Series of 1993 enacted by the Sangguniang Bayan ng Pasig. Respondent wrote petitioner another letter, this time the purpose was allegedly in line with the program of the Municipal Government to provide land opportunities to deserving poor sectors of the community. Petitioner sent a reply stating that the intended expropriation is unconstitutional, invalid, and oppressive, as the area of her lot is neither sufficient nor suitable to provide land opportunities to deserving poor sectors of our community. Respondents reiterated that the purpose is to provide sports and recreational facilities to its poor residents. Issue: Whether or not the intended expropriation is valid. Held: NO. The purpose is not categorically public. The necessity of public character was also not established. Ratio: Local governments have no inherent power of eminent domain and may exercise it only when expressly authorized by statute. o Sec 19 of the Local Government Code of 1991 (RA 7160) prescribes delegation by Congress of the power of eminent domain to LGUs and lays down the parameters for this exercise. Judicial review of the exercise of eminent domain is limited to the following areas of concern: a) Adequacy of the compensation b) Necessity of the taking c) Public use character of the purpose of the taken

***Note: Determination of whether there is genuine necessity for the exercise of eminent domain is justiciable question. However, when the power is exercised by the Legislature, the question of necessity is essentially a political question.

Mactan Cebu Intl Airport v. Lozada Jr FACTS: Petitioners initiated a complaint for the recovery of possession and reconveyance of ownership of Lot No. 88. Lot No. 88 in Lahug, Cebu was subject to expropriation proceedings for the expansion and improvement of the Lahug Airport. On 1961, the trial court rendered judgment in favor of the Republic and ordered the latter to pay Lozada the fair market value. Lozada received the amount of P3,018.00 by way of payment. The affected landowners appealed. During the pendency of the appeal, the parties entered into a compromise settlement that the subject property would be resold to the original owner at the same price when it was expropriated in the event that the Government abandons the Lahug Airport. The projected improvement and expansion plan of the old Lahug Airport, however, was not pursued. Lozada then requested to repurchase the lots, as per previous agreement. On 1989, then President Corazon C. Aquino, through a Memorandum to the Department of Transportation and Communications (DOTC), directed the transfer of general aviation operations at the Lahug Airport to the Mactan-Cebu International Airport Authority. On 1999, the RTC rendered its Decision in favour of plaintiffs. Aggrieved, petitioners interposed an appeal to the CA. CA affirmed RTC decision. ISSUE: (1) WON respondents utterly failed to prove that there was a repurchase agreement or compromise settlement between them and the Government (2) WON said lot should be reconveyed to respondents HELD: (1) (2) NO. A compromise agreement was, in fact, entered into between the Government and respondents, with the former undertaking to resell Lot No. 88 to the latter if the improvement and expansion of the Lahug Airport would not be pursued YES.

1) The purpose is not categorically public Certification issued by the Caniogan Bgy Council, the basis for the passage of Ordinance 42, authorizing the expropriation, indicates that the intended beneficiary is the Melendres Compound Homeowners Association (private/non-profit org), not the residents of Caniogan. The members of the association are just desirous of having their own private playground and recreational facility, and petitioners lot is the nearest vacant space available. 2) There is no genuine necessity to justify the expropriation The very foundation of the right to exercise eminent domain is a genuine necessity and that necessity must be of a public character. Necessity does not mean an absolute but only a reasonable or practical necessity such as would combine the greatest benefit to the public with the least inconvenience and expense to the condemning party and the property owner consistent with such benefit.

RATIO: (1) Petitioners anchor their claim to the controverted property on Fery v. Municipality of Cabanatuan which declared that the Government acquires only such rights in expropriated parcels of land as may be allowed by the character of its title over the properties

CONSTI II DIGESTS: 1D COMPILATION

CHUA, ALIMANGOHAN, CONTRERAS, BRIONES, LENCIO, RELOJO, TENGCO, LIM, DUMA, DANAO, MELLA, TONGSON

)#
- taking of private property, consequent to the Governments exercise of its power of eminent domain, is always subject to the condition that the property be devoted to the specific public purpose for which it was taken. If this particular purpose or intent is not initiated or not at all pursued, and is peremptorily abandoned, then the former owners, if they so desire, may seek the reversion of the property, subject to the return of the amount of just compensation received. (2) The right of respondents to repurchase Lot No. 88 may be enforced based on a constructive trust/implied trust constituted on the property held by the government in favor of the former. Art. 1454 of the Civil Code, If an absolute conveyance of property is made in order to secure the performance of an obligation of the grantor toward the grantee, a trust by virtue of law is established.If the fulfillment of the obligation is offered by the grantor when it becomes due, he may demand the reconveyance of the property to him. In the case at bar, petitioners conveyed Lots No. 916 and 920 to the government with the latter obliging itself to use the realties for the expansion of Lahug Airport; failing to keep its bargain, the government can be compelled by petitioners to reconvey the parcels of land to them, otherwise, petitioners would be denied the use of their properties upon a state of affairs that was not conceived nor contemplated when the expropriation was authorized. On the matter of the repurchase price, while petitioners are obliged to reconvey Lot No. 88 to respondents, the latter must return to the former what they received as just compensation for the expropriation of the property, plus legal interest to be computed from default, which in this case runs from the time petitioners comply with their obligation to respondents. SEC 9: JUST COMPENSATION EPZA v. DULAY Facts: On January 15, 1979, the President of the Philippines, issued Proclamation No. 1811, reserving a certain parcel of land of the public domain situated in the City of Lapu-Lapu, Island of Mactan, Cebu and covering a total area of 1,193,669 square meters, more or less, for the establishment of an export processing zone by petitioner Export Processing Zone Authority (EPZA). The San Antonio Development Corporation was the owner of a piece of land in Lapu-Lapu City which the EPZA expropriated in 1979. The commissioners appointed by the trial court recommended that the San Antonio Development Corp. be paid P15.00 per square meter. EPZA filed a petition for certiorari, arguing that under PD 1533 the compensation should be the fair and current market value declared by the owner or the market value determined by the assessor, whichever is lower. Issue: Whether or not Presidential Decrees Numbered 76, 464, 794 and 1533 have repealed and superseded Sections 5 to 8 of Rule 67 of the Revised Rules of Court, such that in determining the just compensation of property in an expropriation case, the only basis should be its market value as declared by the owner or as determined by the assessor, whichever is lower. Held:

"

If x x x land is expropriated for a particular purpose, with the condition that when that purpose is ended or abandoned the property shall return to its former owner, then, of course, when the purpose is terminated or abandoned the former owner reacquires the property so expropriated.If x x x land is expropriated for a public street and the expropriation is granted upon condition that the city can only use it for a public street, then, of course, when the city abandons its use as a public street, it returns to the former owner, unless there is some statutory provision to the contrary. x x x.If, upon the contrary, however, the decree of expropriation gives to the entity a fee simple title, then, of course, the land becomes the absolute property of the expropriator, whether it be the State, a province, or municipality, and in that case the non-user does not have the effect of defeating the title acquired by the expropriation proceedings. x x x. When land has been acquired for public use in fee simple, unconditionally , either by the exercise of eminent domain or by purchase, the former owner retains no right in the land, and the public use may be abandoned, or the land may be devoted to a different use, without any impairment of the estate or title acquired, or any reversion to the former owner. x x x. [8] Court had ruled otherwise in Heirs of Timoteo Moreno and Maria Rotea v. Mactan-Cebu International Airport Authority: The acquisition by the Republic of the expropriated lots was subject to the condition that the Lahug Airport would continue its operation.The condition not having materialized because the airport had been abandoned, the former owner should then be allowed to reacquire the expropriated property. Ruling in Fery must be revisited as it was not decided pursuant to our now sacredly held constitutional right that private property shall not be taken for public use without just compensation. Taking of private property by the Governments power of eminent domain is subject to two mandatory requirements: (1) it is for a particular public purpose (2) just compensation be paid to the property owner. These requirements partake of the nature of implied conditions that should be complied with to enable the condemnor to keep the property expropriated.

(Fery v. Municipality of Cabanatuan: An expropriation suit commenced upon parcels of land to be used as a site for a public market. Instead of putting up a public market, respondent Cabanatuan constructed residential houses for lease on the area. Claiming that the municipality lost its right to the property taken since it did not pursue its public purpose, petitioner Juan Fery, the former owner of the lots expropriated, sought to recover his properties. However, as he had admitted that, in 1915, respondent Cabanatuan acquired a fee simple title to the lands in question, judgment was rendered in favor of the municipality, following American jurisprudence.)
- the expropriator should commit to use the property pursuant to the purpose stated in the petition for expropriation filed, failing which, it should file another petition for the new purpose.If not, it is then incumbent upon the expropriator to return the said property to its private owner, if the latter desires to reacquire the same.Otherwise, the private property owner would be denied due process of law, and the judgment would violate the property owners right to justice, fairness, and equity.

CONSTI II DIGESTS: 1D COMPILATION

CHUA, ALIMANGOHAN, CONTRERAS, BRIONES, LENCIO, RELOJO, TENGCO, LIM, DUMA, DANAO, MELLA, TONGSON

)$
is the measure of the indemnity, not whatever gain would accrue to the expropriating entity.'

"

NO. The method of ascertaining just compensation under PD 1533 constitutes impermissible encroachment on judicial prerogatives. Although the court technically would still have the power to determine the just compensation for the property, following the decree, its task would be relegated to simply stating the lower value of the property as declared either by the owner or the assessor. Just compensation means the value of the property at the time of the taking. It means a fair and full equivalent for the loss sustained. All the facts as to the condition of the property and its surroundings, its improvements and capabilities should be considered. In this case, the tax declarations used as basis for the just compensation were made long before the declaration of martial law when the land was much cheaper. To peg the value of the lots on the basis of those documents which are outdated would be arbitrary and confiscatory. We are convinced and so rule that the trial court correctly stated that the valuation in the decree may only serve as a guiding principle or one of the factors in determining just compensation but it may not substitute the court's own judgment as to what amount should be awarded and how to arrive at such amount. A return to the earlier well-established doctrine, to our mind, is more in keeping with the principle that the judiciary should live up to its mission "by vitalizing and not denigrating constitutional rights." The doctrine we enunciated in National Housing Authority v. Reyes, supra, therefore, must necessarily be abandoned if we are to uphold this Court's role as the guardian of the fundamental rights guaranteed by the due process and equal protection clauses and as the final arbiter over transgressions committed against constitutional rights. Ratio: Various factors can come into play in the valuation of specific properties singled out for expropriation. The values given by provincial assessors are usually uniform for very wide areas covering several barrios or even an entire town with the exception of the poblacion. Individual differences are never taken into account. The value of land is based on such generalities as its possible cultivation for rice, corn, coconuts, or other crops. Very often land described as "cogonal" has been cultivated for generations. Buildings are described in terms of only two or three classes of building materials and estimates of areas are more often inaccurate than correct. Tax values can serve as guides but cannot be absolute substitutes for just compensation. Prior to the promulgation of P.D. Nos. 76, 464, 794 and 1533, this Court has interpreted the eminent domain provisions of the Constitution and established the meaning, under the fundametal law, of just compensation and who has the power to determine it. Thus, in the following cases, wherein the filing of the expropriation proceedings were all commenced prior to the promulgation of the aforementioned decrees, we laid down the doctrine onjust compensation:

Garcia v. Court of appeals: in estimating the market value, afl the capabilities of the property and all the uses to which it may be applied or for which it is adapted are to be considered and not merely the condition it is in the time and the use to which it is then applied by the owner. An the facts as to the condition of the property and its surroundings, its improvements and capabilities may be shown and considered in estimating its value. Republic v. Santos : According to section 8 of Rule 67, the court is not bound by the conunissioners' report. It may niake such order or render such judgment as shall secure to the plaintiff the property essential to the exercise of his right of condemnation, and to the defendant just compensation for the property expropriated. This Court may substitute its own estimate of the value as gathered from the record
However, the promulgation of the aforementioned decrees practically set aside the above and many other precedents hammered out in the course of evidence-laden, well argued, fully heard, studiously deliberated. and judiciously considered court proceedings. The decrees categorically and peremptorily limited the definition of just compensation thus:

P.D. No. 76: For purposes of just compensation in cases of private property acquired by the government for public use, the basis shall be the current and fair market value declared by the owner or administrator, or such market value as determined by the Assessor, whichever is lower. P.D. No. 464: Section 92. Basis for payment of just compensation in expropriation proceedings. In determining just compensation which private property is acquired by the government for public use, the basis shall be the market value declared by the owner or administrator or anyone having legal interest in the property, or such market value as determined by the assessor, whichever is lower. P.D. No. 794: Section 92. Basis for payment of just compensation in expropriation proceedings. In determining just compensation when private property is acquired by the government for public use, the same shall not exceed the market value declared by the owner or administrator or anyone having legal interest in the property, or such market value as determined by the assessor, whichever is lower. P.D. No. 1533:

Municipality of Daet v. Court of Appeals : well-settled (rule) that just compensation means the equivalent for the value of the property at the time of its taking. Anything beyond that is more and anything short of that is less, than just compensation. It means a fair and fun equivalent for the loss sustained, which

CONSTI II DIGESTS: 1D COMPILATION

CHUA, ALIMANGOHAN, CONTRERAS, BRIONES, LENCIO, RELOJO, TENGCO, LIM, DUMA, DANAO, MELLA, TONGSON

)%

"

Section 1. In determining just compensation for private property acquired through eminent domain proceedings, the compensation to be paid shall not exceed the value declared by the owner or administrator or anyone having legal interest in the property or determined by the assessor, pursuant to the Real Property Tax Code, whichever value is lower, prior to the recommendation or decision of the appropriate Government office to acquire the property.

CONSTI II DIGESTS: 1D COMPILATION

CHUA, ALIMANGOHAN, CONTRERAS, BRIONES, LENCIO, RELOJO, TENGCO, LIM, DUMA, DANAO, MELLA, TONGSON

)&
government by virtue of which contractual relations are worth while, a government which retains adequate authority to secure the peace and good order of society. The States retain adequate power to protect the public health against the maintenance nuisances despite insistence upon existing contracts. Legislation to protect the public safety comes within the same category of the reserved power.

"

SEC 10 Home Building and Loan Assn. v. Blaisdell (1934) Facts: The Act provides that, during the emergency declared to exist, relief may be had through authorized judicial proceedings with respect to foreclosures of mortgages, and execution sales, of real estate; that sales may be postpones and periods of redemption may be extended. The Supreme Court of Minnesota sustained statute, and the case comes here on appeal. The State court upheld the statute as an emergency measure. Although conceding that the obligations of the mortgage contract were impaired, the court decided that what it thus described as an impairment was, not withstanding the contract clause of the Federal Constitution, within the police power of the State as that power was called into exercise by the public economic emergency which the legislature had found to exist.

Facts: On August 20, 1941, Royal L. Rutter (Petitioner) sold to Placido J.Esteban (Respondent) two parcels of land situated in the City of Manila for the sum of P9,600. P4,800 was paid outright, and the balance of P4,800 was made payable as follows: P2,400 on or before August 7, 1942, and P2,400 on or before August 27, 1943, with interest at the rate of 7 percent per annum. To secure the payment of said balance of P4,800, a mortgage over the same parcels of land was constituted in favor of the plaintiff. Respondent failed to pay the two installments as agreed upon, as well as the interest that had accrued. On August 2, 1949, Royal L. Rutter instituted this action in the Court of First Instance of Manila to recover the balance due, the interest due thereon, and the attorney's fees stipulated in the contract. The complaint also contained a prayer for sale of the properties mortgaged. Respondent set up the defense of the moratorium clause embodied in Republic Act No. 342. Section 2 of Republic Act No. 342 provides that all debts and other monetary obligations contracted before December 8, 1941, shall not be due and demandable for a period of eight (8) years from and after settlement of the war damage claim of the debtor by the Philippine War Damage Commission. Section 3 of the said Act provides that should the provision of section 2 be declared void and unenforceable, then as regards the obligation affected thereby, the provisions of Executive Order No. 25 as amended by Executive Order No. 32, relative to debt moratorium, shall continue to be in force and effect.

Rutter v. Esteban (1953) Reserved Power of the State to impair Contracts/Requisites and Limitations

Petitioners Appeal: Appellant contests the validity of Chapter 399 of the Laws of Minnesota, called the Minnesota Mortgage Moratorium Law, as being repugnant to the contract clause (Art. I, 10) and the due process and equal protection clauses of the Fourteenth Amendment, of the Federal Constitution. Issue: Whether the provision for this temporary and conditional relief exceeds the power of the State by reason of the clause in the Federal Constitution by prohibiting impairment of the obligations of contracts. Decision of Supreme Court of Minnesota upheld. The obligation if a contract is the law which binds the parties to perform their agreement. The laws which subsist at the time and place of the making of a contract, and where it is to be performed, enter into and form a part of it, as if they were expressly referred to or incorporated in its terms. Principle embraces alike those, which affect its validity, construction, discharge and enforcement. The ideas of validity and remedy are inseparable, and both are parts of the obligation, which is guaranteed by the Constitution against invasion. Without impairing the obligation of the contract, the remedy may certainly be modified, as the wisdom of the nation shall direct. The constitutional provision qualified by the measure of control which the State retains over remedial processes. State also continues to possess authority to safeguard the vital interests of its people. It does not matter that the legislation appropriate to that end has the result of modifying or abrogating contracts already in effect. Not only are existing laws read into contracts in order to fix obligations as between the parties, but the reservation of essential attributes of sovereign power is also read into contracts as a postulate of the legal order. The policy of protecting contracts against impairment pre-suppose the maintenance of a

Held: Ratio:

Respondent Argument: The subject obligation is a prewar obligation contracted on August 20, 1941 and that he is a war sufferer, having filed his claim with the Philippine War Damage Commission for the losses he had suffered as a consequence of the last war.
Under Section 2 of said Republic Act No. 342, payment of his obligation cannot be enforced until after the lapse of eight years from the settlement of his claim by the Philippine War Damage Commission. This period has not yet expired.

Petitioner Argument: RA 342 is null and void for violating the non-impairment clause of the Constitution.

CONSTI II DIGESTS: 1D COMPILATION

CHUA, ALIMANGOHAN, CONTRERAS, BRIONES, LENCIO, RELOJO, TENGCO, LIM, DUMA, DANAO, MELLA, TONGSON

)'
2. The Court determined that there was no longer an emergency to justify the effectivity of the moratoria. The Court took due notice of factual circumstances that showed the improving conditions of the country and referred to public statements by the Chief Executive indicating the growth of the national economy. The Court held: It can now be safely stated that in the main the financial condition of our country and our people, individually and collectively, has practically returned to normal notwithstanding occasional reverses caused by local dissidence and the sporadic disturbance of peace and order in our midst. Business, industry and agriculture have picked up and developed at such stride that we can say that we are now well on the road to recovery and progress. In light of these circumstances, there is no longer any justification for the continued operation of the debt moratoria imposed by RA 342, EO 25 and EO 32. [For Reference:

"

Issue: W/N RA 342 is constitutional. Ruling: No. RA 342 was declared null and void for being unreasonable and oppressive. EO 25 and 32 were also declared null and void for the same reasons. Ratio: Reserved Power of the State

Home Building and Loan Association vs. Blaisdell


All contracts are subject to an implied reservation of the protective power of the state. Statutes which validly exercise this reserved power, rather than impairing the obligations of an existing contract, are comprehended within the obligations. Laws altering existing contracts constitute an impairment within the meaning of the contract clause only if they are unreasonable in the light of the circumstances occasioning their enactment. Limitations on the Reserved Power Also from Blaisdell 5. 6. 7. 8. The statute imposing the impairment must be addressed to a legitimate purpose. The impairment should only refer to the remedy and not to a substantive right. (The State may postpone the enforcement of the obligation but cannot destroy it by making the remedy futile) The alteration or change must not be burdened with unreasonable restrictions and conditions. (Extensions must not be so piled-up as to make the remedy a shadow) The statute must be justified by an emergency, temporary in nature.

Blaisdell Summary: Appellant contested the validity of charter 339 of the laws of Minnesota of 1993, approved April 13, 1933, called the Minnesota Mortgage Moratorium Law, as being repugnant to the contract clause of the Federal Constitution. The statute was sustained by the Supreme Court of Minnesota as an emergency measure. "Although coceding that the obligations of the mortgage contract was impaired, the court decided that what it thus described as an impairment was, notwithstanding the contract clause of the Federal Constitution, within the police power of the State as that power was called into exercise by the public economic emergency which the legislative had found to exist". This theory was up-held by the Supreme Court.]

Ortigas & Co. vs. Feati Bank (1979) Facts: Plaintiff is a limited partnership and defendant Feati Bank and Trust Co. is a corporation organized and existing in accordance with the laws of the Philippines. Plaintiff is engaged in real estate business developing and selling lots to the public, particularly the Highway Hills Subdivision along EDSA, Mandaluyong, Rizal. Plaintiff, as vendor and Augusto and Natividad Angeles, as vendees entered into separate agreements of sale on installments over 2 parcels of land known as Lots Nos. 5 and 6 of the Highway Hills Subdivision. Later, the vendees transferred their rights and interests over the 2 lots in favor of Emma Chavez. Plaintiff executed the corresponding deeds of sale in favor of Chavez. The agreements and the deeds of sale contained restrictions that: 1. the parcel of land shall be used by the buyer exclusively for residential purposes and she shall not be entitled to remove soil, stones or gravel from it. 2. All buildings and other improvements which may be constructed in the lot must be of strong materials and properly painted, provided with sanitary installations, and shall not be at a distance of less than 2 meters from its boundary lines. Eventually, defendant acquired lot nos. 5 and 6 and the building restrictions were also annotated in the TCTs. Plaintiff claims that the restrictions annotated in the TCTs were imposed as part of its general building scheme designed for the beautification and development of the Highway Hills

Application to this case 1. RA 342, EO 25 and EO 32 have the combined effect of extending the debt moratorium to 12 years. This is an unreasonably long time for the creditors to wait for the settlement of the outstanding obligations. The Court held: While the purpose of Congress is plausible, and should be commended, the relief accorded works injustice to creditors who are practically left at the mercy of the debtors. Their hope to effect collection becomes extremely remote, more so if the credits are unsecured. And the injustice is more patent when, under the law, the debtor is not even required to pay interest during the operation of the relief, unlike similar statutes in the United States.

CONSTI II DIGESTS: 1D COMPILATION

CHUA, ALIMANGOHAN, CONTRERAS, BRIONES, LENCIO, RELOJO, TENGCO, LIM, DUMA, DANAO, MELLA, TONGSON

)(
between a province, city or municipality and a third party in which case the original terms of the contract should govern. But these exceptions do not apply in the case. 2. While non-impairment of contracts is constitutionally guaranteed, the rule is not absolute since it has to be reconciled with the legitimate exercise of police power which is the power to prescribe regulations to promote the health, morals, peace, education, good order and general welfare of the people. The exercise of the power may be judicially inquired into and corrected only if it is capricious, whimsical, unjust, or unreasonable, there having been a denial of due process or a violation of any other applicable constitutional guarantee. The resolution was passed by the Municipal Council in the exercise of police power to safeguard or promote the health and safety of the people in the locality. EDSA, a main traffic artery which runs through several cities in Metro Manila, supports an endless stream of traffic and the resulting noise and pollution are hardly conducive to the health and welfare of the residents. In Helvering vs. Davis, it was held that needs that were narrow or parochial a century ago may be interwoven in our day with the well-being of the nation. What is critical or urgent changes with the times.

"

Subdivision which forms part of the estate of plaintiff where commercial and industrial sites are also designated.

Defendants arguments: 1. The area along the western part EDSA from Shaw Boulevard to Pasig River has been declared a commercial and industrial zone per Resolution No. 27 of the Municipal Council of Mandaluyong. 2. The plaintiff completely sold and transferred to third persons all lots in the subdivision facing EDSA and the subject lots were acquired by it more than 2 years after the area had been declared a commercial and industrial zone.
Defendant then commenced the construction of a building on Lots Nos. 5 and 6, to be devoted to banking purposes but defendant also claims that it could also be devoted to residential purposes. Plaintiff demanded that defendant stop the construction of the commercial building. The defendant refused to comply contending that the building was being constructed in accordance with zoning regulations. The trial court ruled in favor of the defendant and held that the restrictions found in the TCTs were subordinate to the municipal resolution. It held that its was a valid exercise of police power of the municipality. Issues: 1. WON Resolution No. 27 is a valid exercise of police power. 2. WON the resolution can nullify or supersede the contractual obligations assumed by the defendant. Held: 1. 2. Ratio: 1. Yes. Yes. In the first place, the validity of the resolution was never questioned before the trial court. It is a rule that parties must present all questions and issues to the lower court before they can be presented to the appellate court in order for the lower court to pass thereon and so that the appellate court upon appeal may determine whether or not such ruling was erroneous. Section 3 of RA 2264 known as the Local Autonomy Act empowers a Municipal Council to adopt zoning and subdivision ordinances or regulations for the municipality. The law does not restrict the exercise of power through an ordinance. Section 12 of the law reveals that the implied power of a municipality should be liberally construed in its favor and that any reasonable doubt as to the existence of the power should be interpreted in favor of the local government and shall be presumed to exist. The general welfare clause is also liberally interpreted in case of doubt so as to give more power to local governments in promoting the economic conditions and social welfare of the people in the community. The only exceptions under Section 12 are existing vested rights arising out of a contract

Juarez v Court of Appeals (1992) Facts: Virgina Juarez leased a lot (originally leased by her uncle and mother) which she subleased to Roberto Capuchino in 1976, reserving only one room for her personal belongings. The said lot was sold by the owners to a realty corporation which also sold it to herein private respondent Cetus Dev't Corporation. After acquiring the land, Cetus filed a complaint for ejectment against the petitioner on the ground that the lessee had subleased the property without its consent in violation with BP 877. BP 877 provides that: all residential units the total monthly rental of which does not exceed P400 as of the effectivity of the Act shall be covered by the Act.

Petitioner's arguments: BP 877 should not be given retroactive application because it would violate the impairment clause and the prohibition against ex post facto laws. (Note: Petitioner subleased the lot in 1976, and the act was only enacted in 1985.) BP 877 is an ex post facto law.
Issue: Whether or not the retroactive application of BP 877 in the case violates the impairment clause. Held: NO.

CONSTI II DIGESTS: 1D COMPILATION

CHUA, ALIMANGOHAN, CONTRERAS, BRIONES, LENCIO, RELOJO, TENGCO, LIM, DUMA, DANAO, MELLA, TONGSON

))

"

Ratio:

The impairment clause is no longer inviolate o Before: Any interference with the purely private agreements was an unwarranted intrusion that could be properly struck down. o Now: Interests of the public have become involved in what are supposed to be still private agreements. These agreements which have been removed from the protection of the impairment clause have come within the embrace of police power. o As long as the contract affects the public welfare one way or another so as to require the interference of the State, then must the police power be asserted, and prevail, over the impairment clause. Housing is one of the most serious social problems of the country o Regulation of rentals has long been the continuing concern of the govt as manifested by the succession of laws on the subject. o Court has consistently recognized the need for such legislation to prevent the lessor from imposing arbitrary conditions on the lessee while at the same time deterring the lessee from abusing the statutory benefits accorded to him. Petitioners contention that BP 877 is an ex post facto law must also be rejected. o The act is not penal in nature, and the fact that it contains penal provisions does not make it so. o She is not prosecuted under the said penal provisions. She was sued in a civil complaint to eject her from the lot on the ground that she had unlawfully subleased it.

CONSTI II DIGESTS: 1D COMPILATION

CHUA, ALIMANGOHAN, CONTRERAS, BRIONES, LENCIO, RELOJO, TENGCO, LIM, DUMA, DANAO, MELLA, TONGSON

)*
custodial interrogation questioning initiated by law enforcement officers after a person has been taken into custody or otherwise deprived of his freedom of action in any significant way Procedural safeguards: 1. Prior to any questioning, the person must be warned that he has a right to remain silent, that any statement he does make may be used as evidence against him, and that he has a right in the presence of an attorney, either retained or appointed 2. The defendant may waive effectuation of these rights, provided the waiver was made voluntarily, knowingly and intelligently. 3. If he indicates in any manner and at any stage of the process that he wished to consult with an atty before speaking there can be no questioning 4. If the individual is alone and indicates in any manner that he does not wish to be interrogated, police may not question him 5. The mere fact that he may have answered some questions or volunteered some statements on his own does not deprive him of the right to refrain from answering any further inquiries until he has consulted with an atty and thereafter consents to be questioned

"

SEC 12: ORIGINS & RATIONALE Miranda v. Arizona (1966) FACTS: The cases raise questions regarding the admissibility of statements obtained from individual subjected to police interrogation and the necessity for procedures which assure rights to individuals. 1. Miranda v. Arizona the police arrested the defendant and took him to a special interrogation room where they secured a confession 2. Vignera v. NY defendant made oral admitting to police after interrogation and signed an inculpatory statement upon being questioned by asst district atty 3. Westover v. US defendant was handed over FBI. Federal officers had obtained signed statements from defendant 4. California v. Stewart, the local police held the defendant 5 days in the station and interrogated him on 9 separate occasions before they secured inculpatory statement They contend that modern practice of in-custody interrogation is psychologically rather than physically oriented as the manual of police officers instructs the police 1. That the principal psychological factor contributing to successful interrogation is privacy 2. To display an air of confidence in the suspects guilt and from outward appearance to maintain only an interest in confirming certain details 3. That the major qualities of an interrogation should possess are patience and perseverance 4. That the suspect be offered legal excuses for his actions in order to obtain initial admission of guilt 5. When techniques described above prove unavailing, it must be alternated with a show of some hostility 6. How to handle the individual who refuses to discuss the matter entirely or who asks for an atty or relatives ISSUE: WON incommunicado interrogation of individuals in a police-dominated atmosphere results in self-incriminating statements without warning of constitutional rights HELD: Even without employing brutality, the very fact of custodial interrogation exacts a heavy toll on individual liberty and trades on the weaknesses of individuals. The records do not evince over physical coercion or patented psychological ploys; however, none of these cases did the officers undertake to afford appropriate safeguards at the outset of the interrogation to insure that the statements were truly product of free choice. RATIO: The interrogation environment is created for no purpose other than to subjugate the individual to the will of his examiner. This atmosphere carries its own badge of intimidation. The current practice of incommunicado interrogation is at odds with the right against selfincrimination. Unless adequate protective devices are employed to dispel the compulsion inherent in custodial surroundings, no statement obtained from the defendant can truly be the product of his free choice. The prosecution may not use statements , whether exculpatory, stemming from custodial interrogation of the defendant unless it demonstrates the use of procedural safeguards effective to secure the privilege against self-incrimination.

Magtoto v. Manguera (1976) Facts: Magtoto was accused in a criminal Case (CFI of Occidental Mindoro) of murder for the death of Ignacio Calara and Eduardo Calara in two informations both dated February 23, 1973. During the joint trial of these cases, his extrajudicial confession dated November 25, 1972 was, in the Court's order of June 18, 1973, admitted in evidence over the objection of the defense on the ground that it was taken while the accused was in the preventive custody of the PC without his having been informed of his right to remain silent and to counsel. Issue: W/N the confession of Magtoto is admissible Held: Yes. The confessions of the accused were taken before the effectivity of the New Constitution in accordance with the rules then in force, no right had been violated as to render them inadmissible in evidence although they were not informed of "their right to remain silent and to counsel," "and to be informed of such right," because, We repeat, no such right existed at the time. Ratio: Section 20, Article IV No person shall be compelled to be a witness against himself. Any person under investigation for the commission of an offense shall have the right to remain silent and to counsel, and to be informed of such right. No force, violence, threat, intimidation, or any other means which vitiates the free will shall be used against him. Any confession obtained in violation of this section shall be inadmissible in evidence

CONSTI II DIGESTS: 1D COMPILATION

CHUA, ALIMANGOHAN, CONTRERAS, BRIONES, LENCIO, RELOJO, TENGCO, LIM, DUMA, DANAO, MELLA, TONGSON

*+
18. He must be informed that he has the right to waive any of said rights provided it is made voluntarily, knowingly and intelligently and ensure that he understood the same; 19. In addition, if the person arrested waives his right to a lawyer, he must be informed that it must be done in writing AND in the presence of counsel, otherwise, he must be warned that the waiver is void even if he insist on his waiver and chooses to speak; 20. That the person arrested must be informed that he may indicate in any manner at any time or stage of the process that he does not wish to be questioned with warning that once he makes such indication, the police may not interrogate him if the same had not yet commenced, or the interrogation must ceased if it has already begun; 21. The person arrested must be informed that his initial waiver of his right to remain silent, the right to counsel or any of his rights does not bar him from invoking it at any time during the process, regardless of whether he may have answered some questions or volunteered some statements; 22. He must also be informed that any statement or evidence, as the case may be, obtained in violation of any of the foregoing, whether inculpatory or exculpatory, in whole or in part, shall be inadmissible in evidence. SEC 12: WHEN THE RIGHTS BECOME AVAILABLE

"

We hold that this specific portion of this constitutional mandate has and should be given a prospective and not a retrospective effect. Consequently, a confession obtained from a person under investigation for the commission of an offense, who has not been informed of his right (to silence and) to counsel, is inadmissible in evidence if the same had been obtained after the effectivity of the New Constitution on January 17, 1973. Conversely, such confession is admissible in evidence against the accused, if the same had been obtained before the effectivity of the New Constitution, even if presented after January 17, 1973, and even if he had not been informed of his right to counsel, since no law gave the accused the right to be so informed before that date.

People v. Mahinay [ratio lang yung nasa case book. As in guidelines lang. I just added facts, just in case she asks] Facts: Mahinay killed and raped a 12 yr old girl named Ma. Victoria. He was apprehended by the police and made an extra judicial confession with the assistance of Atty. Viernes. He was later on charged and later on convicted for killing and raping Ma. Victoria. He claims that his extrajudicial confession was executed in violation of his constitutional right to counsel. Held: There was no violation of his right to counsel. The consequences of his action were clearly explained by Viernes before he executed the extra-judicial confession. Ratio: Guidelines in the Miranda Rights 12. must be informed in a language known to and understood by him of the reason for the arrest; must be shown the warrant of arrest, if any; every other warning, information or communication must be in a language known to and understood by said person 13. person arrested must be told that he has a right to remain silent and that any statement he makes may be used as evidence against him 14. He must be informed that he has the right to be assisted at all times and have the presence of an independent and competent lawyer, preferably of his own choice; 15. He must be informed that if he has no lawyer or cannot afford the services of a lawyer, one will be provided for him; and that a lawyer may also be engaged by any person in his behalf, or may be appointed by the court upon petition of the person arrested or one acting in his behalf; 16. That whether or not the person arrested has a lawyer, he must be informed that no custodial investigation in any form shall be conducted except in the presence of his counsel or after a valid waiver has been made; 17. The person arrested must be informed that, at any time, he has the right to communicate or confer by the most expedient means telephone, radio, letter or messenger with his lawyer (either retained or appointed), any member of his immediate family, or any medical doctor, priest or minister chosen by him or by any one from his immediate family or by his counsel, or be visited by/confer with duly accredited national or international non-government organization. It shall be the responsibility of the officer to ensure that this is accomplished;

People vs Taylaran (1981) Facts: Taylaran invokes Art. IV Sec 20 to block the admissibility of his declaration that he killed Ofremia Atup because of her alleged vow to kill him. He contends that constitutional safeguards have not been made available to him. Issue: WON he can invoke Art. IV Sec 20 in the present case Held: NO. In the present case, no written confession was sought to be presented in evidence as a result of formal investigation. It is but natural for one who surrenders to the police to give explanation for his act of surrendering. It can hardly be said that he is already under investigation at the moment of surrender. Solicitor General: If he voluntarily admits the killing and it was precisely because he surrendered to admit it, the constitutional safeguards to be informed of his rights to silence and to counsel may not be invoked.

People v. Marcos (1987) Facts: The appellant was charged with the crime of kidnapping. He maintains that his extrajudicial confession before the NBI is inadmissible to evidence against him and that his waiver of his right to counsel is without legal effect because BOTH was obtained from him without the assistance of a counsel/lawyer. Issue: WON the sworn statement is valid. Held: YES! The sworn statement is valid because he was not under police custody when he VOLUNTARY gave his sworn statement before the NBI agent; He was merely invited for questioning so he can shed light on the kidnapping of Benedict. He was even allowed to go

CONSTI II DIGESTS: 1D COMPILATION

CHUA, ALIMANGOHAN, CONTRERAS, BRIONES, LENCIO, RELOJO, TENGCO, LIM, DUMA, DANAO, MELLA, TONGSON

*!
confession and the testimony of the witnesses, we hold that the alleged confession is inadmissible and must perforce be discarded. 4. Ratio: a. A confession is admissible in evidence if it is satisfactorily shown to have been obtained within the limits imposed by the 1987 Constitution.31 Sec. 12, Art. III thereof states in part, to wit: i. SEC. 12. (1) Any person under investigation for the commission of an offense shall have the right to be informed of his right to remain silent and to have competent and independent counsel preferably of his own choice. If the person cannot afford the services of counsel, he must be provided with one. These rights cannot be waived except in writing and in the presence of counsel. (2) No torture, force, violence, threat, intimidation or any other means which vitiate the free will shall be used against him. Secret detention places, solitary, incommunicado, or other similar forms of detention are prohibited. (3) Any confession or admission obtained in violation of this or Section 17 hereof shall be inadmissible in evidence against him.

"

home, after the investigation. (Note: Appellant admitted that he voluntary gave the sworn statement.)

People vs. Rapeza (2007) 1. Facts: a. This treats of the appeal from the Decision dated 1 July 2005 of the Court of Appeals affirming the Consolidated Judgment dated 24 July 2001 of the Regional Trial Court (RTC) of Palawan, Puerto Princesa City in Criminal Case Nos. 13064 and 13202 where Jerry Rapeza (appellant) was found guilty of two (2) counts of murder and sentenced to the penalty of reclusion perpetua for each count, plus a total of P100,000.00 as indemnity for the heirs of the two (2) victims. Facts according to the Prosecution: i. in the afternoon of 21 October 1995, an unidentified woman went to the Culion Municipal Station and reported a killing that had taken place in Sitio Cawa-Cawa, Barangay Osmea, Culion, Palawan. The officer-in-charge, SPO2 Ciriaco Gapas, sent an investigating team led by SPO2 Crisanto Cuizon, Jr. and PO2 Isidro Macatangay, to the victims house. There they saw two bloodied bodies. The victims were later identified as Priscilla Libas and Cesar Ganzon. Upon information supplied by a certain Mr. Dela Cruz that appellant wanted to confess to said crimes, SPO2 Gapas set out to look for appellant. He found appellant fishing in Asinan Island and invited the latter for questioning. Appellant expressed his willingness to make a confession in the presence of a lawyer. Appellant was then brought to the police station after which SPO2 Gapas requested that the appellant be provided with a lawyer. The following day, appellant was brought to the house of Atty. Roberto Reyes, the only available lawyer in the municipality. At the house of Atty. Reyes, in the presence of Vice-Mayor Emiliano Marasigan of Culion, two (2) officials of the Sangguniang Barangay, SPO2 Cuizon and an interpreter, SPO2 Gapas proceeded with the custodial investigation of appellant who was assisted by Atty. Reyes. Appellant was expressly advised that he was being investigated for the death of Libas and Ganzon.

ii.

b.

iii. b.

Republic Act No. 7438 has reinforced the constitutional mandate protecting the rights of persons under custodial investigation. The pertinent provisions read: i. f. As used in this Act, "custodial investigation" shall include the practice of issuing an "invitation" to a person who is investigated in connection with an offense he is suspected to have committed, without prejudice to the liability of the "inviting" officer for any violation of law.

ii.

c.

If the extrajudicial confession satisfies these constitutional standards, it must further be tested for voluntariness, that is, if it was given freely by the confessant without any form of coercion or inducement. Thus, the Court has consistently held that an extrajudicial confession, to be admissible, must conform to the following requisites: 1) the confession must be voluntary; 2) the confession must be made with the assistance of a competent and independent counsel, preferably of the confessants choice; 3) the confession must be express; and 4) the confession must be in writing. If all the foregoing requisites are met, the confession constitutes evidence of a high order because it is presumed that no person of normal mind will knowingly and deliberately confess to a crime unless prompted by truth and conscience. Otherwise, it is disregarded in accordance with the cold objectivity

d.

2.

Issue: a. Held: a.

WON appellants extrajudicial confession is admissible in evidence to warrant the verdict of guilt. NO. There is no direct evidence of appellants guilt except for the alleged extrajudicial confession. Upon careful examination of the alleged extrajudicial

e.

3.

CONSTI II DIGESTS: 1D COMPILATION

CHUA, ALIMANGOHAN, CONTRERAS, BRIONES, LENCIO, RELOJO, TENGCO, LIM, DUMA, DANAO, MELLA, TONGSON

*#
to prove that said rights were translated in a language understood by accused-appellant.

"

of the exclusionary rule. The latter situation obtains in the instant case for several reasons. f. Appellant was not informed of his constitutional rights in custodial investigation. i. We note that appellant did not voluntarily surrender to the police but was "invited" by SPO2 Gapas to the police station. There he was detained from 11 oclock in the morning of 22 October 1995 up to the morning of 23 October 1995 before his extrajudicial statement was allegedly taken. At this juncture, appellant should have been informed of his constitutional rights as he was already considered a suspect, contrary to the finding of the trial court that the mandatory constitutional guidelines only attached when the investigators started to propound questions to appellant on 23 October 1995 in the house of Atty. Reyes. In People v. Dueas, Jr.,40 we ruled, to wit: Custodial investigation refers to the critical pre-trial stage when the investigation ceases to be a general inquiry into an unsolved crime but has begun to focus on a particular person as a suspect. Facts:

People v. Judge Ayson

Private Respondent Ramos, a ticket freight clerk in Philippine Airlines, was allegedly involved in irregularities in the sales of plane ticket. He was then notified by PAL of an investigation regarding the matter. A day before the scheduled investigation, he sent a handwriiten note wherein he stated that hes willing to settle irregularities allegedly charged him in the amount of 75 K. At the investigation conducted by the PAL Branch Manager in Baguio City, Ramos was informed about the findings of the Audit Team. Thereafter, his answers in response to the questions were taken down in writing. Ramos answers were to the effect that: 1) 2) 3) 4) 5) he had not indeed made disclosure of the said tickets: the proceeds had been misused by him; his plan on paying back the money was prevented by shame; he is still willing to settle his obligation; that he was willing to sign his statement.

g.

Even supposing that the custodial investigation started only on 23 October 1995, a review of the records reveals that the taking of appellants confession was flawed nonetheless. i. It is stated in the alleged confession that prior to questioning SPO2 Gapas had informed appellant in Tagalog of his right to remain silent, that any statement he made could be used in evidence for or against him, that he has a right to counsel of his own choice, and that if he cannot afford the services of one, the police shall provide one for him. However, there is no showing that appellant had actually understood his rights. He was not even informed that he may waive such rights only in writing and in the presence of counsel. In this case, it was established that at the time of the investigation appellant was illiterate and was not well versed in Tagalog. This fact should engender a higher degree of scrutiny in determining whether he understood his rights as allegedly communicated to him, as well as the contents of his alleged confession. The extra-judicial confession was allegedly made in Tagalog when accused-appellant is admittedly not well versed in said language. Even if the confession was made in the presence of an interpreter, there is no showing that the rights of a person under investigation were effectively explained and/or interpreted to accused-appellant. The interpreter was not even presented in Court

Two months later, an information was filed against him with the crime of estafa. The prosecutors presented the evidence which included the statements made by Ramos before PAL. Ramos attorneys objected and said that the statement, which appears to be a confession, were inadmissible in evidence because it was taken without the accused being represented by a lawyer. The respondent judge considered all other evidence but not the statement/confession by Ramos that was obtained by PAL through administrative investigation. According to the respondent judge, it is inadmissible in evidence because the accused was not reminded of his right to remain silent and to have counsel. The accused was not assisted by counsel when he made said admission. Respondent judge denied the motion for reconsideration of the private prosecutors. Hence this petitions for certiorari and prohibition. Issue: Whether or not the respondent judge erred in treating the said admission/statement inadmissible in evidence Held: Yes. The writ of certiorari was granted annulling and setting aside the orders of respondent judge.

ii.

iii.

CONSTI II DIGESTS: 1D COMPILATION

CHUA, ALIMANGOHAN, CONTRERAS, BRIONES, LENCIO, RELOJO, TENGCO, LIM, DUMA, DANAO, MELLA, TONGSON

*$
The rights that were included in Sec 20 of Art VI of the 1973 Constitution are already separated and situated in different sections. The right against self incrimination is in Sec 17 of Art III of the 1987 Consti The rights in custodial interrogation is in Sec 12 of Art III of the 1987 Consti

"

Ratio: -The respondent judge based his decision on Sec 20 of the Art VI of the 1973 Constitution which deals with 2 sets of rights: 1) The right against self-incrimination; and 2) The rights of a person in custodial interrogation.

On the Right Against Self Incrimiination -this right of the private respondent was not violated because he willingly answered all the questions asked in the investigation.
-the judge/other officer presiding over a trial, hearing or investigation does not have the duty to remind the witness/accused of his right against self-incrimination. It is a right that a person should know by himself and shall be claimed for it is not selfexecuting or automatically operational. Failure to claim it at the appropriate time is equivalent to waiving of his right. Facts:

People v. Marra (1994) Sgt. Reynaldo de Vera, a witness presented by the prosecution, narrated the sequence of events that happened after he and his companions went to the crime scene to conduct an investigation. De Vera and his companions, having received information that a man in a security guard's uniform was involved in the incident, sought information from a security guard of a nearby bus terminal who pointed them to Marra, who at that time was eating in a carinderia nearby. Marra informed them that his gun was at his house and they all went to his residence to het the same. After receiving said firearm, De Vera asked appellant why he killed Tandoc but Marra initially denied any participation in the killing. Nevertheless, when confronted with the fact that somebody saw him do it, Marra admitted the act although he alleged it was done in self-defense. He was not assisted by counsel nor was he informed of his rights (to remain silent and to counsel). Issue: WON Marra was under custodial investigation when he admitted the killing but invoked selfdefense, hence not admissible to evidence. Held: NO! The appellant was not under custodial investigation when he made the admission. He made it spontaneously given in an ordinary manner with no coercion whatsoever to compel him to make such a statement. He was also not deprived of his freedom of action when he made such statement. Ratio:

On the Right in Custodial Interrogation


The following rights should be observed when a person is in custodial interrogation: He shall have the right to remain silent and to counsel, and to be informed of such right No force, violence, threat, intimidation, or any other means which vitiates the free will shall be used against him Any confession obtained in violation of these shall be inadmissible in evidence.

-The abovementioned rights only in questioning initiated by law enforcement officers after a person has been taken into custody or otherwise deprived of his freedom of action in any significant way. -It is clear that Ramos was not in any sense under custodial interrogation prior to or during the administrative inquiry into the discovered irregularities in ticket sales. -It is also clear that Ramos had voluntarily answered questions posed to him on the first day of the administrative investigation and agreed that proceedings should be recorded. -The respondent judge misapprehended the nature and import of the disparate rights set forth in Sec 20, Art IV of the 1973 Constitution. He has taken them as applying to the same juridical situation, equating one with the other. In so doing, he has grossly erred. NOTE:

Custodial investigation involves any questioning initiated by law enforcement officers after a person has been taken into custody or otherwise deprived of his freedom of action in any significant way. It is only after the investigation ceases to be a general inquiry into an unsolved crime and begins to focus on a particular suspect, the suspect is taken into custody, and the police carries out a process of interrogations that lends itself to eliciting incriminating statements that the rule begins to operate. In the case at bar, Marra could have refused to answer questions from the very start when the policemen requested that they all go to his residence. The police inquiry had not yet reached a level wherein they considered him as a particular suspect. They were just probing into

CONSTI II DIGESTS: 1D COMPILATION

CHUA, ALIMANGOHAN, CONTRERAS, BRIONES, LENCIO, RELOJO, TENGCO, LIM, DUMA, DANAO, MELLA, TONGSON

*%
Issue: WON a confession taken in violation of Sections 12 and 17 can be invoked by appellants. Held: No. The provisions apply to the confessant.

"

a number of possibilities.

People v. Maqueda (1995) Facts: The trial court admitted the Sinumpaang Salaysay of Maqueda although it was taken without the assistance of counsel. The trial court was of the opinion that since an information had already been filed and he was already arrested, the Sinumpaang Salaysay was not taken during custodial investigation. The admissibility should not be tested under Art III Sec 12 but on the voluntariness of its execution. Maqueda was not able to overcome the presumption of voluntariness. Issue: WON the Sinumpaang Salaysay is admissible in evidence Held: NO. The Court also held that the Sinumpaang Salaysay is an extrajudicial admission, not an extrajudicial confession. Ratio: Admission of a Party (Sec 26 Rule 130 of the Rules of Court) Act, declaration or omission of party as to a relevant fact, tending, in connection with proof of other facts, to prove his guilt; Do not directly involve an acknowledgement of guilt or of criminal intent Insufficient to authorize a conviction

Ratio: A confession taken in violation of Sections 12 and 17 of Article III of the Constitution shall be inadmissible in evidence against him, the confessant. This objection can be raised only by the confessant whose rights have been violated as such right is personal in nature. Note: When the rights cease to be available The rights are available during the period of the police investigation. Once the case is filed, it comes under the jurisdiction of the court and the Section 12 right cease. o The accused is protected by Sections 14 and 17 Should the police attempt to further investigate outside the control of the court, Section 12 should apply. SEC 12: POLICE LINE-UPS; PARAFFIN TEST; SIGNATURE Gamboa v. Judge Cruz (1988) Facts: Petitioner was arrested for vagrancy, without a warrant of arrest. Thereafter, petitioner was brought to Precinct 2, where he was booked for vagrancy and then detained therein together with several others. The following day, during the line up of five (5) detainees, including petitioner, complainant pointed to petitioner and said, that one is a companion. After the identification, petitioner was ordered to stay on, while the police investigator was interrogating the complainant, petitioner was told to sit down in front of her. Information for robbery was filed against the petitioner. Petitioner, by counsel, instead of presenting his defense, manifested in open court that he was filing a Motion to Acquit or Demurrer to Evidence. Petitioners Argument: On the ground that the conduct of the line-up, without notice to, and in the absence of, his counsel violated his constitutional rights to counsel and to due process. Issue: Held: Whether or not Petitioners right to counsel had been accrued. The police line-up (at least, in this case) was not part of the custodial inquest; hence, petitioner was not yet entitled, as such stage, to counsel. Petitioner in the course of his identification in the police line-up had not yet been held to answer for criminal offense, he was, therefore, not deprived of his right to be assisted by counsel because the accusatory process had not yet set in.

Confession (Sec 33) Declaration of an accused acknowledging in express terms his guilt

To accept the trial courts view would result in a restrictive application of Sec 12, consequently diminishing his rights under Sec 14. The exercise of the rights to remain silent and to counsel are not confined to the period prior to the filing of information but also at that stage when a person is under investigation for the commission of an offense.

People v. Balisteros (1994) Facts: The three accused were charged with murder for the killing of Romeo Abad in Pandi, Bulacan. During the trial, the prosecution moved for the exclusion of accused Ernesto Galvante from the information for murder so that he could become a witness for the Government. The lower court discharged the accused from the information to be utilized as state witness. Galvante positively identified and pointed to Salvador Balistetos and Nilo Avestroz as the killers. The two were found guilty. Appellants, the two accused, claim that there has been a violation of Section 12 of the Constitution because Galvante executed a sworn statement wherein he categorically admitted his guilt but without the assistance of counsel. Appellants also claim that Galvante made his sworn statement in the presence of Atty. Alejandro, the counsel of plaintiff, Mr. Matic, the 2 brothers of the deceased, the Mayor of Pandi and another person whom he did not know. With this battery of persons surrounding him, we could imagine the pressure exerted on Galvante.

Ratio:

CONSTI II DIGESTS: 1D COMPILATION

CHUA, ALIMANGOHAN, CONTRERAS, BRIONES, LENCIO, RELOJO, TENGCO, LIM, DUMA, DANAO, MELLA, TONGSON

*&
Issue: W/N there was a violation of Loverias right to counsel (and his other Miranda rights); Ruling: No. At the time he was identified by the complainant in the police line-up, Loveria was not yet under investigation. Ratio: 1. Miranda rights may be invoked by an accused only when he is under custodial investigation. Custodial investigation - questioning initiated by law enforcement officers after a person has been taken into custody or otherwise deprived of his freedom of action in any significant way.

"

The right to counsel attached upon the start of an investigation. When the investigating officer starts to ask questions to elicit information and/or confessions or admissions from the respondents/accused. At such point or stage, the person being interrogated must be assisted by counsel to avoid pernicious practice of extorting false or coerced admissions or confessions for the lips of the person undergoing interrogation, for the commission of an offense. Counsel must among other things, assist any person under investigation. They leave no room for equivocation; no custodial investigation shall be conducted unless it be in the presence of counsel, engaged by the person arrested, or by any person in his behalf, or appointed by the court upon petition either of the detainee himself, or by anyone in his behalf. It was held when the process has not yet shifted from the investigatory to the accusatory as when police investigation does not elicit as a confession the accused may not yet avail of the services of his lawyer. When he was identified in the police line-up by complainant he did not give any statement to the police. He was, not interrogated at all as he was not facing a criminal charge. It was not he but the complainant who was being investigated at that time. He was ordered to sit down in front of the complainant while the latter was being investigated Constitutional right to counsel did not attach until judicial criminal proceedings were initiated, and that the exclusionary rule relating to out-of-court identifications in the absence of counsel did not apply to identification testimony based upon a police station show-up which took place before the accused had been indicted or otherwise formally charged with any criminal offense.

Gamboa v. Cruz applies. (There was also a police line-up where the accused was identified. The accused made the same argument.)
In the police line-up conducted, it was the complainant who was being investigated and who gave a statement to the police while the accused was not questioned at all. The accused could not, during the line-up, invoke his right to counsel because he was not under custodial interrogation. The situation is similar here. The appellant was not investigated when Manzanero was in the process of identifying him, he cannot claim that his right to counsel was violated because at that stage, he was not entitled to the constitutional guarantee invoked. 2. There was evidence enough to convict the accused even without the sworn statement of Manzanero (Exh. B) Even assuming that the process of identification of the appellant by Manzanero at the PC headquarters was attended by constitutional infirmities, only Manzanero's sworn statement (Exh. B) where he identified appellant and which was taken by Pat. Ayun, would be excluded for being inadmissible in evidence. The testimony of Manzanero made in open court positively identifying the appellant, as well as those of Richard Bales (conductor) and Betty Apolinario (passenger), would not be affected. These testimonies, taken together with the other evidence on record, would be sufficient to sustain the trial court's judgment of conviction.

People v. Loveria (1990) Miranda Rights/May be invoked only while under Custodial Investigation Facts: Cerilo Manzanero was the driver of a jeepney which the accused, David S. Loveria and two other companions robbed. On the occasion of the robbery the accused stabbed Manzanero multiple times. Manzanero was taken to the hospital were he made a full recovery. Subsequently, upon learning that certain suspects of a different robbery were being detained at the 225th PC Company, Manzanero proceeded to the police headquarters to check. There, he was able to identify the accused Loveria from a line-up. Pat. Ayun then took the sworn statement of Manzanero which was presented in court as evidence (Exh. B). Loveria was convicted by the Regional Trial Court, Branch CLIX (159) of the crime of Robbery with Homicide and Frustrated Homicide. In the appeal before the Supreme Court, accused-appellant assailed the manner in which he was identified by Manzanero at the headquarters of the 225th Philippine Constabulary (PC) in Cogeo, Antipolo, Rizal. He claimed a violation of his constitutional right to counsel.

People vs. Hatton Facts: The appellant argues that when he was presented in a line up of sorts, he was not represented by counsel. He argues that his constitutional right to counsel during custodial investigation has been violated.

CONSTI II DIGESTS: 1D COMPILATION

CHUA, ALIMANGOHAN, CONTRERAS, BRIONES, LENCIO, RELOJO, TENGCO, LIM, DUMA, DANAO, MELLA, TONGSON

*'
Ratio: Appellants reliance on Hassan is misplaced. The court acquitted the accused (in Hassan) because, among other things, of its peculiar factual milieu: there was no lineup of suspects. There was only the accused. o In the case at bar: Frago was singled out by Jicelyn in a line-up composed of 10 persons of different physical attributes. The right to counsel attaches upon start of an investigation when investigation officer starts to ask questions to elicit information and/or confessions or admissions from the respondent/accused. At such point, person being interrogated must be assisted by counsel to avoid pernicious practice of extorting false or coerced admissions or confessions. o In the case at bar: As aptly observed by the solicitor general, the police lineup is not part of the custodial inquest. Petitioner had not been held yet to answer for a criminal offense. Escobedo v Illinois: When the process had not yet shifted from the investigatory to the accusatory as when police investigation does not elicit a confession, the accused may not avail of the services of his lawyer. o In the case at bar: ! Accusatory process had not set in. ! Confrontation between the State and him had not begun. ! He did not give any statement to the police when he was identified. ! He was not interrogated at all as he was not facing a criminal charge. ! Police did not except a confession to be used against him for it was not he but the complainant who was being investigated at that time.

"

Issue: WON accuseds constitutional right has been violated. Held: No. Ratio: In US vs. Wade, it was held that presence of counsel is indispensable in a postindictment line-up. The accused in this case had already been arrested and a lawyer was appointed to represent him. It was 15 days after his arrest and the appointment of a lawyer when he was presented in a police line-up to be identified by the prosecution witness. Thus, the accused had already been under custodial investigation where his right to counsel already attached. Since presence of counsel can avert a prejudice and assure a meaningful confrontation at trial, the post-indictment lineup was a critical stage of the prosecution for Wade and is entitled to the aid of counsel. In the case at bar, Hatton was brought to the police station only to be identified by a witness to the killing of Algarme. Thus, he was not yet under custodial investigation. The police line-up was not part of the custodial inquest and petitioner was not entitled to counsel. When petitioner was identified by the complainant at the line-up, he had not been held to answer for a criminal offense and the accusatory process had not yet set in. The right to counsel attaches upon the start of an investigation wherein the investigating officer starts to ask questions to elicit information or confessions or admissions from the accused. At this stage, the accused must be assisted by counsel to avoid the pernicious practice of extorting false or coerced admissions or confessions. When he was identified by the complainant, he did not give any statement to the police. Therefore, he was not interrogated at all as he was not facing a criminal charge. The police did not exact a confession to be used against him. It was not him who was being investigated but the complainant. He was ordered to sit down in front of the complainant while the latter was being investigated.

People v Frago (1994) Facts: The court a quo found appellant Orlando Frago guilty of the crime of Rape against Jicelyn Lansap. In holding appellant liable for rape, the trial court based its decision mainly on its finding that the accused was positively identified by his victim and that there was no ill motive on her part to testify against him.

People v. Gamboa (1991) (Essential in the success of the prosecution of an offense is the proof of identity of offender. Absent such primary evidence, the prosecution generally falls back to other evidence such as murder weapon, handwriting expert, extrajudicial confession etc. Circumstantial evidence is resorted to ascertain the accused as the person responsible for the commission of the crime.) FACTS: The appellant was subjected to a paraffin test but not in a presence of his lawyer. Gamboa was found guilty of murder. In his appeal, he raised (among many) that the trial court erred in not rejecting the paraffin test results as inadmissible evidence. ISSUE: WON it is violative of his right against self-incrimination. HELD: No. RATIO: Right against self-incrimination is not violated by the taking of the paraffin test of his hands. This constitutional right extends only to testimonial compulsion and not when the body of the accused is proposed to be examined as in the case.

Appellants argument: It was during his detention, when he was not assisted by counsel, that he was identified by the victim. Thus, he invokes Peo v Hassan where the court affirmed the right of the accused to counsel at all stages of the proceedings, the most crucial of which is his identification, and denial thereof entitles him to acquittal.
Issue: Whether or not appellant was denied his right to assistance by a counsel. Held: NO.

People v. Linsangan (1991) Facts: In early November, 1987, police operatives of the Drug Enforcement Unit, Police Station No. 3 of the Western Police District were informed that there was rampant drug using and pushing on Dinalupihan Street, Tondo, Manila. They conducted a buy-bust operation. On November 13, 1987

CONSTI II DIGESTS: 1D COMPILATION

CHUA, ALIMANGOHAN, CONTRERAS, BRIONES, LENCIO, RELOJO, TENGCO, LIM, DUMA, DANAO, MELLA, TONGSON

*(
official duties in a regular manner. Their task of apprehending persons engaged in the deadly drug trade is difficult enough without legal and procedural technicalities to make it doubly so.

"

at 10 o'clock in the morning, before the group left the office for the area of operation, two (2) ten-peso bills were given to Pat. Corpuz who had marked them with his initials "T.C." He gave one of the marked bills to the informer. Then, they proceeded to Dinalupihan. Additional info: As Patrolman Corpuz and the confidential informant walked together, they conversed about the suspect. Pat. Corpuz asked the informant where the suspect was and the informer pointed to the appellant, who was seated by the gutter about six (6) meters away from them, seemingly waiting for someone. He was wearing blue-and-green shorts and a sando (undershirt). The informer raised his hand as a signal to the appellant, who rose and walked toward them. They walked toward a wooden house with a wooden fence and a store on the left side. The informer told the appellant: "Kukuha ako." The informer asked? "Magkano?" The informer told the accused that he would buy P10 worth of marijuana while his "compadre" (referring to Patrolman Tomasito Corpuz), would also get P10 worth. P20 would fetch ten (10) cigarette sticks of hand-rolled marijuana at P2.00 per stick. The policeman and the informer impressed upon the accused that they were in dire need of marijuana. The accused took the P20 from Pat. Corpuz and tucked it in his front waist. The accused went inside the wooden house, while Pat. Corpuz and the informer waited outside. The accused emerged shortly and handed over to Pat. Corpuz ten (10) cigarette sticks of handrolled marijuana. Pat. Corpuz took them with his right hand and at the same time he grabbed the accused with his other hand, saying: "Pulis ito, h'wag kang pumalag!" Pat. Ruiz saw the signal and rushed over to them. The accused tried to resist but was overpowered. The informer took to his heels (pp. 1-35, t.s.n., Jan. 13, 1988). Pat. Ruiz frisked Linsangan and retrieved the marked ten-peso bills (Exhs. A-1 and A2) tucked in his waist. He asked the appellant to sign his name on the two P10 bills. They boarded the jeep and returned to the police station. Pat. Ruiz prepared a letter-request to the NBI for the laboratory examination of the ten suspected marijuana sticks. The appellant also put his initials "C.L." on each stick. Linsangan initialed the bills found tucked in his waist without the presence of counsel. He acquired the marked money for the sale of marijuana during a buy-bust operation. Issue: W/N his right against self-incrimination was violated and was denied due process. Held: No. The appellant was not denied due process during the custodial investigation. Although he was not assisted by counsel when he initialed the P10-bills that the police found tucked in his waist, his right against self-incrimination was not violated for his possession of the marked bills did not constitute a crime. The subject of the prosecution was his act of selling marijuana a cigarettes. His conviction was not based on the presence of his initials on the P10-bills, but on the fact that the trial court believed the testimony of the policemen that they arrested him while he was actually engaged in selling marijuana cigarettes to a member of the arresting party. The trial court gave more credence to their categorical declarations than to the appellant's denials. That is as it should be for as law enforcers, they are presumed to have performed their

People v. de las Marinas (1991) FACTS: De las Marinas was found guilty of violating Sec 15, Art III of RA 6425. In his appeal, he raised that the trial court erred in admitting the evidence exhibits F and F1 as they were inadmissible for having been taken in violation of the Constitution. Appellant was made to sign the Receipt for Property Seized acknowledging that they are the owners of seized properties (shabu) when it was the police officer who confiscated should sign the same. ISSUE: WON such confession was obtained in violation of the Constitution and is inadmissible in evidence HELD: Yes. RATIO: Making the accused sign said receipt is a form of extra-judicial confession of commission of offense. It is a violation of his constitutional right to remain silent and was made to admit the commission of the offense without informing him of his consti rights. (Take note that the court also relied on the testimonial and docu evidence of the prosecution.)

People v Enrique (1991) Facts: An informer told that a person named Bong was selling marijuana cigarettes. An entrapment operation was formed. The team included Natividad who acted as the poseur buyer. Thereafter, Bong approached Natividad and handed him the cigarettes who in turn gave a marked 10-peso bill. After confirming that it was a marijuana cigarette, he signaled his team to arrest Bong whose real name was Antonio Enrique. During the investigation, Enrique unassisted by counsel, wrote his name on the rolled marijuana cigarettes. He argues that such admission is inadmissible in evidence. Issue: Whether or not the cigarettes must be inadmissible in evidence. If yes, whether or not he is acquitted. Held: The cigarettes are inadmissible in evidence. Despite this, is still guilty because his conviction did not rely solely on his admission. He has completely failed to rebut by contrary evidence the testimonies and documentary evidence presented by the prosecution. Arresting officer argued that Enrique did so voluntarily and that this was a security against substitution. Yet they did not exert efforts to inform him of his rights under custodial investigation. In effect, Enriques act amounted an admission of his culpability, which renders the cigarettes inadmissible in evidence.

CONSTI II DIGESTS: 1D COMPILATION

CHUA, ALIMANGOHAN, CONTRERAS, BRIONES, LENCIO, RELOJO, TENGCO, LIM, DUMA, DANAO, MELLA, TONGSON

*)
There was an apparent violation of the right to counsel of the accused. The right to counsel attaches upon the start of the investigation, i.e. when the investigating officer starts to ask questions to elicit information or confession or admission from the accused.

"

People v. Bandin (1993) Facts: (Bandin was arrested in a buy-bust operation for selling marijuana.) Appellant signed on the Booking Sheet and Arrest Report at the police station. He also signed on the Receipt of Property Seized (marijuana) without the assistance of a counsel. Issue: WON the appellants signature on the Booking Sheet and the Receipt is inadmissible as evidence because it was given without the assistance of counsel. Held: Booking sheet and Arrest Report: No! Appellants signature on the Booking Sheet does not admit the commission of an offense nor confess to any incriminating circumstance. It is not an extrajudicial statement and cannot be the basis of a judgment of conviction. It is merely a statement of the accused's being booked (in the police station) and its date. It is a police report and may be useful in charges of arbitrary detention against the police themselves.

PEOPLE v. BOLANOS 211 SCRA 262 Facts: Oscar Pagdalian was murdered in Marble Supply, Balagtas Bulacan. According to Patrolman Rolando Alcantara and Francisco Dayao, deceased was with two companions on the previous night, one of whom was Bolanos(accused) and the other Claudio Magtibay who had a drinking spree with the deceased. When they apprehended the accused they found the firearm of the deceased on the chair where the accused was allegedly seated. They boarded Bolanos along with Magtibay, other accused on the police vehicle and brought them to the police station. While in the vehicle Bolanos was asked by the police if he killed the victim because the victim was abusive and Bolanos admitted that he killed the deceased. RTC convicted him admitting as evidence the confession saying it was given freely before the investigation hence the appeal. Issue: WON accused-appellant deprived of his constitutional right to counsel. Held: Yes. Being already under custodial investigation while on board the police patrol jeep on the way to the Police Station where formal investigation may have been conducted, appellant should have been informed of his Constitutional rights under Article III, Section 12 of the 1987 Constitution, more particularly par. 1 and par. 3. (Miranda Rights). Given that the extra-judicial confession is inadmissible and that it was the only evidence used mainly to charge him of the crime, his guilt was not proven beyond reasonable doubt and the accused must be acquitted.

Receipt of Property Seized: Yes! It is inadmissible to evidence. His signature on the Receipt is a tacit admission of the crime charged, for mere unexplained possession of prohibited drugs is punished by law; and it is tantamount to an uncounselled extra-judicial confession outlawed by the Bill of Rights.
It is, therefore, inadmissible as evidence for any admission wrung from the accused in violation of his constitutional rights is inadmissible against him. SEC 12: RIGHT TO COUNSEL [Estacio] People v De Jesus Facts: Investigation officer Pfc. Rogelio Lorbes admitted that the two accused, Tupas and de Jesus were turned over to him for investigation and that he interviewed them and solicited from them facts and information surrounding the robbery hold-up with homicide without the assistance of a lawyer. The facts and information were later reduced to writing in the presence of CLAO lawyer, a certain Atty. Saldivar. Issue: Whether or not admissions obtained during custodial interrogation without the benefit of Miranda warnings and in the absence of counsel are admissible in evidence when the same are later reduced into writing and signed in the presence of counsel. Held: No. Ratio:

People v. Lucero (1995) Facts: Pfc. Pursal informed the CIS Legal Department about Luceros need for a lawyer after the latter said that he had no lawyer. Atty Peralta then conferred with Lucero. It was noted that Peralta observed no reaction when he explained to Lucero his constitutional rights. When the CIS investigator began asking Lucero preliminary questions, Peralta left to attend a friends wake. The next morning, Lucero was accompanied by 2 CIS agents to Peraltas residence to present to the latter Luceros extrajudicial statement. Peralta signed the document when Lucero said that the statements were given voluntarily. Lucero claims however that he signed the statement under duress; that Atty Peralta was not around during the custodial investigation. The trial court convivted Lucero Issue: WON the extrajudicial confession suffers constitutional infirmities Held: Yes. Conviction cannot be sustained based on his extrajudicial confession. Ratio:

CONSTI II DIGESTS: 1D COMPILATION

CHUA, ALIMANGOHAN, CONTRERAS, BRIONES, LENCIO, RELOJO, TENGCO, LIM, DUMA, DANAO, MELLA, TONGSON

**
Subsequently, accused was apprehended. He was provided with a lawyer from the Public Attorneys Office - Atty. Generoso. In a private meeting, Atty. Generoso asked the accused if he committed the crimes he was accused of. Atty. Generoso also advised the accused that if he did not commit the crime then he should not to make any admission or statement as this would be prejudicial to him. The accused, however, said, "Attorney, aaminin ko na ho total ginawa ko naman." The accused then told Atty. Generoso that he was the one who took the television set but denied having killed Veronia Borja. The accused then made a statement in the presence of the Atty. Generoso confessing to the crime. The statement was entered as evidence in the trial. Accused was convicted of Robbery with Homicide. Accused argument: He now argues in this appeal that he was not fully and duly assisted by a counsel engaged by him. Hence, his extrajudicial confession is constitutionally infirm and inadmissible as evidence. (i.e. That the counsel who helped him wasnt one he chose.) Issue: W/N accuseds right to counsel was violated, and consequently, W/N the extrajudicial confession was inadmissible as evidence. Ruling: Accused right to counsel was NOT violated. The extrajudicial confession was admissible as evidence. Ratio:

"

Substantial compliance with the requirements of the right to counsel is far from the intent of the Constitution. It was during Atty Peraltas absence that Lucero gave an uncounselled confession. The uncounselled confession was supposed to be cured by bringing Lucero to Peraltas residence. The Constitution, in requiring the right to counsel, means effective and vigilant counsel. Because Peralta left, Lucero received no effective counseling at the crucial point (when the interrogation was just starting).

People v. Parojinong Facts: Petitioner (Parojinong) claims that his constitutional right to counsel during his custodial investigation by the police was violated. He claims that the lawyer assigned to him did not actually attend the investigation as the latter went out and that said lawyer was not his choice and was only forced on him. Issue: WON petitioners right to counsel was violated. Held: No. Ratio: The investigating informed the accused of his right and asked him if he is willing to be assisted by counsel of the government and petitioner said yes. Fiscal Luzviminda V. Uy before whom the accused swore his testimony said that accused told her that he was assisted by Atty. Fuentes III during the investigation. She asked the accused if the answers in the affidavit were his, accused said yes. While the initial choice of the lawyer is naturally lodged in the police investigators in cases where the accused cannot afford the services of counsel, the accused really has the final choice as he may reject the counsel chosen for him and ask for another one. " No objection was voiced by the accused throughout the entire investigation. " He apparently acquiesced to the choice of the investigators. " He complained for the first time that Atty. Fuentes was not his choice only during the trial. [Pamon] People v. Baello G.R. No. 101314, 14 July 1993

People v. Parojinog A person under investigation has the right to choose his own counsel. However, if he cannot afford the services of a lawyer, he must be provided with one. While the initial choice of counsel is lodged in the police investigators, the accused has the final decision. He may reject the counsel provided. People v. Masongsong Every lawyer is presumed to have knowledge of the law as well as the training in procedure sufficient to enable him to protect his client. Accuseds failure to request for another counsel negates his claim of denial of the right to choose his lawyer; especially where he was given sufficient time to choose his own counsel had he opted to so. People v. Pinzon A sworn statement is admissible as evidence where it was made and signed with the aid of a counsel whose free legal services the accused accepted,.
In this case, exhibit L completely belies the allegation that the accused was not fully assisted by Atty. Generoso during the investigation. Exhibit L discloses that the accused was informed of his constitutional rights by Atty. Generoso. Atty. Generoso conferred with the accused, warned the latter of the consequences of his confession and even advised him not to make any. However,

Failure to ask for new counsel negates a claim for denial of the right to choose a counsel
Facts: On 10 October, 1990 in the Municipality of Pasig, Metro-Manila, John Baello, accused, and a coconspirator, entered the house of Eustaquio Borja y Reyes thru the window at the second floor of the said house and stole several items. On the occasion of the robbery, the accused stabbed Veronica Borja y Ramos several times and killed her.

CONSTI II DIGESTS: 1D COMPILATION

CHUA, ALIMANGOHAN, CONTRERAS, BRIONES, LENCIO, RELOJO, TENGCO, LIM, DUMA, DANAO, MELLA, TONGSON

!++
Atty. Cajucom can hardly be said to have been voluntarily accepted by the appellant as his counsel. It is clear that he was foisted upon the appellant since the city fiscal immediately suggested the availability of Cajucom without first asking the appellant if he had a counsel of his own choice and if he had one, whether he could hire such counsel and if he could not, whether he would simply exercise his right to remain silent and to counsel. After the appellant said he wanted to be assisted, the city fiscal immediately informed him that Cajucom was ready. Appellant was also threatened with death if he would not admit knowing Jun and Sonny and his participation in the crime. The presence of the military officers and the continuing fear that if he did not cooperate, something would happen to him. Even if he agreed to be assisted by Cajucom, he was unable to assist the appellant in a satisfactory manner. He admitted on cross-examination and at the witness stand that he was an associate of Atty. Galace. There are also misgivings on whether Cajucom was understood by appellant when he informed appellant of his rights in English and Tagalog since the appellant was a fourth grader, farmer and could only understand Ilocano.

"

the accused insisted on going ahead with his confession, although he only confessed to the robbery.

People vs. Agustin Facts: Appellant claims that his extrajudicial confession was taken in violation of his rights under Section 11, Article III. Appellants arguments: 1. The lawyer who assisted him, Atty. Cajucom, was not of his own choice but foisted upon him by the City Fiscal. 2. Cajucom is a law partner of the private prosecutor, Atty. Gallace. 3. Cajucom conferred with him in English and Tagalog although he understood only Ilocano. 4. When Cajucom conferred with him and when the city Fiscal interrogated him, his military escorts were present. He argues that the lawyer who assists the suspect under custodial interrogation should be of his choice, not one foisted on him by the police investigator or other parties. When there are doubts on the voluntariness of the extrajudicial confession, the doubts must be resolved in favor of the accused. He concludes that his extrajudicial confession is inadmissible and his conviction cannot stand because there is no other evidence linking him to the crimes charged. Issue: WON appellants rights were violated. Held: Yes. The admission is inadmissible because it was taken in violation of Section 2. Ratio: It was not extrajudicial confession but it was extrajudicial admission which the appellant made. Appellant was also unlawfully arrested in violation of Section 2. In confession, there is an acknowledgement of guilt of the crime charged by the accused in a criminal case. Admission is applied in criminal cases to statements of facts which do not directly involve an acknowledgement of guilt of the accused. It is a statement by the accused of facts pertinent to the issue and tending to prove his guilt in connection with proof of other facts. It is something less than a confession and is insufficient by itself to authorize a conviction. In the case at bar, nothing indicates that he expressly acknowledged his guilt. He merely admitted some facts which in themselves are insufficient to authorize a conviction and which only tend to establish the ultimate fact of guilt. But the distinction is irrelevant because section 12 refers to both confession and admission. The rule on inadmissibility expressly includes admissions and not just confessions.

People v Guillermo (2004) Facts: Appellant Eric Guillermo was found guilty by the RTC of the crime of murder and was sentenced to suffer the death penalty.

Appellants argument: His conviction was based on inadmissible evidence(confession made before SPO1 Carlito Reyes where appellant was not assisted by a counsel). There is no clear showing that he was informed of his constitutional rights nor was he made to understand the same by the police investigators.
Issue: Whether or not appellants confession should be admissible in evidence. Held: No. The confession appellant made falls short of the protective standards laid down by the Constitution. Ratio: Under Article 3 of the Constitution, a confession to be admissible must satisfy the following requisites: a) The confession must be voluntary b) The confession must be made with the assistance of competent and independent counsel c) The confession must be express d) The confession must be in writing

CONSTI II DIGESTS: 1D COMPILATION

CHUA, ALIMANGOHAN, CONTRERAS, BRIONES, LENCIO, RELOJO, TENGCO, LIM, DUMA, DANAO, MELLA, TONGSON

!+!
RATIO: Appellant cannot claim that he was not advised of his consti rights. He manifested on record his desire to have Atty Giduqio as counsel. He was assisted by said lawyer during the investigation.) [sorry pero di ko gets bakit diniscuss yung ratio below sa case na to. Yung atty of his choice naman niya talaga yung nasunod]
preferably his own choice (as provided in Sec 12, Art III) does not mean that the choice of lawyer by a person under investigation is exclusive as to preclude other equally competent and independent attys from handling the defense OTHERWISE custodial investigation may be obstructed by accused by selecting a lawyer who is not available to protect his interest People v. Continente: When accused cannot afford lawyer or his preferred lawyer is not available, accused still has final choice as he may reject the counsel chosen for him and ask for another one Right to counsel is intended to preclude the slightest coercion as would lead the accused to admit something is false

"

The testimony of Reyes on cross-examination clearly shows the cavalier treatment by the police of said constitutional guarantees which could be gleaned from the transcript of Reyes testimony, which includes that: " Upon arriving at police station, he asked Guillermo to read his constitutional rights written on the wall. " Guillermo said he understood what was written on the wall. " Reyes proceeded with the investigation but the investigation was not recorded or written down. " Appellant was informed that he had to have a counsel. " There was no lawyer or counsel because it was a Sunday afternoon and appellant already admitted that he perpetrated the crime. Appellants alleged confession at the police station lacks the safeguards required by the Bill of Rights. " The investigation officer made no serious effort to make appellant aware of his basic rights under custodial investigation (he was just made to read the rights written in the wall). " Officer exerted no effort to provide him with counsel on the flimsy excuse that it was a Sunday. " Despite absence of counsel, officer proceeded with the investigation. " Record is bare of any showing that appellant had waived his constitutional rights in writing and in the presence of counsel. Even if admission or confession of an accused is gospel truth, if it was made without the assistance of counsel, it is inadmissible in evidence regardless of the absence of coercion or even if it had been voluntarily given. The right of a person under interrogation to be informed implies a correlative obligation on the part of the police investigator to explain and contemplates an effective communication that results in an understanding of what is conveyed. " Absent that understanding, there is a denial of the right to be informed, as it cannot be said that the person has been truly informed of his rights. *Note: However, the inadmissibility of the appellants confession as evidence does not necessarily lead to his acquittal. The rights in Art 3 Sec 12 are meant to preclude the slightest use of the States coercive power as would lead an accused to admit something false. But it is not intended to prevent him from freely and voluntarily admitting the truth outside the sphere of such power, and the facts in the case show that appellant admitted the commission of the crime not just to the police but also to private individuals.

People v. Sayaboc (2004) Facts: Sayaboc was convicted for murder of Joseph Galam. During the custodial investigation, he was represented by PAO lawyer Atty. COrnejo who remained silent during the entire proceeding. As a result, Sayaboc executed an extrajudicial confession, in Ilocano dialect. He therein confessed to killing Joseph Galam at the behest of Marlon Buenviaje for the sum of P100,000. He likewise implicated Miguel Buenviaje and Patricio Escorpiso. The confession was also signed by Atty. Cornejo and attested to by one Fiscal Melvin Tiongson.

Appellants argument: That the extrajudicial confession of Sayaboc may not be admitted in evidence against him because Atty. Cornejo, the PAO lawyer who was his counsel during the custodial investigation, was not a competent, independent, vigilant, and effective counsel. He was ineffective because he remained silent during the entire proceedings. He was not independent, as he was formerly a judge in the National Police Commission, which was holding court inside the PNP Command of Bayombong, Nueva Vizcaya.
Issue: 1. W/N it was taken without the assistance of a competent and independent counsel and an effective and vigilant counsel 2. W/N the extra judicial confession of the accused is admissible Held: 1. NO. We likewise rule that Sayaboc was not afforded his constitutional right to a competent counsel. While we are unable to rule on the unsubstantiated claim that Atty. Cornejo was partial to the police, still, the facts show through the testimonies of Sayaboc and prosecution witness SPO4 Cagungao that Atty. Cornejo remained silent throughout the duration of the custodial investigation. The trial court attributed the silence of Atty. Cornejo to the garrulous nature and intelligence of Sayaboc, SC found this argument unacceptable. That Sayaboc was a "garrulous" man who would "do what he wanted to do regardless of the advice of others" is immaterial. The waiver of a right is within the rights of a suspect. What is lacking is a showing, to the

People v. Mojello (2004) (FACTS: Mojello was charged and was found guilty of rape. In his appeal, he raised that the extrajudicial confession by appellant is inadmissible as he was not apprised of his constitutional rights. ISSUE: WON the extrajudicial confession by appellant is admissible HELD: Yes.

CONSTI II DIGESTS: 1D COMPILATION

CHUA, ALIMANGOHAN, CONTRERAS, BRIONES, LENCIO, RELOJO, TENGCO, LIM, DUMA, DANAO, MELLA, TONGSON

!+#

"

satisfaction of this Court, of a faithful attempt at each stage of the investigation to make Sayaboc aware of the consequences of his actions. If anything, it appears that Sayabocs counsel was ineffectual for having been cowed by his clients enthusiasm to speak, or, worse, was indifferent to it. The right to a competent and independent counsel means that the counsel should satisfy himself, during the conduct of the investigation, that the suspect understands the import and consequences of answering the questions propounded. In People v. Deniega, we said: The desired role of counsel in the process of custodial investigation is rendered meaningless if the lawyer merely gives perfunctory advice as opposed to a meaningful advocacy of the rights of the person undergoing questioning. If the advice given is so cursory as to be useless, voluntariness is impaired. This is not to say that a counsel should try to prevent an accused from making a confession. Indeed, as an officer of the court, it is an attorneys duty to, first and foremost, seek the truth. However, counsel should be able, throughout the investigation, to explain the nature of the questions by conferring with his client and halting the investigation should the need arise. The duty of a lawyer includes ensuring that the suspect under custodial investigation is aware that the right of an accused to remain silent may be invoked at any time. The purpose of the stringent requirements of the law is to protect all persons, especially the innocent and the weak, against possible indiscriminate use of the powers of the government. Any deviation cannot be tolerated, and any fruit of such deviation shall be excluded from evidence. 2. NO. the extrajudicial confession of Sayaboc cannot be used in evidence against him. (Section 12 of Article III of the 1987 Constitution). Jurisprudence provides that extrajudicial confessions are presumed to be voluntary. The condition for this presumption, however, is that the prosecution is able to show that the constitutional requirements safeguarding an accuseds rights during custodial investigation have been strictly complied with, especially when the extrajudicial confession has been denounced. The rationale for this requirement is to allay any fear that the person being investigated would succumb to coercion while in the unfamiliar or intimidating environment that is inherent in custodial investigations. Therefore, even if the confession may appear to have been given voluntarily since the confessant did not file charges against his alleged intimidators for maltreatment, the failure to properly inform a suspect of his rights during a custodial investigation renders the confession valueless and inadmissible. The right to be informed requires "the transmission of meaningful information rather than just the ceremonial and perfunctory recitation of an abstract constitutional principle." It should allow the suspect to consider the effects and consequences of any waiver he might make of these rights. More so when the suspect is one like Sayaboc, who has an educational attainment of Grade IV, was a stranger in Nueva Vizcaya, and had already been under the control of the police officers for two days previous to the investigation, albeit for another offense.

CONSTI II DIGESTS: 1D COMPILATION

CHUA, ALIMANGOHAN, CONTRERAS, BRIONES, LENCIO, RELOJO, TENGCO, LIM, DUMA, DANAO, MELLA, TONGSON

!+$
SEC 12: WAIVER People v. Rous (1990) Facts: (The petitioner was convicted with the crime of highway robbery with homicide.) The petitioner contends that his waiver of his right to remain silent and right to counsel was invalid because they were not made with the assistance of a counsel. He relies heavily on the prevailing rule in the case of People v. Calit. Issue: WON the petitioners waiver of his right to remain silent and to assistance by counsel was valid.

"

People v Olermo (2003) Facts: Appellant notes that during the presentation of the prosecution 1st witness, he was represented by Atty Domingo who was not her retained counsel (Atty Yuseco who was in Cagayan then). During the other hearings, she was represented by a de officio counsel, Atty Perez. Olermo claims that she was deprived of her right to a competent and independent counsel because Atty Domingo and Perez were not familiar with her case, hence not able to adequately protect her interests. Held: Preferably of his own choice in Sec 12 does not mean that the choice of lawyer by appellant is exclusive as to preclude other equally competent and independent attorneys. Otherwise the tempo of justice would be within appellants control who could impede the judicial process by merely selecting a lawyer who is not available. SEC 12: RIGHT TO BE INFORMED People v Pinlac (1988) Facts: Accused was convicted for robbery and robbery with homicide. He assails his conviction on the contention that the court erred in admitting his extrajudicial confession as evidence since he was not informed of his constitutional right to remain silent and to counsel. Issue: Whether or not his due process was observed during the custodial investigation, Held: No. Correct procedure for officers in making arrest and in custodial investigation o Inform him of the reason for the arrest o Show the warrant of arrest o Inform the person of his constitutional rights to remain silent and to counsel and that any statement he might make could be used against him ! He shall have the right to communicate with his lawyer, relative or anyone he chooses by the most expedient means o Officer must see to it that the abovementioned right is accomplished o The investigation shall only be conducted in the presence of counsel engaged by the person arrested, or by any person on his behalf, o appointed by the court upon petition by him or by another ! Right to counsel may be waived but shall not be valid unless made with assistance of counsel o *Any statement obtained in violation of this procedure, whether exculpatory or inculpatory shall be inadmissible in evidence. Right to be informed contemplates transmission of information not just recitation of the constitutional principle o It is not sufficient that officer repeat the provision of the constitution but he must also explain their effects in practical terms. People v Ramos: Court ruled: Verbal admission of accused was inadmissible because prosecution failed to show that those rights were explained to him, such that it could be said that the appraisal was sufficiently manifested and intelligently understood by the accused. In short, prosecution has the burden of proving that the investigating officer complied with the obligation.

Held: Yes! Although the waivers were not made in the presence of a counsel, the defect was cured and the requirement laid down in the Galit case was substantially complied with when petitioners lawyer, Atty. Madarietta, arrived at the closing stage of the interrogation. Records show that Atty. Madarietta read the statement and talked to the petitioner before the latter signed it. The accused voluntary waived his constitutional rights prior to the investigation; for he did not change his mind and avail of his right to remain silent after his counsel arrived and read the statement before he signed it. He could easily have refused to sign the statement and demand possession of the unsigned statement, if he wanted to. SEC 13: RIGHT TO BAIL OR RECOGNIZANCE [Nitcha] Cardines v. Rosete Facts: Respondent Judge Rosete is being charged with misconduct and ignorance of the law for allegedly granting bail to the accused when he fully knew that the charge of illegal recruitment was punishable with life imprisonment hence non-bailable. Respondent judge refutes the charge and maintains that under Sec 3, Rule 114, of the 1985 Rules on Criminal Procedure, the accused charged with an offense penalized with life imprisonment should be granted bail as a matter of right. Issue: Whether or not the respondent judge erred in granting bail to the accused. Held: No Ratio: -The law existing at the time of the alleged illegal recruitment when and when the accused applied for bail was the 1985 Rules on Criminal Procedure which states that a person under detention shall before conviction be granted bail as a matter of right. Two exceptions were recognized:

CONSTI II DIGESTS: 1D COMPILATION

CHUA, ALIMANGOHAN, CONTRERAS, BRIONES, LENCIO, RELOJO, TENGCO, LIM, DUMA, DANAO, MELLA, TONGSON

!+%
2. Invokes the right to change abode and travel; that the hold-departure order is already sufficient guarantee that he will not escape, thus the certification requirement is not necessary Issue: WON the bail is excessive. Held: YES. The bail pending appeal was reduced to 200k. Ratio: Imposing bail in excessive amount could render meaningless that right to bail thus the SC will not hesitate to exercise its supervisory powers over lower courts should the latter effectively deny the right to bail by imposing a prohibitory sum or exacting unreasonable conditions. The 5.5M is unreasonable, excessive, and constitutes an effective denial of petitioners right to bail. The CA order releasing petitioner on bail constitutes lawful order as contemplated by Sec 6. The condition imposed was simply consistent with the nature and function of a bail bond which is to ensure that Yap will make himself available at all times whenever the Court requires his presence. Petitioner is not prevented from changing abode but merely required to inform the court in case he does so.

"

1) when the person was charged with a capital offense; or 2) when the offense charged was punishable with reclusion perpetua -Life imprisonment was not among the exceptions, which means that persons accused of crimes punishable with life imprisonment were entitled to bail as a matter of right. Life imprisonment is different from reclusion perpetua o L.I. is invariably imposed for serious offenses penalized by special laws while reclusion perpetua is prescribed under RPC. o o L.I., unlike reclusion, does not carry with it any accessory penalty L.I. does not appear to have any definite extent or duration while reclusion entails incarceration for at least 30 years after which the convict becomes eligible for pardon.

-Although Rule 114 was recently amended denying any person who is charged with an offense punishable with death, reclusion perpetua or life imprisonment the right to bail, it cannot be retroactively applied in the present case since it would be unfavorable to the accused. NOTE: New Rule (when the imposable penalty for the offense charged is death, reclusion perpetua or life imprisonment)-admission to bail is addressed to the sound discretion of the court depending on whether the evidence of guilt is strong. -The prosecution has the burden of showing that evidence of guilt is strong so as to deprive an accused of the grant to bail. [Paderanga] Yap v CA

Facts:

Narciso v. Sta. Romana-Cruz (2000)

Petitioner was charged with parricide which is punishable with reclusion perpetua. He claims that he was entitled to bail because the evidence of his guilt was not strong. He argues that the prosecutors conformity to his Motion for Bail was tantamount to a finding that the prosecution evidence against him was not wrong. CA ruled that there was no basis for such finding since no hearing had been conducted on the application for bail. CA argues that only ten minutes had elapsed between the filing of the Motion and the Order granting bail, a lapse of time which is not sufficient for the court to receive and evaluate any evidence. Issue: WON petitioner is entitled to bail based on the prosecutors conformity to his motion.

Facts: Petitioner was convicted by the Pasig RTC of estafa and was sentenced to prision correccional to prision mayor. His motion to be allowed provisional liberty was denied by the trial court. He also file the same motion when the case was transmitted to the CA. The CA however upheld the Solicitor Generals recommendation to allow Yap to post bail in the amount of 5.5M, require Yap to produce a certification from the Mayor of the place of his residence and to remain to be so until final judgment.

Held: No. Ratio: Basco v. Rapatalo: The determination whether or not the evidence of guilt is strong, being a matter of judicial discretion, remains with the judge. o Since the discretion is directed to the weight of evidence and since evidence cannot properly be weighed if not duly exhibited or produced before the court, a proper exercise of judicial discretion requires that the evidence of guilt be submitted to the court . A hearing, whether summary or otherwise in the discretion of the court must actually be conducted. o Summary hearing - such brief and speedy method of receiving and considering the evidence of guilt as is practicable and consistent with the purpose of hearing which is merely to determine the weight of evidence for the purposes of bail

Petitioners Argument: 1. That the proposed bill is violative of his right against excessive bail; that the Rules of Court never intended for the civil liability to be a guideline for determining the amount of Bail; that the bail be reduced to 40k, the maximum amount that can be posted for estafa under the 1996 Bail Bond Guide.

CONSTI II DIGESTS: 1D COMPILATION

CHUA, ALIMANGOHAN, CONTRERAS, BRIONES, LENCIO, RELOJO, TENGCO, LIM, DUMA, DANAO, MELLA, TONGSON

!+&
Issue: W/N arraignment is necessary before bail hearings may be conducted. Ruling: The issue was moot since petitioner had already been arraigned on July 10, 2001 and a plea of not guilty had been entered by the Sandiganbayan on his behalf. HOWEVER, the Court took the opportunity to discuss the controlling precepts on the issue. Ratio: The Court agreed with the petitioner. The arraignment of an accused is not a prerequisite to the conduct of hearings on his petition for bail. A person is allowed to petition for bail as soon as he is deprived of his liberty by virtue of his arrest or voluntary surrender. An accused need not wait for his arraignment before filing a petition for bail. Bail is a matter of right and an accused may apply for and be granted bail even prior to arraignment.

"

No trial on the merits Left to the discretion of the court which may confine itself to receiving evidence on just substantial matters The absence of objection from the prosecution is never a grant of bail in such cases. Basco v. Rapatulo: Duties of the trial judge in petitions for bail 1. Notify the prosecutor of the hearing of the application for bail or require him to submit his recommendation. 2. Conduct a hearing of the application for bail regardless of whether or not the prosecution refuses to present evidence to show that the guilt of the accused is strong for the purpose of enabling the court to exercise its sound discretion. 3. Decide whether the evidence of guilt of the accused is strong based on the summary of evidence of the prosecution. 4. If the guilt of the accused is not strong, discharge the accused upon the approval of the bailbond. [Cabral] Serapio v. Sandiganbayan G.R. No. 148468, 28 January 2003 # #

Bail is a matter of right. An accused may apply for and be granted bail even prior to arraignment.
Facts: Petitioner was a co-accused in the plunder trial of President Joseph Estrada. Petitioner raised a petition before the high court contending that the Sandiganbayan committed gadalej when it deferred the hearing of his petition for bail to July 10, 2001 arraigned him on said date and entered a plea of not guilty for him when he refused to be arraigned. Respondents opposed the petition. Respondents, insist that arraignment is necessary before bail hearings may be commenced, because it is only upon arraignment that the issues are joined. Specifically, they argue that: 1. 2. It is only when an accused pleads not guilty that he may file a petition for bail. And that if he pleads guilty to the charge, there would be no more need for him to file said petition. Since it is during arraignment that the accused is first informed of the precise charge against him, he must be arraigned prior to the bail hearings to prevent him from later assailing the validity of the bail hearings on the ground that he was not properly informed of the charge against him. The arraignment of accused prior to bail hearings diminishes the possibility of an accused's flight from the jurisdiction of the Sandiganbayan because trial in absentia may be had only if an accused escapes after he has been arraigned. The conduct of bail hearings prior to arraignment would extend to an accused the undeserved privilege of being appraised of the prosecution's evidence before he pleads guilty for purposes of penalty reduction.

In Lavides the Court ruled that an application for bail in a case involving an offense punishable by reclusion perpetua to death may also be heard even before an accused is arraigned. Further, if the court finds in such case that the accused is entitled to bail because the evidence against him is not strong, he may be granted provisional liberty even prior to arraignment; for in such a situation, bail would be "authorized" under the circumstances. In fine, the Sandiganbayan committed gadalej in ordering the arraignment of petitioner before proceeding with the hearing of his petition for bail. [Leviste]

United States v Puruganan (2002) Facts: The United States of America requested the extradition of Mark Jimenez under the RP-US extradition treaty. Respondent's arguments: He should be allowed to bail and go on provisional liberty in accordance with Art 3 Sec 12 of the Constitution. Issue: Whether or not respondent Jimenez who was undergoing extradition proceedings should be allowed to bail. Held: NO. Bail is not a matter of right in extradition cases (exceptions in the last portion). Ratio:

3. 4.

CONSTI II DIGESTS: 1D COMPILATION

CHUA, ALIMANGOHAN, CONTRERAS, BRIONES, LENCIO, RELOJO, TENGCO, LIM, DUMA, DANAO, MELLA, TONGSON

!+'
As suggested by the use of the word conviction in Art 3 Sec 12 of the 1987 Constitution, the constitutional provision on bail applies only when a person has been arrested and detained for violation of Philippine criminal laws. It does not apply to extradition proceedings because they do not render judgments of conviction or acquittal. Constitutional right to bail flows from the presumption of innocence. Presumption of innocence is not at issue in extradition cases. It matters not if offenses for which Jimenez is sought are bailable in the US, extradition proceedings are separate and distinct from the trail for the offenses for which he is charged. Denial of bail would encourage the accused to voluntarily surrender to the requesting state to cut short their detention here. Notes: Issue on due process: Petitioners argument:His detention prior to the conclusion of the extradition proceedings does not amount to a violation of his right to due process. Court: Doctrine does not always call for a prior opportunity to be heard. Subsequent opportunity to be heard is enough. Also, before US requested his extradition, proceedings had already been conducted in that country. But because he left the jurisdiction of the US before those proceedings could be completed, it was hindered from contunuing with the due processes. Exceptions to the No Bail Rule: 1. That, once granted bail, the applicant will not be a flight risk or a danger to the community 2. That there exists special, humanitarian and compelling circumstances including, as a matter of reciprocity, those cited by the highest court in the requesting state when it grants provisional liberty in extradition cases therein. But since these exceptions have no express or specific statutory basis, applicant bears the burden of proving the requirements.

"

Issues in the case require an interpretation or construction of the treaty and law on extraditon. PD 1069 is intended as a guide for the implementation of extradition treaties to which the Philippines is a signatory, understanding the following postulates of extradition will aid us in properly deciding the issues. 1. Extradition is a major instrument for the suppression of crime " Extradition treaties are entered into for suppressing crimes by facilitating the arrest and the custodial transfer of a fugitive from one state to another " With the advent of easier and faster means of international travel, flight of affluent criminals from one country to another to commit crimes and evade prosecution has become more frequent. 2. The requesting state will accord due process to the accused " Our duly authorized representatives signature on the extradition treaty signifies our confidence in the capacity and willingness of the other state to protect the basic rights of the person sought to be extradited. 3. The proceedings are sui generis " Extradition proceedings are not criminal in nature. Extradition Proceedings Proceeding is sui generis in a class by itself where constitutional rights of the accused are not at fore. Does not involve determination of the guilt or innocence of the accused. His guilt will be adjudged in the court of the state where he will be extradited. (Constitutional rights are only relevant to determine the guilt or innocence of an accused.) Summary in nature Fugitive may be extradited upon showing of the existence of a prima facie case Our courts may adjudge an individual extraditable but the president has the final discretion to extradite him. Extradition court is not called upon to ascertain the guilt or innocence of the person. Person charged/convicted of a crime is restored to a jurisdiction with the best claim to try that person. 4. Criminal Proceedings Constitutional rights or the accused are at fore.

Full-blown trial Requires proof beyond reasonable doubt Judgment becomes executory upon being rendered final.

5.

Compliance shall be in good faith. " We are bound by pacta sunt servanda to comply in good faith with our obligations under the treaty. Otherwise, it will paint a bad image of our country before the world community. There is an underlying risk of flight. " Premise: persons sought to be extradited have the propensity to flee. " Respondent demonstrated that he has the capacity and the will to flee.

Rodriguez v. Judge (2006) FACTS: US Govt filed a petition for extradition against petitioners. RTC granted the application for bail for cash bond of 1M each; bail was subsequently cancelled upon the MR of US govt. Without prior notice of hearing, the trial court cancelled the cash bond and ordered issuance of warrant of arrest. Petitioner filed MR of the cancellation of their bail. Motion was heard and denied. Petitioners: (1) bail bond cannot be cancelled w/o due process (2) They are entitled to prior notice and hearing by analogy to Rule 114, Sec 21 of Rules of Court (surety is required to be notified and show cause why bail bond should not be canceled) (3) Their situation falls within the exception to general rule of no-bail Respondent: (1) prior notice and hearing is not required to cancel petitioners bail

CONSTI II DIGESTS: 1D COMPILATION

CHUA, ALIMANGOHAN, CONTRERAS, BRIONES, LENCIO, RELOJO, TENGCO, LIM, DUMA, DANAO, MELLA, TONGSON

!+(
that rebellion is a capital offense under pd nos 1996, 942 and 1834 which amended article 135 of RPC. On June 5, 1987, Pres. Aquino issued EO 187 repealing PD 1996,942 and 1834 and restoring in full effect art 135 of RPC. Thus, the original penalty for rebellion, prison mayor was re-imposed. Taking into consideration EO 187, Judge Donato ganted Salas petition for bail. Issue: WON Salas is entitled to bail Held: No. he waived his right. Ratio: The right to bail should not be rendered nugatory by requiring an excessive amount. The sole function of bail is to assure the accuseds presence at the trial. Bail cant be denied to Salas because the crime of rebellion is punishable only by prison mayor therefore making it a bailable offense under the constitution. However, Salas waived his right to bail. When Salas entered into an agreement with the government wherein his petition for habeas corpus will be withdrawn in exchange for the release of his co-accused, he expressly waived his right to bail. SEC 13: EXCESSIVE BAIL De la Camara v Enage (1971)

"

(2) Issuance of warrant of arrest ex parte against extradite is not a violation of DPC (3) Prior notice and hearing would defeat purpose of arrest warrant as it gives warning that respondent would be arrested and encourage them to flee (fallacy) (4) Granting that they are entitled to prior notice, such was afforded because in the grant of bail in the order given by trial court it reserved that such is subject to whatever ruling the SC may render in similar extradition case of US v. Purganan ISSUE/S: WON a prospective extradite is entitled to notice and hearing before the cancellation of his bail WON respondent judge committed grave abuse of discretion tantamount to lack or excess of jurisdiction in not considering special circumstances attendant to present case as exception to general rule of no bail in extradition cases HELD: (1) YES. Absent prior notice and hearing, bails cancellation was violation of her right to due process. (2) YES. Cancellation of petitioners bail without prior notice and hearing is considered a violation of right to due process tantamount to grave abuse of discretion RATIO: (1) Sec of Justice v. Lantion: SC ruled that notice and hearing should be afforded to extradite when possible extradition is still being evaluated US v. Purganan: possible extradite is not entitled to notice and hearing before issuance of warrant of arrest The issue here is whether a prospective extradite is entitled to notice and hearing before cancellation of bail. The trial courts immediate cancellation of bail of petitioners is contrary to ruling in Purganan; it misread and misapplied the directive therein. (2) bail may be granted to possible extradite only upon clear and convincing evidence that (1) he will not be a flight risk or danger to community (2) that there exists special, humanitarian and compelling circumstances In this case, co petitioner offered to go on voluntary extradition; her husband had already gone voluntary extradition and is undergoing trial in US; her passport is in possession of authorities; she never attempted to flee; there is a hold-departure order against her; she is now in her 60s, sickly and in medical treatment. [Olalia] SEC 13: WAIVER OF THE RIGHT People v Judge Donato & Rodolfo Salas Facts Rodolfo Salas alias commander bilog was granted bail by Judge Donato in the criminal case of rebellion filed against him. Prior to the granting of bail, Salas entered into an agreement with the prosecution under which he will remain under legal custody and will face trial before the court having custody over his person while the warrants for the arrest of his co-accused (Josefina Cruz and Jose Conception) will be deemed recalled and they shall be immediately released. On may 9, 1987, Salas filed a petition for bail but the prosecution opposed the petition that on the ground

Facts: De la Camara, mayor of Magsaysay, Misamis Oriental was arrested and detained of multiple murder and multiple frustrated murder. He then applied for bail because he says that he is innocent and nothing links him to that fatal incident. Respondent Judge Enage granted petitioners application but fixed the bail at an excessive amount of Php 1,195,200. Subsequently Secretary of Justice Vicente Abad Santos sent a telegram to the judge stating that the bond is excessive and instead Php 40k would be reasonable. Judge remained adamant. Petitioner Mayor then asks for relief to nullify judges order. Judges arguments: His decision finds support in circulars of the Department of Justice. Case has become moot and academic since mayor had escaped from the provincial jail

Counsel for petitioner argues that the issue here is the power of the trial courts in the fixing of bail. Issue: Whether or not bail of Php 1,195,200 is excessive. Held: Yes but he is not entitled for relief because of his subsequent escape. Case is dismissed for being moot and academic. Before conviction, every person is bailable except if charged with capital offenses when evidence of guilt is strong. This flows from the presumption of innocence in favor of the accused. A bail is intended as a mode of short confinement which woul insure the attendance of the accused for subsequent trial. Although the right to bail exists, it should not be rendered nugatory by requiring a sum that is excessive.

CONSTI II DIGESTS: 1D COMPILATION

CHUA, ALIMANGOHAN, CONTRERAS, BRIONES, LENCIO, RELOJO, TENGCO, LIM, DUMA, DANAO, MELLA, TONGSON

!+)

"

It is clear then that the amount of Php 1,195,200 is violative of the constitutional provision. There being two offenses charged, amount could not exceed Php 50k for murder and Php 25k for frustrated murder. Guidelines in fixing of bail: (Villasenor v Abao) 1. Ability of accused to give bail 2. Nature of the offense 3. Penalty for the offense 4. Character and reputation of accused 5. Health of accused 6. Character and strength of evidence 7. Probability of accused appearing in trial 8. Forfeiture of other bonds 9. Whether accused was a fugitive from justice when arrested 10. If accused is under bond for appearance at a trial in other cases Discretion is with the court called upon to rule on the question of bail. But if bail would amount to refusal of the conditions, the Supreme Court may exercise its supervisory powers to provide the required remedy. o

CONSTI II DIGESTS: 1D COMPILATION

CHUA, ALIMANGOHAN, CONTRERAS, BRIONES, LENCIO, RELOJO, TENGCO, LIM, DUMA, DANAO, MELLA, TONGSON

!+*
either his life or his liberty), the accused shall be entitled to, among others, a trial by judicial process, which is contemplated by the due process clause of the Constitution, and not a trial by executive or military process.

"

MILITARY TRIBUNALS Olaguer v. Military Commission No. 34 (1987) Due process contemplates judicial process / Civil courts have jurisdiction over civilians. Facts: Olaguer and several others [all civilians] were arrested by military authorities and were charged for subversion. They were initially detained at Camp Crame. All were transferred to Camp Bagong Diwa except for petitioner Olaguer who remained in detention at Camp Crame. Chief of Staff of AFP created Military Commission No. 34 to try the criminal case against the petitioners. An amended charge sheet was filed for 7 offenses including: Unlawfull possession of explosives Conspiracy to assassinate Pres. Marcos and others Arson of 9 buildings Attempted murder Conspiracy and proposal to commit rebellion and inciting to rebellion The respondent Military Commission found them guilty and sentenced them death by electrocution. Thus, petitioners filed this Petition and seek to enjoin the respondents from taking any further action on the case against the petitioners, and from implementing the judgment of conviction rendered by the respondent Military Commission No. 34.

Military commissions or tribunals are not courts within the Philippine judicial system. Judicial power exists only in the courts, which have 'exclusive power to hear and determine those matters which affect the life or liberty or property of a citizen.' In Toth v. Quarles, the U.S. SC further stressed that 'the assertion of military authority over civilians cannot rest on the President's power as Commander-in-Chief or on any theory of martial law.'' The presiding officer at a court martial is not a judge whose objectivity and independence are protected by tenure and undiminished salary and nurtured by the judicial tradition, but is a military law officer. Moreover, military tribunals pertain to the Executive Department of the Government and are simply instrumentalities of the executive power, provided by the legislature for the President as Commander-in-Chief to aid him in properly commanding the army and navy and enforcing discipline therein, and utilized under his orders or those of his authorized military representatives. Following the principle of separation of powers underlying the existing constitutional organization of the Government of the Philippines, the power and the duty of interpreting the laws (as when an individual should be considered to have violated the law) is primarily a function of the judiciary. We agree with the dissenting views of then Chief Justice Claudio Teehankee and madame Justice Cecilia Munoz-Palma in Aquino, Jr. v. Military Commission No. 2. in so far as they hold that military commissions or tribunals have no jurisdiction to try civilians for alleged offenses when the civil courts are open and functioning. The issue on the jurisdiction of military commissions or tribunals to try civilians for offenses allegedly committed before, and more particularly during a period of martial law, as well as the other issues raised by the petitioner have been ruled upon by a divided SC in Aquino, Jr. where the Court held that respondent Military Commission No.2 has been lawfully constituted and validly vested with jurisdiction to hear the cases against civilians, including the petitioner. And further held: The military tribunals were vested with jurisdiction 'exclusive of the civil courts,' among others, over crimes against public order, violations of the Anti-Subversion Act, violations of the laws on firearms, and other crimes which, in the face of the emergency, are directly related to the quelling of the rebellion and preservation of the safety and security of the Republic. It is essential that in a martial law situation, the martial law administrator must have ample and sufficient means to quell the rebellion and restore civil order. 'Public danger warrants the substitution of executive process for judicial process.' ***** additional info***** Proclamation No. 2045, officially lifting martial law in the Philippines abolished all military tribunals, created pursuant to the national emergency, effectively divests the respondent Military Commission No. 34 of its supposed authority to try civilians, including the herein petitioners. And all cases against civilians pending therein should eventually be transferred to the civil courts for proper disposition.

Petitioners arguments: The judgment rendered by the respondent is null and void. Military Commissions have no jurisdiction to try civilians for offenses alleged to have been committed during the period of martial law. Proceedings before the respondent court are in gross violation of their constitutional right to due process of law.
Issue: WON military commissions or tribunals have the jurisdiction to try civilians for offenses allegedly committed during martial law when civil courts are open and functioning. Held: NO. And as long as the civil courts in the land remain open and are regularly functioning, as they do so today and as they did during the period of martial law in the country, military tribunals cannot try and exercise jurisdiction over civilians for offenses committed by them and which are properly cognizable by the civil courts. Whether or not martial law has been proclaimed in the country or over a part thereof is of no moment. To have it otherwise would be a violation of the constitutional right to due process of the civilian concerned. Due process clause of the Constitution contemplates trial by judicial process, not executive. Ratio: Due process of law demands that in all criminal prosecutions (where the accused stands to lose

CONSTI II DIGESTS: 1D COMPILATION

CHUA, ALIMANGOHAN, CONTRERAS, BRIONES, LENCIO, RELOJO, TENGCO, LIM, DUMA, DANAO, MELLA, TONGSON

!!+

"

CONSTI II DIGESTS: 1D COMPILATION

CHUA, ALIMANGOHAN, CONTRERAS, BRIONES, LENCIO, RELOJO, TENGCO, LIM, DUMA, DANAO, MELLA, TONGSON

!!!
crime charged. The said section states that any evidence of solicitingsatisfactory to the courtshall be regarded as prima facie evidence that such solicitingwas contrary to law, and shall put upon the accused the burden of providing that such act was innocent and not with unlawful intention. Issue: Whether or not the provisions of Sec. 316 of Act No. 355 violates the presumption of innocence of the accused provided by the Constitution Held: No.

"

Tan vs. Barrios (1990) Facts: In 1987, the Supreme Court held in the Olaguer case that as long as civil courts are open and functioning, military tribunals cannot try and exercise jurisdiction over civilians for offenses committed by them. The present case comes on the heels of the Olaguer decision. Issue: WON the Olaguer decision should be applied prospectively only to future cases and cases still ongoing when the said decision was promulgated. Held: YES Ratio: a. There should be NO retroactive nullification of final judgments rendered by military courts against civilians before the promulgation of the Olaguer decision. Such final judgments should not be disturbed by the State. b. When can the final judgment be nullified and a retrial ordered? i. Only in particular cases where the convicted person or the State can show that there was a serious denial of the Constitutional rights of the accused. ii. Basis % violation of the Constitutional rights of the accused, NOT the Olaguer doctrine In the Olaguer case, Olaguer was rescued from a court martial which sentenced him to death without receiving evidence in his defense. It would be wrong and unjust to use the decision in Olaguer in order to REPROSECUTE civilians regardless: i. Of whether they were accorded a fair trial, unlike Olaguer; ii. Of whether they have already been acquitted and released; and iii. Of whether they have accepted their sentences and commenced serving the same. Not every person who was convicted by a military court, much less those who were acquitted and released, desires to undergo the ordeal of a second trial for the same offense, albeit in a CIVIL COURT. PRESUMPTION OF INNOCENCE United States v. Luling Facts: This is an appeal by the defendant after the Court of First Instance of Manila found him guilty of violating Section 316 of Act No. 355 of the United States Philippines Commission. The complaint alleged that herein appellant willfully, directly, unlawfully, and criminally, solicited and received money from one Rufino Elorz or Rufino Elord in order that he may secure through custom brokers, the importation and delivery to the Rufino Elorz or Elord, of certain rolls of paper in which a large quantity of opium was hidden. In this case, the appellant contends that the provisions of Sec. 316 of Act No. 355, in so far as it requires the accused to prove his innocence, is unconstitutional, and further that the evidence adduced during the trial of the cause does not show that he is guilty of the

Ratio: --While the rule is that every man is presumed to be innocent until his guilt is proved beyond reasonable doubt, it has been frequently decided that no constitutional provision is violated by a statute providing that proof by the state of some material fact of facts shall constitute prima facie evidence of guilt, and that the burden is shifted to the defendant for the purpose of showing that such act or acts are innocent and are committed without lawful intention. --The state having the right to declare what acts are criminal, within certain well defined limitations, has a right to specify what act or acts shall constitute a crime, as well as what proof shall constitute prima facie evidence of guilt, and then to put upon the defendant the burden of showing that such acts are innocent and are committed without unlawful intention.

c.

d.

People v. Mingoa Facts: Mingoa was a municipal treasurer entrusted with public funds, failed to account for the same claiming the monies were lost. He was charged with malversation. He now contends that he was convicted on mere presumption and that there was no direct evidence of actual misappropriation of public funds. He also questions the constitutionality of Art. 217 of the Revised Penal Code, which provides that in the event that the public officer accountable for public funds fails to produce such funds upon demand, such will be prima facie evidence that he put the same to personal use. He avers that it impairs his right to be presumed innocent. Issue: WON Art 217 of RPC violates the constitutional presumption of innocence. Held: NO. Ratio: The Supreme Court, citing Cooley, held that there is not constitutional objection of a law providing that the presumption of innocence may be overcome by a contrary presumption founded upon the experience of human conduct, and enacting what evidence shall be sufficient to overcome such presumption of innocence. Such is valid as long as there is a rational connection between the facts proved and the ultimate fact presumed. Art. 217 of the RPC merely creates a presumption of guilt once certain facts are proved and it merely establishes a prima facie evidence which the accused can rebut.

CONSTI II DIGESTS: 1D COMPILATION

CHUA, ALIMANGOHAN, CONTRERAS, BRIONES, LENCIO, RELOJO, TENGCO, LIM, DUMA, DANAO, MELLA, TONGSON

!!#
groupings are based on rational classification, which means that groupings are based on reasonable and real differentiations.

"

Dumlao v Comelec Facts: This is a petition to enjoin COMELEC from implementing Batas Pambansa Blg. 51,52,53 for being unconstitutional. Sec. 4 BP blg. 52 provides that a retired elective official of a province, city or municipality who has received payment of retirement benefits and who is 65 yrs old at the commencement of the office to which he seeks to be elected, shall be disqualified to run for the same position from which he retired. Also it provides that a candidate is disqualified from running for public office on the ground alone that charges have been filed against him before a civil or military tribunal Petitioners are Dumlao, who is a former governor of Nueva Vizcaya, who filed his certificate of candidacy for the 1980 elections and he questions the 1st paragraph of the provision; and Igot and Salapantan, who question the 2nd paragraph.

Teehankee, J., separate opinion: Provision is an arbitrary discrimination and is grossly violative of the equal protection clause because: classification (that would bar 65-yr old retires from running for the same elective office is not germane nor relevant to the alleged purpose of infusing new blood because old blood retirees may continue in local governments since they are not disqualified at all to run for other local elective office as long as it is not the same office from which they retired Age has never been a yardstick for qualification or disqualification. It is the right of the people to elect the candidate of their choice no matter his age. Persons similarly situated should be similarly treated.

Petitioners Arguments: (Dumlao)


Provision is directed against him (intentional discrimination) and that classification is based on purely arbitrary grounds and therefore, class legislation Provision is contrary to the safeguard of equal protection

Issues: Whether or not the 1st and 2nd paragraphs of Sec. 4 BP blg 52 are valid. Held: 1st paragraph is valid and 2nd paragraph is invalid. A partial declaration of nullity of objectionable portion is mandated Ratio: Validity of 1st Paragraph Argument of intentional discrimination is unacceptable because several petitions for disqualification of other candidates on the same ground have already been filed. Argument that it is contrary to the safeguard of equal protection is not well taken because a 65 yr old who has retired from an elective office and who has received retirement benefits is a reasonable classification because there is a need for new blood in political elective echelons and because of the tiredness of the retiree. There is also no clear showing of the invalidity of said provision. Laws shall not be declared invalid unless the conflict with the Constitution is clear beyond reasonable doubt. Invalidity of 2nd Paragraph All reasonable grounds should be resolved in favor of constitutionality however the challenged proviso contravenes the constitutional presumption of innocence. It condemns before one is fully heard. The effect is that there is no distinction between one convicted and one against whom charges have been filed. There is clear and present danger because of the proximity of elections and there would be no time to rebut charges filed against such candidate. Also, it is best that evidence be aired before the Courts rather than before an administrative body such as COMELEC Doctrine: Equal protection clause does not forbid all legal classification. What is proscribed is a classification which is arbitrary and unreasonable. This is not violated if: classification is germane to the purpose of the law and applies to all of the same class.

Marquez Jr v. COMELEC (1995) Facts: Marquez was a defeated candidate for the governorship of Quezon. Before the elections, he filed a petition for the cancellation Rodriguez certificate of candidacy on the ground of the latters disqualification under Sec 40(e) of the Local Government Code (LGC) which disqualifies from running for any elective local position fugitives from justice in criminal or nonpolitical cases here or abroad. A criminal charge (against Rodriguez) of 10 counts of insurance fraud or grand theft of personal property was still pending before the Los Angeles Municipal Court. An arrest warrant was yet to be served on account of his alleged flight. It was dismissed by the COMELEC so Marquez brought the case to the SC. The SC likewise dismissed the case, explaining that the disqualification had ceased to be a pre-proclamation case since Rodriguez had already been proclaimed. Nevertheless, the dismissal was without prejudice to the post-election quo warranto proceedings. Marquez then filed a quo warranto proceeding before the COMELEC which was dismissed. Hence this petition.

Petitioners Argument The disqualification under Sec40(e) is clear and needs no further interpretation. Fugitive from justice covers not only those who flee after conviction, but also those who after being charged, flee to avoid prosecution. (Solicitor General) Respondents Argument The construction placed upon by the Oversight Committee, which was in charge of formulating the Implementing Rules and Regulations (IRR) of the LGC, deserves great and considerable weight. An excerpt from the deliberations show that the committee had apprehensions on the possible constitutional infirmity if the disqualification would be made to embrace those who were merely facing criminal charges The committee finally defined it as a person who has been convicted by final judgment in Art 73 of the IRR of the LGC.

CONSTI II DIGESTS: 1D COMPILATION

CHUA, ALIMANGOHAN, CONTRERAS, BRIONES, LENCIO, RELOJO, TENGCO, LIM, DUMA, DANAO, MELLA, TONGSON

!!$
The court did not inform the accused of his right to have an attorney nor did it ask him if he desired the aid of one. o The court did not inquire whether the accused was to employ an attorney, to grant him reasonable time to procure one or to assign an attorney de officio. The question asked by the court to the accused was Do you have an attorney or are you going to plead guilty? o The question was so framed that it could have been construed by the accused as a suggestion from the court that he plead guilty if he had no attorney. o This is a denial of fair hearing in violation of due process. Under our rules of procedure, it is not enough for the Court to apprise an accused of his right to have an attorney, but it is essential that the court should assign one de officio for him if he so desires and he is poor or grant him a reasonable time to procure an attorney of his own. o

"

Issue: WON fugitive from justice should only be confined to persons who have been convicted by final judgment Held: NO. [The questioned COMELEC resolutions were thus reversed and the case was remanded to the COMELEC.] Ratio: An administrative rule or regulation can neither expand nor constrict the law but must remain congruent to it. Confining fugitive from justice to the definition under Art 73 was an inordinate and undue circumscription of the law. RIGHT TO COUNSEL People v. Holgado (1950) Facts: Frisco Holgado was charged in the CFI of Romblon with slight illegal detention. According to the information, Holgado, being a private person, feloniously and without justifiable motive, kidnap and detain one Artemia Fabreag in the house of Antero Holgado for about eight hours thereby depriving said Artemia Fabreag of her personal liberty.

United States v. Ash (1973) Look at all the Facts I give: One morning, two men with a stocking mask entered a bank in Washington, D.C., they ordered an employee to hang up the telephone and instructed all others present not to move, scooped up money from tellers' drawers into a bag, and left. Both men escaped through an alley. The robbery lasted three or four minutes. A Government informer, told authorities that he had discussed the robbery with Charles J. Ash, Jr., the respondent here. Acting on this information, an FBI agent, showed five black-and-white mug shots of Negro males of generally the same age, height, and weight, one of which was of Ash, to four witnesses. All four made uncertain identifications of Ash's picture. In preparing for trial, the prosecutor decided to use a photographic display to determine whether the witnesses he planned to call would be able to make in-court identifications. The five-member majority of the Court of Appeals held that Ash's right to counsel, guaranteed by the Sixth Amendment, was violated when his attorney was not given the opportunity to be present at the photographic displays conducted before the trial. Court of Appeals relied exclusively on that portion of the Sixth Amendment providing, In all criminal prosecutions, the accused shall enjoy the right to have the Assistance of counsel for his defense. The right to counsel has a rich historical heritage. We re-examine that history in an effort to determine the relationship between the purposes of the sixth Amendment guarantee the purposes the risks of a photographic identification.

IMPORTANT QUOTABLE QUOTES: Court: Do you have an attorney or are you going to plead guilty? Accused: I have no lawyer and I will plead guilty.
Issue: WON the accused was afforded due process. Held: No. Ratio: & KEYS TO SUCCESS Under Rule 112, Section 3 of the Rules of Court, when a defendant appears without attorney, the court has four duties to comply with: 1. It must inform the defendant that it is his right to have an attorney before being arraigned. 2. After giving him such information the court must ask him if desires the aid of an attorney. 3. If he desires and is unable to employ attorney, the court must assign attorney de officio to defend him. 4. If the accused desires to procure an attorney of his own the court must grant him a reasonable time therefor. In criminal cases there can be no fair hearing unless the accused be given (1) an opportunity to be heard (2) by counsel. o Without counsel, the accused may be convicted not because he is guilty but because he does not know how to establish his innocence.

Respondents arguments: This post-indictment identification provides the basis for respondent Ash's claim that he was denied the right to counsel at a "critical stage" of the prosecution.
Issue: Whether the Sixth Amendment grants an accused the right to have counsel present whenever the Government conducts a post-indictment photographic display, containing a picture of the accused, for the purpose of allowing a witness to attempt an identification of the offender.

&

AS APPLIED IN THIS CASE Not one of the duties stated in the provision had been complied with by the trial court.

CONSTI II DIGESTS: 1D COMPILATION

CHUA, ALIMANGOHAN, CONTRERAS, BRIONES, LENCIO, RELOJO, TENGCO, LIM, DUMA, DANAO, MELLA, TONGSON

!!%
In Coleman v. Alabama(1970), the accused was confronted by his adversary at a "critical stage" preliminary hearing at which the uncounseled accused could not hope to obtain so much benefit as could his skilled adversary. The analogy between the unrepresented accused at the pretrial confrontation and the unrepresented defendant at trial this court has stated: The trial which might determine the accuseds fate may well not be that in the courtroom but that at the pretrial confrontation The function of counseling is rendering Assistance continued at the lineup. Although the accused was not confronted there with legal questions, the lineup offered opportunities for prosecuting authorities to take advantage of the accused. Counsel present at line-up would be able to remove disabilities of the accused in precisely the same fashion that counsel compensated for the disabilities of the layman at trial. o

"

Held: The Sixth Amendment does not grant an accused the right to have counsel present when the Government conducts a post-indictment photographic display, containing a picture of the accused, for the purpose of allowing a witness to attempt an identification of the offender. Ratio from the book?! I did my best, I did my best: Powell v Alabama, the court discuss the English common law rule that severely limited the right of a person accused of a felony to consult with counsel at trial. o Sixth Amendment counsel guarantee thus was derived from colonial statutes and constitutional provisions designed to reject the English common law rule. o One consideration was the inherent irrationally of the English limitation. Since the rule was limited to felony proceedings, the result, absurd and illogical, was that an accused misdemeanant could fully rely on counsel, but the accused felon, in theory at least, could consult counsel only on legal questions that the accused proposed to the court. This Court has recognized the function of counsel as a guide through complex legal technicalities; an unaided layman had little skill in arguing the law or in coping with an intricate procedural system. Another factor contributing to the colonial recognition of the accuseds right to counsel was the adoption of the institution of the public prosecutor from the Continental inquisitorial system. o Innovation was brought about by the lack of lawyers, particularly in the newly settled regions, and by the increasing distances between the colonial capitals on the eastern seaboard and the ever-receding western frontier. The result was that the accused in the colonies faced government official whose specific functions it was to prosecute, and who was incomparably more familiar than the accused with the problems of procedure, the idiosyncrasies of juries, and, last but not the least the personnel if the court. o Thus an additional motivation for the American rule was a desire to minimize the imbalance in the adversary system that otherwise resulted with the creation of a professional prosecuting official. Historical background suggests that the core purpose of the counsel guarantee was to assure Assistance at trial, when the accused was confronted with both the intricacies of the law and the advocacy of the public prosecutor. Developments have led this Court to recognize that Assistance would be less than meaningful if it were limited to the formal trial itself. Extension of the right to counsel to events before trial has resulted from changing patterns of criminal procedure and investigation that have tended to generate pretrial events that might appropriately be considered to be parts of the trial itself. Court consistently has applied a historical interpretation of the guarantee, and has expanded the constitutional right to counsel only when new contexts appear presenting the same dangers that gave birth initially to the right itself. Recent cases demonstrate the historical method of this expansion. o In Hamilton v. Alabama(1961) and in White v. Maryland(1963), the accused was confronted with the procedural system and was required, with definite consequences, to enter a plea. o In Massiah v. United States(1964), the accused was confronted by prosecuting authorities who obtained, by ruse and in the absence of defense counsel, incriminating statements.

People v. Liwanag (2001) Effective counsel means reasonableness under prevailing professional norms Facts: Accused Liwanag was charged with Highway Robbery with Multiple Rape. He was provided with counsel de officio in Atty. Uy who assisted him during arraignment and trial. In the middle of the trial, accused-appellant obtained the services of counsel de parte Atty. Bienvenido R. Brioso, replacing Atty. Uy. Accused was convicted and sentenced to reclusion perpetua. He challenged his conviction arguing that he was not provided with intelligent counsel during arraignment and at the beginning of the trial. (Particular arguments in ratio) Issue: W/N accuseds right to counsel was violated. Ruling: NO. (See ratio for specific rulings.) Ratio: In assessing the effectiveness of counsels assistance, the Strickland (explained below) standard invoked by accused-appellant is too stringent for application in Philippine judicial setting. Strickland only seeks to ensure that the adversarial testing process is present in a case by requiring that the assistance rendered by counsel be "effective". The presence of an adversarial testing process, in other words, ensures that the trial is fair by according the accused due process through the "effective" assistance of counsel. While fairness is likewise the object of Article III, Section 14 (2) of the Philippine Constitution, the assistance afforded by counsel to an accused in light of the Philippine constitutional requirement need only be in accordance with the pertinent provisions of the Rules of Court, the Code of Professional Responsibility and the Canons of Professional Ethics. In Philippine judicial setting, a counsel assisting an accused is

CONSTI II DIGESTS: 1D COMPILATION

CHUA, ALIMANGOHAN, CONTRERAS, BRIONES, LENCIO, RELOJO, TENGCO, LIM, DUMA, DANAO, MELLA, TONGSON

!!&
error. The technicality cannot render the subsequent proceedings void and deprive the State of its right to convict the guilty when the facts on the record point to the culpability of the accused. AA 3: Accused was deprived of his right to a preliminary investigation. Had the counselde oficio been effective, counsel should have filed the proper motion on behalf of the accused. Court: Accused-appellant submitted himself to the jurisdiction of the trial court. He is deemed to have waived his right to preliminary investigation. People v. Buluran: The failure to accord appellants their right to preliminary investigation did not impair the validity of the information nor affect the jurisdiction of the trial court. While the right to preliminary investigation is a substantive right and not a mere formal or technical right of the accused, nevertheless, the right to preliminary investigation is deemed waived when the accused fails to invoke it before or at the time of entering a plea at arraignment.

"

presumed to be providing all the necessary legal defenses which are reasonable under the circumstances in accordance with said norms. Accused argument 1: The right to be effectively assisted by counsel necessarily means the right to be assisted by intelligent counsel. Court: Under the law, the requirement is not for counsel to be "intelligent", but to be effective. The meaning of effective counsel was defined in People v. Lucero, to wit: xxx [T]his Court has held in the same case that when the Constitution requires the right to counsel, it did not mean any kind of counsel but effective and vigilant counsel. The requirements of effectiveness and vigilance of counsel during that stage before arraignment were for the purposes of guarding against the use of duress and other undue influence in extracting confessions which may taint them and render them inadmissible. The right to be heard by counsel simply refers to the right to be assisted by counsel for the purpose of ensuring that an accused is not denied the collateral right to due process, a fundamental right which cannot be waived by an accused. AA 2: According to Strickland v. Washington, the assistance rendered by counsel is ineffective or defective if any of the following elements are present:

a. b.

counsels performance was deficient, which requires a showing that counsel was not functioning as the counsel guaranteed the defendant by the Sixth Amendment; the deficient performance prejudiced the defense, which requires a showing that counsels errors were so serious as to deprive the defendant of a fair trial.

People vs. Larranaga, et al. Facts: Appellants assailed the trial court for the following reasons: 1. For appointing counsel de oficio despite their insistence to be assisted by counsel of their own choice 2. For refusing to suspend trial until they shall have secured the services of new counsel They requested either one month or three weeks to look for new counsel. Issue: WON the appellants were denied their right to counsel Held: No. Ratio: There is no denial of the right to counsel where a counsel de oficio was appointed during the absence of the accuseds counsel de parte, pursuant to the courts desire to finish the case as early as practicable under the continuous trial system. It was the strategic machinations of appellants and their counsel de parte which prompted the trial court to appoint counsel de oficio The unceremonious withdrawal of appellants counsel de parte during the proceedings and their stubborn refusal to return to the court for trial undermines the continuity of the proceedings The appointment of counsel de oficio under such circumstances is not proscribed by the Constitution. The preference in the choice of counsel pertains specifically to a person under investigation rather than an accused in a criminal prosecution Even if the application of the concept of preference in the choice of counsel is extended to an accused in a criminal prosecution, such preferential discretion is not absolute as

Accused argued that the assistance afforded him by his counsel during the course of the trial was "ineffective" since the counsel de officio failed to safeguard his rights necessary for the reversal of his conviction. In particular, accused-appellant contends his counsel de officio failed to safeguard was his right to be secure in his person against unreasonable searches and seizures as enshrined in the Bill of Rights. He claims that his right was violated when he was arrested without a warrant which his counsel should have contested.
Court: Any objection involving a warrant of arrest must be made before he enters his plea, otherwise the objection is deemed waived. People v. Costelo: Appellants failure to quash the information, his participation in the trial and presenting evidence in his behalf, placed him in estoppel to make such challenge. He has patently waived any objection or irregularities and is deemed as having submitted himself to the jurisdiction of the court. It should be noted that the legality of arrest affects only the jurisdiction of the court over the person of the accused. Consequently, if objection on such ground is waived, the illegality of the arrest is not sufficient reason for setting aside an otherwise valid judgment rendered after the trial, free from

CONSTI II DIGESTS: 1D COMPILATION

CHUA, ALIMANGOHAN, CONTRERAS, BRIONES, LENCIO, RELOJO, TENGCO, LIM, DUMA, DANAO, MELLA, TONGSON

!!'
The killing of Desilos, according to the trial court, was qualified as murder by the circumstances of treachery and evident premeditation. Hence, appellant was convicted of the complex crime of murder with assault upon an agent of a person in authority. Issue: Whether or not defendants could be convicted of murder with assault upon an agent of a person in authority. Held: NO. Convicting the appellant of a crime not properly alleged in the body of the information filed violates his right to be informed of the nature and cause of the accusation against him. Ratio: Information filed against appellant did not allege the essential element of assault that the accused then knew that, before or at the time of the assault, the victim was an agent of a person in authority. o The information barely alleged that it was done with deliberate intent to kill, with evident premeditation and treachery and taking advantage of nighttime, and that deceased, who was a member of the Philippine Constabulary , was attacked while he was then in the performance of his official duty. The mere fact that the attack was done while the deceased was in the performance of his official duties cannot be an adequate substitute for the essential averment to justify a conviction of the complex crime, which necessarily requires the imposition of the maximum period of the penalty prescribed for the graver offense. Subject information cannot be cured or validated by doctrine in Balbar (see notes) because unlike in the latter case, there are no allegations of facts from which it can be implied that the accused then knew that, before or at the time of assault, the victim was an agent of a person in authority. The fact the the crime of assault was established by the evidence of the prosecution without any objection on the part of accused cannot likewise cure the defect in the information. *Notes: Details on the sufficiency of information are dealt with in the Rules of Court and much of the litigation on the subject matter makes no reference to the constitutional guarantee. Preoccupation of the Court has been exclusively with the procedural aspect of the right. Hence, there has been no attempt, unlike the practice in American courts, to subsume (include) that void for vagueness characterization of statutes under this constitutional guarantee. People v Balbar: The information sufficiently alleges that the accused knew the fact that victim was a teacher, since she was in her classroom and engaged in the performance of her duties. He therefore knew that she was a person in authority, as she was so by specific provision of law. It matters not that such knowledge on his part is not expressly alleged, complainant's status as a person in authority being a matter of law and not of fact, ignorance whereof could not excuse non-compliance on his part.

"

would enable him to choose a particular counsel to the exclusion of other equally capable Reason: the word preferably in Section 12 does not convey that the choice of a lawyer by a person under investigation is exclusive as to preclude other equally competent and independent attorneys from handling his defense because if the rule were otherwise, the tempo of a custodial investigation will be solely in the hands of the accused who can impede or obstruct the progress of the interrogation by simply selecting a lawyer who is not available

The choice of counsel by the accused in a criminal prosecution is also not a plenary one. If the chosen counsel deliberately makes himself scarce, the court is not precluded from appointing a de oficio counsel whom it considers competent and independent to enable the trial to proceed until the counsel of choice enters his appearance There is also no violation of the right to counsel just because the trial court did not grant their request for suspension of the hearing pending their search for new counsel. An application for a continuance in order to secure the services of counsel is ordinarily addressed to the discretion of the court The right of the accused to select his own counsel must be exercised in a reasonable time and manner The period asked by the appellants are unreasonable. They could have hired new lawyers at a shorter time had they wanted to. They should have been diligent in procuring new counsel. It has been held that where the accused declined the courts offer to appoint counsel and elected to defend himself, the denial of his motion made toward the end of the trial for a continuance so that he could obtain counsel of his own choice was not an infringement of his constitutional rights While the accused has the right to discharge or change counsel at any time, this right is subject to supervision by the trial court, particularly after the trial has commenced The court may deny accuseds application to discharge his counsel where it appears that such application is not made in good faith but is made for purposes of delay RIGHT TO BE INFORMED

People v Regala (1982) Facts: Defendants Rudy Regala and Delfin Torres were charged with the crime of murder with assault upon an agent of a person in authority in an information filed by the provincial fiscal of Masbate with the Court of First Instance of Masbate, which reads: xxx The accused, conspiring together xxx, with deliberate intent to kill, with evident premeditation and treachery and taking advantage of nighttime, did then and there willfully, unlawfully and feloniously attack and stab with a knife one Sgt. Juan Desilos, Jr., a member of the Philippine Constabulary while he was then in the performance of his official duty, inflicting upon the latter serious stab wounds xxx which injury directly caused his instantaneous death.

Enrile v. Salazar (1990) FACTS:

CONSTI II DIGESTS: 1D COMPILATION

CHUA, ALIMANGOHAN, CONTRERAS, BRIONES, LENCIO, RELOJO, TENGCO, LIM, DUMA, DANAO, MELLA, TONGSON

!!(
People v. De la Cruz Facts: The information charged the accused (De la Cruz) of willfully, unlawfully and feloniously commit sexual abuse on his daughter either by raping her or committing acts of lasciviousness on her Issue: Whether or not the information is a valid one Held: Allegation in the information is not sufficient averment of the acts constituting the offense Ratio:

"

Sen Juan Ponce Enrile, spouses Rebecco and Erlinda Panlilio, and Gregorio Honasan were charged with crime of rebellion with murder and multiple frustrated murder allegedly committed during the period of the failed coup attempt from Nov26-Dec 10, 1990. Enrile was taken and held overnight at NBI headquarters, without bail, none having been recommended in the info and none fixed in the arrest warrant. Enrile filed, through counsel, for petition for habeas corpus alleging that he was deprived of constitutional right. ISSUE: WON Enrile was deprived of Constitutional right in being held to answer for criminal offense which does not exist in the statute books. HELD: Yes, petitioners contention was technically correct. Based on the doctrine People v. Hernandez, questioned info filed against Enrile et al, must be read as charging simple rebellion only. RATIO: In vote of 11-2, Hernandez remains binding doctrine operating to prohibit the complexing of rebellion with any other offense committed in the occasion thereof, either as means to its commission or as an unintended effect of an activity that constitutes rebellion. People vs. Hernandez: Murders, arsons, robberies are mere ingredients of rebellion as means necessary for the perpetration of offense of rebellion.

People v. Legazpi (1995) Facts: Pamela was charged under 2 separate informations, one for double murder, the other for violation of RA No. 6539 (Anti- Carnapping Act). She was convicted of the complex crime of robbery w/ double homicide, combining and complexing the two crimes alleged in separate informations. Issue: W/N the accuseds right to information was violated Held: Yes. There conviction can only be limited to the crime alleged or necessarily included in the allegations in the separate informations. What controls is the description of the offense, as alleged in the information. While the court can hold a joint trial of two or more cases and can render a consolidated decision, the court cannot combine 2 crimes charged in 2 separate information to form a complex crime and convict the accused therefor. It violates the right of the accused to be informed of the nature and cause of the accusations against him.

Facts in the information do not constitute an offense o Does not cite sections or subsections of RA No. 7610 o Does not state acts and omissions constituting the offense, or any special or aggravating circumstances attending the same Section 8, Rule 110 o Designation of the Offense: ! shall state designation of offense given by the statute, acts or omissions constituting the offense ! reference shall be made to the section or subsection of the statute punishing it o Allegation in the information does not constitute requirements under Section 8 ! These are conclusions of law, not facts ! Information is therefore void for being violative of accusedappellants constitutionally-guaranteed right to be informed of the nature and cause of the accusation against him

Note: Qualifying Circumstance Must be alleged in the information to be appreciated People v. Tagud: failure to allege in the information the relationship of the victim to the offender bars the accused-appellants conviction for rape in its qualified form o there must be complete allegations on the particulars of the indictment ! requirement is based on the right of the accused to be fully informed of the nature of the charges against him People v. Mendez: no less than an accurate description in the information of the inculpatory relationship that would aggravate the offense to one of qualified rape o This is to satisfy the constitutional requirement that an accused should be properly informed of the nature and cause of the accusation

CONSTI II DIGESTS: 1D COMPILATION

CHUA, ALIMANGOHAN, CONTRERAS, BRIONES, LENCIO, RELOJO, TENGCO, LIM, DUMA, DANAO, MELLA, TONGSON

!!)
People v. Purazo (2003) Time need not be stated accurately in the complaint for the crime of rape. Facts: The accused Purazo was convicted in the case of incestuous rape. He argues that the criminal complaint filed against him failed to specify the specific time when the alleged rape was committed violating his right to b informed of the nature and cause of the accusation against him; and that the trial court erred in giving full faith and credence to the testimonies of the prosecution witnesses. Issue: WON the allegation that the rape was committed without giving the specific time is so vague that it did not give the accused a chance to make an intelligent defense.

"

People v Esperanza Facts: An information alleges that accused committed rape as follows: In Albay, Nelson Esperanza, thru force and intimidation, and with lewd design, did then and there willfully, unlawfully and feloniously had sexual intercourse with his niece, IRMA P. ESPERANZA, who is of tender age, she being only 13-years old, against her will and consent. The tc found Nelson guilty of 8 counts of rape against her niece and sentencing him in each count to death penalty.

Nelsons Argument: Granting without admitting that he committed rape, the tc erred in imposing the supreme penalty of death. While the informations alleged that Irma was 13 years old at the time the rapes were committed, they did not allege that Nelson is a relative of Irma within the third civil degree of consanguinity. With this, Nelson should have been meted the penalty of reclusion perpetua.
Issue: Whether or not penalty of death can be imposed on Nelson. Held: NO. The informations charges Nelson with simple rape only because while it alleges that the victim was only 13 yrs old, there is no allegation that Nelson is her relative by consanguinity within the 3rd civil degree. Twin circumstances of minority and relationship under Article 335 of the RPC, are in the nature of qualifying circumstances because they alter the nature of the crime of rape and increase the penalty. As special qualifying circumstances they must be specifically pleaded or alleged with certainty in the information; otherwise, the death penalty cannot be imposed. o In these cases, the allegation that Irma is Nelsons niece is not specific enough. If the offender is merely a relation - not a parent, ascendant, stepparent, guardian, or common law spouse of the mother of the victim the specific relationship must be alleged in the information. The informations in these cases merely allege that Irma is the niece of Nelson. She could be a niece beyond the third civil degree either of consanguinity or affinity. Hence, the informations are fatally defective in this respect. Although granting that relationship within the third civil degree either of consanguinity or affinity was duly proved, such proof cannot be appreciated against Nelson to justify the imposition of the death penalty because he would be denied of his constitutional right to be informed of the nature and cause of the accusation against him. He cannot be charged with rape in its simple form and then be tried and convicted of rape in its qualified form. Although it was alleged that Irma was 13, her birth certificate reveals that she was only 11. These cases could have been cases of statutory rape. Nelson cannot be convicted of statutory rapes. o

Held: No. The allegation that sometime in the month of March 1997, and for sometime subsequent thereto, is sufficient to uphold the conviction of the accused for a rape he had committed one Sunday of April 1997. The 2 are not so remote to each other as to surprise and prejudice the accused. The date is not an essential element of the crime of rape, for the essence of the complaint is carnal knowledge of a woman; thus time or place need not be accurately stated. Ratio: Conviction may be had on proof of the commission of the crime, even if it appears that the crime was not committed at the precise time or place alleged, provided it appears that the specific crime charged was in fact committed prior to the date of the filing of the complaint or information within the period of the statute of limitations and at the place within the jurisdiction of the court. Sec. 11, Rule 110 of the Rules of Court provides that it is not necessary to state in the complaint the precise time at which the offense was committed, except when time is a material ingredient of the offense. SC reiterated that the time averred in the complaint would only need to meet 2 criteria: 1. it is as near to the actual date of the commission of the offense; and 2. the time ultimately proved should be as so alleged in the complaint.

People vs. Moreno: Accused cannot be convicted under paragraphs 2 or 3 of Article 335 of the RPC in an information charging him with rape under paragraph 1 because none of these modes were alleged in the information. The rationale for this rule is that [t]o convict him under either of these statutory provisions is to deprive him of the constitutional right to be informed of the accusation against him.

Garcia v. People (2003) Facts: Petitioner basically claims that her constitutional right to be informed of the nature and cause of the accusation against her was violated because, although she was charged with estafa under Article 315, Section 2[a], as amended, which penalizes false manifestations or fraudulent representations in defraudation of another, she was instead convicted of estafa under Article 315, Section 2[d] which penalizes the issuance of postdated checks that were not funded or were insufficiently funded. Issue: WON the petitioners constitutional right to be informed was violated.

CONSTI II DIGESTS: 1D COMPILATION

CHUA, ALIMANGOHAN, CONTRERAS, BRIONES, LENCIO, RELOJO, TENGCO, LIM, DUMA, DANAO, MELLA, TONGSON

!!*
--Where a prosecuting officer, without good cause, secures postponements of the trial of a defendant beyond a reasonable period of time, the accused is entitled to relief by a proceeding in mandamus to compel a dismissal of the information, or if he be restrained of his liberty, by habeas corpus to obtain his freedom. NOTE: The Right to Speedy Trial It can work for or against the accused. o Since the prosecution has a burden of proof, delay in the trial occasioning disappearance of witnesses and the dimming of memories can actually lessen the chances of conviction. o Similarly, the accuseds own witness can disappear or suffer similar dimming of memory. Factors to consider: o Length of delay o Reason of delay o The effect of defendants assertion of his right o The prejudice caused the defendant The accused cannot agree to the repeated postponement of the trial of his case and then, when he finds the government absent or unable to go trial on any of the dates of hearing, take advantage of said absence and ask for the dismissal of his case. o The constitutional privilege was never intended as furnishing a technical means for escaping trial. The right to a speedy trial should not be used to deprive state of reasonable opportunity of fairly indicting criminals. The right to speedy trial is relative, subject to reasonable delays and posptponements. (e.g. illness, medical attention etc.) This right may be waived but must not be inferred from mere failure of the accused to urge the trial of the case. o It may only be presumed when the postponement of the trial has been sought and obtained by the accused himself or by his attorney. The right includes right to a prompt rendition of judgment. Delay in a case is counted from the time it is filed. o The accused cannot claim his right to a speedy trial was violated when the case was filed nine months after his arrest and investigation. o A delay in the refilling of the case is not a delay in the trial.

"

Held: NO Ratio: Section 14(2) of Article III of the 1987 Constitution provides that an accused has the right to be informed of the nature and cause of the accusation against him. Indeed, Section 6, Rule 110 of the Revised Rules of Criminal Procedure requires that the acts and omissions complained of as constituting the offense must be alleged in the Information. Section 8 thereof provides that the Information shall state the designation of the offense given by the statute and aver the acts or omissions constituting the offense. The real nature of the crime charged is determined by the facts alleged in the Information and not by the title or designation of the offense contained in the caption of the Information. It is fundamental that every element of which the offense is comprised must be alleged in the Information. What facts and circumstances are necessary to be alleged in the Information must be determined by reference to the definition and essential elements of the specific crimes. A careful reading of the Information clearly shows that petitioner was charged with estafa under Article 315, paragraph 2 (a) of the Revised Penal Code. There is no basis for petitioner to conclude that she was convicted for estafa under Article 315, paragraph 2(d) of the Revised Penal Code. While the typographical error in the dispositive portion of the trial courts decision did not help in clearing this matter, the body of the trial courts decision clearly discusses the elements of estafa under Article 315, paragraph 2(a). RIGHT TO SPEEDY TRIAL Conde v. Rivera Facts: Aurelia Conde has been forced to respond to no less than 5 informations for various crimes and misdemeanors, has appeared with her witnesses and counsel at hearings no less than on 8 different occasions only to see the cause postponed, has twice been required to come to the Supreme Court for protection, and now, after the passage of more than one year from the time when the first information was filed, seems as far away from a definite resolution of her troubles as she was when originally charged. Issue: Whether the petitioner was denied the right to speedy trial Held: Yes. Ratio: --In all criminal prosecutions, the accused shall enjoy the right to have a speedy trial. --The postponement of trial and her dismissal from her humble position was arbitrary and detrimental to the public. --The prosecution could have settled and attended the information, preliminary examination, and the case free from vexations, capricious, and oppressive delays.

Nepomuceno vs. Sec of Defense Facts: Nepomuceno along with several others have yet to be arraigned for their respective offenses before the Military Tribunal. This is due to the fact that they filed a Motion to Quash, and then a

CONSTI II DIGESTS: 1D COMPILATION

CHUA, ALIMANGOHAN, CONTRERAS, BRIONES, LENCIO, RELOJO, TENGCO, LIM, DUMA, DANAO, MELLA, TONGSON

!#+
The motion for suspension was granted but the judge denied the motion for disqualification. Senator Diokno through a resolution, led the SC to require respondent judge to comment with a TRO being issued. Issue: WON the circumstance of a party having subscribed before respondent judge an extrajudicial statement later on repudiated by him as the product of intimidation would suffice to negate the degree of objectivity required by the Constitution [Mej mahaba siya, WON petitioners are entitled to the relief they seek, that is, prohibition against respondent judge] Held: YES. The TRO against respondent judge was made permanent. Ratio: Respondent judge could not be totally immune to Reyes assertion that there was intimidation exerted on him. It was he who attested to the due execution of the extrajudicial statement. Respondent judge was called upon to review a matter on which he had previously given opinion. It is this inroad in ones objectivity that is sought to be avoided by the law on disqualification. 1. Due process is satisfied when the degree of objectivity on the part of the judge is sufficient to reassure litigants of his being just. Justice Dizon: Due process of law requires hearing before an imparial and disinterested trinunal and that every litigant is entitled to nothing less than the cold neutrality of an impartial judge. Geotina v Gonzales: A judge has both the duty of rendering a just decision and the duty of doing it in a manner completely free from suspicion as to its fairness and as to his integrity. Gov v Abella, and Dais v Torres: If it appears that the litigant was not given a fair trial because of the judges bias, a new trial could be granted. 2. The Rules of Court provision on disqualification was revised to contain an additional paragraph A judge may, in the exercise of his sound discretion, disqualify himself from sitting in a case for just or valid reasons other than those mentioned above. Causes other than pecuniary interest, relationship or previous participation in the matter, could erode objectivity. Thus to preserve (1) the judges reputation for probity and objectivity and (2) the ideal of impartiality in the administration of justice, the best course for a judge is to disqualify himself if any such cause arises and becomes hard to resist. Pimentel v Salanga: A judge may not be legally prohibited from sitting in litigation. It is worth noting, however, that judges are subject to the frailties of man. A judges passion may be generated because of serious charges of misconduct against him by a suitor of his counsel. They should therefore exercise great care and caution before making up his mind to act or withdraw from a suit. To inhibit in good grace where another judge can hear the case, and where no prejudice would be occasioned, is to give meaning to Sec 1. 3. To disqualify themselves is not always desirable for it could amount to their being recreant (synonym: chicken haha) to their trust, disqualification is still cogent given that prejudice is unavoidable. Luque v Kayanan: Next in importance to rendering a righteous judgment is doin it in a manner as will beget no suspicion of the judges fairness and integrity. Thus, a judge should inhibit himself if his further continuance would not be in the best interest of justice.

"

Supplemental Motion questioning the constitutionality of the Military Commissions, and even asked for preliminary injunction to suspend the trials. Then they assert that their right to a speedy trial is violated and seek the dismissal of the case. The accused invoke their right to speedy trial as they have not yet been arraigned after a long delay. Issue: WON they can invoke their right to a speedy trial Held: NO. Ratio: Any delay in the disposition of Nepomucenos case is attributable to his own actions. Further any delay in filing the charges was brought about by the exigencies of martial law and by the circumstances of the case. The suspension of the privilege of the writ of Habeas Corpus affords a valid delay in filing of charges. Since the crime charged was of a nature involving a nationwide conspiracy, the suspension and the subsequent non-filing of the writ was justified. What the constitution prohibits are unreasonable and capricious delays. A delay to constitute denial of the right to speedy trial must be vexatious, capricious and oppressive. The delays in this case are neither unreasonable nor capricious and were brought about by the accuseds own actions. Further, the right may be waived. RIGHT TO AN IMPARTIAL TRIAL Mateo Jr v. Villaluz (1972) Facts: Four criminal actions for robbery in band with homicide were filed against Mateo Jr, Cruz, Roberto Martinez, and Ruben Martinez. The Martinezes filed a motion to dismiss prompting the prosecution to file their opposition to said motion. Respondent judge deferred to rule on said motions until the prosecution has presented its evidence against Rolando Reyes, another suspect in the robbery who was being tried separately. It appears that the deferral was due to Reyes extra-judicial statement signed and sworn to before the same respondent judge. Said extrajudicial statement had implicated petitioners. The prosecution filed a motion to present additional evidence. This was opposed by Mateo who claimed that to allow the prosec to do so would deprive him of a fair trial. Respondent judge granted the prosecs motion saying that such matter was within the courts sound discretion. Reyes was called as an additional witness and his extrajudicial statement was presented. Reyes however repudiated the statement saying that he executed it because he had been threatened by a government agent. The 4 petitioners then verbally moved for the suspension to allow them to file a motion to disqualify respondent judge.

Petitioners Contention Respondent judge in the exercise of his sound discretion, should disqualify himself from sitting in the case under Sec 1 Rule 137 of the Rules of Court (Sec 1) because Reyes had repudiated the statement he sworn to before the former. Respondent judge would have to pass upon that repudiation.

CONSTI II DIGESTS: 1D COMPILATION

CHUA, ALIMANGOHAN, CONTRERAS, BRIONES, LENCIO, RELOJO, TENGCO, LIM, DUMA, DANAO, MELLA, TONGSON

!#!
" Recordation, appeal, whatever other institutions might present themselves in the character of checks, would be found to operate rather as cloaks rather than checks; as cloaks in reality, as checks only in appearance.

"

Pareded v Gopengco: The restriction against appeal (or stay) where a trial judge denies a motion for disqualification is not absolute. It does not preclude a resort before the higher courts to determine whether the trial judge committed grave abuse of discretion in refusing to disqualify himself. 4. The discretion in the first instance belongs to the respondent judge but its exercise is subject to the SCs corrective authority. Where there is a possibility of a trial being tainted by partiality, the SC can step in to assure respect for the demands of due process. 5. To lessen the danger of being a participant in any event that might lend itself to the interpretation that impartiality has been compromised, lower court judges are advised to limit themselves to adjudication and to leave to others the role of notarizing declarations. RIGHT TO A PUBLIC TRIAL Re Oliver (1948) *discussion of concepts only. orig case down the abyss, nowhere to be found. Sorry. Angel L

STATUS QUO (a good law student would read this bearing in mind that this is a 1948 US case, so please do) Today, almost without exception, every state by constitution, statute, or judicial decision requires that all criminal trials be open to the public. Without exception, all courts have held that an accused is at the very least entitled to have his friends, relatives, and counsel present, no matter with what offense he may be charged. Gaines v. Washington: A criminal trial conducted in secret would violate procedural requirements of the Fourteenth Amendments due process clause, although there was no actual violation, since the trial courts order barring the general public had not been enforced.

HISTORY OF THE RIGHT TO A PUBLIC TRIAL The practice of guaranteeing a public trial has its roots in our English common law heritage. a. The guarantee to a public trial first appeared in state constitution in 1776. b. Then came the ratification in 1971 of the Federal Constitutions Sixth Amendment: In all criminal prosecutions, the accused shall enjoy the right to a speedy and public trial. The Anglo-American distrust for secret trials has been ascribed to the notorious use of this practice as an instrument for the suppression of political and religious heresies in disregard of the right of an accused to a fair trial, as practiced by: a. the Spanish Inquisition b. the English Court of Star Chamber c. to the French monarchys abuse of the letter de cachet RATIONALE Safeguard: The guarantee has always been recognized as a safeguard against any attempt to employ our courts as instruments of persecution. Effective Restraint: The knowledge that every criminal trial is subject to contemporaneous review in the forum of public opinion is an effective restraint on possible abuse of judicial power. Jeremy Bentham: " Suppose the proceedings to be completely secret, and the court, on the occasion to consist of no more than a single judge that judge will be at once indolent and arbitrary: how corrupt his inclination may be, it will find no check, at any rate no tolerably efficient check, to oppose it. " With publicity, all other checks are insufficient.

Garcia v. Domingo Giving a Fact: For the convenience of the parties the trial was held in the air conditioned chamber of the respondent judge Garcia. The complaint was under the premise that such act is in violation of the right to hold a public trial. A procedure had been agreed to beforehand by the other respondents as defendants, the hearings have been thus conducted on fourteen separate occasions without objections on their part, and without an iota of evidence offered to substantiate any claim as to any other person so minded being excluded from the premises. Issue: Meaning to be accorded the constitutional right to public trial. Did respondent judge commit a grave abuse of discretion stigmatizing as violative of such a guarantee the holding of the trial of the other respondents inside the chambers of city Judge Gregorio Garcia named as petitioner.

Held: It is not in violation of the right to a public trial since the trial was still open to public and there is no showing that the public was deprived to witness the trial proceeding. Ratio: There is no ambiguity on the word employed. The trial must be public. It possesses that character when anyone interested in observing the manner a judge conducts the proceedings in his courtroom may do so. There is to be no ban on such attendance. His being a stranger to the litigants is of no moment. No relationships to the parties need to be shown. The thought that lies behind this safeguard is the belief that thereby the accused is afforded further protection, that his trial is likely to be conducted with regularity and not tainted with the impropriety. There is the well recognized exception though that warrants the exclusion of the public where the evidence may be characterized as offensive to decency or public morals.

CONSTI II DIGESTS: 1D COMPILATION

CHUA, ALIMANGOHAN, CONTRERAS, BRIONES, LENCIO, RELOJO, TENGCO, LIM, DUMA, DANAO, MELLA, TONGSON

!##

"

There was no showing that the public was thereby excluded. It is to be admitted that the size of the room allotted the Judge would reduce the number of those who could be present. Such a fact though is not indicative of any transgression of this right. Courtrooms are not of uniform dimensions. It suffices to satisfy the requirement if a trial being public if the accused could have his friends, relatives and counsel present, no matter with what offense he may be charged.

Issue: W/N respondent Judge acted with gadalej when he denied the motion to serve written interrogatories to Dr. Academia; Ruling: No. Respondent Judge did not act with gadalej. Ratio: A. Comment of the Solicitor General: The constitutional guarantee could not be stretched to include the right to serve interrogatories on a witness living in abroad. 1. 2. Service of written interrogatories is not a compulsory or coercive process. It is be merely the delivery to a proposed deponent of a set or series of questions, the answers to which will constitute his deposition. Service of written interrogatories by itself, does not obtain for the person utilizing them the twofold objectives specified in the constitutional guarantee which are a. to secure the attendance of witnesses; and b. to secure the production of evidence in behalf of the accused.

Notes worth knowing, but not reciting: The 1935 Constitution, explicitly enumerated the right to a public trial to which an accused was entitled. o Merely reiteration of that appeared in the Philippine Autonomy Act of 1916, popularly known as the Jones Law. Such a right found expression in the Philippine Bill of 1902, likewise an organic act. Chairman of the Committee on the Bill of Rights, Delegate, later Justice, Jose P. Laurel, stressed: Trial should also be in public in order to offset any danger of conducting it in a illegal and unjust manner. Where, for reasons of security, the arraignment and the hearing were not in court but in the Bilibid Prison, but where (1) the public was not excluded (2) the accused was not prejudiced, and (3) the accused did not object, there was no denial of the right to a public trial. COMPULSORY PROCESS Fajardo v. Garcia (1980)

It merely apprises the proposed deponent of the questions which he is requested to answer. He may decline to give the deposition, which ultimately will be submitted as his testimony. Clearly, therefore, service of written interrogatories is completely different or worlds apart from the compulsory process that is established in the constitution and accorded as an inviolate right of the accused. B. There is no showing that the denial of the motion for the service of written questions amounted to an infirmity that was fatal to the proceedings. The fact that the petitioners were treated in the hospital by a doctor in question could be testified to by other witnesses, including the nurses who must have been present. The mechanical certificates describing the wounds and the length of the accuseds stay in the hospital could also be sufficient evidence. C. The court however, did not specifically rule on the scope of compulsory process. Court: In the light of what has been stated, it becomes obvious why as of now, there is no need to make a definite pronouncement on the scope of the expanded concept of the constitutional right to secure not only the attendance of witnesses but the production of evidence. All that the decision stands for is that the standard required for the grant of certiorari has not been met.

Note: This ponencia does not actually explain the scope of compulsory process. The Court here based its decision on the standards of certiorari. However, the clear implication it seems (and this is just my opinion, so feel free to ignore it) is that since the Court did not find that the Judge acted capriciously or whimsically (i.e. with grave abuse of discretion) in denying the motion, then the serving of written questions is not necessarily within the scope of compulsory process. This is the conclusion supported by the comment of the Solicitor General (which the Court did not categorically reject).
Facts: Petitioners Farjardo et. al. were charged with the crime of murder. At the time of their arrest, the accused hand wounds in different parts of their body. They were examined and treated by Dr. Academia. The doctor subsequently issued corresponding medical certificates. During the trial, the defense argued that Dr. Academias testimony on the examination and treatment of the wounds on the bodies of all of the accused would be crucial for the defense. By this time however, Dr. Academia had left the country and was residing in the United States. The defense then filed a motion that written questions be sent to Dr. Academia. Judge Garcia, respondent, denied the motion. Hence, this petition for certiorari.

CONSTI II DIGESTS: 1D COMPILATION

CHUA, ALIMANGOHAN, CONTRERAS, BRIONES, LENCIO, RELOJO, TENGCO, LIM, DUMA, DANAO, MELLA, TONGSON

!#$
the court held that there was an exception to the general rule that the accused should be present for purposes of identification even if there was a waiver the exception is when the accused unqualifiedly admits in open court after his arraignment that he is the person named as defendant in the case on trial

"

TRIAL IN ABSENTIA; RIGHT TO BE PRESENT Carredo vs. People Facts: Petitioner was charged with malicious mischief before the Municipal Trial Court of Malabuyoc, Cebu City. He deposited a cash bond for his provisional liberty. Upon arraignment, he entered a plea of not guilty and he filed a written waiver of appearance. In the letter, the accused waives his appearance during the trial or any stage thereof and he agrees that in case he fails to appear for trial despite due notice, his absence will be deemed as express waiver of his right to be present and the court may proceed with the trial of his case as if he were present. He admits that he could be identified by witnesses who are testifying at the time that the accused was not present. At the hearing, the prosecution moved for the recall of its principal witnesses for the purpose of identifying the accused who was not then present. Hence, the hearing was re-scheduled. Later, a subpoena was issued to petitioner who failed to appear on such date. The defense counsel justified petitioners absence in that the latters presence can no longer be required as he already filed a written waiver of appearance. However, the judge still issued an order for the arrest of petitioner, the confiscation of the cash and order the bondsman (the petitioner) to show cause why no judgment should be rendered against him. The RTC of Cebu City also denied the petition for certiorari and prohibition. Hence, this petition for certiorari. Issue: WON an accused who, after arraignment, waives his further appearance during the trial can be ordered arrested by the court for non-appearance upon summons to appear for purposes of identification. Held: Yes. The exception held in the case of presiding judge is not present. Ratio: In Aquino Jr. vs. Military Commission, a similar issue was presented, it was held that the order of respondent requiring the presence of the petitioner at all times should be modified in the sense that his presence should be required only when he is to be identified. Petitioner only admitted that he could be identified by witnesses who have testified at the time the accused was not present. The court held that accused should appear in order to be identified. In People vs. Presiding Judge, the accused was charged with murder before the RTC of Pangasinan. Upon his arraignment, he manifested orally in open court that he is waiving his right to be present during trial. The prosecuting official moved that the accused be compelled to appear and be present at the trial so he can be identified by the prosecution witnesses. The court sustained the position of the accused and held that the accused cannot be compelled to appear because of the presence of the exception. it was held that the accused was conferred a fundamental right to ignore the terms of the bond posted by him in accordance with his constitutional right to bail

Thus, the rulings in Aquino and Presiding Judge are different due to the statement of the accused. The exception in the case of people vs. presiding judge is not present in the case of Aquino. The statement of the accused in Aquino is not such unqualified admission contemplated in Presiding Judge. In the case at bar, petitioner only admits that he can be identified by the prosecution witnesses in his absence. He did not admit that he is the very person named as defendant in the case on trial. His admission is vague and far from unqualified. He thus cannot seek the benefit of the exception recognized in presiding judge. The provision of the Constitution authorizing the trial in absentia of the accused in case of his non-appearance after arraignment despite due notice simply means that he thereby waives his right to meet the witnesses face to face. An express waiver of appearance after arraignment is of the same effect. However, such waiver of appearance and trial in absentia does not mean that the prosecution is deprived of its right to inquire the presence of the accused for purposes of identification by its witnesses which is vital for the conviction of the accused. Such waiver does not mean a release of the accused from his obligation under the bond to appear in court whenever so required. In the case of People vs. Prieto, it was held that the 1973 Constitution now unqualifiedly permits trial in absentia even of capital offenses, provided that after arraignment he may be compelled to appear for the purpose of identification by the witnesses of the prosecution, or provided he unqualifiedly admits in open court after his arraignment that he is the person named as the defendant in the case on trial. The reason for requiring the presence of the accused, despite his waiver, is if allowed to be absent in all the stages of the proceedings without giving the peoples witnesses the opportunity to identify him in court, he may in his defense say that he was never identified as the person charged in the information and therefore, is entitled to an acquittal.

CONSTI II DIGESTS: 1D COMPILATION

CHUA, ALIMANGOHAN, CONTRERAS, BRIONES, LENCIO, RELOJO, TENGCO, LIM, DUMA, DANAO, MELLA, TONGSON

!#%
quasi-judicial hearings However, the right to a speedy disposition of a case, like the right to speedy trial, is deemed violated only when: o the proceedings is attended by vexatious, capricious, and oppressive delays, or o unjustified postponements of the trial are asked for and secured, or o even without cause or justifiable motive a long period of time is allowed to elapse without the party having his case tried. Equally applicable is the balancing test used to determine whether a defendant has been denied his right to a speedy trial, or a speedy disposition of a case for that matter o Conduct of both the prosecution and the defendant is weighed, and such factors as: ! length of the delay ! reasons for such delay ! assertion or failure to assert such right by the accused ! prejudice caused by the delay The delay in this case disregarded the Ombudsman's duty, as mandated by the Constitution and Republic Act No. 6770, to enforce the criminal liability of government officers or employees in every case where the evidence warrants in order to promote efficient service to the people. o The failure of said office to resolve the complaints that have been pending for almost four years is clearly violative of this mandate and the rights of petitioner as a public official. Circumstances obtaining in the instant case do not warrant or justify the length of time (4 years) it took the Ombudsman to resolve preliminary investigation. o There was an unexplained interval or inactivity close to 4 years, prior to the issuance of the resolution finding probably cause and directing the filing of the corresponding informations. SEC 17: RIGHT AGAINST SELF-INCRIMINATION US v. Navarro (1904) FACTS: The defendants Baldomero Navarro et al, were charged with the crime of illegal detention. The court convicted them to life imprisonment (Art 483). Art 481: A private person who shall lock up or detain another, or in any way deprive him of his liberty shall be punished with the penalty of prision mayor. Art 483: One who illegally detains another and fails to give information concerning his whereabouts, or does not prove that he set him at liberty, shall be punished with cadena temporal in its maximum degree to life imprisonment.

"

SEC 16: SPEEDY DISPOSITION OF CASES Lopez Jr v Ombudsman (2001) Facts: In this case, the preliminary investigation was resolved close to 4 years from the time all the counter and reply affidavits were submitted to the Office of the Ombudsman. After the last replyaffidavit was filed on February 28, 1995, it was only on July 17, 1998 that a resolution was issued recommending the filing of the corresponding criminal informations against the petitioner and the others. It took eight months or on February 27, 1999 for Deputy Ombudsman Margarito P. Gervacio, Jr. to approve the same and close to another year or on April 30, 1999 for Ombudsman Aniano Desierto to approve the recommendation. *Timeline: December 22, 1993: The Office of the Ombudsman received verified audit report of the COA special audit team relative to the purchase by DECS Region XII Office of school equipment and laboratory apparati . Finding the same sufficient in form and substance was docketed as OMB Case No. 34-93-2791. On the basis thereof, a preliminary investigation was conducted. March 1, 1994: The respondents were ordered to file their counter-affidavits. May 10, 1994: The Office of the Ombudsman received the counter-affidavits of respondents. February 29, 1995: The Office of the Ombudsman received the reply-affidavit of COA. July 17, 1998: Resolution was issued where graft investigation officer Ladrera recommended the filing of 30 informations against petitioner. February 27, 1999: The said recommendation was approved by Deputy Ombudsman for Mindanao Gervacio, and April 30, 1999: Recommendation was approved by Ombudsman Aniano Disierto. May 5, 1999: The informations were filed with the Sandiganbayan. Issue: Whether or not there was undue and unjustifiable delay on the part of the Ombudsman in resolving the complaint filed against the petitioner, which violated his constitutional right to a speedy disposition of the complaint against him. Held: Yes. The court resolves to directly dismiss the informations already filed before the Sandiganbayan against petitioner in the interest of the speedy disposition of cases (and considering that the long and unexplained delay in the resolution of criminal complaints against petitioner was not corrected by the eventual filing of the information). Ratio: Under the Constitution, any party to a case may demand expeditious action on all officials who are tasked with the administration of justice. o The constitutional right to a "speedy disposition of cases" is not limited to the accused in criminal proceedings but extends to all parties in: ! all cases, including civil cases administrative cases ! and in all proceedings, including judicial hearings

One convicted of simply depriving a person of his liberty may be imprisoned for a term of from six to twelve years and one convicted of depriving a person of his liberty and who shall not state his whereabouts or prove that he had set said person at liberty may be punished by imprisonment for a term of seventeen years four months and one day, to life, as in this case.
The counsel for the defendants claims that such is illegal, since 1902 Act passed by Congress, provides that:

CONSTI II DIGESTS: 1D COMPILATION

CHUA, ALIMANGOHAN, CONTRERAS, BRIONES, LENCIO, RELOJO, TENGCO, LIM, DUMA, DANAO, MELLA, TONGSON

!#&
Precisely the same of law (Boyd) applies to the case at bar. If the defendant does not do certain things, if he does not make certain statements or proofs, he is severely punished. It may be said that the defendant is only required to speak on one point in the case, that the prosecution must prove the illegal detention, and that the burden of showing the whereabouts only is put upon the defendant.

"

a. b. c. d.

Sec 5: ". . . no person shall be compelled in any criminal case to be a witness against himself." Sec 57 of General Orders, No. 58: defendant in a criminal case shall be presumed to be innocent until the contrary is proved Section 59: burden of proof of guilt shall be upon the prosecution. These provisions are in conflict with those of Article 483 they have the effect of repealing that section.

ISSUE: WON these provisions (Art 481, 483) violate constitutional rights of defendants (forcing a defendant to become a witness in his own behalf or to take a much severer punishment) HELD: Yes. The judgment is reversed and the defendants are found guilty of the crime defined and punished by Art 482 of the Penal Code. RATIO: It is the duty of the prosecution, in order to convict one of a crime, to produce evidence showing guilt beyond a reasonable doubt; and the accused cannot be called upon either by express words or acts to assist in the production of such evidence; nor should his silence be taken as proof against him. He has a right to rely on the presumption of innocence until the prosecution proves him guilty of every element of the crime with which he is charged. Acc to Mr. Justice Bradley (Boyd case): "Any compulsory discovery by extorting the party's oath . . . to convict him of a crime . . . is contrary to the principles of free government; it is abhorrent to the instincts of an Englishman; it is abhorrent to the instincts of an American. It may suit the purposes to despotic power but it cannot abide the pure atmosphere of political liberty and personal freedom." The provision that no one is bound to incriminate himself was established on the grounds of public policy and humanity of policy. If the party were required to testify, it would place the witness under the strongest temptation to commit the crime of perjury, and of humanity, because it would prevent the extorting of confessions by duress. Purpose: to wipe out such practices as formerly prevailed in these Islands of requiring accused persons to submit to judicial examinations, and to get testimony regarding the offense with which they were charged. If the disclosure thus made would be capable of being used against him as a confession of crime, or an admission of facts tending to prove the commission of an offense, such disclosure would be an accusation against himself.

In People vs. Courtney: ISSUE: whether or not a law permitting a person charged with crime to testify in his own behalf was constitutional or not. HELD: his omission or refusal to testify "should create no presumption against him." "A law which, while permitting a person accused of a crime to be a witness in his own behalf, should at the same time authorize a presumption of guilt from his omission to testify, would be a law adjudging guilt without evidence, and while it might not be obnoxious to the constitutional provision against compelling a party in a criminal case to give evidence against himself, would be a law reversing the presumption of innocence, and would violate the fundamental principles binding alike upon the legislature and the courts."

U.S. v. Tan Teng (1912) Facts: The defendant herein raped Oliva Pacomio, a seven-year-old girl. Tan Teng was gambling near the house of the victim and it was alleged that he entered her home and threw the victim on the floor and place his private parts over hers. Several days later, Pacomio was suffering from a disease called gonorrhea. Pacomio told her sister about what had happened and reported it to the police. Tan Teng was called to appear in a police line-up and the victim identified him. He was then stripped of his clothing and was examined by a policeman. He was found to have the same symptoms of gonorrhea. The policeman took a portion of the substance emitting from the body of the defendant and turned it over to the Bureau of Science. The results showed that the defendant was suffering from gonorrhea. He assails the admissibility of the evidence claiming that it was obtained in violation of his right against self incrimination. Issue: W/N his right against self-incrimination was violated Held: No. The right against self-incrimination is a prohibition against extracting from the defendants own lips, against his will, an admission of guilt. It does not cover all forms of compulsion but only testimonial compulsion. The examination does not call upon the accused to be a witness, nor does it compel him to render testimony or answer questions; the evidence obtained from him is not testimonial in nature and is not covered by the right. The accused was not compelled to make any admission or answer any questions, and the mere fact that an object found upon his body was examined seems no more to infringe the rule invoked than would the introduction of stolen property taken from the person of a thief.

controlling doctrine: Boyd vs. The United States (116 U.S. 616): The court stated that a compulsory production of a man's private papers to establish a criminal charge against himself, or to forfeit his property is unconstitutional. It is tantamount to compelling their production for the prosecution will always be sure to state the evidence expected to be derived from them as strongly as the case will admit of."

CONSTI II DIGESTS: 1D COMPILATION

CHUA, ALIMANGOHAN, CONTRERAS, BRIONES, LENCIO, RELOJO, TENGCO, LIM, DUMA, DANAO, MELLA, TONGSON

!#'

"

Mr. Wigmore, in his valuable work on evidence, in discussing the question before us, said: If, in other words, it (the rule) created inviolability not only for his [physical control] in whatever form exercised, then it would be possible for a guilty person to shut himself up in his house, with all the tools and indicia of his crime, and defy the authority of the law to employ in evidence anything that might be obtained by forcibly overthrowing his possession and compelling the surrender of the evidential articles a clearreductio ad absurdum. In other words, it is not merely compulsion that is the kernel of the privilege, . . . but testimonial compulsion. (4 Wigmore, sec. 2263.) Ratio: The main purpose of the provision of the Philippine Bill is to prohibit compulsory oral examination of prisoners before trial. or upon trial, for the purpose of extorting unwilling confessions or declarations implicating them in the commission of a crime. (People vs. Gardner, 144 N. Y., 119.) To admit the doctrine contended for by the appellant might exclude the testimony of a physician or a medical expert who had been appointed to make observations of a person who plead insanity as a defense, where such medical testimony was against necessarily use the person of the defendant for the purpose of making such examination. (People vs. Agustin, 199 N.Y., 446.) The doctrine contended for by the appellants would also prevent the courts from making an examination of the body of the defendant where serious personal injuries were alleged to have been received by him. The right of the courts in such cases to require an exhibit of the injured parts of the body has been established by a long line of decisions. Holt vs. U.S. But the prohibition of compelling a man in a criminal court to be a witness against himself, is a prohibition of the use of physical or moral compulsion, to extort communications from him, not an exclusion of his body as evidence, when it may be material. The objection, in principle, would forbid a jury (court) to look at a person and compare his features with a photograph in proof. Moreover we are not considering how far a court would go in compelling a man to exhibit himself, for when he is exhibited, whether voluntarily or by order, even if the order goes too far, the evidence if material, is competent. State vs. Miller It was not erroneous to permit the physician of the jail in which the accused was confined, to testify to wounds observed by him on the back of the hands of the accused, although he also testified that he had the accused removed to a room in another part of the jail and divested of his clothing. The observation made by the witness of the wounds on the hands and testified to by him, was in no sense a compelling of the accused to be a witness against himself. If the removal of the clothes had been forcible and the wounds had been thus exposed, it seems that the evidence of their character and appearance would not have been objectionable. (clothing and handprint was also used as evidence against the victim)

Villaflor v. Summers (1920) Facts: Emeteria Villaflor and Florentino Souingco were charged with the crime of adultery. The judge of the trial court, Hon. Pedro Concepcion, ordered the defendant to submit her body to examination of one or two competent doctors to determine is she was pregnant or not. The accused refused to obey the order on the ground that such examination of her person was a violation of the constitutional provisions relating to self-incrimination. She was found in contempt of court and was ordered to be committed to Bilibid Prison until she should permit the medical examination required by court.

Petitioners Argument: such bodily exhibition is an infringement of his right against selfincrimination as provided in the constitution
Issue: Whether or not the right against self-incrimination extends to the inspection of the body or is limited to testimony Held: The court held that the constitutional guaranty is limited to a prohibition against compulsory testimonial self-incrimination. An ocular inspection of the body of the accused is permissible provided several limitations. Ratio: Conflicting Jurisprudences: o People v. McCoy: forced to examination by physicians; court held that proceeding was in violation of the spirit and meaning of the Constitution wherein no person shall be compelled in any criminal case to be a witness against himself o State v Height (Justice McClain): court should lay general rule that a defendant can be compelled to disclose only those parts of the body which are not usually covered o Holt v US (Justice Holmes): prohibition of the use of physical or moral compulsion to extort communication from him, not an exclusion of his body as evidence when it may be material o Philippine Jurisprudence: constitutional limitation was said to be simply a prohibition against legal process to extract from the defendants own lips, against his will, an admission of guilt Court adheres to the proposition that the constitutional provision was and is merely to prohibit testimonial compulsion o Basis: history of the provision o Privilege as disclosed by the text writers was testimonial compulsion o Criminal laws are not to protect the guilty but to protect the innocent ! No evidence of physical facts can for any substantial reason be held to be detrimental to the accused except in so far as the truth is to be avoided in order to acquit a guilty person o On the defense that every person has a natural and inherent right to the possession and control of his own body:

CONSTI II DIGESTS: 1D COMPILATION

CHUA, ALIMANGOHAN, CONTRERAS, BRIONES, LENCIO, RELOJO, TENGCO, LIM, DUMA, DANAO, MELLA, TONGSON

!#(
[NOTE: There was yet no information filed against the petitioner for the supposed falsification, nor was he on trial under examination. The order was only for an investigation prior to the filing of information.] Issue: (1) WON the constitutional provision invoked by the petitioner prohibits the compulsion to execute the assailed order. (2) WON taking of sample of petitioners handwriting by the fiscal to determine whether he wrote certain documents supposed to be falsified, constitutes evidence against himself within the scope and meaning of the constitutional provision under examination. Held: YES and YES. The constitutional privilege is not limited precisely to oral testimony, but extends to all giving and furnishing of evidence disclosing of any fact which the accused had the right to hold secret.

"

o o

Between a sacrifice of ascertainment of truth to personal consideration, between disregard of the public welfare for refined notions of delicacy, law and justice cannot hesitate Constitutional guaranty is limited to a prohibition against testimonial selfincrimination Ocular inspection of the body is permissible: ! Provided that torture and force shall be avoided ! Due care should be taken not use violence and not to embarrass the patient any more than is absolutely necessary !

US vs Ong Siu Hong 1917 Facts: Counsel of Ong Siu Hong contends that his client was compelled to be a witness against himself. This was the result of forcing the accused to discharge the morphine from his mouth. Issue: Whether or not this constitutes self-incrimination. Held: No To force a prohibited drug from the person of an accused is like requiring him to exhibit himself before the court; or putting in evidence papers and other articles taken from the room of an accused in his absence; This is like US vs Tan Teng where there was a taking of substance from the body of the accused to be used in proving his guilt. It would be a forced construction to hold that any article, substance, or thing taken from a person accused of crime could not be given in evidence. The main purpose of this Right against Self-incrimination is to prohibit testimonial compulsion by oral examination in order to extort unwilling confessions from prisoners implicating them in the commission of a crime.

Writing is something more than moving the body, or the hand, or the fingers; writing is not a purely mechanical act, because it requires the application of intelligence and attention; and in the case at bar writing means that the petitioner herein is to furnish a means to determine whether or not he is the falsifier, which is against the right conferred by the constitutional provision as this case involves the compulsion of the petitioner to make, prepare, or produce, by sampling his handwriting, evidence not yet in existence; in short, to create this evidence which may seriously incriminate him.
Ratio: The text of the constitutional provision invoked by the petitioner is not limited to declaracion but says to be a witness. The rights intended to be protected by the constitutional provision is so sacred, and the pressure toward their relaxation so great when the suspicion of guilt is strong and the evidence obscure, that it is duty of courts liberally to construe the prohibition in factor of personal rights, and to refuse to permit any steps tending toward their invasion. Although there is a similarity between one who is compelled to produce a document, and one who is compelled to furnish a specimen of his handwriting, for in both cases, the witness is required to furnish evidence against himself; the latter is more serious because here the witness is compelled to write and create, [by means of writing] evidence which does not exist, and which may identify him as the falsifier.

Beltran v. Samson (1929)

Writing is not a purely mechanical act, because it requires the application of intelligence and attention, therefore, it constitutes an evidence against the accused.
Facts: Beltran was ordered by the respondent judge, upon the petition of the fiscal, to appear before the latter for the purpose of taking a sample of his handwriting to determine if it is he who wrote certain documents which were supposed to be falsified. Respondent contend that the order in question is based on the provisions of section 1687 of the Administrative Code which provides that a judge,, upon motion of the fiscal, may compel witnesses to be present at the investigation of any crime but this power should be exercised w/o prejudice to the constitutional right of the accused to appear. Petitioner invokes his constitutional right under the Jones Law and incorporated in General Order No. 58 which provides that no man accused of a crime shall be compelled to be a witness against himself.

Bermudez v. Castillo (1937) Facts: There was an investigation which was being conducted by the office of the Solicitor-General against the respondent, in connection with an administrative case. In his defense, the respondent filed, six letters as evidence. He then contends that said six letters are the complainant's, but the latter denied it while she was testifying as a witness in rebuttal.

CONSTI II DIGESTS: 1D COMPILATION

CHUA, ALIMANGOHAN, CONTRERAS, BRIONES, LENCIO, RELOJO, TENGCO, LIM, DUMA, DANAO, MELLA, TONGSON

!#)
Facts: Petitioner Chavez was convicted for conspiring with Dy Sun Hiok y Lim in taking a motor vehicle without the consent of the owner, with intent to gain and abuse of confidence. He contends that he should be freed from imprisonment upon the ground that he was denied his constitutional right not to be compelled to testify against himself in trial. During the trial, the counsel of the petitioner already told the court that the petitioner doesnt want to testify for the prosecution but the court was firm that the petitioner should do so. COURT: What he will testify to does not necessarily incriminate himand there is the right of the prosecution to ask anybody to act as witness on the witnessstand including the accusedBut surely, counsel could not object to have the accused called on the witness stand. The court further added that if there should be question that is incriminating then that is the time for counsel to interpose his objection and the court will sustain him if and when the court feels that the answer of this witness to the questions would incriminate him. Issue: Whether or not the petitioner was denied his constitutional right against self-incrimination Held: Yes

"

she admitted, however, that 3 other letters admitted as evidence (not included in the initial 6 letters) were in her own handwriting. For purposes of comparison between the initial 6 letters and the other 3 letters, the respondent required her to copy them in her own handwriting in the presence of the investigator. The complainant, upon advice of her attorney, refused to submit to the trial to which it was desired to subject her, invoking her right not to incriminate herself and alleging that the other letters already in the respondent's possession, were more than sufficient for what he proposed to do. The investigator, upholding the complainant, did not compel her to submit to the trial required, thereby denying the respondent's petition. Hence, respondent instituted this proceeding. Issue: WON the complainant could be required by the respondent to copy said letters in her own handwriting in the presence of the investigator. Held: NO Ratio: a. Art 3 Sec. 17 % Although the provision makes reference only to criminal cases, the privilege has consistently been held to extend to all proceedings sanctioned by law and to all cases in which punishment is sought to be visited upon a witness, whether a party or not. b. This doctrine which has universal judicial acceptance, leads to several important consequences: i. (a) A clause exempting a person from being "a witness against himself" protects as well a witness as a party accused in the cause; that is, it is immaterial whether the prosecution is then and there "against himself" or not. So also a clause exempting "the accused" protects equally a mere witness. ii. (b) A clause exempting from self-criminating testimony "in criminal cases" protects equally in civil cases, when the fact asked for is a criminal one. iii. (c) The protection, under all clauses, extends to all manner of proceedings in which testimony is to be taken, whether litigious or not, and whether "ex parte" or otherwise. It therefore applies in all kinds of courts . . . in all methods of interrogation before a court, . . . and in investigations by a legislature or a body having legislative functions. c. Initially, Court ruled that the constitutional provision relates solely to testimonial compulsion; however, the Court has already enlarged the application of the provisions by holding that a person may not be compelled to produce specimens of his handwriting for purposes of confrontation with certain documents supposed to have been falsified by him. Court said that "writing is something more than moving the body, or hand, or the fingers; writing is not a purely mechanical act, because it requires the application of intelligence and attention; and in the case at bar writing means that the petitioner herein is to furnish a means to determine whether or not he is the falsifier, as the petition of the respondent fiscal clearly states.

Ratio: An accused may altogether refuse to take the witness stand and refuse to answer any and all questions. o For in reality, the purpose of calling an accused as a witness for the People would be to incriminate him. o The damaging facts forged in the decision were drawn directly from the lips of Chavez as a prosecution witness. There was no genuine consent when the petitioner testified. o He was coerced by the judged to testify. The petitioner did not waive his constitutional immunity from being called to testify against himself. His testimony is not of his own choice. o He did not volunteer to take the stand. o He did not offer himself as a witness. o On the contrary, He claimed the right upon being called to testify.

Castro, J., separate opinion:


The judge had no right to compel the accused to make any statement whatever. The constitutional right against self-incrimination may, on occasion, save a guilty man from his just deserts, but it is aimed against a more far-reaching evilrecurrence of the Inquisition and the Star Chamber, even if not in their stark brutality. o Consideration on the dignity and intrinsic importance of the individual man The government must establish guilt by evidence independently and freely secured; it can not by coercion prove a charge against an accused out of his own mouth.

Chavez v. Court of Appeals

CONSTI II DIGESTS: 1D COMPILATION

CHUA, ALIMANGOHAN, CONTRERAS, BRIONES, LENCIO, RELOJO, TENGCO, LIM, DUMA, DANAO, MELLA, TONGSON

!#*

"

CONSTI II DIGESTS: 1D COMPILATION

CHUA, ALIMANGOHAN, CONTRERAS, BRIONES, LENCIO, RELOJO, TENGCO, LIM, DUMA, DANAO, MELLA, TONGSON

!$+
Pascual Jr v. Board of Medical Examiners (1969) Facts: In an administrative proceeding for alleged immorality against Pascual Jr, the complainants announced that Pascual Jr would be presented as the first witness. Pascual Jr objected, relying on the right to be exempt from being a witness against himself. The Board of Examiners said that Pascual Jr would have to testify unless he could secure a restraining order. The lower court issued an injunction commanding the Board to refrain from further proceeding with the case.

"

Cabal v. Kapunan Facts: Col. Jose C. Maristela of the Philippine Army filed a letter-complaint charging petitioner Manuel F. Cabal, then Chief of Staff of the Armed Forces of the Philippines, with graft, corrupt practices, unexplained wealth, conduct unbecoming of an officer and gentlemen, dictatorial tendencies, giving false statements of his assets and liabilities in 1958 and other equally reprehensible acts. The President created a committee to investigate the charge of said unexplained wealth and submit its reports and recommendations. The committee ordered Cabal to take the witness stand and be sworn to as witness for Maristela, in support of his charge of unexplained wealth. Cabal objected, invoking his constitutional right against self-incrimination. Issue: WON the proceeding before the aforementioned Committee is civil or criminal in character. Held: Criminal Ratio: The accused in a criminal case may refuse not only to answer incriminatory questions, but, also, to take the witness stand. The purpose of the charge against petitioner is to apply the provisions of the Anti-Graft Law, which authorizes the forfeiture to the State of property of a public officer or employee which is manifestly out of proportion to his salary as such public officer or employee and his other lawful income and the income from legitimately acquired property. Such forfeiture has been held to partake of the nature of a penalty. As a consequence, proceedings for forfeiture of property are deemed criminal or penal, and hence the exemption of defendants in a criminal case from the obligation to be witnesses against themselves is applicable thereto. No person shall be compelled in any criminal case to be a witness against himself. This prohibition against compelling a person to take the stand as a witness against himself applies only to criminal, quasi-criminal, and penal proceedings, including a proceeding civil in form for forfeiture of property by reason of the commission of an offense, but not a proceeding in which the penalty recoverable is civil or remedial in nature. The privilege of a witness not to incriminate himself is not infringed by merely asking the witness a question which he refuses to answer. The privilege is simply an option of refusal, and not a prohibition of inquiry. However, where the position of the witness is virtually that of an accused on trial, it would appear that he may invoke the privilege in support of a blanket refusal to answer any and all questions. * Right Against Self-Incrimination is available: 1) criminal cases 2) civil cases 3) administrative cases -as long as the penalty is penal in nature.

Pascual argued that compelling him to testify is tantamount to grave abuse of discretion for violating his right against self-incrimination. The Board on the other hand argued that the right is available only when a question calling for an incriminating answer is asked; that his remedy is to object once he is in the witness stand. The lower court ruled in favor of Pascual Jr and prohibited the Board from compelling him to testify for the complainant.
Issue: WON the Board may compel Arsenio Pascual Jr, the person proceeded against, to take the witness stand without his consent Held: NO. Ratio: Cabal v Kapunan Petitioner was charged under the Anti-Graft Act. He was charged for contempt by Judge Kapunan for his refusal to be sworn as witness. The accused in a criminal case may refuse not only to answer incriminatory questions but also to take the witness stand. " Although the proceeding against Pascual Jr is administrative, it possesses a criminal aspect since he could suffer the revocation of his license as medical practitioner. o The constitutional guarantee extends to lawyers and other individuals. It should not be watered down by imposing disbarment and deprivation of livelihood for asserting said right. This principle is applicable to Pascuals case wherein he could possibly lose his medical license. The constitutional guarantee protects the right to silence, silence which cannot be used as presumption of his guilt. The government must accord to the dignity and integrity of its citizens in its duty of punishing crime and revealing the truth. Its means should not be offensive to the high sense of respect accorded the human personality. The Self-Incrimination clause enables citizens to create a zone of privacy which the government may not force to surrender to his detriment.

CONSTI II DIGESTS: 1D COMPILATION

CHUA, ALIMANGOHAN, CONTRERAS, BRIONES, LENCIO, RELOJO, TENGCO, LIM, DUMA, DANAO, MELLA, TONGSON

!$!
TO COME UNDER THE BAN, the punishment must be flagrantly and plainly oppressive, wholly disproportionate to the nature of the offense as to shock the moral sense of the community.

"

Bermudez v Castillo: The Self-Incrimination clause must be given a liberal and broad interpretation favorable to the person invoking it, for it to be a real protection.
SEC 19: CRUEL, DEGRADING OR INHUMAN PUNISHMENT; EXCESSIVE FINES People v. Estoista (1953) Facts: Estoista was found guilty of possessing or carrying firearms prohibited under Republic Act No. 4. The penalty imposed by RA 4 for such crime is imprisonment from 5 to 10 years. Issues: 1. WON the penalty of confinement from 5 to 10 years for possessing or carrying firearms is cruel or unusual. WON Republic Act No. 4 is unconstitutional because of the penalty imposed. 2. WON the accused should be sentenced to imprisonment from 5 to 10 years. Held: 1. No. 2. No. Six months was commensurate and just for appellants offense. Ratio: Issue 1 Considering the prevalent conditions which the law proposes to suppress (i.e. rampant lawlessness against property, person, and the security of the Government directly traceable in large measure to promisous carrying and use of powerful weapons), the penalty is not unusual or cruel. Constitutionality: If imprisonment from 5 to 10 years is out of proportion to the present case in view of certain circumstances, the law is not to be declared unconstitutional for this reason. o MOREOVER, the issue of constitutionality was raised for the first time only in the course of the oral argument in the Court of Appeals. It was contested neither at the trial nor in the brief for the appellant.

Issue 2 Taking into consideration the intention and the degree of malice of the accused, application of the law to its full extent would be too harsh. o Note: The court ordered that a copy of its decision be furnished to the President, thru the Secretary of Justice with the recommendation that the imprisonment imposed be reduced to 6 months. Doctrine: It takes more than merely being harsh, excessive, out of proportion, or severe for a penalty to be obnoxious to the Constitution. o The fact that the punishment authorized by the statute is severe does not make it cruel and unusual.

***SERIES OF IMPORTANT NOTES*** ----------NOTE: The measure of cruelty and excess. Guides for determining whether a punishment is cruel and unusual (Furman v. Georgia): 1. A punishment must not be so severe as to be degrading to the dignity of human beings. 2. It must not be applied arbitrarily. 3. It must not be unacceptable to contemporary society. 4. It must not be excessive, i.e., it must serve a penal purpose more effectively than a less severe punishment would. ------------Note: When punishment is deemed cruel and unusual. Punishment may be deemed cruel and unusual for any one of four reasons. 1. There are certain punishments which inherently involve so much pain and suffering that civilized people cannot tolerate them (e.g. the rack, the thumbscrew, or other means of torture). 2. There are unusual punishment in the sense of being previously unknown for a given offense. 3. A penalty may be cruel and unusual because it is excessive and serves no legislative purpose. 4. A punishment that is not excessive and possessing a legislative purpose may nevertheless be invalid if popular sentiment abhors it. -----------Note: The Supreme Court has held that the death penalty is not in itself a cruel or unusual punishment. The 1973 Constitution, by making the imposition of the death penalty automatically reviewable by the Supreme Court, implicitly admitted that per se it is not cruel and unusual. The new Constitution, by allowing the possibility of its restoration, implicitly admits that it need not be cruel and inhuman. o HOWEVER, the circumstances under which a specific law may allow the death penalty may make it cruel and unusual under such law. ------------Note: A fine is excessive when under any circumstance it is disproportionate to the offense.
SEC 19: EFFECT OF ABOLITION ON APPLICATION OF PENAL LAWS People v. Munoz (1989) Facts: Penalty for murder under Article 248 of the Revised Penal Code was resolution temporal in its maximum period to death. Article III, Section 19(1) of the 1987 Constitution, modified this.

CONSTI II DIGESTS: 1D COMPILATION

CHUA, ALIMANGOHAN, CONTRERAS, BRIONES, LENCIO, RELOJO, TENGCO, LIM, DUMA, DANAO, MELLA, TONGSON

!$#
inadvisable penalty. Where do we get the medium period now until such time that Congress gets around to accommodate this amendment? FR. BERNAS: As I said: this is a matter which lawyers can argue with judges about. All we are saying is, the judges cannot impose the death penalty. Note: The court realizes that this interpretation may lead to certain inequities that would not have arisen under Article 248 of the Revised Penal Code before its modification. Thus, a person originally subject to the death penalty and another who committed the murder without the attendance of any modifying circumstances will now be both punishable with the same medium period although the former is concededly more guilty than the latter. But that is the will not of this Court but of the Constitution. That is a question of wisdom, not construction. The problem in any event is addressed not to this Court but to the Congress. Penalties are prescribed by statute and are essentially and inclusively legislative. We should not encroach on this prerogative of the lawmaking body. SEC 19: RESTORATION OF THE DEATH PENALTY People v. Echegaray Facts: Quoted in the case book are the relevant doctrines and concepts on the Death Penalty based on the syllabus of People v. Echegaray.

"

Comfortably, the Court has since February 2,1987 not imposed the death penalty whenever it was called for under the said article but instead reduced the same to reclusion perpetua as amended by the above provision. The three-grade scheme of the original penalty, including death. Was thus maintained except that the maximum period was not imposed because of the constitutional provision. In Peoplev. Guevarra; in view of the abolition if the death penalty under Section 19, Article III of the 1987 Constitution, the penalty that may be imposed for murder is reclusion temporal in its maximum period to reclusion perpetua. Later, without categorically saying so, the Court, People v. Masangkay and People v. Atenecio, divided the modified penalty into three new periods, In People v. Intino, as follows: the lower half of reclusion temporal maximum; the upper half of reclusion temporal maximum as the medium; and reclusion perpetua as the maximum.

Issue: Whether or not they also meant to require a corresponding modification in the other periods as a result of the prohibition against the death penalty. Held: We hereby reverse the current doctrine providing for three new periods for the penalty of murder as reduced by the Constitution. Instead, we return to our original interpretation and hold that Article III, Section 19(1) does not change the periods of the penalty prescribes by Article 248 of the revised penal code except only insofar as its prohibits the imposition of the death penalty and reduces it to reclusion perpetua. The range of the medium and minimum penalties remains unchanged. Accepting arguendo that it was the intention of the framers to abolish the death penalty, we are still not convinced from the debates in the Constitutional Commission that there was also a requirement to adjust the two remaining periods by dividing them into three shorter periods. This is not a necessary consequence of the provision as worded. A reading of Section 19(10) of Article III will readily show that there is really nothing therein which expressly declares the abolition of the death penalty. The provision merely says that the death penalty shall not be imposed unless for compelling reasons involving heinous crimes the Congress hereafter provides for it and, if already imposed, shall be reduced to reclusion perpetua. It is settled rule of legal hermeneutics that if the language under consideration is plain, it is neither necessary nor permissible to resort to extrinsic aids, like the records of the constitutional convention, for its interpretation. MR. REGALADO: That would be reclusion perpetua. But the range of the penalty for murder consists of the three periods. The maximum period of reclusion temporal under the present status is the minimum period for the penalty of murder. The medium period is reclusion perpetua. The maximum period is death. If we now remove the death penalty, we will, therefore, have a range of penalty of 17 years, 4 months and 1 day to 20 years of reclusion perpetua. You cannot divide reclusion perpetua into two. It is

Quick review of facts - Echegaray was accused of raping his daughter Rodessa Echegaray in April 1994. He was convicted by Branch 104 of the Regional Trial Court in Quezon City on September 7, 1994. The death sentence was automatically reviewed by the Supreme Court and was affirmed on June 25, 1996. It was appealed by Echegaray, but the appeal was denied on January 19, 1999. Echegaray was executed on February 5, 1999. Issue: N/A Ruling and Ratio: 1. The State possesses power to secure society against threatened and actual evil. The legislative enacts criminal laws that define and punish illegal acts, the executive enforces these laws, and the judiciary tries and sentences criminals in accordance with these laws. 2. Death is imposable in cases provided by law.

Ratio:

Harden The Death penalty is not cruel, unjust nor excessive.


The courts are not the proper venue to debate the morality or propriety of the death penalty. As long as the penalty remains in statutes, and as long as criminal law provides for its imposition in certain cases, it is the duty of judicial officers to respect and apply the law. 3. Constitutional requirements for the restoration of the Death Penalty

CONSTI II DIGESTS: 1D COMPILATION

CHUA, ALIMANGOHAN, CONTRERAS, BRIONES, LENCIO, RELOJO, TENGCO, LIM, DUMA, DANAO, MELLA, TONGSON

!$$
People vs. Purazo Separate Opinions: o Issue: 1. 2. Held: 1. 2. Vitug, J.: WON RA 7659 in prescribing a single mandatory penalty of death for certain offenses constitutes a legislative infringement on judicial prerogative. Would it be right for the courts to ignore the mitigating circumstances that may be attendant to cases involving the mandatory death penalty? Yes No. Vitug agrees with the ponencia. Judicial power as opposed to ministerial power involves the exercise of judgment and the use of mental faculties that may not be subject of arbitrary legislation. Resolving an issue of law or of fact is a judicial question that may not be interfered with in the guise of regulatory legislation. The US Supreme Court evident in the case of Woodson and Roberts, has declared mandatory sentences as a form of cruel and unusual punishment because it is a process that accords no significance to relevant facets of the character and record of the individual offender or the circumstances of the particular offense, excludes from consideration in fixing the ultimate punishment of death the possibility of mitigating factors and treats all persons convicted of a designated offense not as uniquely equal human beings but as members of a faceless, undifferentiated mass to be subjected to the blind infliction of the penalty of death. Congress enacted into law RA 7659 providing for the imposition of the death penalty for certain offenses that virtually left the courts with hardly any sentencing discretion. In people vs, echegaray, the SC affirmed the constitutionality of the statute. The Philippine Bill of Rights empowers Congress to reinstate the death penalty but only if such re-imposition is: a. for compelling reasons b. confined to heinous crimes According to Bernas, Congress should instead preserve judicial discretion to all capital offenses which are equally heinous. Senator Tolentino acknowledged that neither the legislative qualification or designation of the crime as heinous nor the imposition of the death penalty per se should be conclusive on the judiciary, both matters being not solely legislative but likewise judicial in concept and nature. It is thus submitted that the mandatory character of the death penalty for heinous crimes prescribed in RA 7659 notwithstanding, the courts are not precluded given mitigating factors duly established in evidence from: a. declaring the crime charged to be non-heinous in character or b. concluding that no compelling reasons exist to warrant the imposition of the death penalty Either of which would allow such courts to impose reclusion perpetua.

"

Congress has the power to restore the death penalty. Art III, Section 19 plainly vests in Congress the power to re-impose the death penalty for compelling reasons involving heinous crimes. This power, however, is not subsumed in the plenary legislative power of Congress. It is subject to requirements and limitations. Requirements for the restoration of the Death Penalty by Congress a. Congress must define what are heinous crimes; b. Congress must penalize by death, only crimes that qualify as heinous; and c. Congress, in restoring the death penalty, must be motivated by compelling reasons. 4. Heinous Crime Definition Heinous crimes grievous, odious and hateful offenses which, by reason of their inherent or manifest wickedness viciousness, atrocity and perversity are repugnant and outrageous to the common standards and norms of decency and morality in a just, civilized and ordered society. 5. R.A. 7659 Crimes warranting mandatory death penalty and crimes punishable by reclusion perpetua to death Two types of heinous crimes: a. punished by mandatory death penalty b. punished by reclusion perpetua to death The proper time to determine the heinousness of heinous crimes of the second type is on automatic review. The aggravating circumstances under the Revised Penal Code remain subsist and affect criminal liability. Neither the Constitution nor R.A. 7659 amends the Revised Penal Code. 6. Restoration of the Death Penalty Rationale The death penalty is imposed in heinous crimes because the perpetrators: a. committed unforgivably execrable acts that have so deeply dehumanized a person; b. committed criminal acts with severely destructive effects on national efforts; c. caused irreparable and substantial injury to both their victim and society. Rape, especially one perpetrated against ones own daughter, is an intrinsically evil act and is outrageous to dignity and decency and deserves to be punished by death.

CONSTI II DIGESTS: 1D COMPILATION

CHUA, ALIMANGOHAN, CONTRERAS, BRIONES, LENCIO, RELOJO, TENGCO, LIM, DUMA, DANAO, MELLA, TONGSON

!$%
Facts: This is a consolidation of cases involving prosecution of offenses under BP 22. The defendants in the cases moved seasonably to quash the informations on the ground that the acts charged did not constitute an offense, the statute being unconstitutional.

"

Concurring Opinions: o Panganiban, J.: - concurs that the Death Penalty Law is unconstitutional and voted to impose reclusion perpetua o Carpio, J.: - concurs with the ponencia of Justice Bellosillo finding appellant Purazo guilty of the crime of qualified rape and imposing on him the penalty of reclusion perpetua instead of death - voted to convict Purazo of the crime of qualified rape but voted to impose the penalty of reclusion perpetua because no compelling reasons exist to impose the death penalty The trial court is mandated to impose the death penalty on appellant pursuant to RA 7659 which imposes death penalty for heinous crimes. But on automatic review, the SC is not mandated to impose such just because the accused is guilty of a heinous crime. The Death Penalty Law requires an overriding condition before one adjuged guilty of a heinous crime can be meted out the death penalty. If a majority of the members of the court do not vote to affirm the death penalty, the penalty is automatically reduced to reclusion perpetua. Article 47 of the RPC provides that the death penalty shall be imposed on all cases in which it must be imposed under existing laws. This is mandatory on the trial court but not on the SC. If the court affirms the conviction of the accused, the death penalty is not ipso facto imposed. The law requires the court to hold a second voting on whether to impose the death penalty. If reclusion perpetua is imposed, it necessarily means that majority of the members of court voted that the accused is guilty of a heinous crime but less than a majority voted to impose the death penalty. Rule is: In deciding cases where the penalty to be imposed is death, the court should vote twice. First voting: to decide whether the accused is guilty beyond reasonable doubt of the heinous crime charged. If he is, the court will then proceed to the second voting. Second voting: to decide whether to impose the death penalty, to determine the presence of compelling reasons If less than a majority of the members of court vote to impose the death penalty, the law requires that reclusion perpetua shall be imposed, even if the accused is guilty of a heinous crime. The legislature has defined the heinous crimes that deserve the death penalty. But even if a person is charged and convicted of a heinous crime, the Constitution still requires that there must be compelling reasons for imposing the death penalty. This is the reason the law requires the court to hold a second voting. If there are no compelling reasons, then te law automatically imposes the penalty of reclusion perpetua for the heinous crime. The power to define heinous crimes and provide for their penalty, as well as to prescribe the standard for determining the compelling reasons for imposing the death penalty is a legislative power. SEC 20: IMPRISONMENT FOR DEBT Lozano v Martinez (1986)

Defendants argument: Since the offense under BP is consummated only upon the dishonor or non-payment of the check when it is presented to the drawee bank, it is a bad debt law rather than a bad check law. What is punished is the non-payment of the check, not the act of issuing it. It is a veiled device to coerce payment of a debt under threat of penal sanction.
Issue: Whether or not BP 22 runs counter the inhibition in the Bill of Rights against imprisonment for debt. Held: NO. Ratio: The gravamen of the offense punished by BP22 is the act of making and issuing a worthless check or a check that is dishonored upon presentation for payment, and NOT the non-payment of an obligation, which the law punishes. o Thrust: to prohibit making of worthless checks and putting them in circulation. o The law punishes the act not as offense against property but offense against public order for deleterious effects on public interest. Enactment of BP 22 is a valid exercise of police power. o It is within the prerogative of the lawmaking body to proscribe acts deemed pernicious and inimical to public welfare although it may be constitutionally impermissible for legislature to penalize a person for non-payment of a debt ex contractu. o Law punishes not only acts mala in se but also malum prohibitum in the exercise of its police power. o It is not for us to question wisdom or impolicy of the statute. It is sufficient that a reasonable nexus exists between means and end. o Public concern that prompted its enactment: Approximate value of bouncing checks per day was close to P200M, and when overdrafts were banned, it averaged between P50-80M a day. Checks have become widely accepted as medium of payment. The perception that they are convenient substitutes for currency is based on confidence, which should not be shaken. Cited cases: Ganaway case: Organic provisions relieving from imprisonment for debt were intended to prevent commitment of debtors to prison for liabilities arising from actions ex contractu. Inhibition was never meant to include damages arising in ex delicto, for the reason that damages recoverable therein do not arise from any contract entered into between the parties but are imposed upon the defendant for the wrong he has done and are considered as punishment, nor to fines and penalties imposed by the courts in criminal proceedings as punishments for crimes.

CONSTI II DIGESTS: 1D COMPILATION

CHUA, ALIMANGOHAN, CONTRERAS, BRIONES, LENCIO, RELOJO, TENGCO, LIM, DUMA, DANAO, MELLA, TONGSON

!$&
- E N D -

"

Law involved here is the Code or Procedure in Civil Actions (not a criminal statute!) which authorized the arrest of the defendant in a civil case on grounds akin to those which justify the issuance of a writ of attachment under Rules of Court. People v Vera: Constitutionality of a statutory provision which made illegal and punishable refusal or employer to pay, when he can do so, salaries of employees, was upheld, being within the authority of the legislature to enact such in the exercise of police power. o

Notes: History of the constitutional prohibition: It is a safeguard that evolved during the early part of the 19th century in various parts of the American union as a result of the peoples revulsion at the cruel and inhumane practice which permitted creditors to cause the incarceration of debtors who could of pay their debts. The humanitarian provisions that forbid the practice were eventually enshrined in the constitutitions of various states of the Union, and were transported to our shores by the Americans and embodied in our organic laws. There are conflicting jurisprudence on the constitutionality of the worthless check acts. But the court noted that foreign jurisprudence must be taken with abundant caution since substantial differences exist between our statute and the worthless check acts of those states where the jurisprudence evolved.

In Re Petition for Habeas Corpus (2003) FACTS: Petitioners are tenants of Berlito Taripe. They were arrested by Ormoc City policemen by authority of Warrant of Arrest from a motion filed by Eleuteria Bolano, special administratrix of late Anselma Allers. She prayed that petitioners be held guilty of indirect contempt for not complying with probate courts order which directs petitioners to pay monthly rentals to Bolano. Petitioner contends that they did not comply with the order because they were not certain as to the rightful person to whom to pay the rentals because it was Taripe who had originally leased the property to them. ISSUE: WON payment of rentals is covered by constitutional guarantee against imprisonment HELD: YES! RATIO: Sec 20: No person shall be imprisoned for debt. Debt refers to civil debt and not one arising from criminal offense. It is any liability to pay arising out of contracts, express or implied. Petitioners could not be held guilty of contempt of court for their continued refusal to comply with probate courts orders. In Sura vs. Martin: Court held that Where an order for the arrest and imprisonment of defendant for contempt of court (for failure to satisfy judgment for support on ground of insolvency) would, in effect, violate the Constitution

CONSTI II DIGESTS: 1D COMPILATION

CHUA, ALIMANGOHAN, CONTRERAS, BRIONES, LENCIO, RELOJO, TENGCO, LIM, DUMA, DANAO, MELLA, TONGSON

!$'
Ratio: For a person under prosecution to be placed in jeopardy, he must be placed on trial under the following conditions: 1. Court of competent jurisdiction 2. Upon a valid complaint or information (sufficient in form and substance) 3. After he has been arraigned 4. After he has pleaded to the complaint or the information (issue is properly joined) Purpose: the right against double jeopardy protects the accused not against the peril of second punishment, but rather against the burden of being tried again for the same offense so that after being notified of the dismissal of his case, the accused can rest assured that he will not have to prepare for yet another litigation or prosecution.

"

SEC 21: ATTACHMENT OF JEOPARDY People v. Ylagan (1933) Facts: A complaint for physical injuries in the justice of the peace court of Batangas, Province of Batangas, against Elisea Ylagan. After preliminary investigation, the case forwarded to the Court of First Instance, where the provincial fiscal filed an information charging her with serious physical injuries. Upon arraignment, the defendant pleaded not guilty to the information; whereupon the private prosecutor, with the concurrence of the deputy provincial fiscal, moved for the dismissal of the case, which motion was granted by the court. The attorney for the defendant said nothing about the dismissal of the case. Eleven days later, the acting provincial fiscal filed another information in the same justice of the peace court, charging the same defendant with the same offense of serious physical injuries. After another preliminary investigation, the case was again forwarded to the Court of First Instance, where the information filed in the justice of the peace court was reproduced. Upon arraignment, the defendant entered a plea of double jeopardy, based on section 28 of the Code of Criminal Procedure. After hearing, the court sustained the plea and dismissed the case. From this order of dismissal, an appeal was taken by the Government. Section 28 of the Code of Criminal Procedure read as follows: A person cannot be tried for an offense, nor for any attempt to commit the same or frustration thereof, for which he has been previously brought to trial in a court of competent jurisdiction, upon a valid complaint or information or other formal charge sufficient in form and substance to sustain a conviction, after issue properly joined, when the case is dismissed or otherwise terminated before judgment without the consent of the accused. Issue: W/N it constitutes double jeopardy Held: YES. This court had held that there is no jeopardy until the investigation of the charges has actually been commenced by the calling of a witness; but we are now convinced that such a view should be abandoned. There is no provision or principle of law jeopardy. All that the law requires is that the accused has been brought to trial "in a court of competent jurisdiction, upon a valid complaint or information or other formal charge sufficient in form and substance to sustain a conviction, after issue properly joined." Under our system of criminal procedure, issue is properly joined after the accused has entered a plea of not guilty. The mere calling of a witness would not add a particle to the danger, annoyance, and vexation suffered by the accused, after going through the process of being arrested, subjected to a preliminary investigation, arraigned, and required to plead and stand trial. The rule against double jeopardy protects the accused not against the peril of second punishment, but against being again tried for the same offense.

People v. Balicasan Facts: Aurelio Balisacan was charged with homicide in killing Leoncio Bulaoat. In the arraignment, upon assistance of counsel, he pleaded guilty and was allowed to present evidence to prove mitigating circumstances. After hearing his testimony, the court rendered a decision acquitting the accused. The prosecution appealed and claimed that the trial court erred in acquitting the accused despite of his plea of guilty. Plea of guilty is an unconditional admission of guilt with respect to the offense charged. It forecloses the right to defend oneself from said charge and leaves the court with no alternative but to impose penalty fixed by law under the circumstances. However, Section 2, Rule 122 of the Rules of court provide that The People of the Philippines can not appeal if the defendant would be placed thereby in double jeopardy. This provision applies even if the accused fails to raise the question of double jeopardy. Issue: Whether or not this appeal placed the accused in double jeopardy Held: There having been no standing plea at the time the court a quo rendered its judgment of acquittal, there can be no double jeopardy with respect to the appeal therein. Ratio: The testimony of the accused had the effect of vacating his plea of guilty and the court should have required him to plead anew on the charge or direct that a new plea of not guilty be entered for him: this was not done by the trial court The court decided upon the merits without giving the prosecution opportunity to present evidence or even to rebut the testimony of the defendant o In doing so, court acted without due process of law o For lack of requirement of due process, its action is null o The acquittal being a nullity means that there is no acquittal at all for want of due process o The acquittal can not constitute a proper basis for a claim of double jeopardy

CONSTI II DIGESTS: 1D COMPILATION

CHUA, ALIMANGOHAN, CONTRERAS, BRIONES, LENCIO, RELOJO, TENGCO, LIM, DUMA, DANAO, MELLA, TONGSON

!$(
2. 3. first jeopardy must have been validly terminated;

"

People vs Espinosa (2003) Facts: Separate cases of estafa and attempted corruption of public officers were filed before Sandiganbayan by the Ombudsman against Espinosa, Vasquez and Sanano. Espinosa filed a Motion for Reinvestigation of the cases. During investigation, Espinosa filed a Motion for Leave to Travel Abroad. Then Sandiganbayan issued an Order resetting the hearing and required respondent to be conditionally arraigned on that date before it would act on his Motion to Travel. Espinosa was arraigned and was granted his Motion to Travel. The Order of Arraignment, stated that upon being duly arraigned, he entered a plea of Not Guilty. Ombudsman moved to withdraw ex parte the two cases. Thereafter, it filed in the same court seven Informations for Malversation of Public Funds against Espinosa and several others. Respondents Argument: Double jeopardy had already attached Petitioners Argument: Arraignment was conditional, because it was made for the purpose of accommodating respondents request to travel abroad while the matters were pending reinvestigation. Issue: Whether or not this is a case of double jeopardy as held by the Sandiganbayan Held: Yes, petitioner has not waived his right to invoke double jeopardy, therefore petition should be dismissed. Ratio: Petitioners Argument: Dismissal of latter informations on the ground of double jeopardy has no basis since his arraignment in the earlier cases was only conditional. Sandiganbayans practice of conditionally arraigning the accused pending the Ombudsmans reinvestigation of the case is not provided for in the regular rules of procedure. o Espinosa pleaded simply and unconditionally. Sandiganbayan itself found this simple process inconsistent with its studied manner of conditionally arraigning the accused pending reinvestigation. o The court does not lose control of the proceedings by reason of such review. The rule in this jurisdiction is that once a complaint or information is filed in Court any disposition of the case as to its dismissal or the conviction or acquittal of the accused rests in the sound discretion of the Court. o Conditions attached to an arraignment must be unmistakable, express, informed and enlightened. They must be expressly stated in the Order disposing of the arraignment. Otherwise, the plea should be deemed to be simple and unconditional. Petitioners Argument: Espinosa has waived his right to invoke double jeopardy. Sec 17 of Rule 117 of the Revised Rules of Criminal Procedure, to substantiate a claim for double jeopardy: 1. a first jeopardy must have attached prior to the second;

the second jeopardy must be for the same offense, or the second offense includes or is necessarily included in the offense charged in the first information, or is an attempt to commit the same or a frustration

Legal jeopardy attaches only: (a) upon a valid indictment; (b) before a competent court; (c) after arraignment; (d) when a valid plea has been entered; & (e) case was dismissed or terminated without the express consent of the accused. Such waiver must be clear, categorical, knowing and intelligent. The alleged waiver falls short of the above requirement: o Lawyer for Espinosa was present when the Order of Sandiganbayan was issued. Lawyer must have heard that the hearing of the motion to travel was reset so that the movant could be conditionally arraigned. Arraignment was conditional for if it was not so, Espinosa would have been deemed to have abandoned his recourse for the reevaluation of his cases before the Ombudsman. The dismissal of the estafa and the corruption cases was made upon petitioners ex parte Motion for the withdrawal of the Informations. Respondent was not notified of this and neither was there a hearing. Respondent has shown that he learned of the Motion only after the cases had been dismissed. It is clear that the dismissal, having been secured by petitioner without the express consent of the accused, does not amount to a waiver of the right against double jeopardy. (4th requisite)

SEC 21: TERMINATION OF JEOPARDY Bulaong v. People (1966) First jeopardy for subversion is not yet terminated or dismissed / Defense of double jeopardy is not available Facts: Petitioner Agaton Bulaong a.k.a Ka Era and others were charged with the crime of rebellion. Trial against Bulaong did not proceed until 1958 for he was then at large. The Anti-Subversion Act (RA 1700) was enacted and took effect on 1957. The following year, Bulaong was arrested and he was charged with Rebellion. The information provided, among others, that: he joined the Communist Party of the Philippines (CPP) and the Hukbalahap (HUKS), whose aims and purposes are to remove people and territory of the RP and to overthrow it eventually by rising publicly and taking up of arms, [as an organizer and leader] making armed raids, ambuscades and attacks upon the Constabulary, AFP, Police Forces and other military detachments of the government.

CONSTI II DIGESTS: 1D COMPILATION

CHUA, ALIMANGOHAN, CONTRERAS, BRIONES, LENCIO, RELOJO, TENGCO, LIM, DUMA, DANAO, MELLA, TONGSON

!$)
b. c. d. e. f. g.

"

On the same date another information was filed Bulaong of subversion defined in Section 4 of the Anti-Subversion Act. The information providing that: in Manila, the center of their activity, Bulaong became an officer/leader of CCP and Hukbong Magpapalaya ng Bayan (HMP) which organizations have aimed to overthrow the Government by means of force which organizations have been outlawed and declared illegal Bulaong continued and remained an officer and/or a ranking leader of said organizations until his arrest by making and conducting raids, ambuscades and armed attacks upon the Constabulary, AFP, Police Forces and other military detachments of the government. While his case for subversion is still pending, he was convicted of the crime of rebellion by the lower court. Appellants contentions: Accused contends that rebellion as defined in Art.134 of the RPC is a lesser offense compared to subversion as defined in Section 4 of RA 1700. that since the facts alleged in the information for rebellion and subversion are the same he can not be legally prosecuted for both offenses without being placed twice in jeopardy of being punished for the same acts. Issue: WON Bulaong can interpose the defense of double jeopardy on the ground that the facts alleged in both charges were the same. Held: NO. Rules of Court provides that double jeopardy is available to the accused only where he was either convicted or acquitted or the case against him was dismissed or otherwise terminated without his consent. Ratio: In the case at bar, the accused has not been convicted or acquitted for subversion, neither was the said case dismissed or terminated without his consent, far as stated, charges for subversion is still pending in said court. It is the conviction, acquittal of the accused or dismissal or termination of the case that bars further prosecution of the same offense or any attempt to commit the same or frustration thereof, or for any offense which necessarily includes or is necessarily included in the offense charged in the former complaint or information.

On the same day he was sentenced, petitioner made an express waiver of his right to appeal and started serving his sentence of imprisonment. Three days later, the Fiscal filed a motion for Modification of Penalty. Through counsel, petitioner filed a Motion for Withdrawal of Plea of Guilty and Waiver of Commitment . Then, Judge Coquia (1st Judge) issued an order setting aside his judgment and that the case shall be tried on the merits again. Subsequently, Judge Coquia was transferred to Manila, and the case against petitioner was reassigned to Judge Maceren (2nd Judge), before whom petitioner was re-arraigned. Judge Maceren held a new hearing of the case on the merits and thereafter, Judge Maceren rendered a new judgment against petitioner finding the latter guilty. Petitioner filed a Motion for Reconsideration of the above decision in which motion petitioner questioned the jurisdiction of the trial court to try his case anew after he had fully served the judgment rendered by Judge Coquia against him. Petitioner, in said motion, argued that the judgment of Judge Coquia against him had already become final when he started serving his sentence and that therefore, the Court thereafter lost jurisdiction over his case; and that no amount of waiver or consent on his part could bestow on said court jurisdiction that it had already lost.

Issue: Main issue: WON the petitioners right against double jeopardy was violated. Sub-issue: WON the right against double jeopardy may be waived. Held: YES. YES. Ratio: a. MAIN ISSUE % People v. Sanchez % under Section 7 of Rule 116, Rules of Court, a judgment in a criminal case becomes final: i. ii. iii. after the lapse of the period for perfecting an appeal; when the sentence has been partially or totally satisfied or served; or the defendant has expressly waived in writing his right to appeal.

Under the circumstances, the sentence having become final, no court, not even this High Tribunal, can modify it even if erroneous. b.

Bustamante v. Maceren 1972 Facts: a. The petitioner was accused of murder in an information filed with the Court of First Instance of Laguna. Upon arraignment, petitioner entered a plea of guilty, petitioner was sentenced by the trial court.

SUB-ISSUE % It is true that under sec 10, Rule 117 of the revised Rules of Court, the defense of double jeopardy is waivable, and if not raised or set up at the proper time, is deemed waived, but the same rule provides that % If, however, the defendant learns after he has pleaded or has moved to quash on some other ground that the offense for which he is now charged is an offense for which he has been pardoned, or of which he has been convicted or acquitted or been in jeopardy, the court may in its discretion entertain at any time before judgment a motion to quash on the ground of such pardon, conviction, acquittal or jeopardy."
In the case at bar, considering that defense counsel raised the question of double jeopardy in favor of petitioner during the new trial and before Judge Maceren rendered judgment based on said new trial, it is believed that the above principle can be applied to this case by analogy and that said judge, in the exercise of his discretion, should have entertained said

c.

CONSTI II DIGESTS: 1D COMPILATION

CHUA, ALIMANGOHAN, CONTRERAS, BRIONES, LENCIO, RELOJO, TENGCO, LIM, DUMA, DANAO, MELLA, TONGSON

!$*
o Doctrine of Estoppel in Acierto: ! When the trial court dismisses a case on a disclaimer of jurisdiction, upon instigation of the accused, the latter is estopped on appeal from asserting the jurisdiction of the lower court in support of his plea of second jeopardy.

"

plea of double jeopardy in the interest of justice, especially since at the time such plea was made, petitioner had already fully served the one-year straight sentence imposed upon him by Judge Coquia on December 14, 1970, and was already entitled to be released from custody after such full service of his penalty under said judgment." Thus it would appear there is no legal bar to the remedy prayed for by petitioner.

People v. Obsania Facts: A complaint of rape with robbery was filed in municipal court of Balungao, Pangasinan against the accused Willy Obsania. The accused pleaded not guilty upon arraignment, and forthwith his counsel moved for the dismissal of the case, contending that the complaint was fatally defective for failure to allege lewd designs and that the subsequent information filed by the fiscal which averred lewd designs did not cure the jurisdictional infirmity. The court granted the motion to dismiss hence this appeal by the fiscal. The accused contended that the appeal places him in double jeopardy and cited various cases to support his claim. Issues: Whether or not lewd designs an indispensable element which should be alleged in the complaint Whether or not the present appeal puts the accused in double jeopardy Held: The Court answered both in the negative. Ratio: On the first issue-- Alleging lewd desgns in a complaint for rape is not necessary. The said complaint satisfies the requirements of legal sufficiency of an indictment for rape.

The court cannot agree with the arguments of the accused citing different jurisprudence (Bangalao, Labatete, Villarin, and Cloribel). o

Bangalao was decided solely on the question of jurisdiction. In this case, it was already held that the lower tribunal had jurisdiction. The primary consideration now is to determine whether the dismissal was issued with or without the express consent of the accused.
The case of Labatete cannot abandon the Salico doctrine because the former was not one of dismissal but of acquittal. Had the dismissal not amounted to acquittal, then the doctrine of waiver would have applied and prevailed. The court was not impressed with the Villarin case that treated the motion to dismiss of the counsel different from a motion that can be personally moved for by the defendant. Had the motion to dismiss filed by Villarins counsel been considered as one made by the defendant himself, as should have been done, the Villarin case should have been resolved consistent with the doctrine of waiver in Salico and/or that of estoppel in Acierto. The dismissals in Cloribel case were considered as acquittals because they were predicated on the right of a defendant to a speedy trial and on the failure of the government to prosecute.

o Lascivious intent inheres in rape and the unchaste design is manifest in the very act itself.

The sine qua non conditions for the application of the doctrines of waiver and estoppel are present in this case, these are: o The dismissal must not be on the merits and must not necessarily amount to an acquittal. Such dismissal must be sought or induced by the defendant personally or through his counsel.

On the second issue-- One of the requirements for the protection against double jeopardy to inure in favor of an accused is that he was acquitted, or convicted, or that the case against him was dismissed or otherwise terminated without his express consent. o The case was ordered by the judge upon his motion to dismiss. This action constitutes a waiver of double jeopardy as enunciated in People v. Salico. The Salico doctrine provides that there will not be a bar to another prosecution for the same offense if the accused was the one who had the case dismissed for the reason that he thereby prevents the court from proceeding to the trial on the merits and rendering a judgment of conviction against him. o

Rivera Jr. v. People Facts: Rivera was arrested for attempting to transport marijuana to Manila. The Forensic Chemist who will present the marijuana specimen, despite notice failed to appear. Petitioner through counsel then moved for the dismissal of the case. This was denied by respondent Judge and the hearing was re-set. At the re-scheduled date, Capt. Lina Sarmiento despite notice, was not around

CONSTI II DIGESTS: 1D COMPILATION

CHUA, ALIMANGOHAN, CONTRERAS, BRIONES, LENCIO, RELOJO, TENGCO, LIM, DUMA, DANAO, MELLA, TONGSON

!%+
The grant is left to the courts discretion in ascertaining whether there is competent evidence to sustan indictment. It shall not be disturbed in the absence of grave abuse of discretion. A grant (of demurrer) amounts to an acquittal and any further prosecution would violate the constitutional proscription on double jeopardy. This is an exception to the rule that the dismissal of a criminal case with the accuseds express consent or upon his own motion bars a plea of double jeopardy.

"

necessitating a second call. When the case was called for the second time at around 9:00 a.m. Capt. Sarmiento was still not around. Hence, Atty. Tomas Gorospe, on behalf of petitioner orally moved for the dismissal of the case invoking the right to speedy trial as the petitioner stands confined and that the Government failed to prosecute or adduce evidence due to the nonappearance of a vital prosecution witness. The respondent Judge verbally granted the motion and ordered the immediate release of the accused. While the subsequent calendared cases set for that day was in progress, and in less than an hour after pronouncement of the verbal order of dismissal, Capt. Lina Sarmiento arrived direct from Quezon City. Upon a satisfactory explanation, the respondent Judge issued his now assailed Order setting aside his previous verbal order of dismissal and re-scheduling for continuation of trial. Alleging that the verbal order of dismissal made in open court amounted to the acquittal of petitioner and which order is immediately final and executory, the respondent Judge could no longer set it aside without violating petitioner's constitutional right against double jeopardy. Issue: WON the verbal dismissal by the judge was final. WON the finality of the decision would result to double jeopardy Held: No and No. The verbal dismissal is not final until written and signed by the judge. The Court held that the earlier verbal order of dismissal was not final, in fact, was ineffective, because it left something to be done in line with the decision of this Court. Pursuant to the Rules of Court, `the judgment' and the order of dismissal in question had, if completed, such effect 'must be written . . . personally and directly prepared by the judge, and signed by him . . .

Peo v Velasco (2000): The reason behind the finality-of-acquittal rule The state should not be allowed to make repeated attempts to convict one for an alleged offense, subjecting him to embarrassment, compelling him to live in a continuing state of anxiety and insecurity, and enhancing the possibility that although innocent he may be found guilty

People v Sandiganbayan and Velasco (2002) Facts: The PCGG filed an information against Velasco, then Minister of Energy, for a violation of the Anit-Graft and Corrupt Practices Acts. Velasco pleaded not guilty. After the prosecution rested its case, Velasco filed a Demurrer to Evidence, on the ground of insufficiency of evidence. It was initially denied by the Sandiganbayan, but the demurrer was granted in Velascos Motion for Reconsideration (MR) ergo, the dismissal of the case.

Condrada v. People (2003) Facts: Pablo Conrada was charged with rape. When he was arraigned, petitioner pleaded not guilty. March 1999: (date set by the trial court for the initial hearing) Prosecution moved to postpone the hearing due to the absence of complainant and her witness. April 1999: Prosecution moved to postpone the hearing due to the absence of the complainant and her witness. o Petitioner objected to the motion on the ground that his right to speedy trial was being violated by such postponements. o Court granted petitioners motion and reset the hearing. May 1999: The prosecution requested for another postponement. o Petitioner moved for a temporary dismissal of the case. o Prosecution said it would not object to a temporary dismissal. o The Court issued an order temporarily dismissing the case. Issues: 1. WON the dismissal of the criminal case is permanent in character so as to operate as an acquittal of the petitioner for the crime charged. 2. WON the reinstatement of the criminal case places the petitioner in double jeopardy. Held: 1. 2. Ratio: A. ISSUE ON CHARACTER OF THE DISMISSAL: KEYS TO SUCCESS: A permanent dismissal of a criminal case may refer to: 1. the termination of the case on the merits, resulting in either the conviction or acquittal of the accused 2. the dismissal of the case due to the prosecutions failure to prosecute No. The dismissal was only temporary. No. The reinstatement of the case did not place the petitioner in double jeopardy.

Petitioners Argument: Sandiganbayan committed grave abuse of discretion in dismissing the criminal case. Velasco cannot invoke the rule on double jeopardy since Sandiganbayans Resolution granting the MR is null and void for failing to state a summary of facts proved by the prosecutions evidence.
Issue: WON petitioners petition for certiorari has merit Held: NO Ratio: Sec 23, Rule 119, Revised Rules of Criminal Procedure The trial court may dismiss the action on the ground of insufficiency of evidence upon a demurrer filed by the accused.

&

CONSTI II DIGESTS: 1D COMPILATION

CHUA, ALIMANGOHAN, CONTRERAS, BRIONES, LENCIO, RELOJO, TENGCO, LIM, DUMA, DANAO, MELLA, TONGSON

!%!
given retroactive effect, there is still a need to determine whether the requirements for its application are attendant. Court further held that the reckoning date of the two-year bar had to be first determined whether it shall be from the date of the order of Judge dismissing the cases, or from the dates of receipt thereof by the various offended parties, or from the date of affectivity of the new rule. According to the Court, if the cases were revived only after the two-year bar, the State must be given the opportunity to justify its failure to comply with the said time-bar. That the new rule fixes a time-bar to penalize the State for its inexcusable delay in prosecuting cases already file in court. However, the State is not precluded from presenting compelling reasons to justify the revival of cases beyond the two-year bar.

"

the dismissal of the case on the ground of unreasonable delay in the proceedings, in violation of the accuseds right to speedy disposition or trial of the case against him. A provisional dismissal of a criminal case is a dismissal without prejudice to the reinstatement thereof before the order of dismissal becomes final or to the subsequent filing of a new information for the offense within the periods allowed under the RPC or the Revised Rules of Court. AS APPLIED IN THIS CASE: The dismissal ordered by the trial court (May 1999) was a temporary dismissal of the case, and not a permanent dismissal on the ground that the right of the accused to speedy trial had been violated by the delay in the prosecution of the case. o The court denied petitioners motion on the ground of his right to speedy trial when despite said motion in April 1999, it ordered the resetting of the hearing of the case in May 1999. o The trial court expressly stated that the dismissal was subject to reinstatement within 30 days from the date of temporary dismissal. Otherwise, if the case is not revived within 30 days from dismissal, the case would be considered dismissed permanently. ISSUE ON JEOPARDY KEY TO SUCCESS: The proscription against double jeopardy presupposes that an accused has been previously charged with an offense, and the case against him is terminated either by his acquittal or conviction, or dismissed in any other manner without his consent. AS APPLIED IN THIS CASE The dismissal of the criminal case was provisional or temporary, without prejudice to the revival thereof within 30 days from the date of dismissal. The court finds that the reinstatement of the case did not place petitioner in double jeopardy.

3.

&

B.

&

Petitioners Argument: Section 8, Rule 117 of the Revised Rules of Criminal Procedure is not applicable of Criminal Cases because the essential requirements for its application were not present when Judge issued his resolution. Petitioners maintain that the respondent did not give his express consent to the dismissal by the judge. The respondent allegedly admitted in his pleadings filed with the Court of Appeals and during the hearings thereat that he did not files any motion to dismiss said cases, or even agree to a provisional dismissal thereof. The heirs of the victims were allegedly not given prior notices of the dismissal of the said cases by Judge, the respondents express consent to the provisional dismissal of the cases and the notice to all the heirs of the victims of the respondents motion and the hearing thereon are conditions sine qua non to the application of the time-bar in the second paragraph of the new rule. Respondents Argument: Respondent insists that the respondent himself moved for the provisional dismissal of the criminal cases. Further asserts that the heirs of the victims, were duly notified and said motion and the hearing thereof. He contends that it was a sufficient that the public prosecutor was present during the hearing on the motion for judicial determination of the existence of probable cause because criminal actions are always prosecuted in the name of the People, and the private complainants merely prosecute the civil aspect thereof. Issue: Held: Whether or not, Section 8, Rule 117 of the revised rules of criminal procedure is not applicable to criminal cases against petitioners. Since the conditions sine qua non for the application of the new rule were not present when the judge issued his resolution, the State is not barred by the time limit set forth in the second paragraph of Section 8 of Rule 117 of the Revised Rules of Criminal Procedure. The State can thus revive or refile Criminal Cases Nos. or file new Informations for multiple murders against the respondent. Two rules are distinct, hence, it is not proper to require the element of prior plea in double jeopardy cases in a section 8, Rule 117 situation. Section 8, Rule 117 is a new rule that is complete by itself and should not be construed in light of rules implementing other rights of an accused.

&

Facts:

People v. Lacson (2003) Motion for Reconsideration of the Resolution, for the determination of several factual issues relative to the application of Section 8 of Rule 117 of the Revised Rule of Criminal Procedure on the dismissal of Criminal Cases filed against the respondent and the co-accused with the said court. The respondent and his co accused were charged with multiple murder for the shooting and killing of eleven male persons, bandied as members of the Kuratong Baleleng Gang. Court ruled in the Resolution sought to be reconsidered that the provisional dismissal of Criminal Cases were with the express consent of the respondent as he himself moved for said provisional dismissal when he filed his motion for judicial determination of probable cause and for examination of witnesses. The Court also held therein that although Section 8, Rule 117 of the Revised Rules of the Criminal Procedure could be

CONSTI II DIGESTS: 1D COMPILATION

CHUA, ALIMANGOHAN, CONTRERAS, BRIONES, LENCIO, RELOJO, TENGCO, LIM, DUMA, DANAO, MELLA, TONGSON

!%#
Section 8, Rule 117 should not also be confused with section 3(i), Rule 117 which is the rule of procedure that protects the constitutional right of an accused against double jeopardy. Two rules are distinct; hence, it is not proper to require the elements of prior plea in double jeopardy cases in a section 8, rule 117 situations. Section 8, Rule 117 is a new rule that is complete by itself and should not be construed in light of the rules implementing other rights of the accused. The provisional dismissal under section 9 rule 119 becomes permanent after the lapse of one or two years depending on the gravity of the offense involved. There can be no hedging in the meaning of the word permanent for the new rule used the word without a bit of embroidery. Section 8, Rule 117 regulates the conduct of the prosecution of an offense once the case is filed in court. The new rule regulates the time when the State must compete the prosecution of a pending case after its provisional dismissal. It provides the consequence when the state sleeps on its duty to revive a provisionally dismissed case. If the state loses the right to continue the prosecution of an offense already filed in court, it is not because the rule has amended the prescriptive period of the crime provided by our substantive law. Rather, it is a simple case where the State forfeited its right to prosecute by its own inaction, an inaction that unless justified cannot be allowed to further impair the rights of an accused. The permanent dismissal under section 8, Rule 117 precludes the prosecution of the accused for the same offense under new information. Section 8, Rule 117 changed the old rule that dismissals that are provisional in character lack the imprimatur of finality; hence, they do not bar the revival of the offense charged or the filing of new information for the same offense. The old rule was precisely jettisoned by the Committee and by this Court because of its unfairness to the accused. I do not share the thesis that the refilling of Criminal cases Nos. Q-01-101102 to Q-01101112 is not a revival of criminal cases Nos. Q-99-81679 to Q-99-81689. The cases that were provisionally dismissed for lack of probable cause refer to the eleven (11) Informations for murder filed against the respondent, et al. allegedly for the summary execution of some members of the Kuratong Baleleng Gang, Without Doubt, these are the same cases re-filed against the respondent after another preliminary investigation with the principal difference that respondent is now charged as a principal and no longer as an accessory. The new rule does enhance constitutional rights of an accused to speedy trial and speedy disposition of the case(s) against him. More broadly, the new rule was designed to achieve one of the end-goals of the criminal process to minimize the burdens of accusation and litigation. Section 8, Rule 117 of the Revised rules of Criminal Procedures reads: Section 8. Provisional dismissal A case shall not be provisionally dismissed except with the express consent of the accused and with notice to the offended party. The provisional dismissal of offenses punishable by imprisonment not exceeding six (6) years or a fine of any amount, or both, shall become permanent one (1) year after issuance of the order without the case having been revived. With respect to offenses punishable by imprisonment of more than six (6)years, their provisional

"

Ratio:

Having invoked before the petitioners-panel of prosecutors and before the Court of Appeals, the respondents is burdened to establish the essential requisites of the first paragraph. (1) The prosecution with the express conformity of the accused or the accused moves for a provisional (sin perjuico) dismissal of the case; or both the prosecution and the accused move for a provisional dismissal of the case; (2) The offended party is notified of the motion for a provisional dismissal of the case; (3) The court issues an order granting the motion and dismissing the case provisionally. (4) The public prosecutor is served with a copy of the order of provisional dismissal of the case. Respondent has failed to prove that the first and second requisites of the first paragraph of the new rule were present when The Judge dismissed Criminal Cases, Irrefragably, the prosecution did not file any motion for the provisional dismissal of the said criminal cases. The respondent merely filed a motion for judicial determination of probable cause and for examination of prosecution witnesses. The respondent did not pray for the dismissal, provisional or otherwise of Criminal Cases. Neither did he ever agree, impliedly or expressly, to a mere provisional dismissal of the cases. During the hearing in the Court of Appeals, the respondents, through counsel, categorically, unequivocally, and definitely declared that he did not file any motion to dismiss the criminal cases nor did he agree to provisional dismissal thereof. Respondents admissions made in the course of the corresponding in the Court of Appeals are biding and conclusive on him. The respondent is barred from repudiating this admission absent evidence of palpable mistake in making such admissions. To apply the new rule in Criminal Cases would be to add to or make exceptions from the new rule that are not expressly or impliedly included therein. In crimes involving private interests, the new rule requires that the offended party or parties or the heirs of the victims must be given adequate or prior notice of any notion for the provisional dismissal of the criminal case. Such notice will enable the offended party or the heirs of the victim the opportunity to seasonably and effectively comment on or object to the motion on valid grounds. Even if the respondents motion for a determination of probable cause and examination of witnesses may be considered for the nonce as his motion for a provisional dismissal of Criminal cases, however, the heirs of the victims were not notified thereof prior to the hearing on said motion. There never was any attempt on the part of the trial court, the public prosecutor and/or the private prosecutor to notify all the heirs of the victims of the respondents motion and the hearing thereon and of the resolution of the Judge dismissing said cases. The said heirs were this deprived of their right to be heard on the respondents motion and to protect their interests either in the trial court or in the appellate court.

Note:

PUNO, J., Dissenting

CONSTI II DIGESTS: 1D COMPILATION

CHUA, ALIMANGOHAN, CONTRERAS, BRIONES, LENCIO, RELOJO, TENGCO, LIM, DUMA, DANAO, MELLA, TONGSON

!%$
b. Verdicts of acquittal are to be regarded as absolutely final and irreviewable. The fundamental philosophy behind the constitutional proscription against double jeopardy is to afford the defendant, who has been acquitted, final repose and safeguard him from government oppression through the abuse of criminal processes. People v. Velasco In the absence of a finding of mistrial, i.e. the criminal trial was a sham, a judgment of acquittal is final and unappealable on the ground of double jeopardy, whether it happens at the trial court level or at the Court of Appeals. SEC 21: SAME OFFENSE; ORDINANCE AND STATUTE People vs. Relova Facts: The Batangas City police together with personnel of the Batangas Electric Light System searched and examined the premises of the Opulencia Carpena Ice Plant and Cold Storage owned by private respondent Opulencia. The police discovered that electric wiring, devices and contraptions had been installed without authority from the city government. According to petitioners, there electric devices were designed purposely to lower the readings of electric current consumption in the electric meter of the plant. Opulencia admitted that he had caused the installation of the electrical devices in order to lower the readings of his electric meter. An information was filed against him for violation of Ordinance No.1. The information reads the accused, with intent to defraud the city government of Batangas, without proper authorization, did then and there willfully and feloniously make unauthorized installations of electric wirings to lower the consumption of electric fluid at the plant and as a result, the city government was damaged in the amount of P41,062.16 covering the period from November 1974 to February 1975. The accused pleaded not guilty and filed a motion to dismiss upon the grounds that the crime charged had already prescribed and the motion was granted. He was acquitted. Later, the acting fiscal filed another information against the accused. This time for theft of electric power under Article 308 of the RPC. The information reads the accused, with intent to gain and without knowledge of the Batangas Electric Light System, did then and there willfully and feloniously take and steal electric current valued in the amount of P41,062.16. Before he could be arraigned, Opulencia filed a motion to quash alleging that he had been previously acquitted of the offense charged in the second information and that the filing was violative of his constitutional right against double jeopardy. The judge granted the motion and ordered the case dismissed. A motion for reconsideration was also denied. Hence, this petition for certiorari and mandamus.

"

dismissal shall become permanent two (2) years after issuance of the order without the case having been revived.

People v. Court of Appeals (2004) Facts: Respondents Claudio Francisco, Jr. and Rudy Pacao were charged and convicted of the crimes of homicide and attempted murder for the shooting of Marcial "Boyet" Azada. Their conviction was reversed by Court of Appeals. The people filed a petition for certiorari before the Supreme Court. The petition alleged errors of judgment and misappreciation of testimonial and material evidence on the part of the CA. Specifically, the petition challenged the CAs appreciation of the testimonies of witnesses, the evidence of power burns and the testimonies of ballistic experts. Issue: a. Whether or not the petition for certiorari under Rule 65 of the Revised Rules of Court is the proper legal recourse for the reversal of the assailed decision of the Court of Appeals; b. Whether or not an appeal of the judgment of acquittal by the Court of Appeals violates the Double Jeopardy Clause of the Constitution;

Ruling: a. No. A review of facts and evidence is not the province of the extraordinary remedy of certiorari. b. Yes. Respondents Francisco and Pacao, after having been found not guilty by a court of competent jurisdiction, must be afforded rest and tranquility from repeated attempts by the State at conviction Their acquittal must be accorded finality in faithful adherence to the rule against double jeopardy.

Ratio: a. The petition ascribed to the Court of Appeals errors of judgment, not errors of jurisdiction. Specifically, petitioner delved on the testimonies of witnesses and the opinions of the expert witnesses on certain physical evidence. These are errors that go deeply into the appellate courts appreciation and assessment of the evidence proffered by the parties. These are findings that impinge on errors of judgment and not errors of jurisdiction. Factual matters cannot be inquired into by the Supreme Court in a certiorari proceeding. This Court cannot be tasked to go over the proofs presented by the parties and analyze, assess and weigh them again, in order to ascertain if the trial and the appellate courts were correct in according superior credit to this or that piece of evidence of one party or the other. The mere fact that a court erroneously decides a case does not necessarily deprive it of jurisdiction. Thus, assuming arguendo that a court commits a mistake in its judgment, the error does not vitiate the decision, considering that it has jurisdiction over the case.

Petitioners arguments: 1. The protection against double jeopardy is a protection against a second or later jeopardy of conviction for the SAME OFFENSE. The first information was one for unlawful installation of electric wiring devices which are acts in violation of the ordinance. the purpose for the ordinance is to ensure that electrical installations on residences or buildings be done by persons duly authorized to avoid fires and accident and it is a primarily regulatory measure and not intended to punish theft of electric fluid

CONSTI II DIGESTS: 1D COMPILATION

CHUA, ALIMANGOHAN, CONTRERAS, BRIONES, LENCIO, RELOJO, TENGCO, LIM, DUMA, DANAO, MELLA, TONGSON

!%%
- if the two charges are based on one and same act, conviction or acquittal under either the law or ordinance shall bar a prosecution under the other

"

2. 3.

to constitute an offense under the ordinance, it is not essential to establish any mens rea on the part of the offender

The offense of theft under Article 308 of the RPC has different elements. The theft of electricity can be effected even without illegal installations of any kind. The unauthorized installation punished by the ordinance is not the same as that of theft of electricity under the RPC. the second offense is not an attempt to commit the first or a frustration thereof the second offense is not necessarily included in the offense charged in the first information

Issue: WON the filing of the 2nd information is violative of private respondents constitutional right against double jeopardy. Held: Yes. Petition is denied. Ratio: Petitioners position must be examined not under the first sentence of the provision but under the second sentence. - First sentence: the constitutional protection against double jeopardy is not available where the second prosecution is for an offense that is different from the offense charged in the first or prior prosecution, although both first and second offenses may be based upon the same act. - Second sentence (Exception): the constitutional protection against double jeopardy is available although the prior offense charged under an ordinance be different from the offense charged subsequently under a national statute such as the RPC, provided that both offenses spring from the same act. In Yap vs. Lutero, petitioner Yap was charged with violation of ordinance no. 22. The information charged him with having driving an automobile recklessly thereby endangering other vehicles. Later, Yap was again charged with serious physical injuries through reckless imprudence in violation of the Revised Motor Vehicle Law. Yap moved to quash the second information on the ground that it placed him twice in jeopardy of punishment for the same act. Later, Yap was acquitted for the first information. The SC held that the constitutional protection against double jeopardy was available to petitioner. It regarded the offense of reckless driving under the ordinance and serious physical injuries through reckless imprudence under the Revised Motor Vehicle Law as derived from the same act which is the operation of an automobile in a reckless manner. Second Sentence - contemplates double jeopardy of punishment for the same act - applies even if the offenses charged are not the same owing to the act that one constitutes a violation of an ordinance and the other a violation of a statute

But conviction or acquittal is not indispensable to sustain the plea of double jeopardy of punishment for the offense. So long as jeopardy has attached under one the informations charging said offense, the defense may be availed of in the other case involving the same offense, even if there has been neither conviction nor acquittal in either case. Where the offenses charged are penalized either by different sections of the same statute or by different statutes, the important inquiry relates to the identity of the offenses charged. The protection against jeopardy is available only where an identity is shown to exist between the earlier and the subsequent offenses charged. But the identity of offenses that must be shown need not be absolute identity. The protection is available so long as the acts which constitute or have given rise to the first offense under an ordinance are the same acts which constitute or have given rise to the offense charged under a statute. If the second sentence of the double jeopardy provision had not been written, conviction or acquittal under an ordinance would never constitute a bar to another prosecution for the same act under a statute. In the instant case, the relevant acts took place within the same time frame. During this period, the accused installed electrical wiring in his plant without obtaining a permit. He conceded that such was for the purpose of reducing his power bill. This corrupt intent was thus present from the moment that such unauthorized installation began. The taking of electric current was integral with the unauthorized installation of electric wiring and devices. But the extinction of criminal liability whether by prescription or by the bar of double jeopardy does not carry with it the extinction of civil liability arising from the offense charged. SEC 21: RULE ON SUPERVENING FACTS

First Sentence - prohibits double jeopardy of punishment for the same offense - one may be twice put in jeopardy of punishment of the same act, provided that he is charged with different offenses or the offense charged in one case is not included in the crime charged in the other case

Melo v People (1960) Facts: Petitioner Conrado Melo was charged with frustrated homicide for having allegedly inflicted upon Bejamin Obillo several serious wounds. Two days later, the accused pleaded not guilty, and at the evening of the same day Obillo died from his wounds. An amended information was filed charging accused with consummated homicide.

Petitioners argument: Court should be enjoined from entertaining the amended information as it would result to double jeopardy.
Issue: Whether or not there was double jeopardy.

CONSTI II DIGESTS: 1D COMPILATION

CHUA, ALIMANGOHAN, CONTRERAS, BRIONES, LENCIO, RELOJO, TENGCO, LIM, DUMA, DANAO, MELLA, TONGSON

!%&
RATIO: Melo vs. People expressly repealed the ruling in People v. Tarok and People vs. Manolong. Doctrine in Melo vs. People: where after the first prosecution a new fact supervenes for which the defendant is responsible, which changes the character of the offense, together with the facts existing at the time, constitutes a new and distinct offense but the accused cannot be said to be in second jeopardy for the new offense. In the instant case, no new supervening fact has existed or occurred which transformed the offense from less serious physical injuries to serious physical injuries. Failure to disclose new fact (in x-ray) may be attributed to the examining physician. First examination was superficial as no xray examination was made. Solicitor Gen: People vs. Manolong shall be applied (accused was charged with Less serious physical injuries and serious physical injuries and SC held that there was no double jeopardy. In said case, the course of the healing of an injury may not be determined beforehand; it was only after the healing period when the examination showed that there was deformity and loss of the use of right of hand; therefore, there was a supervening event). No supervening fact had occurred in instant case which justifies the application of Melo and Manoling ruling, for which, double jeopardy must be applied. Thorough physical and medical exam of injuries must be applied to avoid such instances. Because of the constitutional provision of double jeopardy, the accused cannot be held to answer for graver offense committed. NOTE: PEOPLE VS. ADIL (1977) : Accused was convicted of physical injuries which would require healing of 5-9 days. Another case was filed as the victim developed a permanent scar and deformity on the face. DOUBLE JEOPARDY? NO. Scar and deformity were supervening facts not in existence at the time of the first charge and could not have been foreseen. PEOPLE vs. CITY COURT (1983) : Gapay was convicted with serious physical injuries through reckless imprudence. The victim died. Another info was filed. Gapay pleaded double jeopardy. Does supervening event apply? NO. No new facts supervened the arraignment and conviction of accused. SEC 22: EX POST FACTO LAWS

"

Held: NO. Ratio: Double Jeopardy When a person is charged with an offense and the case is terminated either by acquittal or by conviction or in any other manner without consent of the accused, the latter cannot again be charged with the same or identical offense. Same evidence test o Rules of Court: There is identity between two offenses not only when the 2nd offense is exactly the same as the 1st, but also when the 2nd offense is an attempt to commit the first, or a frustration thereof, or when it necessarily includes or is necessarily included in the offense charged in the information. o One who has been charged with an offense cannot be again charged with the same or identical offense though the latter be lesser or greater than the former. Rule of identity does not apply, however, when the second offense was not in existence at the time of the first prosecution. o Exs. Diaz case and People v Espino: accused was charged with physical injuries and after conviction the injured person dies, the charge for homicide against the same accuse does not put him twice in jeopardy. (cases very similar with the case at bar!) ! When the Rules of Court were drafted, there was no intention of abandoning Diaz ruling, since after the Rules were approved, the ruling was reiterated in a later case (Espino). Rule 106, Section 13 rightly allows the information to stand. Under this provision, it was proper for the court to dismiss the first information and order the filing of a new one for the reason that the proper offense was not charged in the former and the latter did not place the accused in a second jeopardy for the same or identical offense. RULE: Where after the first prosecution a new fact supervenes for which the defendant is responsible, which changes the character of the offense and, together with the facts existing at the time, constitutes a new distinct offense, the accused cannot be said to be in 2nd jeopardy if indicted for the new offense.

People vs. Buling (1960) FACTS: Buling was found guilty of less serious physical injuries having inflicted wounds on Balaba. According to the complaint, wounds would require medical attendance for 1015 days. Balabas injuries however did not heal in the estimated period so another information was filed against the accused charging him for serious physical injuries. Accused was found guilty of serious physical injuries. ISSUE: WON the prosecution and conviction of Balaba for less serious physical injuries is a bar to the prosecution for serious physical injuries HELD: YES. No new fact supervened which changes the character of the offense into one which was not in existence at the time the case of less serious physical injuries was filed.

Lacson v. Executive Secretary (1999) Facts: Eleven persons believed to be members of the Kuratong Baleleng gang, an organized crime syndicate involved in bank robberies, were slain by elements of the Anti-Bank Robbery and Intelligence Task Group (ABRITG). Among those included in the ABRITG were petitioners and petitioner-intervenors. Acting on a media expose of SPO2 Eduardo delos Reyes, a member of the Criminal Investigation Command, that what actually transpired was a summary execution and not a shoot-out between the Kuratong Baleleng gang members and the ABRITG, Ombudsman Aniano Desierto formed a panel of investigators to investigate the said incident. Said panel found the incident as a legitimate police operation.

CONSTI II DIGESTS: 1D COMPILATION

CHUA, ALIMANGOHAN, CONTRERAS, BRIONES, LENCIO, RELOJO, TENGCO, LIM, DUMA, DANAO, MELLA, TONGSON

!%'
controlled corporations, in relation to their office as may be determined by law. The said special court is retained in the new (1987) Constitution under the following provisions in Article XI, Section 4: Sec. 4. The present anti-graft court known as the Sandiganbayan shall continue to function and exercise its jurisdiction as now or hereafter may be provided by law

"

However, a review board modified the panels finding and recommended the indictment for multiple murder against twenty-six respondents including herein petitioner, charged as principal, and herein petitioner-intervenors, charged as accessories. After a reinvestigation, the Ombudsman filed amended informations before the Sandiganbayan, where petitioner was charged only as an accessory. The accused filed separate motions questioning the jurisdiction of the Sandiganbayan, asserting that under the amended informations, the cases fall within the jurisdiction of the Regional Trial Court pursuant to Section 2 of R.A. 7975. They contend that the said law limited the jurisdiction of the Sandiganbayan to cases where one or ore of the principal accused are government officals with Salary Grade 27 or higher, or PNP officials with rank of Chief Superintendent or higher. Thus, they did not qualify under said requisites. However, pending resolution of their motions, R.A. 8249 was approved amending the jurisdiction of the Sandiganbayan by deleting the word principal from the phrase principal accused in Section 2 of R.A. 7975. Petitoner now questions the constitutionality of Section 4 of R.A. 8249, including Section 7, which provides that the said law shall apply to all cases pending in any court over which trial has not begun as of the approval hereof. Issues: 1. 2. W/N the Sandiganbayan has jurisdiction on this case Whether or not Sections 4 and 7 of R.A. 8249 violate the petitioners right to equal protection clause of the Constitution as the provisions seemed to have been introduced for the Sandiganbayan to continue to acquire jurisdiction over the Kuratong Baleleng case. Whether or not said statute may be considered as an ex-post facto law, thereby depriving them to their right to due process as they can no longer avail of the twotiered appeal, which they allegedly have under R.A. 7975.

Petitioner and intervenors, relying on R.A. 7975, argue that the Regional Trial Court, not the Sandiganbayan, has jurisdiction over the subject criminal cases since none of the principal accused under the amended information has the rank of Superintendent or higher. On the other hand, the Office of the Ombudsman, through the Special Prosecutor who is tasked to represent the People before the Supreme Court except in certain cases, contends that the Sandiganbayan has jurisdiction pursuant to R.A. 8249. R.A 7975 (old) Sec 4. Jurisdiction The Sandiganbayan shall exercise exclusive original jurisdiction in all cases involving: a. Violations of Republic Act No. 3019, as amended, otherwise known as the AntiGraft and Corrupt Practices Act, Republic Act No. 1379, and Chapter II, Section 2, Title VII, Book II of the Revised Penal Code, where one or more of the principal accused are afficials occupying the following positions in the government, whether in a permanent, acting or interim capacity, at the time of the commission of the offense: xxx R.A. 8249 (new) Sec. 4. Jurisdiction The Sandiganbayan shall exercise exclusive original jurisdiction in all cases involving: a. Violations of Republic Act No. 3019, as amended, otherwise known as the Anti-Graft and Corrupt Practices Act, Republic Act No. 1379, and Chapter II, Section 2, Titile VII, Book II of the Revised Penal Code, where one or more of the accused are officials occupying the following positions in the government, whether in a permanent, acting or interim capacity, at the time of the commission of the offense: xxx

3.

Held: 1. YES. The established rule is that every law has in its favor the presumption of constitutionality, and to justify its nullification there must be a clear and unequivocal breach of the Constitution, not a doubtful and argumentative one. The burden of proving the invalidity of the law lies with those who challenge it. That burden, we regret to say, was not convincingly discharged in the present case. The creation of the Sandiganbayn was mandated in Section 5, Article XIII of the 1973 Constitution, which provides: Sec. 5. The Batasang Pambansa shall create a special court, to be known as Sandiganbayan, which shall have jurisdiction over criminal and civil cases involving graft and corrupt practices and such other offenses committed by public officers and employees including those in government-owned or

In enacting R.A. 8249, the Congress simply restored the original provisions of P.D. 1606 which does not mention the criminal participation of the public officer as a requisite to determine the jurisdiction of the Sandiganbayan.

2. NO. Petitioner and intervenors posture that Sections 4 and 7 of R.A. 8249 violate their right to equal protection of the law is too shallow to deserve merit. No concrete evidence and convincing argument were presented to warrant such a declaration. Every classification made by the law is presumed reasonable and the party who challenges the law must present proof of arbitrariness. The classification is

CONSTI II DIGESTS: 1D COMPILATION

CHUA, ALIMANGOHAN, CONTRERAS, BRIONES, LENCIO, RELOJO, TENGCO, LIM, DUMA, DANAO, MELLA, TONGSON

!%(
1. PET: In particular, it is stressed that the Senator had expressed strong sentiments against those officials involved in the Kuratong Baleleng cases during the hearings conducted on the matter by the committee headed by the Senator. Petitioner further contends that the legislature is biased against him as he claims to have been selected from among the 67 million other Filipinos as the object of the deletion of the word "principal" in paragraph a, Section 4 of P.D. 1606, as amended, and of the transitory provision of R.A. 8249. COURT: On the perceived bias that the Sandiganbayan Justices allegedly had against petitioner during the committe hearings, the same would not constitute sufficient justification to nullify an otherwise valid law. Their presence and participation in the legislative hearings was deemed necessary by Congress since the matter before the committee involves the graft court of which one is the head of the Sandiganbayan and the other a member thereof. The Congress, in its plenary legislative powers, is particularly empowered by the Constitution to invite persons to appear before it whenever it decides to conduct inquiries in aid of legislation. 2. PET: violation of the one-title-one-subject provision of the Constitution COURT: Much emphasis is placed on the wording in the title of the law that it "defines" the Sandiganbayan jurisdiction when what it allegedly does is to "expand" its jurisdiction. The expantion in the jurisdiction of the Sandiganbayan, if it can be considered as such, does not have to be expressly stated in the title of the law because such is the necessary consequence of the amendments. The requirement that every bill must only have one subject expressed in the title is satisfied if the title is comprehensive enough, as in this case, to include subjects related to the general purpose which the statute seeks to achieve. Such rule is liberally interpreted and should be given a practical rather than a technical construction. ** please check p.896 for enumeration of what an ex post facto law is. SEC 22: BILLS OF ATTAINDER People v. Ferrer Facts: In this case the constitutionality of the Anti-Subversion Act is assailed Anti-Subversion Act: outlaws the Communist Party of the Philippines and other subversive associations, and punishes any person who knowingly, willfully and by overt acts affiliates himself with, becomes or remains a member of the Party of any other similar subversive organization Argument: Anti-Subversion Act is a bill of attainder because it tars and feathers the Communist Party of the Philippines as a continuing menace to the freedom and security of the country; its existence, a clear, present and grave danger to the security of the Philippines Issue:

"

reasonable and not arbitrary when the following concur: (1) it must rest on substantial distinction; (2) it must be germane to the purpose of the law; (3) must not be limited to existing conditions only, and (4) must apply equally to all members of the same class; all of which are present in this case.
Paragraph a of Section 4 provides that it shall apply to all cases involving certain public officials and under the transitory provision in Section 7, to all cases pending in any court. Contrary to petitioner and intervenors argument, the law is not particularly directed only to the Kuratong Baleleng cases. The transitory provision does not only cover cases which are in the Sandiganbayan but also in any court. Since it is within the power of Congress to define the jurisdiction of courts subject to the constitutional limitations, it can be reasonably anticipated that an alteration of that jurisdiction would necessarily affect pending cases, which is why it has to provide for a remedy in the form of a transitory provision. 3. NO. There is nothing ex post facto in R.A. 8249. Ex post facto law, generally, provides retroactive effect of penal laws. R.A. 8249 is not a penal law. It is a substantive law on jurisdiction, which is not penal in character. Penal laws are those acts of the Legislature, which prohibit certain acts and establish penalties for their violations or those that define crimes and provide for their punishment. R.A. 7975, as regards the Sandiganbayans jurisdiction, its mode of appeal and other procedural matters, has been declared by the Court as not a penal law, but clearly a procedural statute, one which prescribes rules of procedure by which courts applying laws of all kinds can properly administer justice. Not being a penal law, the retroactive application of R.A. 8249 cannot be challenged as unconstitutional. Petitioner's and intervenors' contention that their right to a two-tiered appeal which they acquired under R.A. 7975 has been diluted by the enactment of R.A. 8249, is incorrect. The same contention has already been rejected by the court several times considering that the right to appeal is not a natural right but statutory in nature that can be regulated by law. The mode of procedure provided for in the statutory right of appeal is not included in the prohibition against ex post facto laws. R.A. 8249 pertains only to matters of procedure, and being merely an amendatory statute it does not partake the nature of an ex post facto law. It does not mete out a penalty and, therefore, does not come within the prohibition. Moreover, the law did not alter the rules of evidence or the mode of trial. It has been ruled that adjective statutes may be made applicable to actions pending and unresolved at the time of their passage. In any case; R.A. 8249 has preserved the accuseds right to appeal to the Supreme Court to review questions of law. On the removal of the intermediate review of facts, the Supreme Court still has the power of review to determine if he presumption of innocence has been convincing overcome.

*SOME MINOR ARGUMENTS

CONSTI II DIGESTS: 1D COMPILATION

CHUA, ALIMANGOHAN, CONTRERAS, BRIONES, LENCIO, RELOJO, TENGCO, LIM, DUMA, DANAO, MELLA, TONGSON

!%)
Elements of crime of joining the Communist Party of the Philippines or any other subversive association: o In case of subversive organization other than the CPP: ! Purpose of organization should be to overthrow the present Government and establish totalitarian regime ! That accused joined such organization ! He did so knowingly, willfully and by overt acts o In case of the CPP: ! That CPP continues to pursue the objectives which led to declare it to be an organized conspiracy to overthrow the government ! Accused joined the CPP ! He did so willfully, knowingly and by overt acts

"

Whether or not the Anti-Subversion Act is a Bill of Attainder? Held: The Anti-Subversion Act is NOT a bill of attainder, however the SC gave guidelines in its enforcement Ratio: -bill of attainder: legislative act which inflicts punishment without trial -constitutional ban against bills of attainder serves to implement the principle of separation of powers by confining legislatures to rule-making 1. The act, viewed in its actual operation, does not specify the Communist Party of the Philippines of its members for punishment thereof it simply declares the Party to be an organized conspiracy for the overthrow of the government for purposes of prohibition against membership in the outlawed organization Communist Party of the Philippines: term is used solely for definitional purposes focus is not on individuals but the conduct Argument of the Appellant: under the statute organizational guilt is imputed despite requirement of proof knowing membership o It is precisely the nature of conspiracy, where all who participate in the criminal covenant are liable o Contention is correct IF statute punishes mere membership devoid of any specific intent to further the unlawful goals of the Party Element of membership is what distinguishes it from guilty knowledge o It must be shown to have been acquired knowingly, willfully and by overt acts 2. Assuming that the act specifies individuals and not activities, it is not enough to render it a bill of attainder it is only when a statute applies either to named individuals or to easily ascertainable members of a group in such a way as to inflict punishment on them without a judicial trial does it become a bill of attainder New York Bryant v. Zimmerman: legislature passed law requiring every secret, oathbound society with a membership of at least twenty to register, and punishing any person who joined or remained a member of such a society failing to register 3. It is not enough that the statute specify persons or groups in order that it may fall within the ambit of prohibition against bills of attainder; it is also necessary that it must apply retroactively and reach past conduct to be a bill of attainder, it must be proved that: o it reaches past conduct ! [not proven, because] prohibition applies only to acts committed after the approval of the act o the penalties it imposes are inescapable ! [not proven, because] Act expressly provides that such renunciation shall operate to exempt such persons from penal liability, therefore penalties are not inescapable Guidelines for enforcement of the act

Justice Fernando Dissenting: Anti-Subversion Act falls within the ban of the bill of attainder clause 1. concern of state safety and security should be ignored There must be greater understanding for the governmental response to situations of that character There was insufficient appreciation of the compulsion of the constitutional commands against bills of attainder and abridgment of free speech 2. Legislative Acts that apply either to named individuals or easily ascertainable members of a group in such a way as to inflict on them punishment amounting to a deprivation of any right civil or political, without judicial trial are bills of attainder Cummings v. Missouri: provision was that lawyers, doctors, ministers and other professionals must disavow that they had ever manifested a desire for nations enemies or a sympathy; SC condemned provision as a bill of attainder because it singles out the people to be punished United States v. Brown: making it a crime for a member of Communist Party to serve as an officer or an employee of a labor union Communist Party v. Subversive Activities Control Board: statute requiring the Communist Party of the United States to register was upheld to be valid by the US SC because it attaches to its activities and not its members It is clear that the very title of the Act to outlaw the Communist Party of the Philippines and other similar associations is tainted with invalidity Note:

Anti-Subversion Act was repealed by the Revised Anti-Subversion Law o However, Revised Anti-Subversion Law provides that acts committed before PD 885 shall be prosecuted in accordance with the Anti-Subversion Act Revised Anti-Subversion law in turn has been repealed, but the constitutional doctrine in Ferrer remains

PEO V SANDIGANBAYAN

CO V. COURT OF APPEALS

CONSTI II DIGESTS: 1D COMPILATION

CHUA, ALIMANGOHAN, CONTRERAS, BRIONES, LENCIO, RELOJO, TENGCO, LIM, DUMA, DANAO, MELLA, TONGSON

!%*

"

- E N D (of 1st year huhu) -

Cultivate the habit of being grateful for every good thing that comes to you, and to give thanks continuously. And because all things have contributed to your advancement, you should include all things in your gratitude. ! Ralph Waldo Emerson
Goodluck!

Вам также может понравиться